Текст
                    И.М.Гелъфгат, Л.Э.Генденштейн, Л.А.Кирик
1001 ЗАДАЧА ПО ФИЗИКЕ С РЕШЕНИЯМИ
Центр «Инновации в науке, технике, образовании» 1998, Харьков-Москва
Книга содержит задачи по всему курсу элементарной физики. Они
градуированы следующим образом: задачи средней трудности, повышенной
трудности и олимпиадные задачи. К большинству задач приведены подробные
решения обучающего характера. Для итогового самоконтроля предложены тесты.
Книга предназначена для учащихся средних школ, лицеев и гимназий (в том
числе физико-математического профиля), студентов педагогических вузов,
преподавателей, а также лиц, занимающихся самообразованием.
СОДЕРЖАНИЕ
Механика
1.	Кинематика	6
2.	Законы Ньютона	17
3.	Закон всемирного тяготения	23
4.	Импульс, работа, энергия. Законы сохранения	25
5.	Статика	34
6.	Гидростатика и аэростатика	42
7.	Механические колебания и волны	46
8.	Комбинированные задачи по механике	52
Молекулярная физика
9.	Молекулярно-кинетическая теория. Свойства газов	62
10.	Пары, жидкости и твердые тела	70
11.	Законы термодинамики	77
Электричество и магнетизм
12.	Электростатика	86
13.	Законы постоянного тока	99
14.	Электрический ток в различных средах	116
15.	Магнитное поле. Сила Лоренца	119
16.	Электромагнитная индукция	124
17.	Электромагнитные колебания и волны. Переменный ток	131
Оптика
18.	Законы геометрической оптики	140
19.	Оптические системы и приборы	145
20.	Волновая оптика	154
Теория относительности и атомная физика
21.	Релятивистская механика	162
22.	Кванты, атомы, ядра, частицы	165
Тесты	172
Ответы, указания, решения	185
Приложения	588

ТЕМ, КТО ХОЧЕТ НАУЧИТЬСЯ РЕШАТЬ ЗАДАЧИ ПО ФИЗИКЕ Ньютон писал, что примеры при обучении полезнее правил. В справедливости этих слов убеждается каждый, кому приходится учить или учиться. Поэтому самый эффективный способ научить решать задачи — это просто показывать, как они решаются, а самый эффективный способ научиться решать задачи — это просто их решать! Но что делать, если «просто решать» не получается? В таком случае советуем начать с изучения решений задач и разобраться на примерах, «как это делается». И пусть вас не смущает, что вы изучаете готовые решения, а не решаете сами: «аппетит придет во время еды», и вам захочется попробовать срои силы. А желание решать задачи — это главное условие для их решения! В первую очередь попробуйте решать задачи без звездочек — это задачи сред- него уровня трудности. Одной звездочкой отмечены задачи повышен- ной трудности, двумя звездочками — олимпиадные. Задачи среднего уровня трудности подобраны так, что они достаточно полно представ- ляют весь курс элементарной физики; поэтому те читатели, которые не ставят себе целью углубленную подготовку по физике, вполне могут ограничиться задачами без звездочек. В то же время задачи со звездочками предоставляют обширный материал для занимающихся в специализированных классах, лицеях и гимназиях, а также для подготовки к олимпиадам по физике. При изложении решений многих задач мы ставили целью пройти весь путь вместе с читателем: искали, с чего начать решение (часто это — самое трудное), подробно проводили расчеты, анализировали результаты, а также старались предостеречь от типичных ошибок. После задач приведены тесты, которые помогут вам самостоятель- но оценить свой уровень подготовки. (В последние годы тесты все чаще используются при проверке знаний в школах и вузах.) Величины, указанные на рисунках к условиям задач, считаются заданными. Для удобства читателей в книге приведен справочный материал, в том числе математическое приложение, содержащее наиболее упот- ребительные приемы приближенных вычислений и часто используе- мые формулы. Некоторые задачи по механике были подсказаны нам бароном Мюнхаузеном. Условия этих задач мы честно изложили от его имени и отметили особым значком. (В других разделах физики барон, видимо, еще не разобрался.) Распределение материала между авторами было следующим: — И. М. Гельфгат и, Л. Э. Генденштейн — задачи и решения; — И. М. Гельфгат и Л. А. Кирик — те(|ту. . ; j •; s 1।; • - •
Равномерное прямолинейное движение s = vt at2 Равноускоренное движение s = Vot + и V2 Центростремительное ускорение а = -= К I закон Ньютона F = О ♦-> v = const II закон Ньютона F = та III закон Ньютона F12 = -F21 п „ ТП1ТП2 Закон всемирного тяготения F = G -2 * R Импульс р — mv „г ти2 Кинетическая энергия Ич = £t kx^ Потенциальная энергия WP = mgh, WP = I условие равновесия ^F - О II условие равновесия У'М = О Сила Архимеда Fa = Гармонические колебания а = - <в2х, х = Асов(<в£ + фо) Математический маятник Т = 2iv/l/g Груз на пружине Т = 2ivlrn/k
МЕХАНИКА КИНЕМАТИКА ЗАКОНЫ НЬЮТОНА ЗАКОН ВСЕМИРНОГО ТЯГОТЕНИЯ ИМПУЛЬС, РАБОТА, ЭНЕРГИЯ ЗАКОНЫ СОХРАНЕНИЯ СТАТИКА ГИДРО- И АЭРОСТАТИКА МЕХАНИЧЕСКИЕ КОЛЕБАНИЯ И ВОЛНЫ КОМБИНИРОВАННЫЕ ЗАДАЧИ ПО МЕХАНИКЕ.
Задачи 6 1. КИНЕМАТИКА Прямолинейное равномерное движение f* 1.1. Когда моя любимая лошадь подворачивает ногу, я обычно взваливаю лошадь на себя, и мы продолжаем движение, но медленнее: когда я вверху, наша скорость щ = 120 км/ч, а когда я внизу, ог = 30 км/ч. Чему равна наша средняя скорость, если: а) я еду полпути, а потом несу лошадь? б) я еду половину времени, а потом несу лошадь? 1.2. Против течения мы плывем медленнее, чем в стоячей воде; зато по течению — быстрее. Возникает вопрос: где удастся скорее проплыть одно и то же расстояние туда и обратно — в реке или в озере? 1.3* . Автомобиль проехал половину пути со скоростью Vi — 60 км/ч. Половину оставшегося времени движения он ехал со скоростью v2 = 15 км/ч, а последний участок пути — со скоростью Уз = 45км/ч. Чему равна средняя скорость автомобиля на всем пути? <1.4. Недавно я разминался, бегая вдоль железной дороги. На- встречу мне промчались два поезда — один через t = 6 мин после другого. Я знал, что они оба идут Со скоростью и = 60 км/ч, причем второй поезд отправился со станции через t = 10 мин после первого. Я тут же достал блокнот и ручку и прямо на бегу вычислил по этим данным свою скорость V. Если и вы сможете ее определить, то увидите, что бегаю я неплохо! 1.5. Моторная лодка проходит расстояние между двумя пунктами А и В по течению реки за время ti = 3 ч, а плот — за время t = 12 ч. Сколько времени ti затратит моторная лодка на обратный путь? 1.6. Эскалатор поднимает стоящего человека за ti = 1 мин; если эскала- тор стоит, а человек идет по нему сам, на тот же подъем уходит tz = 3 мин. Сколько времени понадобится на подъем, если человек будет идти по движущемуся эскалатору? 1.7. Человек, идущий вниз по опускающемуся эскалатору, затра- чивает на спуск 1 мин. Если человек будет идти вдвое быстрее, он затратит на 15 с меньше. Сколько времени он будет спускаться, стоя на эскалаторе?
7 Механика 1.8. Самолет летит из пункта А в пункт В и обратно со скоростью v = 390 км/ч относительно воздуха. Пункты А и В находятся на расстоянии s = 1080 км друг от друга. Сколько времени потратит самолет на весь полет, если на трассе полета непрерывно дует ураганный ветер со скоростью и = 150 км/ч? Рассмотрите два случая: а) ветер дует вдоль прямой АВ; б) ветер дует под прямым углом к прямой АВ. 19‘ Однажды, стоя под дождем, я задумался: чему равна ско- ejv.V Рость капель? Прежде всего я взял отвес и убедился, что дат капли падают строго вертикально. Затем я сел в карету и обнаружил, что во время езды следы капель на стекле кареты наклонены под углом а — 60° к горизонту. Тогда я спросил у лошади, с какой скоростью едет карета, и узнал, что ее скорость и — 30 км/ч. Тут же я рассчитал скорость v капель относительно земли. Чему она равна? 1.10. Идет отвесный дождь. Скорость капель и. По асфальту со ско- ростью v катится мяч. Другой такой же мяч лежит неподвижно. На какой мяч попадает больше капель? Во сколько раз? 1.11. На лодке переплывают реку, отправляясь из пункта А (см. рисунок). Скорость лодки в стоячей воде v = 5 м/с, скорость течения реки и = 3 м/с, ширина реки s = 200 м. В какой точке пристанет лодка к противоположному берегу, если держать курс перпендикулярно берегам? Какой курс следует держать, чтобы попасть в точку В? Для обоих случаев опре- делите время переправы. 1.12. -------------------т------------------ I I I -------------------1------------------ Рыбак плыл на моторной лодке по реке, зацепил шляпой за мост, и она свалилась в воду. Рыбак поплыл дальше, но через полчаса солнце так нагрело ему голову, что пришлось повернуть обратно за шляпой. Лодка догнала ее на 4 км ниже моста. Чему равна скорость течения реки? В какую сторону плыл вначале рыбак — по течению или против? 1.13* . 1 Ъ Два автомобиля двигались с постоянными ‘ Т vt скоростями Ui и и2 по дорогам, пересекающим- дТ —.""?g ся под. прямым углом (см. рисунок). Когда I < - —J
Задачи 8 первый из них достиг перекрестка, второму оставалось проехать до этого места расстояние I. Спустя какое время t расстояние между автомобилями будет наименьшим? Чему равно это рассто- яние Smin? 1.14* *. Между пунктами А и В, находящимися на противоположных берегах реки, курси- рует катер. При этом он все время нахо- дится на прямой АВ (см. рисунок). Точки А и В находятся на расстоянии s = 1200 м друг от друга. Скорость течения реки и — 1,9 м/с. Прямая АВ составляет с направлением течения реки угол а = 60°. С какой скоростью v относительно воды и под какими углами pi и р2 к прямой АВ должен двигаться катер в обе стороны, чтобы пройти из А в В и обратно за время t = 5 мин? 1.15* . От пристани А к пристани В по реке плывет лодка со скоростью Vi = 3 км/ч относительно воды. От пристани В по направлению к пристани А одновременно с лодкой отходит катер, скорость кото- рого относительно воды Ог = 10 км/ч. За время движения лодки между пристанями катер успевает пройти это расстояние четыре раза и прибывает к В одновременно с лодкой. Определите направ- ление и скорость течения реки. 2Н 1.16* Записывая свои воспоминания, ш.и я засиделся до поздней ночи при свечах. Обе свечи одинаковой длины I я зажег одновременно и поставил, как показано на рисунке. Скоро я заметил, что тень первой свечи на левой стене неподвижна, а тень вто- рой свечи на правой стене укорачивает- ся со скоростью и. Я тут же определил, когда я останусь при одной свече и когда — в полной темноте. Попробуйте и вы ответить на эти вопросы. 1.17*. Каким будет ответ в задаче 1.16, если тень на левой стене поднимается со скоростью щ, а на правой — опускается со скоростью С>2? 1.18**. Человек находится на берегу озера в точке А к хочет в кратчайшее время попасть в точку В, находящуюся на озере
9 Механика (см. рисунок). Скорость движения человека в воде щ, а по берегу v2. По какой траектории следует двигаться человеку, если > Щ? 1.19**. По прямому шоссе со скоростью щ = 16 м/с движется автобус. На расстоянии d = 60 м от шоссе и s = 400 м от автобуса находится человек. Человек может бежать со скоростью и2 = 4 м/с. В каком направлении он должен бежать, чтобы успеть «перехватить» автобус, который к нему приближается? При какой наименьшей скорости человека v2min это вообще возможно? В каком направ- лении следует при этом бежать? 1.20*. Автобус движется по прямому шоссе со скоростью щ. Человек может бежать с меньшей скоростью и2. Определите геометрическое место точек, в которых может находиться первоначально человек, чтобы успеть «перехватить» автобус. 1.21*. Две прямые, пересекающиеся под углом а (см. рисунок), движутся перпендикулярно самим себе со скоростями щ .и v2. Определите скорость v точки пересечения прямых. 1.22**. Атомное ядро летит со скоростью v и распадается на два одинако- вых осколка. Определите максимально возможный угол а между скоростью ядра и скоростью осколка, если известно, что при распаде покоящегося ядра каждый из осколков приобретает ско- рость и. 1.23**. В каких пределах может изменяться угол 0 разлета осколков (см. задачу 1.22)?
Задачи 10 Прямолинейное равноускоренное движение 1.24. Докажите, Что при прямолинейном равноускоренном движении без начальной скорости выполняется «закон нечетных чисел»: пути, проходимые телом за последовательные равные промежутки времени, относятся, как последовательные нечетные числа: Si : s2 : . . . : sn = 1 : 3 : . . . : (2п - 1). 1.26. Свободно падающее тело прошло последние s = 30 м за время т = 0,5 с. С какой высоты Н падало тело? 1.26. В последнюю секунду свободного падения тело прошло половину своего пути. С какой высоты Н и какое время t падало тело? 1.27. От движущегося поезда отцепляют последний вагон. Поезд про- должает двигаться с той же скоростью. Сравните пути, пройден- ные поездом и вагоном к моменту остановки вагона. Ускорение вагона можно считать постоянным. 1.28. На наклонную плоскость после начального толчка снизу вверх вкатывается шарик. На расстоянии I = 30 см от начала пути шарик побывал дважды: через й = 1си#2 = 2с после начала движения. Считая движение равноускоренным, определите начальную ско- рость Vo и ускорение а. 1.29. Тело падает с высоты 100 м без начальной скорости1’. За какое время тело проходит первый и последний метры своего пути? Какой путь проходит тело за первую секунду своего движения? За последнюю? 1.30. С каким промежутком времени оторвались от карниза крыши две капли, если спустя 2с после начала падения второй капли рассто- яние между каплями равно 25 м? 1.31. Тело брошено вертикально вверх с начальной скоростью по. Когда оно достигло высшей точки траектории, из той же начальной точки с той же начальной скоростью брошено вверх другое тело. На какой высоте h они встретятся? Здесь и далее в разделе 1 при решении задач можно пренебречь сопротивлением воздуха.
11 Механика 1.32. Два тела брошены вертикально вверх из одной и той же точки с одинаковой начальной скоростью оо = 19,6 м/с с промежутком времени т = 0,5 с. Через какое время t после бросания второго тела и на какой высоте h тела встретятся? 1.33. Тело начинает свободно падать с высоты Н = 45 м. В тот же момент из точки, расположенной на высоте h — 24 м, бросают другое тело вертикально вверх. Оба тела падают на землю одновременно. Определите начальную скорость п0 второго тела, приняв g —10 м/с2. 1.34*. Лифт начинает подниматься с ускорением а — 2,2 м/с2. Когда его скорость достигла v = 2,4 м/с, с потолка кабины лифта начал падать болт. Чему равны время t падения болта и перемещение болта при падении относительно Земли? Высота кабины лифта Н = 2,5 м. 1.35* . На клин, плоскость которого составляет угол , 4 А . а с горизонтом (см. рисунок), положили тело А. Какое ускорение а необходимо сообщить клину в горизонтальном направлении, чтобы «выбить» клин из-под тела, т. е. чтобы тело А свободно Падало? 1.36. Лыжник скатился с горы длиной Si = 60 м за = 15 с, а затем проехал по горизонтальному участку еще s2 = 30 м до остановки. Найдите скорость щ лыжника в конце спуска и ускорение аг на горизонтальном участке. Постройте график зависимости скорости от времени. 1.37* . Поезд прошел расстояние между двумя станциями 8 = 17 км со средней скоростью 1>ср = 60 км/ч. При этом на разгон в начале движения и торможение перед остановкой он потратил в общей сложности ti = 4 мин, а остальное время двигался с постоянной скоростью V. Чему равна эта скорость? 1.38* С высоты Н на упругую горизонтальную плиту свободно падает стальной шарик. Постройте графики зависимости высоты и ско- рости шарика от времени. Продолжительностью соударения с плитой можно пренебречь. 1.39* . На рисунке приведен график зависимости vx(t) для тела, движу-
Задачи 12 щегося вдоль оси х. Постройте графики зависимости от време- ни ускорения ах, перемещения sx и пройденного пути I. Криволинейное движение 1.40. Определите скорость v и ускорение а точек земной поверхности в Харькове за счет суточного вращения Земли. Географические координаты Харькова: 50° северной широты, 36° восточной долго- ты. Радиус Земли 2? = 6400 км. 1.41. Тело брошено с начальной скоростью ь>о под углом а к горизонту. Сколько времени длится полет? На каком расстоянии от места бросания упадет тело? При каком значении угла а дальность полета будет наибольшей? Найдите уравнение траектории тела. 1.42. С отвесного берега высотой h произведен выстрел в горизонталь- ном направлении. Начальная скорость пули равна v0. Определите модуль й направление скорости пули v при вхождении в воду. 1.43. Под каким углом а к горизонту нужно направить струю воды, чтобы высота ее подъема была равна дальности? 1.44. Снаряд вылетает из дальнобойной пушки с начальной скоростью i>o — 1000 м/с под углом а = 30” к горизонту. Сколько времени t снаряд находится в воздухе? На какую высоту Н поднимается? На каком расстоянии L от пушки он упадет на землю? 1.45. Камень бросают горизонтально с вершины горы, склон которой образует угол а с горизонтом. С какой скоростью v0 нужно бросить
13 Механика камень, чтобы он упал на склон горы на расстоянии L от вершины? 1.46. Самолет летит горизонтально на высоте h со скоростью v. Летчик должен сбросить груз в точку А, находящуюся на земле впереди по курсу самолета. На каком расстоянии s (по прямой) от точки А следует произвести сброс? 1.47 . Два тела (1 и 2) падают с одной и той же высоты Н. На пути тела 2 находится расположенная под углом 45° к горизонту площадка, от которой это тело упруго отражается. Как различаются времена и конечные скорости падения этих тел? На какой высоте надо разместить площадку, чтобы отскочившее от нее тело упало как можно позже? 1.48 . Под углом а = 60” к горизонту брошено тело с начальной ско- ростью ио = 20 м/с. Через какое время t оно будет двигаться под углом р = 45° к горизонту? 1.49 *. Из шланга, лежащего на земле, бьет под углом а = 45° к горизонту вода с начальной скоростью и0 = 10 м/с. Площадь сечения отвер- стия шланга S = 5 см2. Определите массу т струи, находящейся в воздухе. 1.50 **. Какую наименьшую начальную скорость должен получить при ударе футбольный мяч, чтобы перелететь через стену высотой Н, находящуюся на расстоянии s? 1.51 *. Как направлено ускорение шарика, раскачива- ющегося на нити (см. рисунок), в точках А, В, . С? Точка А — крайняя точка траектории. Для Jp точки В достаточно дать качественный ответ. ‘ у " "в Комбинированные задачи по кинематике 1.52 *. Человек, стоящий на крутом берегу озера, тянет за веревку нахо- дящуюся на воде лодку. Скорость, с которой человек выбирает веревку, постоянна и равна v0. Какую скорость v будет иметь лодка в момент, когда угол между веревкой и вертикалью равен а? 1.53 *. х Концы каната А и В (см. рисунок) тянут вниз с одинаковой
Задачи 14 скоростью v. Какую скорость и имеет груз в тот момент, когда угол между канатами в точке его закрепления равен 2а? 1.54 *. Нижний край опирающейся о стену лестницы скользит по полу со скоростью 2 м/с. Определите скорость верхнего края лестницы, когда она образует со стеной угол а = 60”. В А Трактор движется со скоростью v = 36 км/ч. С какой скоростью (см. рисунок) движутся относительно Земли: а) точка А на нижней части гусениц; б) точка В на верхней части гусениц; в) точка С? 1.56. с I Сплошной диск радиусом R катится без проскальзывания с постоянной скоро- стью v по горизонтальной поверхнос- / \ ти (ем. рисунок), а) Определите модули / v \ и направления скоростей и ускорений Вт 01 ID точек А, В, С, D на ободе диска от- \ / носительно неподвижного наблюдателя. \ / б) Какие точки дисйа имеют ту же по х. модулю скорость, что и центр диска О? '777777777777^7777777 1.57*. А Катушка с намотанной на ней нитью лежит на горизонтальном столе и может катиться по нему без скольженйя. Внутренний радиус катушки равен г, внешний R. С какой скорЬстыо и будет перемещаться ось катушки, если конец нити тянуть в горизон- тальном направлении со скоростью о? Рассмотреть два случая (см. рис. а я б). 1.58*. Выпущенный вертикально вверх снаряд в верхней точке траек- тории разорвался на множество мелких, осколков, разлетающихся
IS Механика с одинаковой по модулю начальной скоростью Оо в разные сторо- ны. Как будет меняться со временем форма «облака» из осколков? / Л гл\ 1 It \ ( (/о) ) ( ( *о) ) \ 7 ! г \ ] 777^'77'77'77 Z7 7^7^777 К задаче 1.57, рис. а К задаче 1.57, рис. б 1.59*. С высоты Н на наклонную плоскость, образующую угол а с горизонтом, свободно падает мяч и упруго отражается. Определите расстояния между местами 1-го и 2-го, 2-го и 3-го, . . ., n-го и п+1-го ударов о плоскость. 1.60*. Два автомобиля движутся друг за другом по дороге с одинаковой скоростью v = 72 км/ч. При каком минимальном расстоянии I между ними камешек, застрявший между сдвоенными шинами переднего грузового автомобиля, не может попасть на задний автомобиль? 1.61*. На киноэкране демонстрируется движущаяся повозка. Радиус колес R = 0,4 м, каждое колесо имеет N = 6 спиц. Съемка производилась со скоростью 24 кадра в секунду. При какой минимальной скорости движения повозки колеса на экране будут казаться: а) вращающимися «не в ту» сторону; б) неподвижными относительно повозки? 1.62**. Четыре черепахи находятся в углах квадрата со стороной а. Черепахи начинают двигаться одновременно с одинаковой и пос- тоянной по модулю скоростью v. При этом первая черепаха все время держит курс на вторую, вторая — на третью, третья — на четвертую, четвёртая — на первую. Через какое время t черепахи встретятся? Ответьте на тот же вопрос для трех черепах, нахо- дящихся в углах правильного треугольника со стороной а. fl.63**. Однажды на охоте я наблюдал забавную сцену. Беспечный заяц, ничего вокруг не замечая, бежал с постоянной ско- ростью по прямой тропинке вдоль поля, а на поле на расстоянии L от тропинки сидела голодная лиса. Она
Задачи 16 увидела зайца, когда он находился в ближайшей к ней точке тропинки, и тут же пустилась в погоню. Со свойственной мне наблюдательностью я заметил, что лиса бежала с такой же по величине скоростью, что и заяц, и при этом все время «держала курс» на зайца. Через некоторое время лиса оказалась почти на тропинке и расстояние между ней и зайцем перестало меняться. Каким стало это расстояние? 1.64**. Через сколько времени t лиса (см. задачу 1.63) догонит зайца, если ее скорость и превышает скорость зайца и? 1.65*. Шкив радиусом R = 20 см приводится во вращение грузом, подвешенным на нити, постепенно сматывающейся со шкива (см. рисунок). В начальный момент груз был неподвижен, а затем стал опускаться с ускорением а = 2 см/с2. Какова угловая скорость ® шкива в тот момент, когда груз пройдет путь s = 1 м? Найдите ускорение аА точки А в этот момент. 1.66. Мальчик бьет ногой по футбольному мячу, лежащему на расстоя- нии I — 3 м от стены. При этом мяч приобретает скорость v0 = 10 м/с по направлению к стене под углом а = 45° к горизонту. Когда происходит удар мяча о стену — при подъеме или опускании? На какой высоте й? Чему равна скорость v при ударе? Где упадет мяч? Удар мяча о стену можно считать упругим. 1.67**. Муха заметила на столе каплю меда, пролетая точно над ней горизонтально со скоростью vQ на высоте Н. Как надо двигаться мухе, чтобы как можно быстрее добраться до меда? Сколько времени t для этого понадобится? Считайте, что муха способна развивать ускорение а в любом направлении. j ZZZZ2^ZZZZ А I Ifl □ К задаче 1.65 К задаче 1.68
17 Механика 1.68*. С какой скоростью v должен ехать автомобиль, чтобы сорвав- шийся с его колеса в точке А (см. рисунок) застрявший в шине камешек попал в ту же точку колеса, от которой оторвался? Радиус колеса R = 20 см. 2. ЗАКОНЫ НЬЮТОНА f',^2.1? Я принадлежу к числу тех людей, которые умеют изменять свой вес почти мгновенно. Для этого мне достаточно войти в кабину лифта и нажать кнопку. Каков, по-вашему, мой вес Р в тот момент, когда скорость лифта направлена вверх и равна v = 1 м/с, а ускорение направлено вниз и равно а = 1,8 м/с2? Моя масса т = 80 кг. 2.2. Автомобиль массой т = 3,3 т проходит со скоростью v = 54 км/ч по выпуклому мосту, имеющему форму дуги окружности ра- диусом R — 75 м. С какой силой автомобиль давит на мост, проходя его середину? С какой силой автомобиль давил бы на середину вогнутого моста с таким же радиусом кривизны? ,2.3*. На подставке лежит груз, прикрепленный легкой пружиной к потолку. В начальный момент пружина не растянута. Подставку начинают опускать вниз с ускорением а. Через какое время t груз оторвется от подставки? Жесткость пружины k, масса груза т. 2.4. Человек, стоя на платформе весов, быстро приседает и выпрям- ляется. Как изменяются в ходе этого движения показания весов? Л5.) К концам шнура, перекинутого через блок, подвешены грузы с массами mi = 100 г и m2 = 150 г. Определите ускорения грузов, силу натяжения шнура Т и показание F динамометра, на котором висит блок. Шнур можно считать невесомым и нерастяжимым; массой блока и трением в оси блока можно пренебречь. 2.6* . Через невесомый блок, укрепленный на ребре призмы (см. рисунок), перекинута /у ^'^УЧ невесомая нить с грузами на концах. Ljmi ^'"ч'У Определите ускорение грузов а и силу В>4 натяжения нити Г. Трением пренебречь. ,1ч
Задачи 18 2.7. Определите ускорения щ и аг показанных на рисунке грузов и силу натяжения Т нерастяжимой нити. Массой блоков и нити можно пренебречь; трение отсутствует. 2.8. На однородный стержень длины I действуют две силы Fi и F2, приложенные к его концам и направленные в противоположные стороны (см. рисунок). С какой силой F растянут стержень в сечении, находящемся на расстоянии х от одного из его концов? 2.9. В вагоне поезда, движущегося со скоростью v = 72 км/ч, взве- шивают на пружинных весах тело массой m = 5 кг. Определите показание Р пружинных весов, когда поезд движется по закруг- лению радиусом R = 400 м. 2.10. С какой силой Р давит гонщик на кресло гоночного автомобиля на вираже, если масса гонщика пг = 70 кг, скорость автомобйля v = 200 км/ч, радиус закругления дороги R = 50 м? Во сколько рад эта сила превосходит вес неподвижного гонщика? На гладком столе лежат два бруска (см. рисунок) с массами Ш = 400 г и гпг = 600 г. К одному из них приложена горизон- тальная сила F = 2 Н. Определите силу Т натяжения нити, если сила приложейа: а) к первому бруску; б) ко Второму бруску. 2.12*. Два груза с массами mi и тп2, связанные легким шнуром, лежат на горизонтальйой поверхности. Шнур выдерживает силу натя- жения Т. Коэффициент трения между каждым из грузов и повер- хностью равен ц. С какой силой F можно тянуть первый груз
19 Механика параллельно шнуру, чтобы шнур не разорвался? В начальный момент шнур не натянут. 2.13*. Два груза с массами mi и т2 связаны легкой нитью, перекинутой через не- подвижный блок (см. рисунок). Груз массы mi отпускают без толчка. С каким ускорением а относительно стола движутся грузы, если коэффициент трения второго груза о стол равен ц? Какова сила Т натяжения^ити? Как изменится ответ, если^док* система находится в лифте, движущемся с ускорением а0, направленным вверх? 2.14*. Чугунное ядро массой т падает в воде с постоянной скоростью v. С какой силой F надо тянуть его вверх, чтобы оно поднималось со скоростью 2и? Сила сопротивления прямо пропорциональна ве- личине скорости. 2-15. _ Из верхней точки вертикального диска радиуса R прорезан желоб (см. рисунок). Как зависит от угла 0 время t скольжения грузика по желобу? Трением пренебречь. 2.16. Тело соскальзывает без начальной скорости с наклонной плос- кости. Угол наклона плоскости к горизонту а = 30”, длина наклонной плоскости Z = 2 м. Коэффициент трения тела о плос- кость ц — 0,3. Каково ускорение тела? Сколько времени длится соскальзывание? 2.17*. В показанной на рисунке системе а = 20”, mi = 2 кг, т2 = 1 кг; коэффициент трения между первым грузом и наклонной плос- костью р.1 — 0,1. Нить и блок можно считать невесомыми, нить — К заваче 2.15 • К заОаче 2.17
Задачи 20 нерастяжимой, трением в блоке пренебречь. Грузы отпускают без начальной скорости. Определите ускорение а системы грузов и силу натяжения нити Т. Как изменится результат, если коэф- фициент трения увеличится до цг = 0,3? 2.18**. Определите ускорение системы грузов (см. задачу 2.17) при произ- вольных значениях mi, m2 и ц. В начальный момент грузы неподвижны. 2.19*. На горизонтальном участке дороги от равномерно идущего поезда массой М = 1000 т оторвался последний вагон массой m = 40 т, проехал расстояние зв = 200 м и остановился. Какое расстояние зп проехал поезд за время торможения вагона? Решить задачу в двух случаях: а) скорость поезда осталась неизменной; б) сила тяги локомотива осталась неизменной. Во всех случаях считать, что сила сопротивления пропорциональна массе. 2.20. На тележке установлен штатив, на котором подвешен шарик на нити. Тележка движется горизонтально с ускорением а. Найти угол а отклонения нити от вертикали и силу Т натяжения нити. 2.21*. На тележке стоит сосуд с водой. Тележка движется горизонтально с ускорением а. Определите форму поверхности воды в сосуде. 2.22. По наклонной плоскости, образующей угол а с горизонтом, съез- жает без трения тележка, на которой установлен штатив. К штативу подвешен на нити шарик массой т. Найти угол 0 отклонения нити от вертикали и силу Т натяжения нити. 2.23*. По наклонной плоскости, образующей угол а с горизонтом, сколь- зит без трения сосуд с водой. Найти форму поверхности воды в сосуде. 2.24**. Кирпич массой m лежит на горизонтальном столе. Коэффициент трения между кирпичом и столом равен ц. К кирпичу приложена горизонтальная сила F. а) Выразите аналитически и графически зависимость силы трения FTp и ускорения кирпича а от величины силы F. б) Сделайте то же самое для случая, когда сила F направлена под углом а к плоскости стола (учитывая случаи а > 0 и а < 0). 2.26*. Угол а наклонной плоскости с горизонталью постепенно уве-
21 Механика личивается от О до 90°. На плоскости находится ящик массы т. Коэффициент трения равен ц. Постройте график зависимости силы трения Ftp от угла а. Чему равно максимальное значение силы трения Fmax? 2.26. Если наклонить доску под углом а к горизонту, кирпич движется по ней практически равномерно. За какое время t кирпич проедет всю доску, если наклонить ее под углом 0 > а? Длина доски равна I. 2.27*. Санки толкнули вверх по ледяной горке, составляющей угол а — 30° с горизонтом. Санки въехали на некоторую высоту и спустились обратно. Время спуска tc в п = 1,2 раза превышает время подъема in. Чему равен коэффициент трения? 2.28**. На ледяном склоне, составляющем угол а с горизонтом, находится доска массой М. Как должен бежать по этой доске человек массой т, чтобы доска оставалась в покое? При каком коэффициенте трения ц между подошвами и доской это возможно? Трение между доской и льдом пренебрежимо мало. 2.29*. На диск проигрывателя на расстоянии г от оси положили монету массой т. Диск вращается с частотой п. Коэффициент трения между монетой и диском ц. Найдите зависимость силы трения, действующей на монету, от расстояния г. 2.30. Шарик на нити длиной I равномерно движется по окружности в горизонтальной плоскости (см. рисунок). При этом нить все время образует с вертикалью угол а (такую систему называют коническим маятником). Найдите период Т вращения ша- рика.
Задачи 22 2.31. К диску проигрывателя прикреплен высокий вертикальный стер- жень, а к его вершине подвешен шарик на нити длиной I = 48 см. Расстояние стержня от оси вращения диска г = 10 см (см. рисунок). После включения проигрывателя нить отклоняется от вертикали на угол а = 45°. Определите угловую скорость и частоту вращения диска. 2.32. На вираже летчик поворачивает корпус самолета вокруг направ- ления движения на угол а = 10°. Скорость полета и = 360 км/ч. Определите радиус поворота R. 2.33* *. На вертикальной оси электродвигателя укреплен отвес — ма- ленький шарик на нити длиной I — 12,5 см. При медленном вращении двигателя нить остается вертикальной, а при быстром вращении шарик движется как конический маятник. При какой частоте ni вращения двигателя нить начинает отклоняться от вертикали? Чему равен угол ее отклонения <ра при частоте вра- щения Пг = 3 с1? 2.34* . Металлическая замкнутая цепочка длиной I — 62,8 см насажена на деревянный диск (см. рисунок). Диск раскручивают с по- мощью электродвигателя. Когда частота вращения диска достигает п = 60 с-1, цепочка соскакивает с диска. Она ведет себя как жесткий обруч: может, например, катиться по столу (пока вращение не замедлится). Определите силу Т натяжения цепочки в тот момент, когда она соскакивает с диска. Масса цепочки m = 40 г.
23 Механика 3. ЗАКОН ВСЕМИРНОГО ТЯГОТЕНИЯ 3.1. Оцените массу Солнца, считая расстояние R от Земли до Солнца равным 1,5 108 км. 3.2. Оцените, во сколько раз отличаются силы притяжения вашего тела к Земле и к Солнцу. Расстояние до Солнца считайте равным 1,5 108 км. 3.3. К равноплечим рычажным весам подвешивают груз и гири с помощью невесомых нитей различной длины (см. рисунок). Оце- ните, при какой разнице h длин нитей весы будут^вать погреш- ность Атп = 0,01 г при взвешивании груза массой т = 10 кг. Радиус Земли Л = 6400 км. Вращение Земли не учитывать. К задаче 3.4 3.4* . Найдите силу F притяжения маленького шарика массой т vl большого однородного шара массой М, в котором имеется сфе- рическая полость (см. рисунок). 3.5* *. Однажды в своих космических странствиях я попал на удивительную планету: внутри она была пуста, т. е. имела форму сферической оболочки постоянной толщины. Оби- татели планеты селились на ее внутренней поверхности и, самое удивительное, перелетали из одного места в другое, чуть- чуть подпрыгнув: внутри планеты совершенно не ощущалась сила тяжести! Как это объяснить? 3.6* . Известно, что при подъеме с поверхности Земли сила притяжения F тела к Земле падает. А как обстоит дело при погружении в
Задачи 24 глубокую шахту, доходящую до центра Земли? Постройте график зависимости F(r) для тела массы т, где г — расстояние тела от центра Земли. Считайте, что плотность земного шара всюду одинакова.* 3.7. Спутник движется по круговой орбите на высоте h от поверхности Земли. Выразите скорость спутника и и период его обращения Т через й, радиус Земли R и ускорение силы тяжести на поверхности Земли g. 3.8. Чтобы установить постоянную радиосвязь через искусствен- ный спутник Земли, удобно, чтобы он все время «висел» над одной и той же точкой земной поверхности. Определите ра- диус орбиты г такого спутника, его скорость v и период обращения Т. В какой плоскости должна лежать траектория полета? 3.9. Определите скорость движения Луны относительно Земли и пе- риод ее обращения вокруг Земли. Считайте, что Луна движется по круговой орбите радиуса 384 000 км. 3.10. В момент солнечного затмения действующие на Луну силы при- тяжения Земли и Солнца направлены противоположно. Какая из этих сил больше и во сколько раз? Используйте данные, приве- денные в задачах 3.1 и 3.9. 3.11. К какому уменьшению веса тел на экваторе по сравнению с полюсом приводит вращение Земли? В каком направлении вдоль экватора и с какой скоростью v должен лететь самолет, чтобы на нем этот эффект не наблюдался? 3.12. На экваторе некоторой планеты тела весят втрое меньше, чем на полюсе. Период обращения этой планеты вокруг своей оси равен Т = 55 мин. Определите среднюю плотность р планеты. <3.13. Мне захотелось увеличить скорость суточного вращения Земли так, чтобы на экваторе ощущалась невесомость. Во сколько раз следует «укоротить» сутки? Не будет ли каких- либо неприятных побочных эффектов от этого? 3.14. Справедливы ли законы Паскаля и Архимеда в состоянии невесомости?
25 Механика 3.15. Спутник обращается по круговой орбите на небольшой высоте над планетой. Период его обращения равен Т. Достаточно ли этих данных, чтобы определить среднюю плотность планеты р? >3.16**. Летая вокруг Земли в спутнике по круговой орбите А (см. рисунок), я решил приземлиться. Одним могучим ударом в заднюю стенку кабины я уменьшил скорость спутника так, что он перешел на орбиту В, касающуюся поверхности Земли. Через какое время t после этого я приземлился? Радиус круговой орбиты был втрое больше радиуса Земли. 4. ИМПУЛЬС, РАБОТА, ЭНЕРГИЯ. ЗАКОНЫ СОХРАНЕНИЯ Закон сохранения импульса 4.1. Мяч массой тп = 300 г упал с высоты Н = 1,23 м на асфальт и подскочил на ту же высоту. Продолжительность удара об асфальт t = 0,1 с. Определите среднюю силу удара Вер. Как изменится средняя сила удара, если мяч ударится о твердую поверхность, наклоненную под углом а = 30? к горизонту? Какой будет Fcp, если в обоих случаях мяч заменить пластилиновым шаром той же массы? Продолжительность удара — та же. 4.2* . Струя сечением В = 6 см2 ударяет из брандспойта в стенку под углом а = 60? к нормали и под тем же углом упруго «отражается» от нее. Скорость течения воды в струе и = 15 м/с. С какой силой F она давит на стену? и : •
Задачи 26 <4.3. Летя на ядре со скоростью v = 50 м/с, я так удачно соскочил с него, что моя скорость относительно Земли стала равной нулю. Какова стала скорость ядра после того, как я с него соскочил? Моя масса вдвое превышает массу ядра. 4.4* . Ракета попала в пылевое облако. Пылинки оказались липкими и соударялись с ракетой неупруго. Чтобы скорость движения v относительно облака не падала, пришлось включить двигатель, развивающий силу тяги F. Как нужно изменить силу тяги двига- теля, чтобы: а) двигаться со скоростью 2и; б) сохранить скорость v неизменной при попадании в область, где концентрация частиц (число частиц в единице объема) в три раза больше? 4.5. На гладком льду лежит цилиндрическое однородное бревно дли- ной Н. К одному из его концов прикрепили веревку и стали медленно поднимать. Когда угол между бревном и поверхностью льда стал равным а, вертикально натянутая веревка оборвалась. На какое расстояние сместится при падении бревна его нижний конец? 4.6. На сколько сместится неподвижная лодка массой М = 280 кг, если человек массой т = 70 кг перейдет с ее носа на корму? Расстояние от носа до кормы I = 5 м, сопротивление воды пренебрежимо мало. 4.7. Два рыбака ловят рыбу в озере, сидя в неподвижной лодке. На сколько сместится лодка, если рыбаки поменяются местами? Масса лодки М = 280 кг, масса одного рыбака mi = 70 кг, масса второго т2 = 140 кг, расстояние между рыбаками I = 5 м. Соп- ротивлением воды пренебречь. 4.8. Масса пушки М = 800 кг. Пушка выстреливает ядро массой т = 10 кг с начальной скоростью v0 = 200 м/с относительно Земли под углом а = 60? к горизонту. Какова скорость и отката пушки? Трением пренебречь. 4.9. Обледеневший склон холма, об- ~1 разующий угол а с горизонтом, заканчивается небольшим обры- вом (см. рисунок). Под обрывом на горизонтальной площадке „ стоит тележка массы М. Какую ,, Ж М, I скорость и приобретет тележка,
27 Механика если в нее с высоты Н соскользнет с холма камень массой т? Трением пренебречь. 4.10* . На гладкой горизонтальной поверхности лежит обруч массы М и радиуса R. На обруче сидит жук массы т. По каким траек- ториям будут двигаться жук и центр обруча, если жук поползет по обручу? 4.11* . По длинному склону, образующему угол а с горизонтом, съезжает тележка, на которой установлен бак с водой. Через отверстие площадью S в задней стенке бака вытекает вода со скоростью v относительно бака. Поверхность воды установилась параллельно склону. Определите коэффициент сопротивления движению ц. Масса тележки с баком М, за время спуска вытекает лишь небольшая часть воды. f* 4.12*. Однажды в открытом космосе на моей ракете закончились запасы топлива, и мне пришлось разгонять ее, выплевывая назад через люк вишневые косточки массой тп со скоростью v относительно ракеты. Определите скорость ракеты ип после выплевывания n-й косточки. Первоначальная масса всей ракеты М, начальная скорость равна нулю. Работа, мощность, энергия 4.13. Какая работа А совершается при подъеме на крышу веревки длиной I — 40 м и массой т — 6 кг? Первоначально веревка свешивалась вертикально с края крыши. <4.14. Убив гигантскую змею, я вытянул ее поперек дороги (см. рисунок), измерил и взвесил. Длина ее оказа- Дорога Трава лась L, масса М. Чтобы освободить дорогу, мне пришлось перетащить змею на траву. Какую работу я при этом совершил? Коэффициент трения змеи о дорогу ць о траву — ц2. 4.15*. Поезд движется по прямому участку пути со скоростью и. На крышу
Задачи 28 вагона, проезжающего под мостом, опускают ящик массой т. Ящик скользит по крыше и останавливается, пройдя расстояние L относительно вагона. Определите полную работу А силы трения: а) в системе отсчета, связанной с поездом; б) в системе отсчета, связанной с Землей. Коэффициент трения между ящиком и кры- шей вагона равен ц. 4.16. Автомобиль массой М = 2 т разгоняется с места в гору с уклоном 0,02. Коэффициент сопротивления ц = 0,05. Автомобиль набрал скорость v — 97,2 км/ч на отрезке s = 100 м. Какую среднюю полезную мощность Рср развивает двигатель? 4.17* . Перед самой посадкой ракета массой М с работающим двигателем неподвижно «зависла» над землей. Скорость вытекающих из ракеты газов и. Какова мощность N двигателя? 4.18. Ведро с водой массой М = 10 кг поднимают на высоту Л = 10 м, прикладывая постоянную силу F = 200 Н. Какую работу А при этом совершают? Чему равно изменение потенциальной энергии AWP? Как согласуются эти результаты с законом сохранения энергии? 4.19. Санки массой m = 20 кг поднимают по гладкому склону на высоту h = 2,5 м, прикладывая силу F = 300 Н, направленную вдоль склона. Санки движутся с ускорением а — 3 м/с2. Какая работа А совершается при подъеме? Чему равно изменение потенциальной энергии AWP санок? Чему равна кинетическая энергия Wk санок на вершине? 4.20. Автомобиль при полностью включенных тормозах (колеса не вращаются^ может удержаться на участке горной дороги с наклоном " до а = 30°. Каков тормозной путь s этого авто- мобиля на горизонтальном участке той же дороги при скорости и = 72 км/ч? 4.21*. Бассейн, имеющий площадь S = 100 м2, разделен пополам под- вижной вертикальной перегородкой и заполнен водой до уровня Л = 2 м. Перегородку медленно передвигают так, что она делит бассейн в отношении 1 : 3. Какую работу А пришлось совершить? Вода не проникала через перегородку и не переливалась через край бассейна.
29 Механика 4.22. Автомобиль поднимается по склону со скоростью v = 60 км/ч. Спускаясь по тому же склону с выключенным мотором, он дви- жется равномерно с той же скоростью. Какую мощность Р раз- вивает двигатель на подъеме? Уклон равен 0,05; масса автомобиля т = 2 т. >4.23*. Однажды мне понадобилось вта- щить на холм ящик с пушеч- г ными ядрами (см. рисунок). Я А прицепил этот ящик тросом к своему автомобилю, который двигался JHP' с грузом довольно медленно. К моему удивлению, скорость не изменилась и '----------------— — после того, как ящик прошел край склона (точку А) и стал двигаться горизонтально. Каков коэффициент трения ц между ящиком и землей? Мощность двигателя все время оставалась неизменной. Склон холма образует угол а = 45° с горизонтом. Трением между тросом и дорогой можно пренебречь. 4.24*. Автомобиль движется вверх по пологому подъему со скоростью щ = 6 м/с и спускается по тому же пути со скоростью = 9 м/с, не меняя мощности двигателя. С какой скоростью v будет ехать этот автомобиль по горизонтальному участку той же дороги, если мощ- ность двигателя неизменна? Сопротивлением воздуха пренебречь. 4.25. Шлифовальный диск диаметром d = 30 см вращается с частотой п = 120 об/мин. Металлический брусок прижимают к цилиндричес- кой поверхности диска с силой F = 100 Н. Коэффициент трения между бруском и диском ц = 0,2. Какую мощность N развивает двигатель, вращающий диск? Закон сохранения энергии. Упругие и неупругие столкновения 4.26*. Обезьяна раскачивалась на длинной тонкой лиане так, что мак- симальный угол отклонения лианы от вертикали составлял а. Когда обезьяна находилась в нижней точке траектории, лиана зацепилась серединой за ветку дерева. Каким стал максимальный угол Р отклонения лианы от вертикали, если а < 60?? Что изменится, если 60? < а < 90??
Задачи 30 4.27*. Санки съезжают с горы, длина основания которой а = 5 м, а высота Н = 2 м. После этого они проезжают до остановки еще з = 35 м по горизонтальной площадке. Определите коэффициент трения ц, считая его одинаковым на всем пути. Переход склона горы в горизонтальную поверхность считайте достаточно плавным. 4.28. Легкий теннисный мяч ударили ногой, и он полетел в направлении движения ноги. Скорость движения ноги при ударе и = 10 м/с. Какую скорость v приобрел мяч? 4.29. За счет чего увеличивается потенциальная энергия поднимающе- гося воздушного шара? Не нарушается ли при этом закон сохра- нения энергии? 4.30*. Лежащий на гладком горизонтальном столе клип имеет массу М и высоту й. Угол наклона поверхности клина к горизонту равен а. С клина соскальзывает без трения небольшое тело массой т. Какую скорость и приобретет тело в конце спуска? Определите величину и направление скорости. 4.31. Деревянный брусок висит на веревке длиной I. В брусок выст- релили, пуля застряла в нем, и веревка отклонилась от вертикали на угол а. Какова скорость пули п0? Масса бруска М, масса пули т. Пуля летела горизонтально. >4.32*. Однажды во время охоты я увидел летящую головку сыра. Когда она села на ветку, я тут же выстрелил и, конечно, попал. Головка сыра упала на землю,.но оказалось, что она всего лишь ранена. Пришлось еще раз выстрелить в нее — сверху вниз. Когда я разрезал сыр, то обнаружил, что первая пуля вощла в сыр на глубину Si = 10 см, а вторая — на глубину 32 — 10,1 см (измерения я делал очень тщательно!). Благодаря этому я сразу же определил массу М головки сыра, даже не взвешивая ее! Как я это сделал? (Добавлю, что массу пули я знал: т = 10 г; кроме того, я предполагал, что сила сопротивления F, действующая на пулю внутри сыра, постоянна). 4.33. Человек прыгает в воду со скалы высотой й = 10 м. На какую глубину Н он бы при этом опустился, если бы можно было пренебречь силами сопротивления воздуха и воды? Масса челове- ка т = 60 кг, объем V = 66 л..
31 Механика f4.34*. Я загрузил в вагон, стоявший на кольцевом железнодорож- ном пути в точке А, мешок с порохом. От случайной искры мешок взорвался, да так, что вагон раскололся на две неравные части и они понеслись по рельсам друг от друга. Обе части столкнулись снова в некоторой точке, и буферные пружины опять «растолкнули» части вагона. При втором же ударе обе части соединились друг с другом намертво, как будто взрыва и не было! Где произошли эти столкновения? Массы частей вагона mi и ш2, длина кольцевого пути L. Сопротивлением движению частей вагона можно пренебречь. >4.35*. Обычно я стараюсь одной пулей убивать хотя бы несколько куропаток: жду, когда они в полете выстроятся по прямой линии, и стреляю вдоль нее. Однажды в линию выст- роились 20 куропаток. Я выстрелил и обратил внимание, что пуля, прострелив первую куропатку, потеряла 5% начальной скорости. Тут же я предсказал, какова будет моя добыча в этот раз. Попробуйте и вы это сделать. 4.36. Два одинаковых шара, двигавшихся со скоростями Vj и с2 вдоль одной прямой, испытывают лобовой упругий удар. Определите их скорости ui и и2 после удара. 4.37*. При нецентральном ударе биллиардного шара о такой же, но неподвижный, оба шара разлетаются всегда под одним и тем же углом. Найдите этот угол. 4.38*. Шар массой mi налетает на неподвижный шар массой т2. Про- исходит лобовое упругое соударение. Как зависит доля а передан- “ г. min нои при соударении энергии от отношения масс шаров к = —? Ш-2 Постройте график зависимости а(А). 4.39*. На сортировочной горке сталкиваются два медленно движущихся в одну сторону железнодорожных вагона. Пружины буферов смяг- чили удар и потом «растолкали» вагоны, так что удар можно считать упругим. Какова была максимальная энергия W упругой деформации пружин? Вагоны имеют массы mi и т2, их скорости до столкновения равны щ и. и2. 4.40. Шар массой тп, имеющий скорость и, налетел на покоящийся шар
Задачи 32 массой zn/2 и после упругого удара изменил направление своего движения на угол а = 3(У\ С какими скоростями стали двигаться шары после удара? 4.41*. Одинаковые биллиардные шары подве- шены на нитях равной длины, как показано на рисунке. Расстояния меж- ду соседними шарами малы. Что про- изойдет, если отклонить крайний шар и отпустить? Если проделать это сразу с двумя шарами? >4.42*. Как вы знаете, мне случалось пересаживаться в полете с одного пушечного ядра на другое, встречное. На сколько изменяется суммарная кинетическая энергия моего тела и ядер при такой пересадке? Какова причина этого изме- нения? Рассмотрите два случая: 1) остается неизменной скорость оставленного мною ядра; 2) остается неизменной скорость ядра, на которое я пересел. Ядра имеют одинаковую скорость v0 = 1000 м/с и массу М = 100 кг; моя масса т = 80 кг. 4.43*. По горизонтальной плоскости может перемещаться без трения гладкая горка высотой Н и массой М (см. рисунок). На неподвижную горку налетает скользящее по плоскости неболь- шое тело массой т. Как зависит результат столкновения от начальной скоростщи налетающего тела? При движении по горке тело не отрывается от нее. 4.44*. Докажите, что при соударении двух тел изменение кинетической энергии сис- темы не зависит от того, в какой сис- теме отсчета рассматривается процесс. 4.45*. Два упругих стальных шара массами 1 х **4 mi = 200 г и тг = 100 г подвешены рядом '"а так, что их центры находятся на одном уровне (см. рисунок). Отклонив первый К задаче 4.45
33 Механика шар на высоту Н = 18 см, его отпускают. На какую высоту поднимется каждый из шаров после удара? 4.46. Железнодорожная платформа массой М и длиной L катится со скоростью и. На передний край платформы ставят ящик массой т. Ящик скользит по платформе и останавливается у противопол- ожного края. Определите коэффициент трения ц между ящиком и платформой. 4.47*. Докажите, что кинетическую энергию системы движущихся мате- Ми; риальных точек можно представить в виде W = —+ Wгде М — суммарная масса всех материальных точек, vc — скорость центра масс системы точек, WUM — кинетическая энергия мате- риальных точек в системе отсчета их центра масс. 4.48*. Как движутся частицы после столкновения, если максимально возможная часть их кинетической энергии перешла во внутрен- нюю? 4.49*. Потенциальная энергия взаимодействия двух материальных то- С чек имеет вид Wv = —, где г — расстояние между точками. Докажите, что эти материальные точки взаимодействуют с силой F, обратно пропорциональной квадрату расстояния между ними. При каких значениях С взаимодействие является притяжением? 4.50*. Какую минимальную скорость нужно сообщить телу, чтобы оно покинуло Землю насовсем? Суточное вращение Земли не учиты- вать. Радиус Земли R = 6400 км. 4.51*. Какова кинетическая энергия Wk обруча массой т, катящегося без проскальзывания со скоростью v по плоской поверхности? 4.52*. Однородный куб надо переместить вдоль горизонтальной плос- кости на расстояние, значительно превышающее длину ребра куба. В каком случае потребуется совершить меньшую работу: при перемещении «волоком» (прикладывая силу горизонтально) или кантованием (опрокидыванием через ребро)? Коэффициент трения равен ц.
Задачи 34 5. СТАТИКА 5.1. Фонарь массой т = 10 кг висит посередине улицы шириной I = 10 м. Допустимая сила натяжения каната Т = 500 Н. На какой высоте Н могут быть закреплены концы каната, чтобы точка подвеса фонаря находилась на высоте h — 5 м? 5.2. С какой силой F надо тянуть за веревку, привязанную к ящику массой т = 40 кг и образующую угол а = 30° с горизонтом, чтобы ящик двигался по горизонтальной поверхности равномерно? Коэффициент трения между ящиком и площадкой ц = 0,27. 5.3. Ящик толкают по горизонтальной плоскости, прикладывая к нему силу, как это показано на рисунке. Масса ящика т, коэффициент -гг г> трения ц. При каком значении силы F ящик будет двигаться равномерно? 5.4* . Груз массой т необходимо равномерно перемещать по горизон- тальной плоскости. Какую минимальную силу Fmin нужно для этого приложить и под каким углом а к плоскости следует ее направить? Коэффициент трения равен ц. 5.5. Каков должен быть коэффициент трения ц для того, чтобы зако- лоченный в бревно клин не выскакивал из него? Угол при вершине клина а — ЗО9. 5.6. Каким должен быть коэффициент жесткости пружины, которая может заменить систему из двух пружин с коэффициентами жес- ткости ki и ki, соединенных: а) параллельно; б) последовательно? 5.7. Найти равнодействующую п приложенных в одной точке сил, каждая из которых равна по модулю F и образует угол 2л/п с соседними (ближайшими по направлению) силами. 5.8. Невесомые стержни АВ и ВС шарнирно закреплены в точках А,
35 Механика В, С (см. рисунок). Чему равны действующие на стержни силы, если а = 609, а масса подвешенного в точке В фонаря т = 3 кг? 5.9* . Найти силы, действующие на шарнирно закрепленные стержни ВС и АС, если АВ = 60 см, АС = 1,2 м, ВС = 1,6 м (см. рисунок). Масса груза 50 кг, массой стержней можно пренебречь. 5.10. На концах нити, перекинутой через два неподвижных блока, висят два одинаковых груза (см. рисунок). К середине нити прикрепили третий такой же груз. На какое расстояние h опус- тится этот груз после установления равновесия? Расстояние меж- ду осями блоков равно 21. Трением в осях блоков пренебречь. К задаче 5.10 ' К задаче 5.11 5.11* . К концам нити, перекинутой через два блока, подвесили два груза пц и тг (см. рисунок). Какой груз тпз надо подвесить к нити между блоками, чтобы при равновесии угол а был равен 1209? Рассмот- реть случаи: a) mi = тя = 4 кг; б) mi = 3 кг, zn2 = 5 кг.
Задачи 36 5.12* . Груз массой т.2 находится на наклонной плоскости (см. рисунок), образующей угол а с горизонтом. Коэффициент трения равен ц. На нити, привязанной к грузу и переброшенной через блок, подвешен груз массой mi. При какой величине mi система будет находиться в равновесии? 5.13* . Цепочка массой т подвешена к потолку (см. рисунок). При каком угле а сила натяжения цепочки в ее нижней точке равна весу цепочки? Чему будет равна при этом сила Т натяжения в точке подвеса? <5.14. Однажды я подстрелил удивительно большого гуся (ди- кого!) и множество обычных (совершенно одинаковых!) куропаток. Меня заинтересовало, во сколько раз гусь тя- желее куропатки. По моей просьбе слуга вырезал ровную палку и, привязав к ней веревку, сделал рычажные весы. Сначала я подвесил гуся слева; чтобы уравновесить его,справа пришлось подвесить связку из 40 куропаток. Однако когда я подвесил гуся справа, то, к моему удивлению, для уравновешивания хватило всего 10 куропаток! Я, конечно, тут же сообразил, что мой слуга сделал весы неравноплечими. Более того, я сразу определил, во сколько раз гусь на самом деле тяжелее куропатки, да к тому же — во сколько раз одно плечо самодельных весов длиннее другого. Попробуйте сделать это и вы (чтобы не затруднять вас слишком, разрешаю массой палки пренебречь). 5.15. Два муравья толкают кусок древесной коры с силами Fi = 2 м! и Fi — 4 мН. Силы направлены в одну сторону, расстояние между линиями действия сил равно 1,8 мм. В каком месте и какую силу должен приложить третий муравей, чтобы кусок коры оставался в покое? Трением можно пренебречь.
37 Механика 5.16. Каким станет ответ в задаче 5.15, если первый муравей будет не толкать кусок коры, а тянуть его на себя с той же силой? 5.17. К стержню длиной I = 120 см и массой т = 8 кг подвешены два груза: к левому концу — весом Pi = 30 Н, а к правому — весом Р2 = 90 Н. Стержень подвесили горизонтально на одной нити так, что он находится в равновесии. На каком расстоянии от левого конца стержня находится точка подвеса? 5.18. К балке массой mi = 400 кг и длиной Z = 7 м подвешен груз массой т2 = 700 кг на расстоянии а = 2 м от одного из концов. Балка своими концами лежит на опорах. Какова сила давления на каждую из опор? 5.19. Чтобы узнать массу линейки, на один из ее концов положили груз массой 250 г и начали выдвигать этот конец за край стола. Линейка находилась в равновесии до тех пор, пока ее не вы- двинули на четверть длины. Чему равна масса линейки? На- сколько можно было бы выдвинуть линейку, если бы масса груза была 125 г? 5.20. В доске сделали лунку и положили в нее шар (см. рисунок). Глубина лунки в два раза меньше радиуса шара. На какой угол а нуж- но наклонить доску, чтобы шар выскочил из - ..г лунки? 5.21. На конце вала, проходящего через д,___________ В i р подшипники А и В (см. рисунок), Г* *Т**| укреплен массивный диск С. Масса Ш Уд И вала — 7 кг, масса диска — 28 кг, гГ ~ Г И АВ = 70 см, ВС = 10 см. Чему равны силы давления вала на подшипники | | I А и В? 5.22. Доска массой т = 70 кг имеет длину I = 1,6 м. Она лежит на двух опорах, расположенных на расстояниях ах = 40 см и а2 = 20 см от ее концов. Какую вертикальную силу надо приложить к каждому концу доски, чтобы приподнять этот же конец или противопо- ложный? Считать g = 10 м/с2. 5
Задачи 38 5.23. Гладкий невесомый стержень АС длиной 1 м вставлен горизон- тально с малым зазором по толщине на глубину АВ = 0,2 м в вертикальную стену (см. рисунок). К концу С стержня подвешен груз весом Р = 100 Н. Определите силы реакции стенки в точках А и В. 5.24* . Гладкий невесомый стержень АС длиной 1 м вставлен под углом а = 30° к горизонту в вертикальную стену (см. рисунок). К концу С стержня подвешен груз весом Р = 100 Н. Определите силы реакции боковых стенок отверстия в точках А и В. С какой силой стержень сжат? Расстояние АВ равно 0,2 м. К задаче 5.23 5.25**. Тонкий од УДе стержень укреплен на шарнире в точке А и я в равновесии горизонтальной нитью (см. рисунок), в са стержня т = 1 кг, угол его наклона к горизонту а = 45°. Найдите величину и направление силы N реакции шарнира. К задаче 5.26
39 Механика 5.26*. Невесомые стержни АВ и ВС соединены шарнирно между собой и с вертикальной стеной (см. рисунок); угол между стержнями равен а. К середине стержня АВ подвешен груз массой т. Определите силы Fa и Fb давления стержня АВ на шарниры А и В. 5.27. Рычаг изогнут так, что стороны его АВ, a g ВС и CD равны между собой и образуют | ......... друг с другом прямые углы (см. рисунок). FX Ось рычага — в точке В. В точке А прило- У жена сила F перпендикулярно плечу ры- И 1 , -i чага АВ. Определите минимальное зна- ® чение силы, которую нужно приложить в точке D, чтобы рычаг находился в равновесии. Изменится ли ответ, если ось рычага переместить в точку С? Массой рычага пренебречь. 5.28. Колесо радиусом R и массой т стоит перед ступенькой высотой й. Какую горизонтальную силу F надо приложить к оси колеса, чтобы оно въехало на ступеньку? 5.29. Решите задачу 5.28, если h < R и сила F может быть направлена произвольно. Как именно она должна быть направлена? 5.30* . На земле лежат вплотную друг к другу два одинаковых бревна цилиндрической формы. Сверху кладут такое же бревно. При каком коэффициенте трения ц между бревнами они не раскатят- ся? По земле бревна не скользят. 5.31. Однородный шар массы т и радиуса R подвешен на нити длиной I к гладкой вертикальной стене. Определите силу Т натяжения нити и силу F давления шара на стену. 5.32*. j Тяжелый однородный шар радиуса R подве- шен на нити длиной I, конец которой закреп- лен на вертикальной стене. Точка крепления нити к шару находится на одной вертикали с центром шара. Каков должен быть коэф- фициент трения между шаром и стеной, что- • бы шар находился в равновесии? 5.33. Шкаф массой т = 30 кг опирается ножками д на платформы двух весов (см. писунок). С
Задачи 40 Ширина шкафа АВ = 1,2 м, высота AD = 2 м. Определите показания весов, если в точке D приложена горизонтальная сила F = 59 Н. Центр тяжести шкафа совпадает с его геометрическим центром. 5.34*. Каков должен быть минимальный коэффициент трения gmin между кубом и горизонтальной плоскостью, чтобы однородный куб мож- но было опрокинуть через ребро горизонтальной силой, приложен- ной к верхней грани? Какая минимальная сила Fmin для этого потребуется? Масса куба т. 5.35*. Какой минимальной силой Fm\a можно опрокинуть через ребро однородный куб, находящийся на горизонтальной плоскости? Каков должен быть при этом минимальный коэффициент трения Цтт между кубом и плоскостью? Масса куба т. 5.36*. Лестница стоит наклонно, опираясь на гладкую вертикальную стену. Коэффициент трения между ножками лестницы и по- лом р. Какой наибольший угол может образовывать лестница со стеной? Центр тяжести лестницы расположен в ее середине. 5.37. На горизонтальной плоскости стоит однородный цилиндр радиуса R и высоты h. Если медленно наклонять плоскость, то в какой-то момент цилиндр опрокидывается. При каком угле наклона а это происходит? При каком коэффициенте трения ц между цилин- дром и плоскостью это возможно? 5.38*. Высокий прямоугольный однородный брусок с квадратным осно- ванием стоит на горизонтальной поверхности. Как определить коэффициент трения между бруском и поверхностью, располагая только линейкой? 5.39*. Шкаф с невысокими ножками неподвижно стоит на наклонной плоскости (см. рисунок). Масса шкафа т, центр тяжести совпа- дает с геометрическим центром. Определите силы нормального давления на наклонную плос- кость левых и правых ножек шкафа.
41 Механика 5.40*. Шкаф с невысокими ножками поставили на наклонную плоскость (см. рисунок к задаче 5.39). Определите силы давления на наклон- ную плоскость левых и правых ножек шкафа, если коэффициент трения ц меньше tga. 5.41*. Пять кирпичей длиной I кладут без раствора ।--------1 один на другой так, что каждый кирпич вы- ।—'-----г—' ступает над нижележащим (см. рисунок). На ।—'----1—' какое наибольшее расстояние правый край ।—*----1—' самого верхнего кирпича может выступать |—*--г-। над правым краем самого нижнего кирпича? '---1 >5.42**. Решая предыдущую задачу, я задумался: а что, если взять не пять кирпичей, а больше? Можно ли, имея достаточный запас кирпичей, уложить их друг на друга так, чтобы край самого верхнего кирпича выступал над краем самого ниж- него кирпича на целую милю? Этот вопрос не давал мне покоя, поэтому» я раздобыл побольше кирпичей и взялся за дело. И что вы думаете — я достиг успеха! Как мне это удалось? 5.43. В вершинах треугольника помещены шарики массы т каждый. Найдите положение центра тяжести системы. 5.44. В двух вершинах треугольника помещены шарики массы т каж- дый. В третьей вершине помещен шарик массой 4т. Найдите положение центра масс системы. 5.45. Одна треть цилиндрического стержня изготовлена из меди, а две трети — из I медь I алюминий I алюминия (см. рисунок). В какой из частей стержня расположен его центр тяжести? 5.46*. Десять шариков с массами 100, 200, 300, ..., 1000 г укреплены на невесомом стержне длиной 90 см так, что расстояние между центрами двух соседних шариков равно 10 см. Найдите положение центра масс системы. 5.47*. Пользуясь только линейкой без делений и не про- изводя никаких вычислений, найдите построе- нием положение центра тяжести С показанной на рисунке однородной пластинки.
Задачи 42 5.48*. Докажите, что центр тяжести плоской однородной треугольной пластины находится в точке пересечения медиан. 5.49*. Однородная тонкая пластина имеет форму круга радиуса R, в котором вырезано круглое отверстие радиуса R/2 (см. рисунок). Где на- ходится центр тяжести пластины? 5.50**. Найдите центр тяжести тонкой однородной проволоки, согнутой в виде полуокружности радиуса г. 6. ГИДРОСТАТИКА И АЭРОСТАТИКА ел. Три сосуда (см. рисунок) имеют одинаковую площадь дна S. Срав- ните силы давления на дно каждого из сосудов, если в них налито оди- наковое количество воды. В каж- дом из трех случаев сравните силу давления F на дно с весом Р на- литой воды. 6.2 *. В воду опущен сосуд с приставным дном, имеющий форму усечен- ного конуса. Если в сосуд налить 200 г воды, то дно отпадет. Отпадет ли дно, если: а) в его центр поставить гирю массой 200 г; б) налить 200 г бензина; в) налить 200 г ртути? 6.3 . Аквариум имеет форму куба со стороной а и заполнен доверху водой. Определите силы давления воды на дно и боковые стенки аквариума. Атмосферное давление не учитывать. 6.4 . Аквариум имеет форму куба со стороной а = 60 см. До какой высоты h следует налить в него воду,, чтобы сила давления на боковую стенку была в 6 раз меньше, чем на дно? Атмосферное давление не учитывать. 6.5 *. В колена (7-образной трубки налиты вода и спирт, разделенные ртутью (см. рисунок). Уровень ртути в обоих коленах одинаков. На высоте ho = 24 см от уровня ртути колена соединены тонкой горизонтальной трубкой с закрытым краном. Высота столба воды
К задаче 6.5 43, Механика hi = 40 см. Определите высоту стол- ба спирта ht. Какое движение воз- никнет в жидкости после того» как откроют кран? 8.6**. Как изменятся уровни всех жидкос- тей после открывания крана (см. за- дачу 6.5), если спирт заменить ке- росином? Керосин имеет ту же плот- ность, что и спирт, но, в отличие от него, не смешивается с водой. Капил- лярные эффекты не учитывать. 6.7 *. В стакане плавает кусок льда. Как изменится уровень воды, когда лед растает? Изменится ли ответ, если во льду находится кусочек пробки? Стальная гайка? 6.8 *. В небольшом бассейне плавает лодка. Изменится ли (и как) уровень воды в бассейне, если лежащий на дне лодки камень бросить в воду? 6.9 *. В небольшом бассейне плавает полузатопленная лодка. Уровень воды в лодке такой же, как в бассейне. Из лодки зачерпнули ведро воды и вылили в бассейн. Где теперь выше уровень воды — в лодке или в бассейне? Как изменился уровень воды в бассейне? 6.10 . Потонет ли в воде стеклянная бутылка, дойерху наполненная водой? Бутылка со ртутью — в ртути? 6.11 *. В воде плавает льдина с площадью поперечного сечения S = 5 м2 и высотой Н = 0,5 м. Какую работу надо совершить, чтобы полностью погрузить льдину в воду? 6.12 . Стеклянный шар уравновешивается на точных рычажных весах латунными гирями массой mi = 170 г. Взвешивание производится на уровне моря при температуре 0рС. Какова масса шара т? Гири какой массы тг потребуются для уравновешивания шара на высоте h = 5,5 км, где плотность воздуха уменьшается вдвое? 6.13 . Воздушен, шар объемом V — 300 м® парит вблизи поверхности
Задачи 44 Земли. С шара сбросили балласт, и шар поднялся на высоту, где плотность воздуха вдвое меньше. Какова масса Дтп балласта, если объем шара при подъеме увеличился в полтора раза? Температуру воздуха считайте равной (ТС. 6.14 *. На дне реки установлена бетонная конструкция грибовидной формы (см. рисунок). С какой силой давит конструкция на дно реки? Атмосферное давление не учитывать. К задаче в.14 К задаче 6.16 6.15 . Пластмассовый брусок плавает в воде. Как изменится глубина погружения бруска в воду, если поверх воды налить слой масла, полностью покрывающий брусок? 6.16 . Тело находится на границе двух жидкостей (см. рисунок). Вы- числите действующую на него силу Архимеда Fa. 6.17 *. Тело плотностью р плавает на границе раздела двух жидкостей с плотностями pi и р2, причем pi > р2. Какая часть объема тела погружена в «нижнюю» жидкость? 6.18 *. Доска толщиной 5 см плавает в воде, погрузившись на 70%. Поверх воды разливается слой нефти толщиной 1 см. На сколько будет выступать доска над поверхностью нефти? 6.19 *. Стальной кубик с длиной ребра а = 10 см плавает в ртути. Поверх ртути наливают воду вровень с верхней гранью кубика. Какова высота Н слоя воды? 6.20 . Однородное тело подвешено к динамометру. Показание динамометра в воздухе Pi = 8,4 Н, а в воде Р2 = 5 Н. Чему равна п^ррр^т^ тела р?
45 Механика 6.21 *. В лифте находится ведро с водой, в котором плавает мяч. Как изменится глубина погружения мяча, если лифт будет двигаться с ускорением а, направленным вверх? Вниз? 6.22 . На рычажных весах уравновешен гирями сосуд с водой. Нару- шится ли равновесие, если в воду погрузить подвешенный на нитке стальной брусок так, чтобы он не касался дна? 6.23 . На сколько нужно увеличить массу гирь (см. задачу 6.22), чтобы восстановить равновесие весов, если брусок погружается в воду полностью? Размеры стального бруска 5 см х 6 см х 8 см. 6.24 . Как изменится ответ в задаче 6.23, если верхний конец нитки не держать в руках, а привязать к перекладине того же сосуда (см. рисунок)? 6.25 . В озере на некоторой глубине плавает полый шар, полностью погруженный в воду. Можно ли считать, что шар находится в состоянии невесомости, поскольку его вес в воде «полностью исчез»? Будет ли ощущать невесомость челбвек, находящийся внутри шара? 6.26 *. Вернется ли плавающий шар (см. задачу 6.25) на прежнюю глубину, если его погрузить ниже и отпустить? 6.27 . Сплошные шары — алюминиевый и железный — уравновешены на рычаге. Нарушится ли равновесие, если оба шара погрузить в воду? Рассмотрите два случая, когда шары имеют: а) одинаковую массу; б) одинаковый объем. 6.28 *. Оцените массу атмосферы Земли (радиус Земли R = 6400 км). 6.29. Пробковый поплавок массой тп, привязанный нитью к тяжелому «якорю», находится на глубине h под водой. Какое количество теплоты выделится после перерезания нити? 6.30 . Тело массой т = 3 кг и объемом V = 2 дм3 находится в воде на глубине й = 3 м. Какую работу необходимо совершить при мед- ленном подъеме тела на высоту Н = 5 м над поверхностью воды? Равна лй'зШр&бота изменению потенциальной энергии тела?'
Задачи 46 6.31 *. Перевернутая кастрюля массы т и ра- диуса И стоит на резиновой прокладке (см. рисунок), плотно прилегая к ней краями. В дне кастрюли — круглое отверстие радиусом г, в которое плотно вставлена легкая вертикальная трубка. В кастрюлю через трубку наливают жидкость. Когда длина столба жид- кости в трубке становится равной Л, жидкость начинает вытекать из-под кастрюли. Определите плотность жидкости р. 7. МЕХАНИЧЕСКИЕ КОЛЕБАНИЯ И ВОЛНЫ Гармонические колебания 7.1. Математический маятник длиной 1 м колеблется с амплитудой 1 см. За какое время он пройдет путь в 1 см, если начнет движение из положения равновесия? За какое время он пройдет: а) первую половину этого пути; б) вторую половину этого пути? 7.2. Амплитуда гармонических колебаний маятника 6 см. Какую часть периода груз маятника находится не далее 3 см от положения равновесия? 7.3* . Рассмотрите движение конического маятника . (груз на нити движется по окружности в гори- / зонтальной плоскости) и выразите период дви- жения по окружности через длину нити (и / / угол а отклонения от вертикали (см. рисунок). / Докажите, что при малых углах а периоды / г конического маятника и обычного математи- £ зд-------- ческого маятника с той же длиной нити равны. — — — — <7.4*. Как-то, гуляя, я невзначай забрался в горы. Мне, конечно, захотелось узнать, на какой я высоте. В моей сумке слу- чайно оказались маятниковые часы. Держа их в правой руке, я начал сравнивать их ход с ходом электронных часов на левом запястье. Ровно через час я обнаружил, что маятниковые часы отстали на пять секунд, и сразу понял, на какой высоте
47 Механика нахожусь. Попробуйте и вы узнать эту высоту. Маятник моих часов сделан из такого сплава, что длина его от температуры не зависит, а мои электронные часы, как известно, самые точные в мире. Радиус Земли Л — 6400 км. 7.5. Стоя на вершине горы, барон Мюнхаузен (см. задачу 7.4) решил восстановить точность своих маятниковых часов. На какую часть длины он должен укоротить маятник? 7.6. При какой скорости поезда о подвешенный в вагоне маятник длиной I = 1 м особенно сильно раскачивается, если длина рельсов L = 25 м? 7.7* . Чему равен период Т колебаний математического маятника дли- ной I: а) в лифте, ускорение которого направлено вверх и равно а; б) в лифте, ускорение которого направлено вниз (а < g)‘, в) в поезде, движущемся горизонтально с ускорением а; г) на тяжелой тележке, съезжающей без трения с наклонной плоскости под углом а к горизонту? Чему равен во всех этих случаях период колебаний пружинного маятника? 7.8* . В кабине лифта, находящегося на верхнем этаже небоскреба, раскачивается подвешенный на нити шарик. Нить привязана к гвоздю, вбитому в стену кабийы. Трос лифта обрывается, и лифт начинает падать. Опишите движение шарика относительно лиф- та на протяжении всего движения лифта. Учтите при этом, что сила сопротивления воздуха быстро растет с увеличением ско- рости. 7.9. Подвешенный груз растягивает легкую пружину на Л/ = 16 см. Чему равен период колебаний груза на этой пружине? 7.10. Найдите период Т ко- лебаний систем, изоб- раженных на рисунках а, б, в. Коэффициенты жесткости пружин ki и k2, масса груза т. Трение отсутствует.
Задачи 48 7.11* На два быстро вращающихся одинако- вых валика положили горизонтально доску с массой т (см. рисунок). Расстоя- ние между осями валиков равно L, коэф- фициент трения между доской и вали- । ками равен ц. Какое движение будет совершать доска? Как изменится ответ, если оба валика изменят направление вращения? 7.12**. Два грузика массами mi и т2 связаны пружиной. Коэффициент жесткости пружины k. Каков период Т свободных колебаний такой системы, если при колебаниях грузики движутся вдоль одной прямой? 7.13**. На гладком столе лежат два грузика с массами mi = 100 г и т» = 300 г, соединен- / ные пружиной жесткостью k = 50 Н/м. / Один из грузиков касается стенки (см. / щ, I рисунок). Грузики связаны нитью длиной Ьттгпгп ТН'ГГП Г) Г) ггг. 1о = 6 см. При этом пружина сжата на AZ — 2 см. Опишите движение грузиков после того, как нить пережигают. Массой пружины можно пренебречь. 7.14*. Однажды, путешествуя вблизи Северного полюса, я очу- тился один на отколовшейся плоской льдине площадью S = 5 м2. От огорчения я подпрыгнул, и льдина вместе со мной начала колебаться, совершая одно колебание в секун- ду. Это меня сразу успокоило: зная свою массу (т = 80 кг), я тут же определил, что льдина достаточно толстая. Какова ее толщина? 7.15**. На рисунке показано положение равновесия коле- бательной системы (математического маятника с пружинной связью). Определите период Т малых колебаний системы. Как изменится ответ, если пружину заменить полоской эластичной резины, которая имеет те же длину и жесткость? 7.16*. л k Во сколько раз изменится частота малых коле- ФттШАф m баний небольшого груза на легком стержне (см. л рисунок на следующей стр.), если к середине стержня прикрепить горизонтальную пружину с жесткостью А? В равновесии стержень занимает вертикальное положение. чь t >
49 Механика 7.17*. Два математических маятника длиной I каждый связаны неве- сомой пружиной с жесткостью k. На рисунке показано поло- жение равновесия системы. Маятники отклоняют в плоскости рисунка на одинаковые углы и отпускают. Определите период Т малых колебаний связанных маятников, если: а) маятники отклонены в одну сторону (колебания в одной фазе); б) маятники отклонены в противоположные стороны (колебания в противо- фазе). <7.18*. Когда я наконец добрался по льду до самого Северного полюса, меня ожидало открытие: из глубокого колодца торчала земная ось! Я ухватился за нее, как за шест, и быстро заскользил вниз — ось оказалась очень гладкой. К счастью, я не расшибся: колодец оказался без дна, и скоро я вынырнул вверх ногами на Южном полюсе! Жестокий антар- ктический мороз не помешал мне тут же вычислить время t, которое понадобилось для такого удивительного путешествия с одного полюса на другой, а также скорость о, с которой я пролетал через центр Земли. Определите их и вы, считая Землю однород- ным шаром и пренебрегая сопротивлением воздуха. 7.19*. Санки длиной I = 80 см скользят горизонтально по снегу и останавливаются, частично выехав на асфальт. Определите время торможения, если трение о снег отсутствует, а коэффициент трения об асфальт р. = 0,4. Масса санок распределена по их длине равномерно. 7.20*. Найдите период колебаний жидкости в U-образном сосуде посто- янного сечения. Длина всего столба жидкости равна 2Н.
Задачи 50 7.21**. Найдите период Т малых колебаний изображенного на рисунке маятника. Массой стержня можно пренебречь. 7.22*. Невесомый стержень с закрепленными на нем грузами (см. ри- сунок) может вращаться вокруг горизонтальной оси О. Определите период Т его малых колебаний. 7.23*. Горизонтальная подставка с лежащим на ней предметом соверша- ет горизонтальные гармонические колебания с периодом Т = 0,5 с. Коэффициент трения предмета о подставку ц = 0,1. При какой- амплитуде А колебаний предмет проскальзывает на подставке при колебаниях? 7.24. Горизонтальная подставка совершает вертикальные гармоничес- кие колебания с амплитудой А = 5 мм. При какой частоте колебаний v лежащий на подставке предмет не отрывается от нее? 7.25*. На пружине, имеющей жесткость k, неподвижно висит очень легкая чашка (см. рисунок). На чашку с высоты h без начальной скорости пластилиновый шарик с массой ш. Определите амплитуду А возникающих колебаний. Механические волны 7.26. По шнуру бежит незатухающая гармоническая волна, так что поперечное смещение точки О шнура .изменяется по закону у — Асов cot. По какому закону изменяется смещение у точки X, расположенной на расстоянии х от точки О? Волна бежит от О к X со скоростью о. . ;г л \№1 <
51 Механика 7.27. Как изменяются частота и длина волны звука при переходе из воздуха в воду? 7.28* . Найдите собственные частоты колебаний воздушного столба в закрытой с обоих концов трубе длиной I = 3,4 м. 7.29* . Над цилиндрическим сосудом высотой Н = 1 м звучит камертон, имеющий собственную частоту колебаний v = 340 Гц. В сосуд медленно наливают воду. При каких положениях уровня воды в сосуде звучание камертона значительно усиливается? 7.30. Камертон колеблется с частотой v = 440 Гц. Какую минимальную длину может иметь резонаторный ящик («подставка» камертона) для усиления звука? Не противоречит ли закону сохранения энергии тот факт, что из двух одинаковых камертонов, возбуж- денных одинаковыми по силе ударами, намного громче звучит тот, который установлен на резонаторе? 7.31. При какой глубине озера в нем могут «раскачаться» физио- логически опасные инфразвуковые колебания с частотой 7,5 Гц? 7.32. Первая резонансная частота открытой с обеих сторон органной трубы равна 300 Гц. Чему равна первая резонансная частота такой же трубы, закрытой с обоих концов? Закрытой с одного конца? 7.33 **. Почему изменяется голос человека в барокамере, заполненной дыхательной смесью из кислорода и гелия? Как изменится в этой атмосфере тон духовых инструментов? Камертона? Камертона на резонаторном ящике? 7.34 *. Какую форму имеет фронт ударной волны, которая возникает в воздухе при полете пули со скоростью и, превышающей скорость звука о? 7.35 *. Реактивный самолет пролетел со скоростью* в два раза превыша- ющей скорость звука, на высоте h = 5 км над наблюдателем. На каком расстоянии от наблюдателя был самолет, когда человек услышал звук? 7.36**. Гоночный автомобиль, включив сирену, мчится со скоростью и -= 306 км/ч. Частота колебаний сирены v0 -= 400 Гц. Впереди на
Задачи 52 обочине лежит другой автомобиль с точно такой же включенной сиреной (водитель, к счастью, остался жив). Каждый из водителей различает звук сирены другого автомобиля, потому что он выше, чем звук его собственной сирены. Кто из них слышит более высокий звук? 7.37*. Морские волны движутся со скоростью и и набегают на берег с частотой vo. Волновой фронт параллелен береговой линии. С какой частотой V волны ударяются о катер, идущий от берега со ско- ростью v, направленной под углом а к береговой линии? Каким станет ответ, если катер изменит направление движения на противоположное? 7.38*. Два одинаковых динамика (А и В) подключены к выходу одного генератора электрических колебаний частотой v = 680 Гц. Рассто- яние между динамиками 4 м. Амплитуда звуковых колебаний в точке С, находящейся посередине отрезка АВ (см. рисунок), максимальна и равна а. Какова амплитуда звуковых колебаний в точках В и В, если CD = 6,25 см, СЕ — 12,5 см? Каким будет ответ, если изменить полярность подключения одного из динамиков? 8. КОМБИНИРОВАННЫЕ ЗАДАЧИ ПО МЕХАНИКЕ 8.1. Чему равен коэффициент полезного действия ц наклонной плос- кости длиной L = 3 м и высотой h = 1,8 м, если коэффициент трения ц = 0,1? 8.2. Тело массой т = 1 кг вращают в вертикальной плоскости на веревке длиной I = 1 м. При прохождении нижней точки окруж- ности сила натяжения веревки Г = 80 Н. Когда скорость тела направлена вертикально вверх, веревку отпускают. На какую высоту Л над нижней точкой окружности поднимется тело?
53 Механцка 8.3* . Подвешенный на нити шарик массой т совершает колебания в вертикальной плоскости. Когда шарик проходит положение рав- новесия, сила натяжения нити Ti = 2mg. На какой максимальный угол а от вертикали отклоняется шарик? Чему равна сила Т2 натяжения нити в момент наибольшего отклонения шарика? 8.4. Шарик массой т, подвешенный на нити, отклоняют до горизон- тального положения нити и отпускают. При каком угле а между нитью и вертикалью сила натяжения нити будет равна mg? Чему равна максимальная сила Ттах натяжения нити? 8.5* . Какой должна быть минимальная прочность нити, чтобы на ней можно было вращать шарик массой т в вертикальной плоскости? Каким будет ответ, если нить заменить невесомым стержнем? 8.6**. На легкой нерастяжимой нити длиной I подвешен маленький шарик массой т. Нить с шариком отводят до горизонтального положения и отпускают. Какой угол а образует нить с вертикалью в тот момент, когда ускорение шарика направлено горизонтально? Каковы в этот момент скорость v шарика и сила Т натяжения нити? 8.7*. Снаряд в верхней точке своей траектории на высоте Н = 2 км разорвался на два одинаковых осколка. Один из них вернулся точно назад и попал обратно в жерло пушки, второй упал на расстоянии s = 8 км от пушки. Какой была начальная скорость Уо снаряда? Сопротивлением воздуха пренебречь. 8.8*. Заторможенный блок, через кото- рый перекинута невесомая нерастя- жимая нить с грузами т.\ и т2 на концах, уравновешен на равнопле- чих рычажных весах (см. рисунок). Как следует изменить массу гирь на правой чашке, чтобы равновесие сохра- нилось после освобождения блока?
Задачи 54 8.9*. Каковы ускорения грузов показанной на рисунке системы? Трение отсутствует, массой блоков и нити можно пренебречь. Участки нити, не лежащие на блоках, вертикальны. Нить нерастяжима. 8.10*. Между двумя неподвижными муфтами может без трения переме- щаться вверх и вниз стержень. Нижним концом он касается гладкой поверхности клина (см. рисунок). Массы клина и стержня одинаковы. Клин лежит на гладком горизонтальном столе. Опре- делите ускорения клина и стержня. 8.11*. Маленький шарик подвешен в точке А на нити длиной I. В точке В на расстоянии 1/2 ниже точки А в стену вбит гвоздь (см. рисунок). Шарик отводят так, что нить занимает горизонтальное положение, и отпускают. В какой точке траектории исчезает сила натяжения нити? До какой найвысшей точки поднимется шарик? К задаче 8.11 К задаче 8.12 8.12*. Небольшое тело массой т соскальзывает с. вершины гладкого неподвижного шара (см. рисунок). С какой силой давит тело на поверхность Шара? 8.13*. На какой высоте Н тело (см. задачу 8.12) отделится от шара? 8.14**. На каком расстоянии от точки касания шара с плоскостью (см. задачу 8.12) упадет тело? i яопР „
55 'Механика 8.15*. Небольшое тело соскальзывает без тре- ния с наклонной плоскости, переходя- щей в «мертвую петлю» радиуса R (см. рисунок). С какой минимальной высо- ты Н должно начинаться движение, чтобы тело прошло «мертвую петлю», не отрываясь от нее? 8.16**. Обруч скатывается без проскальзывания с наклонной плоскости высотой h и длиной I. Определите его скорость v в нижней точке. 8.17*. Галилей определял на опыте ускорение свободного падения, ска- тывая шары с наклонной плоскости. При этом он ошибочно исходил из того, что ускорение шара а = gsina, где а — угол наклона плоскости. На самом деле шары скатываются с меньшим ускорением. В результате Галилей получил заниженное значение g1 ускорения свободного падения. Какое значение g1 получил бы Галилей, скатывая с наклонной плоскости обручи? 8.18**. Небольшое тело массой т соскальзывает без трения с наклонной плоскости, переходящей в «мертвую петлю» радиуса R (см. ри- сунок к задаче 8.15). Начальная высота тела Н = 57?. С какой силой F давит тело на поверхность в верхней точке «мертвой петли»? Каким будет ответ, если в условии задачи соскальзываю- щее тело заменить тонкостенным колечком, скатывающимся без проскальзывания? 8.19**. Легкий стержень длиной I с двумя V шариками на концах (их массы тлх и тп2; \ mi > тп2) может вращаться вокруг го- ' ризонтальной оси, проходящей через 9 т* середину стержня (см. рисунок). Стер- жень приводят в горизонтальное поло- \ жение и отпускают. С какой силой F \ давит стержень На ось в первый момент V после освобождения? При прохождении положения равновесия? 8.20**. Веревка массы т и длины L переброшена через маленький легкий блок и уравновешена. От легкого толчка блок начал вращаться. Какова скорость v веревки в тот момент, когда с одной стороны блока свешивается честь веревки длиной х> L/2? С какой силой F веревка давит на блок в этот момент? Трение в блоке отсутствует.
Задачи 56 8.21*. Гладкий шелковый шнур длины L и массы т лежит на столе перпендикулярно его краю. Шнур начинает без начальной ско- рости соскальзывать с края стола. Определите скорость v и уско- рение а шнура в момент, когда он соскользнул со стола наполо- вину. Определите также силу Т натяжения шнура у края стола в этот момент. 8.22**. Санки скатываются без началь- ной скорости из точки А (см. рису- нок). Наклон изменяется плавно, так что радиус кривизны профиля горы всюду намного превышает высоту й. Коэффициент трения ц = 0,2. Сопротивлением воздуха можно пренебречь. Предложите графический способ определения точки остановки. 8.23**. Санки скатываются без началь- ной скорости из точки А склона (см. рисунок). Радиус кривизны траектории санок всюду намного превышает высоту й. Сопротивле- нием воздуха можно пренебречь. При каком коэффициенте трения ц санки преодолеют горку? 8.24*. С какой скоростью v вытекает жидкость из маленького отверстия у дна сосуда, если высота уровня жидкости относительно дна равна Н? Трением в жидкости пренебречь. 8.25*. Какую форму должен иметь сосуд, чтобы при вытекании через маленькое отверстие в дне уровень жидкости понижался с посто- янной скоростью? (Это были бы идеальные водяные часы, если бы трением в жидкости можно было пренебречь.) 8.26*. Оцените, во сколько раз изменится рекорд по прыжкам в высоту, если спортсмены будут состязаться в зале, расположенном на Луне, где сила тяжести в шесть раз меньше. 8.27*. Оцените, какого размера должна быть малая планета с плотностью Земли, чтобы человек, оттолкнувшись, мог улететь с нее насовсем. 8.28*. Лягушка массой т сидит на конце доски массой М и длиной L. Доска плавает на поверхности пруда. Лягушка прыгает под углом
57 Механика а к горизонту и «приземляется» на другом конце доски. Опре- делите начальную скорость t>o лягушки относительно Земли. 8.29**. Верхний конец тонкой однородной деревянной палочки шарнирно закреплен. Нижний конец ее погружен в воду. Если сосуд с водой медленно поднимать, то с некоторого момента вертикальное поло- жение палочки уже не соответствует устойчивому равновесию. Почему? Покажите, что при дальнейшем подъеме сосуда палочка будет отклоняться так, что длина погруженной в воду части будет оставаться неизменной. Найдите плотность р палочки, если в воду погружена 1/4 часть ее длины. 8.30**. На концах очень длинной нити подвешены одинаковые грузы массой т каждый. Нить перекинута через два малых непод- вижных блока, расположенных на расстоянии 21 друг от друга. К середине нити прикрепляют груз массой 2т (см. рисунок). Найти скорости грузов по истечении достаточно большого промежутка времени. Массой блоков и нити, а также трением можно пренеб- речь. 8.31**. На полу стоит вертикально легкий стержень. На верхнем конце его закреплен массивный шар. Стержень начинает падать без начальной скорости. При этом его нижний конец не проскальзы- вает. Какой угол аШах стержень образует с вертикалью, когда его нижний конец перестанет давить на пол? При каком значении коэффициента трения р. такое движение возможно? На каком расстояний от начальной точки опоры упадет шар? 8.32*. Ось О закреплена горизонтально на уровне поверхности воды. Однородный брусок квадратного сечения (см. рисунок) может вращаться с малым трением вокруг оси О, совпадающей с его осью симметрии. Какое положение займет брусок? ' '' П ТЫНс < ,
Задачи 58 8.33*, Диск массой т лежит на гладкой горизонтальной поверхности. К точке А на ободе диска (см. рисунок) прикладывают силу F перпендикулярно к радиусу ОА. Каково ускорение а центра диска в этот момент? К задаче 8.33 К задаче 8.34 8.34**. Быстро вращающийся вокруг горизонтальной оси цилиндр стал- кивается (см. рисунок) с горизонтальной поверхностью и отлетает от нее вертикально вверх. Цилиндр и поверхность изготовлены из упругих материалов. В какую сторону вращался цилиндр? Чему равен коэффициент трения ц между цилиндром и поверхностью? В каком направлении отлетел бы цилиндр, если бы он вращался в противоположную сторону? 8.35*. Из-за торможения в атмосфере высота орбиты спутника уменьша- ется. Как изменяется при этом скорость спутника? Согласуется ли это изменение с законом сохранения энергии? <8.36**. В одном из своих путешествий я открыл неизвестную доселе планету Солнечной системы. Она, как и наша Земля, имеет радиус г = 6400 км, а радиус ее круговой орбиты составляет 150 миллионов километров. Но пла- нета движется вокруг Солнца поступательно, у нее отсутствует суточное вращение. Я обошел ее всю и при помощи карманного чувствительного динамометра с удивлением обнаружил, что ускорение свободного падения g не всюду одинаково! Объясните это явление и подсчитайте, на сколько максимальное значение g на этой планете отличается от минимального. Согласно моим точнейшим измерениям планета представляет собой однород- ный шар.
59 Механика 8.37*. К проволоке, закрепленной верхним концом, подвешен груз и отпущен без толчка. Под действием груза проволока удлиняется. Сравните изменение потенциальной энергии проволоки AW\ и груза AW2. Как полученный результат согласуется с законом сохранения энергии?
2 _ 1 _ 1 ___ Основное уравнение МКТ газов р = -^пЕъ = -^nniov2 = -^pv2 О о о ______ А - * - 3,_ — таи2 Физическии смысл абсолютной температуры — кТ = Ek = —z— 2л Уравнение Клапейрона = const Уравнение Менделеева-Клапейрона pV = Относительная влажность ср = ——— 100% Рн(П Сила поверхностного натяжения F = al Избыточное давление под сферической поверхностью жидкости Др = 2a/R Закон Гука ст = Е|е| = 1о Линейное тепловое расширение I = Zo(l + at) Работа газа при изобарном изменении его объема А’ - рАУ Первый закон термодинамики Д£7 = Q+ A = Q-A’ 3 Внутренняя энергия идеального одноатомного газа U = -^vRT 2л Количество теплоты, необходимое для нагревания тела Q = cmAt = С At Количество теплоты, необходимое для плавления Q = km Количество теплоты, необходимое для парообразования при неиз- менной температуре Q = Lm Количество теплоты, выделяющееся при сгорании топлива Q = qm т^тттг т«, — Тг КПД цикла Карно
МОЛЕКУЛЯРНАЯ ФИЗИКА МОЛЕКУЛЯРНО-КИНЕТИЧЕСКАЯ ТЕОРИЯ СВОЙСТВА ГАЗОВ ПАРЫ, ЖИДКОСТИ И ТВЕРДЫЕ ТЕЛА ЗАКОНЫ ТЕРМОДИНАМИКИ
Задачи 62 9. МОЛЕКУЛЯРНО-КИНЕТИЧЕСКАЯ ТЕОРИЯ. СВОЙСТВА ГАЗОВ Основные понятия молекулярно-кинетической теории 9.1. Из открытого стакана за время t = 20 сут испарилась вода массой т .= 200 г. Сколько молекул испарялось за т = 1 с? 9.2. Озеро со средней глубиной й = 5,0 м и площадью S = 4,0 км2 «посолили», бросив кристаллик поваренной соли NaCl массой т = 10 мг. Спустя длительное время из озера зачерпнули стакан воды объемом V = 200 см3. Сколько ионов натрия оказалось в этом стакане? 9.3. Рассмотрим кристалл с так назывемой простой кубической решет- кой (см. рисунок). Определите его плотность р, если масса каждого атома равна то, а длина ребра кубической ячейки а. К задаче 9.3 К задаче 9.4 9.4. Кристаллы поваренной соли NaCl кубической системы состоят из чередующихЬя ионов натрия и хлора (см. рисунок). Плотность соли р = 2200 кг/м3. Определите расстояние а между центрами ближайших ионов. 9.5. Многие металлы в твердом состоянии образуют так называемую гранецентрированную кубическую решетку: ионы металла распо- ложены в вершинах кубической ячейки и в центрах ее граней. Определите плотность р кристалла, если молярная масса металла равна Af; а длина ребра кубической ячейки а. 9.6* . В сжатом газе взаимодействие между молекулами становится существенным. Как изменилось бы давление в сосуде со сжатым
63 Молекулярная физика газом, если бы силы взаимодействия между молекулами внезапно исчезли? 9.7* . В закрытом сосуде происходит полное сгорание кусочка угля с образованием углекислого газа. После этого сосуд охлаждают до первоначальной температуры. Сравните конечное давление в сосуде с начальным. Объем угля мал по сравнению с объемом сосуда. 9.8. В закрытом сосуде при давлении ро находится смесь из одного моля кислорода и двух молей водорода. Между ними происходит реакция с образованием водяного пара. Какое давление уста- новится в сосуде после охлаждения до первоначальной темпера- туры? Конденсации пара не происходит. 9.9* . В закрытом баллоне находится смесь из mi = 0,50 г водорода и т-2 — 8,0 г кислорода при давлении pi = 2,35 • 105 Па. Между газами происходит реакция с образованием водяного пара. Какое дав- ление р установится в баллоне после охлаждения до первоначаль- ной температуры? Конденсации пара не происходит. 9.10. Определите средние кинетические энергии Е поступательного движения и средние квадратичные скорости молекул кислорода и водорода при температуре t = 27°С. Свойства газов 9.11. Герметично закрытый сосуд полностью заполнен водой при тем- пературе t = 27°С. Каким стало бы давление р внутри сосуда, если бы силы взаимодействия между молекулами воды внезапно исчезли? 9.12* . При вытекании жидкости из отвер- стия, расположенного у дна открыто- го цилиндрического сосуда, скорость опускания уровня жидкости непосто- янна (см. задачи 8.24 и 8.25). Однако Мариотт придумал, как добиться рав- номерного вытекания жидкости из цилиндрического сосуда. На рисунке изображен «сосуд Мариотта»: через
Задачи 64 пробку пропущена узкая, открытая с обоих концов трубка, не доходящая до дна. При этом жидкость из нижнего отверстия вытекает с постоянной скоростью. Объясните действие «сосуда Мариотта». Что^изменится, если вынуть пробку из малого от- верстия 1, расположенного на уровне нижнего конца трубки? Из отверстия 2, расположенного выше? 9.13. Под водой на глубине Л = 5,0 м отломили нижний конец запаянной стеклянной трубки, и в нее вошло т = 1,95 г воды. Каким было давление ро в запаянной трубке? Объем трубки V — 2,0 см3, атмосферное давление рл = 105 Па. 9.14. Открытую с двух сторон стеклянную трубку длиной Z = 1,0 м наполовину погружают в ртуть. Затем трубку закрывают сверху и вынимают. Какой длины х столбик ртути останется в трубке? Атмосферное давление Н = 750 мм рт. ст. 9.15. В сосуд со ртутью опускают открытую с двух сторон стеклянную трубку, оставляя над поверхностью ее часть длиной I = 60 см. Затем трубку закрывают и погружают еще на а = 30 см. Опре- делите высоту Л столба воздуха в трубке, если атмосферное дав- ление равно Н = 760 мм рт. ст. 9.16. В левом, запаянном колене CZ-образной трубки (см. рисунок) находится столб воздуха высотой hi = 30 см. Ртуть в обоих коленах находится на одном уровне. Какой будет высота Лг столба воздуха в левом колене, если правое долить ртутью доверху? Атмосферное давление Н = 760 мм рт. ст. 9.17. Цилиндрический сосуд высотой Л = 60 см до половины заполнен ртутью и герметично закрыт крышкой с сифонной трубкой, запол- ненной ртутью (см. рисунок). Сифон имеет равные колена, и его трубка кончается у дна сосуда. При каком давлении р воздуха в К задаче 9.16 К задаче 9.17
65 Молекулярная физика сосуде ртуть перестанет вытекать через сифон? На сколько сантиметров понизится уровень ртути за время истечения? Атмос- ферное давление Н — 760 мм рт. ст. 9.18. В центре откачанной и запаянной с обоих концов горизонтальной трубки длиной L = 1,0 м находится столбик ртути длиной h = 20 см. Если трубку поставить вертикально, столбик ртути сместится на 2 = 10 см. До какого давления р0 была откачана трубка? 9.19. Горизонтальный цилиндрический сосуд разделен на две части тонким металлическим поршнем. Одна часть сосуда содержит кислород, другая — такое же по массе количество водорода. Каково равновесное положение поршня, если длина сосуда Z = 50 см? 9.20. Два баллона с объемами Vi и V2 соединены трубкой с краном. Они содержат газы при одинаковой температуре Т и давлениях pi и р2 соответственно. Какое давление р установится в баллонах, если открыть кран? Температура не изменяется, газы в химическую реакцию не вступают. 9.21. Компрессор захватывает при каждом качании объем воздуха AF и нагнетает его в резервуар емкостью V. Первоначально давление в резервуаре равно атмосферному рв. Какое давление рм уста- новится в резервуаре после N качаний компрессора? Считайте, что температура воздуха при сжатии не изменяется. 9.22. Поршневым воздушным насосом с емкостью AF откачивают воз- дух из сосуда емкостью V. Начальное давление равно атмосферно- му рв. Определите давление pN в сосуде после N качаний насоса. Температуру можно считать неизменной. 9.23* . Давление воздуха в шинах автомобиля pi = 500 кПа при темпера- туре 21 = 14°С. Во сколько раз уменьшится площадь соприкосно- вения колес с дорогой, если после поездки температура в шинах повысилась до 22 = 57°С? Атмосферное давление рв = 100 кПа. Изменением объема шины можно пренебречь. 9.24. По газопроводу течет метан (СН4) при давлении р = 2,0- 10е Па и температуре t = 17°С. За время т "= 1 ч транспортируется иг = 32 кг газа. Площадь поперечного сечения трубы, газопровода S = 6,0 см2. Какова скорость и движения газа в трубе?
Задачи 66 9.25. Постройте графики процесса, происходящего с идеальный газом (см. рисунок) в координатах р, Т и р, V. Масса газа постоянна. К задаче 9.25 К задаче 9.26 9.26. Постройте графики процесса, происходящего с идеальным газом (см. рисунок) в координатах р, V nV, Т. Масса газа постоянна. 9.27. Постройте графики процесса, происходящего с идеальным газом (см. рисунок) в координатах р, Т и V, Т. Масса газа постоянна. Участки графика 1-2 и 3-4 соответствуют изотермическим про- цессам. 9.28. Сравните объем идеального газа в состояниях 1 и 2 (см. рисунок). NIacca газа в ходе процесса оставалась неизменной. 9.29*. Как менялось давление идеального газа в ходе процесса, график которого изображен на рисунке? Укажите точки на графике, соответствующие минимальному и максимальному давлению.
67 Молекулярная физика 9.30*. Как менялась температура идеального газа в ходе процесса, график которого изображен на рисунке? Укажите точку, в которой достигалась наибольшая температура (дайте графичес- кое решение). 9.31*. Поршень в цилиндре с газом неплотно прилегает к стенке и пропускает газ наружу. На рисунке показана зависимость объема газа от температуры при изобарном процессе. Укажите направ- ление процесса. 9.32*. Определите температуру идеального газа в состоянии 2 (см. рису- нок), если состояния 2 и 4 лежат на одной изотерме. Температуры Ti и Т3 в состояниях 1 и 3 известны. К задаче 9.31 К задаче 9.32 9.33. Нагревается или охлаждается газ, расширяющийся по закону: а) рУ2 = const; б) р = const; в) p/V = const?
Задачи 68 9.34*. Один моль идеального газа перево- дят из состояния 1 в состояние 2 (см. рисунок). Определите максималь- ную температуру Тщах газа в ходе процесса. 9.36*. Оболочка аэростата объемом 7=800 м, находящегося на поверхности Зем- ли, наполнена водородом на а = 7/8 своего объема при температуре #1 = 17°С. Аэростат поднялся на высоту, где давление рг = 80 кПа и температура #2 = -3°С. Сколько водорода потерял при подъеме аэростат в результате расширения газа? На поверхности Земли атмосферное давление pi = 100 кПа. 9.36*. Во сколько раз изменится подъемная сила воздушного шара, если наполняющий его гелий заменить водородом1’? Весом оболочки шара можно пренебречь. 9.37. В цилиндре под поршней площадью S = 100 см2 и массой mi = 50 кг находится воздух при температуре ti = 7°С. Поршень находится на высоте hi = 60 см от дна цилиндра. Воздух в цилиндре нагревают до #2 = 47°С, а на поршень ставят гирю массой пг2 = 100 кг. На сколько опустится или поднимется поршень? Атмосферное дав- ление рл. = 100 кПа, трением поршня о стенки цилиндра можно пренебречь. 9.38. Сколько молекул воздуха выходит из комнаты объемом V — 80 м3 при повышении температуры от ti = 15°С до.#2 = 27°С? Атмосфер- ное давление рл = Ю5 Па. 9.39. На весах установлены два одинаковых сосуда. Один заполнен сухим воздухом, а другой — влажным (содержащим водяные пары). Температуры и давления в обоих сосудах одинаковы. Какой из сосудов легче? 11 Здесь и далее воздух можно рассматривать как газ с молярной массой равной 0,029 кг/моль. Эта величина соответствует средней массе молекул газов, входящих в состав воздуха (напомним, что воздух состоит примерно на 4/5 Из азота и на 1/5 — из кислорода). <
69 Молекулярная физика 9.40. В сосуде объемом V = 1,0 л находится азот массой т = 0,28 г. Азот нагрет до температуры t = 1500°С, при которой 30% всех молекул азота диссоциируют на атомы. Определите давление р в сосуде. 9.41*. В сосуде находится смесь азота и водорода. При начальной темпе- ратуре Т азот полностью диссоциирован на атомы, а диссоциацией водорода можно пренебречь. При нагревании до температуры 2Т оба газа полностью диссоциируют и давление утраивается по сравнению с начальным. Каково отношение масс азота и водорода в смеси? 9.42*. Герметически закрытый бак высотой h = 5,0 м заполнен водой доверху. На дне его находится пузырек воздуха. Давление на дно бака ро = 0,15 МПа. Каким станет давление р на дно, если пузырек всплывет? Стенки бака считаются абсолютно жесткими, вода — несжимаемой. 9.43**. Сжимаемость воды (относительное уменьшение ее объема при увеличении давления на 1 Па) Р = 5,0 10 10 Па-1. Какой должна быть площадь S основания цилиндрического бака, чтобы при решении задачи 9.42 воду действительнс можно было считать несжимаемой? Начальный объем пузырька Vo = 15 мм3. 9.44**. Герметически закрытый бак высотой Л — 5,0 м заполнен водой доверху. На дне его находятся два одинаковых пузырька воздуха. Давление на дно бака ро = 0,15 МПа. Каким станет это давление, если всплывет один пузырек? Оба пузырька? Стенки бака счита- ются абсолютно жесткими, вода — несжимаемой. 9.45*. Какой радиус г должен иметь наполненный гелием воздушный шар, чтобы он мог подняться в воздух, если масса 1 м2 оболочки шара mQ = 50 г? Температура воздуха t = 27°С, давление рЛ = 100 кПа. 9.46**. Валлон с газом разделен на две части теплоизолирующей перего- родкой с малым отверстием1’. По разные стороны перегородки все время поддерживаются температуры Ti и Т2. Определите отно- шение давлений р\ и р2 в различных частях баллона. ^1 I и»| — .т ii.ii.in *’ Это означает, что молекулы проходят в отверстие только «поодиночке», т. е. макроскопическое движение газа вблизи отверстия не может возникнуть.
Задачи 70 9.47*. Поршень массой т находится в равновесии / / посередине герметично закрытого цилиндра * *U* , * (см. рисунок). В каждой половине цилиндра I находится v молей газа, имеющего абсолют- ную температуру Т. Определите период т ма- I-... , Ж., лых колебаний поршня, считая, что температура газа при коле- баниях остается неизменной. Трением можно пренебречь. 10. ПАРЫ, ЖИДКОСТИ И ТВЕРДЫЕ ТЕЛА Насыщенные и ненасыщенные пары. Влажность воздуха 10.1. При ti = ЗО^С относительная влажность воздуха ipi = 80%. Какова будет относительная влажность <р2, если этот воздух нагреть при постоянном объеме до #2 = 509С? 10.2* . В помещение нужно подать V = 10000 м3 воздуха с температурой ti = 18°С и относительной влажностью <pi = 50%. Воздух снаружи имеет температуру #2 = 109С и относительную влажность <р2 — 60%. Осушать или увлажнять придется наружный воздух^ Сколько воды придется при этом сконденсировать или испарить? 10.3. В закрытом сосуде объемом V = 100 л при t = ЗО^С находится воздух с относительной влажностью <pi = 30%. Какой через некоторое время станет относительная влажность <р2, если в сосуд влить т = 1,0 г воды? Температура не изменилась. 10.4. В герметично закрытом сосуде объемом V = 1,1 л находятся т = 100 г кипящей воды и ее пар при температуре 100°С. Воздуха в сосуде нет. Определите массу пара тп. Зависимостью плотности воды от температуры можно пренебречь. 10.6. На какую высоту h можно поднять кипящую воду поршневом насосом? Считайте, что вода при подъеме не остывает. 10.6**. В открытом неглубоком сосуде находятся — одна над другой — две несмешивающиеся жидкости. При атмосферном давлении температура кипения нижней жидкости 77°С, верхней 100РС. Что
71 Молекулярная физика можно сказать о температуре t, при которой начнется кипение в сосуде? Можно ли ее найти по приведенным в условии данным? 10.7. Температура воздуха ti = 20°С, точка росы t2 = lO^C. Определите относительную влажность воздуха <р. 10.8* . В цилиндре под поршнем — воздух, имеющий температуру ti = 10^С и относительную влажность ipi — 60%. Какой станет относительная влажность фг этого воздуха после нагревания его до температуры t2 — 100^С и уменьшения объема втрое? 10.9. Воздух подвергается изотермическому сжатию, при этом его объем уменьшается в четыре раза. При каком объеме V воздуха начина- ется конденсация водяного пара? Какая масса т росы выпадает? Начальный объем Vo = 2,0 м3, температура t ~ 18°С, относительная влажность воздуха до сжатия ip0 = 50%. 10.10. Какова плотность насыщенного водяного пара при температуре 10(ГС? 10.11*. Сосуд объемом V = 10 л заполнен сухим воздухом при давлении Pi = 100 кПа и температуре ti = lO^ C. Каково будет давление р2 влажного воздуха, если в сосуд налить воду массой т = 10 г и нагреть до температуры t2 = ЮО^С? 10.12. В кухне развесили много выстиранного белья. На улице моросит холодный осенний дозедь. Быстрее ли высохнет белье, если открыть форточку? 10.13* . Два сосуда наполнены водой до разных уровней и соединены трубками с кранами (см. рисунок). Воздух из сосудов откачан. Что произойдет, если открыть нижний кран? Верхний? 10.14*. В теплоизолированном цилиндре под не- весомым поршнем находится вода массой т = 30 г при температуре ti = О^С. Площадь поршня S = 500 см2, внешнее давление р - 108 Па. На какую высоту Л поднимется поршень, если находящийся в цилиндре электронагреватель выделит энергию Q “ 25 кДж?
Задачи 72 10.15. Известно, что если воду подсолить, то температура ее кипения повысится. Как при этом изменится плотность паров в пузырьках кипящей жидкости? 10.16*. В запаянной с одного конца горизонтально лежащей трубке на- ходится воздух с относительной влажностью фо = 80%, отделен- ный от атмосферы столбиком ртути длиной I = 76 мм. Какой станет относительная влажность ф, если трубку поставить вертикально открытым концом вниз? Температура при этом не меняется, атмосферное давление ра = 760 мм рт. ст. Ртуть из трубки при переворачивании не выливается. 10.17*. При какой длине I столбика ртути (см. задачу 10.16) в трубке, поставленной вертикально открытым концом вверх, выпадет роса? 10.18*. Трубку длиной Z = 60 см, запаянную с одного конца, погружают в ртуть вертикально открытым концом вниз. При какой глубине погружения h в трубке выпадет роса? Температура в трубке не меняется. Атмосферное давление ра = 76 см рт. ст., относительная влажность воздуха ф = 80%. 10.19*. Паровой котел частично заполнен водой, а частично — смесью воздуха и насыщенного пара при температуре ti = 100°С. Началь- ное давление в котле pi = Зра = 300 кПа. Найдите давление р2 в котле после понижения температуры до t2 = KFC. 10.20**. В закрытом баллоне объемом V = 5,0 л находится смесь из mi = 0,50 г водорода и т2 = 8,0 г кислорода при давлении pi = 2,35- 10s Па. Между газами происходит реакция с образованием воды. Какое давление р установится в баллоне после охлаждения до первона- чальной температуры? 10.21**. При сжатии некоторой порции влажного воздуха его объем уменьшился в четыре раза, а давление возросло в три раза. Когда воздух сжали еще в два раза, давление стало в пять раз больше первоначального. Температура при сжатии оставалась постоян- ной. Какова была относительная влажность воздуха ф в самом начале?
73 Молекулярная физика Свойства жидкостей 10.22. Будет ли кипеть вода в кастрюле, которая плавает в другой кастрюле с кипящей водой? 10.23. Как заставить воду кипеть без нагревания? Как заставить воду замерзнуть кипением? 10.24*. Можно ли расплавить свинец в воде? 10.25*. Вода в стакане замерзает при охлаждении до 0рС. Почему же в некоторых облаках, представляющих собой скопление маленьких капелек воды, вода не замерзает и при более низких температурах (например, при -5°С)? 10.26. Капиллярная трубка с очень тонкими стенками подвешена вер- тикально к чашке рычажйых весов. Весы уравновешены. К трубке подносят снизу сосуд с водой, так что поверхность воды касается капилляра. Чтобы восстановить равновесие, пришлось увеличить груз на другой чашке весов на т = 0,14 г. Определите радиус г капилляра1’. 10-27- в с Легкая незамкнутая жесткая рамка (см. ри- -----------------1 сунок) плавает на поверхности воды. Какая сила F и в каком направлении будет дейст- вовать на рамку, если капнуть внутрь рамки мыльным раствором? Коэффициент поверх- ностного натяжения чистой воды сть мыль- ного раствора ст2, причем ст2 < сть A I Z I D 10.28*. Действительно ли нельзя носить воду в решете? Пусть тонкие нити решета протянуты на расстоянии d = 1,0 мм друг от друга. Сколько воды можно унести в таком круглом решете радиуса г = 10 см? 10.29. Прямоугольная проволочная рамка с массой т, стороны которой равны о и ft, касается поверхности воды всеми своими сторонами. Какую вертикальную силу F необходимо приложить, чтобы отор- вать рамку от поверхности воды? Рамка смачивается водой. 1’ В задачах 10.26 — 10.47 смачивание и несмачивание считается полным. Поверхность стекла считается чистой, т. е. смачиваемой водой.
Задачи 74 10.30. Для удаления жирных пятен материю проглаживают горячим утюгом, подЛожив под нее лист бумаги. Почему жир при этом впитывается в бумагу, а не расходится по материи? 10.31*. При смазывании лыжных ботинок их нагревают, чтобы мазь лучше впитывалась. Как нужно нагревать ботинки — снаружи или изнутри? 10.32**. Капиллярная стеклянная трубка радиуса г и высоты h соединена с широкой и более высо- ______ кой трубкой (см. рисунок). Широкая трубка н постепенно заполняется каплями воды, пада- ющими через равные промежутки времени, -ч Постройте графики изменения уровня воды в обеих трубках с течением времени и график изменения разности этих уровней. 10.33. Длинная, открытая с обоих концов тонкостенная капиллярная трубка радиусом г = 1,0 мм расположена вертикально. Какова максимально возможная высота h столба воды, находящейся в трубке? 10.34. Смачиваемый водой кубик массы т = 20 г плавает на поверхности воды. Длина ребра кубика а — 3,0 см. На каком расстоянии h от поверхности воды находится нижняя грань кубика? 10.35. На поверхности воды плавают два одинаковых кубика с длиной ребра а = 1,0 см, изготовленные из смачиваемого водой материала. Один из кубиксв натерли парафином. На сколько один из кубиков плавает теперь выше? Какой это кубик? 10.36* . Какой радиус должен иметь алюминиевый шарик, натертый парафином, чтобы он «плавал» в воде, погрузившись ровно напо- ловину? 10.37. Оцените максимальный размер капель воды, которые могут висеть на потолке. 10.38*. Докажите, что избыточное давление в жидкости под ее цилиндри- ческой поверхностью радиуса R равно ст/Я, а под сферической поверхностью 2ст/R.
75 Молекулярная физика 10.39*. Радиус одного колена 17-образной стеклянной капиллярной трубки и = 1,0 мм, радиус другого колена г2 = 2,0 мм. В трубку налита ртуть. К какому колену трубки следует присоединить вакуумный насос и какую разность давлений воздуха Др в коленах трубки должен он создать, чтобы уровни ртути в обоих коленах оказались одинаковыми? Ртуть не смачивает стекло. 10.40*. Куда будет двигаться в горизонтальном коническом капилляре капля смачивающей жидкости? Капля несмачивающей жидкости? 10.41*. В двух длинных открытых с обеих сторон вертикальных капилля- рах находятся столбики воды длиной 2,0 и 4,0 см. Найдите радиус г кривизны нижнего мениска в каждом из капилляров, если их внутренний диаметр d = 1,0 мм. 10.42*. Конец стеклянной капиллярной трубки радиуса г = 0,50 мм опущен в воду на глубину h = 2,0 см. Какое избыточное давление Др необходимо создать в трубке, чтобы выдуть пузырек воздуха через ее нижний конец? 10.43. Предлагается проект вечного двигателя (см. рисунок): тонкая смачиваемая водой изогну- тая трубка вставлена в широкий сосуд с во- 1 д —— дой. Вода поднимается по трубке и стекает Z- ?- Z - Z- Z- Z обратно в сосуд. Вытекающая из трубки вода _ > > - ~ может совершать полезную работу. В чем Гт 7 ~.7 дефект этого проекта? 10.44*. Стеклянная капиллярная трубка, внутренний диаметр которой d — 0,50 мм, вертикально погружена в воду. Верхний конец трубки выступает на h = 2,0 см над поверхностью воды. Какую форму имеет мениск? 10.45. Радиус мыльного пузыря равен R. Найдите давление воздуха р внутри пузыря. Давление воздуха вне пузыря ро, поверхностное натяжение мыльной пленки о. 10.46*. Оцените силу, необходимую для разъединения двух «слипшихся» зеркальных стекол размером 1 м х 1 м, между которыми попала Вода. Среднее расстояние между стеклами d = 0,2 мм. Как можно облегчить разъединение стекол?
Задачи 76 10.47**. На четыре ртутных шарика, лежащих на горизонтальной плос- кости, осторожно кладут квадратную пластинку (на рисунке пока- зан вид сверху). Радиус шариков л> = 1 мм, масса пластинки т = 80 г. Смачивания нет. Определите зазор d между плоскостью и плас- тинкой. К задаче 10.47 К задаче 10.48 10.48*. В три сосуда (см. рисунок) налита до одинакового уровня горячая вода. Как изменится давление на дно сосудов после остывания воды? Свойства твердых тел 10.49. При изготовлении некоторых точных приборов необходимо обес- печить постоянство разности длин двух стержней при изменении температуры. Какие длины должны иметь железный и медный стержни при 0°С, чтобы разность AZ их длин не зависела от температуры и оставалась равной 10 см? 10.50. Металлическое кольцо и диск с выре- /f77777'?'K занным сектором (см. рисунок) нагрева- (//////Тг ют. Изменится ли (и как) внутренний у/л. )//j ч ///ПгГ'Г'т- диаметр кольца? Изменится ли угол xj//м+/г раствора сектора ср? х'<444*' 10.51. Концы стального стержня сечением S = 2,0 см2 прочно закреплены при температуре Л = 15°С. С какой силой F стержень будет действовать на опоры, если его нагреть до t2 = 150рС? 10.52. Стальная проволока натянута горизонтально с небольшим уси- лием между двумя стенками при температуре ti = 0рС. При охлаждении до какой температуры t2 она может лопнуть? ;и эн чу;-
77 Молекулярная физика 10.53* *. Между двумя столбами натянута с не- большим усилием легкая проволока дли- ной 21. К проволоке посередине подве- шивают фонарь массой т. Площадь попе- речного сечения проволоки равна S, мо- дуль упругости материала Е. Определите угол провисания проволоки а (см. ри- сунок), считая его малым. 10.54. 21 Железобетонная колонна сжимается силой F. Какая часть на- грузки приходится на железо, если площадь поперечного сечения железа S» составляет 1/20 площади поперечного сечения бетона S6, а модуль упругости бетона составляет 1/10 модуля упругости железа Еж2 10.55. Из скольких стальных проволок диаметром d = 2,0 мм должен состоять трос, рассчитанный на подъем груза массой т = 16 т? 11. ЗАКОНЫ ТЕРМОДИНАМИКИ Уравнение теплового баланса и фазовые переходы 11.1. В калориметр помещен лед с начальной температурой ti — -10°С и нагреватель постоянной мощности. Постройте график изме- нения в зависимости от времени т температуры t в калориметре (от ti др <2 — Н-КРС). 11.2. Для нагревания некоторой массы воды от температуры ti = 20рС до температуры кипения t2 = 100° С электронагреватель работал в течение Ti = 12 мин. Какое время потребуется после этого, чтобы обратить всю воду в пар? Теплоемкостью сосуда и потерями теплоты пренебречь. 11.3* . В калориметр1’, содержащий тв = 1,5 кг воды при температуре tB = 2QPC, положили тп = 1,0 кг льда, имеющего температуру tx = -10°С. Какая температура 0 установится в калориметре? 11.4* . Решите задачу 11.3 при: а) тя = 0,10 кг; б) тя = 8,0 кг. Остальные числовые значения в условии не изменяются. Определите в каж- ’’ Здесь и далее можно пренебречь теплоемкостью калориметра, если ее значение в условии не приведено.
Задачи 78 дом случае суммарный объем вещества в калориметре после установления теплового равновесия. 11.5* . В калориметре находится вода массой zni = 400 г при темпера- туре ti = 5°С. К ней долили еще zn2 = 200 г воды с температурой <2 = 10рС и положили т3 = 400 г льда с температурой t3 = -60°С. Какая температура 0 установится в калориметре? Как изменится количество льда? 11.6* . В сосуд с водой при температуре t = 20РС поместили тл — 100 г льда с температурой <л = -8°С. Какая установится температура? Теплоемкость сосуда с водой 1,67 кДж/К. 11.7. Калориметр содержит лед массой т„ = 100 г при температуре <л = 0°С. В калориметр впускают пар с температурой tn = 100рС. Сколько воды оказалось в калориметре, когда весь лед растаял? Температура образовавшейся воды равна 0рС. 11.6. Смесь, состоящую из тпл = 5,0 кг льда и тя — 15 кг воды при общей температуре ti = 0рС, нужно нагреть до температуры 0 = 80°С, .пропуская водяной пар с температурой ti = 100рС. Определите необходимую массу пара тпп. 11.9. Калориметр содержит zni = 250 г воды при температуре ti = 15°С. В воду бросили т.2 = 20 г мокрого снега. В результате температура в калориметре понизилась на At = 5°С. Сколько воды было в снеге? 11.10. Нагретый алюминиевый куб положили на лед, и куб полностью погрузился в лед. До какой. температуры t был нагрет куб? Температура льда 0рС, потерями тепла пренебречь. 11.11* . При соблюдении некоторых мер предосторожности воду можно переохладить, т. е. охладить ниже 0р,С. Пробирку, содержащую т = 12 г переохлажденной воды с температурой t = -5°С, встря- хивают. При этом часть воды замерзает. Какова масса тя образо- вавшегося льда? Теплообменом с окружающей средой и теплоем- костью самой пробирки можно пренебречь. 11.12* . В колбе находилась вода при 0°С. Выкачивая из колбы воздух, добились того, что в ней остался только лед. Какая часть воды при этом испарилась? Удельная теплота парообразования воды при (УС равна 2,50 МДж/кг.
79 Молекулярная физика 11.13. Два одинаковых кусочка льда летят навстречу друг другу с равными скоростями и при ударе превращаются в воду. Оцените, при какой минимальной скорости v льДинок перед ударом это возможно. Температура льдинок перед ударом ti — -12°С. 11.14. Свинцовая пуля ударяется о броневую плиту и отскакивает от нее. На нагревание пули идет 60% потерянной ею механической энергии. Скорость пули перед ударом о плиту Vo = 400 м/с, после удара v = 100 м/с. Температура пули перед ударом ti = 50°С. Какая часть пули расплавилась? 11.15. Установка, выделяющая тепловую мощность N = 50 кВт, охлаж- дается проточной водой, текущей по спиральной трубке диамет- ром d = 15 мм. При установившемся режиме проточная вода нагревается на \t — 25°С. Определите скорость v течения воды. 11.16. С какой высоты h должна падать вода, чтобы при ударе о землю она закипала? На нагрев воды идет 50% расходуемой механичес- кой энергии, начальная температура воды ti = 20°С. Газовые процессы 11.17. После включения отопления воздух в комнате нагревается от температуры То до температуры Т. Во сколько раз изменяется внутренняя энергия воздуха, содержащегося в комнате? 11.18. Объем некоторой массы газа увеличивается вдвое. В каком случае газ совершит большую работу — при изобарном расширении или при изотермическом? 11.19*. На рисунке показан график цикли- g ческого процесса с некоторой массой I *1 | идеального газа. На каких этапах ‘1 ’' процесса газ получал тепло, на каких "*•..... отдавал? Какое количество теплоты больше: полученное от нагревателя или отданное холодильнику? ------------------* 11.20. v В цилиндре под поршнем находится воздух. С ним поочередно происходят следующие процессы: изохорное нагревание, изобар-
Задачи 80 ное расширение, изотермическое расширение. Затем воздух изо- барно возвращается в исходное состояние. Начертите график процесса в координатах р, V. Укажите, на каких этапах процесса воздух получает тепло и на каких отдает. 11.21. Газ находится в вертикальном цилиндре, герметично закрытом сверху поршнем. Цилиндр помещают в термостат с высокой температурой. Одинаковое ли количество теплоты получает газ, если: 1) поршень закреплен; 2) поршень легко перемещается? 11.22*. Докажите, что молярные теплоемкости1’ идеального газа при пос- тоянном давлении Ср и при постоянном объеме Су связаны соот- ношением Ср - Су = R, где R — универсальная газовая постоянная. 11.23*. Для нагревания т = 2,0 кг неизвестного газа на ЛГ = 5,0 К при постоянном давлении требуется количество теплоты Qp = 9,1 кДж, а для нагревания при постоянном объеме требуется Qy = 6,5 кДж. Какой это может быть газ? 11.24 *. Чему равны молярные теплоемкости одноатомного идеального газа при постоянном объеме Су и при постоянном давлении Ср? Найдите их отношение у = Ср/Су. Что можно сказать о величине этого отношения для жидкости? 11.25 . Всегда ли температура газа повышается при получении некоторого количества теплоты? 11.26 *. На рисунке изображены два замкнутых процесса, происходящих К задаче 11.26 К задаче 11.27 ’’ Молярная теплоемкость численно равна количеству теплоты, необходимому для нагревания одного моля вещества на 1 К.
81 Молекулярная физика с идеальным газом: 1-2-3-1 и 3-2-4-3. В каком из них газ совершает большую работу? 11.27 **. Определите работу А', совершенную одним молем идеального газа за цикл (см. рисунок). Известны температуры газа Л и Гз в состояниях 1 и 3. Точки 2 и 4 лежат на одной изотерме. 11.28 *. Газ под поршнем сжимают, помещая на поршень груз. Сравните конечную температуру и необходимую для сжатия работу в двух случаях: а) на поршень одну за другой медленно ставят небольшие гири; б) на поршень сразу ставят одну гирю большой массы. Конечный объем газа в обоих случаях одинаков. Сосуд с газом не теплоизолирован. 11.29 . Кислород нагревают при постоянном давлении от температуры ti = 0°С. Какое количество теплоты Q необходимо сообщить газу, чтобы его объем удвоился? Количество вещества v = 1 моль. 11.30*. Газ находится в вертикальном цилиндре с площадью дна S = 10 см2. Цилиндр закрыт перемещающимся без трения поршнем массой т = 9,8 кг. Начальный объем газа Vo = 5,0 л, температура to = (ГС. Давление наружного воздуха рЛ = 100 кПа. Какое количество теплоты Q необходимо затратить для нагревания газа при этих условиях на ДТ = 10 К? Известно, что повышение температуры газа на ту же величину при закрепленном поршне потребовало бы количества теплоты Qi = 90 Дж. 11.31**. Какое количество теплоты получает од- ноатомный газ при нагревании в ходе процесса, изображенного на рисунке? Получает или отдает тепло газ при ох- лаждении? Чему равно это количество теплоты? 11.32* . В длинной, расположенной горизонта- льно теплоизолированной трубе между двумя одинаковыми поршнями (масса каждого равна т) находится v = 1 моль одноатомного газа при температуре То. В начальный момент поршни сближаются, причем скорости поршней направлены в одну сторону и равны 3v и v. До какой максимальной температуры Г нагреется газ? Массой газа по сравнению с массой поршней можно пренебречь. Поршни тепло не проводят. Трение пренеб- режимо мало. Атмосферное давление не учитывать.
Задачи 82 Тепловые машины 11.33. Автомобиль движется со скоростью v = 72 км/ч. Мощность двига- теля N = 60 кВт, его КПД равен 30%. Определите расход бензина на s = X км пути. 11.34. Реактивный самолет имеет четыре двигателя, каждый из которых развивает силу тяги F = 30 кН. Коэффициент полезного действия двигателей равен 25%. Определите расход керосина на перелет длиной I = 4000 км. 11.35. Понизится ли температура в комнате, если открыть дверцу рабо- тающего холодильника? 11.36* *. Из-за несовершенства теплоизоляции холодильник получает от воздуха в комнате количество теплоты Q = 420 кДж за время т = 1 ч. Температура в комнате ti= 2(ГС. Какую минимальную мощность Р должен потреблять холодильник от сети, чтобы под- держивать внутри холодильного шкафа температуру = -5°С? 11.37**. Казалось бы, эффективнее всего отапливать Помещение с помощью электронагревательных приборов: при атом вся потребляемая электроэнергия превращается во внутреннюю и передается возду- ху в помещении, т. е. КПД = 100%. Однако существует нагрева- тель и с большим КПД — так называемый «тепловой насос», который отбирает энергию для обогрева комнаты у наружного воздуха1*. Чему равен КПД теплового насоса, работающего по циклу Карно, если температура воздуха в помещении равна Т, а на улице Т'Ч 11.38. Холодильник, потребляющий мощность Р, за время г превратил воду в лед. Какое количество теплоты Q передал холодильник воздуху в комнате, если масса воды гп, а ее начальная температура t? Теплоемкостью самого холодильника можно пренебречь. 11.39*. Газовая нагревательная колонка потребляет Уо = 1,2 м3 метана (СН4) в час. Найти температуру t подогретой воды, если вытекаю- 1> Наглядной моделью теплового насоса может служить холодильник, поставлен- ный на окно или встроенный в стену здания так, чтобы холодильная камера сообщалась с атмосферой, а полученная из атмосферы и от сети энергия выделя- лась на задней стенке, обращенной в комнату.
83 Молекулярная физика щая струя имеет скорость и = 0,5 м/с. Диаметр струи d = 1,0 см, начальная температура воды и газа to = 11°С. Газ в трубе находится под давлением р = 1,2 атм. КПД нагревателя г] = 0,6. 11.40**. Одноатомный идеальный газ совершает показанный на рисунке цикл из двух изохор и двух изобар. Определите КПД цикла. 11.41**. Рабочим телом тепловой машины является одноатомный идеаль- ный газ. Определите КПД тепловой машины, график цикла кото- рой показан на рисунке. 11.42**. КПД цикла 1-2-3-1 (см. рисунок) равен т|1, а КПД цикла 1-3-4-1 равен т]2. Определите КПД ц цикла 1-2-3-4-1. К задаче 11.42
Закон Кулона F = БГ 4тГЕоЕ< Напряженность поля точечного заряда Е = — 4Л£оЕГ! Напряженность доля равномерно заряженной бесконечной плос- кости Е = ~^ 2&£ Работа электростатических (кулоновских) сил А = qU = g(<pi - <р2) Потенциальная энергия заряда в электрическом поле = qq> _ q Потенциал поля точечного заряда <р = —2—- 4лЕоЕГ Электроемкость конденсатора С = q/U „ ЕоЕВ Электроемкость плоского конденсатора С = СЕТ2 о2 Энергия заряженного конденсатора W, = £t Закон Ома для участка цепи I -U/R I Электрическое сопротивление проводника R - р-= О ё Закон Ома для замкнутой (полной) цепи I =--- Зависимость сопротивления проводника от температуры р = р<>(1 + at) Работа электрического тока А = Ult Закон Джоуля-Ленца Q = I2Rt 1 М Закон электролиза т = kit = —It Закон Ампера F = I • AZBsina = I • AZB± Сила Лоренца F = |g|vBsina = |g|vBi Магнитный поток Ф = BScosa = B„S Закон электромагнитной индукции S, = - ДФ/At ЭДС самоиндукции S.i = -L— At Энергия магнитного поля WM = LF/Z Формула Томсона Т - Zit'lLC Емкостное сопротивление в цепи переменного тока Хс = - фС Индуктивное сопротивление в цепи переменного тока Хъ = ©Z, Действующие значения силы тока и напряжения I = Д=, U = у 2 у2
ЭЛЕКТРИЧЕСТВО И МАГНЕТИЗМ ЭЛЕКТРОСТАТИКА ЗАКОНЫ ПОСТОЯННОГО ТОКА ЭЛЕКТРИЧЕСКИМ ТОК В РАЗЛИЧНЫХ СРЕДАХ Г'"" МАГНИТНОЕ ПОЛЕ \ СИЛА ЛОРЕНЦА (ЭЛЕКТРОМАГНИТНАЯ ИНДУКЦИЯ "ЭЛЕКЪОМАГН ИТН Ы Е КО^ЕБА^ИЯ И ВОЛНЫ '^ПЕРЕМЕННЫЙ ТОК о 6 о
Задачи 86 12. ЭЛЕКТРОСТАТИКА” Закон Кулона. Напряженность электрического поля 12.1. Два металлических шара имеют равные по величине заряды. Зависит ли величина силы их взаимодействия от того, будут эти заряды одноименными или разноименными? Расстояние между центрами шаров в обоих случаях одно и то же. 12.2. Могут ли два одноименно заряженных тела притягиваться? 12.3. Два разноименно заряженных шарика находятся на некотором расстоянии друг от друга. Между ними помещают стеклянный стержень. Как изменится сила взаимодействия шариков? 12.4* . После того, как два маленьких заряженных металлических ша- рика привели в соприкосновение и раздвинули на прежнее рассто- яние, сила их кулоновского взаимодействия увеличилась по мо- дулю в п = 4/3 раза. Одноименными или разноименными были первоначально заряды qi и q2 на шариках? Во сколько раз они отличались по модулю? Радиусы шариков равны. 12.5* *. Может ли точечный заряд находиться в состоянии устойчивого равновесия под действием только кулоновских сил? 12.6. Два разноименных точечных заряда q и -4q q -4q закреплены на расстоянии а друг от друга Ф а ♦ (см. рисунок). Каким должен быть заряд q0 ~ и ₽Де следует его расположить, чтобы вся система находилась в равновесии? 12.7. Решите задачу 12.6, заменив в условии заряд -4g на 4g. 12.8. Три одинаковых одноименных заряда g расположены в вершинах равностороннего треугольника. Какой заряд Q нужно поместить в центре треугольника, чтобы система зарядов находилась в равно- весии? 12.9. В вершинах квадрата находятся четыре одинаковых одноименных заряда д. Какой заряд Q нужно поместить в центр квадрата, чтобы 11В этом разделе используется обозначение к-1/(4лЕо)
87 Электричество и магнетизм система находилась в равновесии? Будет ли это равновесие ус- тойчивым? 12.10. Совпадают ли силовые линии электростатического поля с траек- ториями движения точечного положительного заряда, начальная скорость которого равна нулю? 12.11*. Можно ли создать электростатическое поле, линии напряжен- ности которого имеют вид, показанный на рисунке? 12.12. На рис. а, б, в показаны картины силовых линий трех электри- ческих полей. Как будет вести себя незаряженный шарик, поме- щенный в каждое из этих полей? Рис. а Рис. б Рис. в К задаче 12.11 К задаче 12.12. 12.13**. Три одинаковых положительных заряда q расположены в верши- нах равностороннего треугольника со стороной а. Определите величину напряженности поля Е в точке, лежащей на расстоянии а от каждого из зарядов. 12.14. В однородное электрическое поле вносят металлический незаря- женный шар. Где и какие индуцированные заряды появятся на шаре? Нарисуйте линии напряженности поля и эквипотенциаль- ные поверхности. 12.16*. _ Электрическое поле создается положитель- +q А \ ным точечным зарядом (см. рисунок). Как • • I у изменятся напряженность Е и потенциал <р 4 * * * В * * "" поля в точке А, если справа от нее поместить незаряженный шар? 12.16. Пылинка покоится в пространстве между горизонтальными пластинами плоского конденсатора. Ее масса т = 3,0 IO"11 г, рас-
Задачи 88 стояние между пластинами конденсатора d = 5,2 мм. После облучения ультрафиолетовым излучением пылинка теряет часть заряда и начинает опускаться. Чтобы восстановить равновесие, потребовалось увеличить начальное напряжение Uo = 480 В на ДС7 = 25 В. Какой заряд Aq потеряла пылинка? 12.17*. Два одинаковых одноименно заряженных шарика, подвешенных в одной точке на нитях равной длины, опускают в керосин. При этом угол расхождения нитей не изменяется. Какова плотность р материала шариков? 12.18*. На одинаковых нитях, закрепленных в одной точке, висят два одинаковых маленьких шарика, несущих одинаковые заряды. Шарики разошлись на расстояние а — 9,5 см, которое намного меньше длины нитей. Один из шариков разрядили. Что произой- дет с шариками после этого? При каком расстоянии b между шариками снова установится равновесие? 12.19*. Три маленьких одинаково заряженных шарика массой т = 4,0 г каждый подвешены на шелковых нитях длиной I = 1,0 м. Верхние концы нитей закреплены в одной точке. Расстояние от каждого шарика до двух других одинаково: а — 5,0 см. Каков заряд q каждого шарика? Работа электростатического поля. Потенциал. Проводники и диэлектрики в электростатическом поле 12.20. Два электрона, находящиеся очень далеко друг от друга, движутся навстречу друг другу вдоль одной прямой с одинаковыми по величине скоростями Vo = 1000 км/с. На какое наименьшее расстояние они сблизятся? 12.21*. Два электрона находятся на большом расстоянии друг от друга. Вначале один электрон неподвижен, а другой приближается к нему с начальной скоростью Vo, направленной вдоль соединяющей электроны прямой. На какое наименьшее расстояние они сбли- зятся? С какими скоростями разлетятся? 12.22*. Два одноименных точечных заряда Qi и с массами zni и тг движутся навстречу друг другу. Когда расстояние между ними равно г, их скорости Vi и v2. До какого минимального расстояния ГппП сблизйтся заряды?
89 Электричество и магнетизм 12.23*. Четыре шарика, имеющие одинаковые заряды q, расположены вдоль одной прямой с интервалом а. Какую работу А нужно совершить, чтобы разместить шарики: а) в вершинах квадрата со стороной а; б) в вершинах тетраэдра с ребром а? 12.24**. Два одинаковых металлических шарика радиуса R = 1 мм со- единены длинным тонким проводом. Один из них размещен в разреженном воздухе, а другой — посередине большой вакуумной камеры. На расположенный в вакууме шарик падает с большого расстояния поток электронов с начальной скоростью Vo =3000 км/с. Какой заряд Q можно накопить таким способом на шариках? Каким будет ответ, если увеличить начальную скорость электро- нов до Vo =10 000 км/с? Электрический пробой разреженного воздуха происходит при напряженности электрического поля Ео = 3 104 В/м. 12.25*. По тонкому металлическому кольцу радиуса R равномерно расп- ределен заряд q. Определите напряженность поля Е и потенциал <р в точке А, расположенной на оси кольца на расстоянии h от его центра. 12.2Q*. Электрон находится на оси тонкого кольца радиуса R на рассто- янии h от его центра. Кольцо получает положительный заряд q и начинает притягивать электрон. С какой скоростью v пролетит электрон через центр кольца? Обязательно ли это произойдет? 12.27*. Тонкое проволочное кольцо радиуса R несет на себе электрический заряд q. В центре кольца расположен одноименный с q заряд Q, причем | Q |» | q |. Определите силу Т, с которой растянуто кольцо. 12.28**. Тонкое проволочное кольцо радиуса R имеет электрический заряд +Q. Маленький шарик массой т, имеющий заряд -q, может двигаться без трения по тонкой диэлектрической спице, проходя- щей вдоль оси кольца. Как будет двигаться шарик, если его отвести от центра кольца на расстояние Хо < R и отпустить без начальной скорости? Запишите уравнение движения шарика x(t). Как изменится движение, если убрать спицу? 12.29*. Тонкое неподвижное проволочное кольцо радиуса R имеет заряд +Q. В центре кольца — маленький шарик массы т с зарядом -q.
Задачи 90 Шарику толчком придают начальную скорость vo, направленную вдоль оси кольца. Как зависит характер движения шарика от величины ио? 12.30**. На расстоянии Л от большой плоской проводящей пластины находится точечный заряд +q. С какой силой F действует пластина на заряд? 12.31*. Заряд Q равномерно распределен по объему шара радиусом R из непроводрщего материала. Чему равна напряженность поля Е на расстоянии г от центра шара? Постройте график Е(г). 12.32**. Вернувшись к условию задачи 12.31, определите потенциал поля Ф на расстоянии г от центра шара. Постройте график ф(г). 12.33. Металлический шар радиуса R имеет заряд Q. Чему равны напря- женность поля Е и потенциал ф на расстоянии г от центра шара? Постройте графики зависимостей Е(г) и ф(г). 12.34. Металлический заряженный шар окружен толстым сферическим слоем диэлектрика (е — 2). Нарисуйте картину силовых линий поля. Почему поле скачкообразно изменяется при переходе через границу диэлектрика? 12.35**. На расстоянии г от центра изолированной металлической незаря- женной сферы радиуса R находится точечный заряд q. Определите потенциал сферы ф при г > R. 12.36*. Проводящий шар радиуса R соединен тонкой длинной проволокой с землей. На расстоянии г от его центра размещают точечный заряд +q. Какой заряд Q приобретает шар? Влиянием Проволоки на поле пренебречь. 12.37. Два металлических шара, расположенные далеко друг от друга, имеют радиусы Bi = 5 см, В2 = 15 см и заряды qi = 12 нКл, = -40 нКл. Шары соединяют тонкой проволокой. Какой заряд Д<7 пройдет по проволоке? 12.38*. Внутри полой проводящей незаряженной сферы радиуса R поме- щен (не обязательно в центре) маленький шарик с зарядом +Q. 1) Какие заряды индуцйруются на сфере? Как они распределяются
91 Электричество и магнетизм по ней? 2) Как выглядят силовые линии электрического поля? Чему равен потенциал <р сферы? 3) Будет ли поле действовать на другой точечный заряд +q вне сферы, на расстоянии гот ее центра? Если будет, то с какой силой? 4) Как изменятся распределение зарядов и поле, если сферу заземлить? (Считайте q < Q.) 12.39*. Внутрь полой проводящей незаряженной сферы помещен шарик с зарядом q, после чего сферу на короткое время заземляют, а затем шарик осторожно удаляют из сферы через небольшое отверстие так, что со сферой он не соприкасается. Какой заряд имеет сфера после этого? Как он распределен? Что представляет собой электрическое поле сферы? 12.40*. Внутри тонкой металлической сферы ра- диуса Я = 20 см (см. рисунок) находится / металлический шар радиуса г — R/2 (цент- / /7777Х. \ ры шара и сферы совпадают). Через ма- / (/ r////\ i ленькое отверстие в сфере проходит длин-. I / ный провод, с помощью которого шар \ —jL заземлен. На сферу помещают заряд Q = У _А_ = 20 нКл. Определите ее потенциал <р. 12.41*. Три концентрические тонкие металлические сферы имеют радиу- сы Bi, 1?2, 2?з, причем Ri < Rz < Rz. Первая и третья сферы заземлены, вторая имеет заряд Q > 0. Найдите напряженность электрического поля Е во всех точках пространства. 12.42*. Два одинаковых металлических шарика, имеющие радиусы г и заряды q, расположены на расстоянии R » г друг от друга и очень далеко от Земли. Шары поочередно заземляют на короткое время. Какими станут заряды шаров после N заземлений каждого из них? 12.43. Металлический шар радиуса Ri, заряженный до потенциала <pi, окружают концентрической с ним тонкой проводящей сферичес- кой оболочкой радиуса R-z. Каким станет потенциал шара, если его соединить проводником с оболочкой? Если соединить оболочку с Землей? 12.44**. Два одинаковых металлических шара радиуса R, находящиеся далеко друг от друга, соединены тонким проводом. Каждый из шаров имеет заряд Q и окружен концентрической с ним тонкой
Задачи 92 незаряженной проводящей сферической оболочкой. Радиус одной оболочки равен R + а, радиус другой оболочки равен R + Ъ, причем а « R, Ъ <с R. Какой заряд q пройдет по проводу, если обе оболочки заземлить? 12.45**. По сферической оболочке радиуса R равномерно распределен заряд Q. Какая растягивающая сила f действует на единицу площади оболочки? 12.46**. Капле ртути радиуса R ~ 1 мм, находящейся в вакууме, сообщили заряд Q = 1 нКл. Найдите давление р внутри капли. Электрическая емкость 12.47. Во сколько раз изменится емкость плоского конденсатора, если в Рис. а Рис. б 12.48. Конденсатор состоит из трех полосок фольги площадью S = 4 см2 каждая, разделенных слоями слюды толщиной d = 0,2 мм. Край- ние полоски фольги соединены между собой (см, рисунок). Какова емкость С такого конденсатора? К задаче 12.48 К задаче 12.49
93 Электричество и магнетизм 12.49. Определите заряды на каждом из конденсаторов в цепи, изобра- женной на рисунке, если Ci = 2 мкФ, Сг = 4 мкФ, Cg = 6 мкФ, & = 18 В. 12.50*. Определите емкость Со батареи конденсаторов, изображенной на рисунке. К задаче 12.50 D F К задаче 12.51 12.51*. Определите емкость Со представленной на рисунке батареи одина- ковых конденсаторов. 12.52*. Определите заряд каждого из конденсаторов и разность потен- циалов между точками D и Е (см. рисунок), если Ci = Сг = С3 = С, а С< = 4С. К точкам А и В подведено постоянное напряжение U. К задаче 12.52 зс ФбС D „6С 6C Е 12С К задаче 12.53 12.53*. Определите емкость Со показанной на рисунке батареи конден- саторов. 12.54*. Из проволоки сделан куб, в каждое ребро которого вставлен конденсатор с емкостью С. Куб подключен к цепи противоположными вершинами, как показано на рисунке. Определите ем- кость Со получившейся батареи конден- саторов. в
Задачи 94 12.55. Два одинаковых плоских конденсатора соединены параллельно и заряжены до напряжения Uo = 240 В. После отключения от источника тока расстояние между пластинами одного из конден- саторов уменьшили в три раза. Каким станет напряжение U на конденсаторах? 12.56. Два одинаковых плоских воздушных конденсатора соединены последовательно, и к ним подведено постоянное напряжение U. Первый конденсатор заполняют диэлектриком с диэлектрической проницаемостью е. Во сколько раз изменится напряженность электрического поля в каждом из конденсаторов? 12.57* . Два конденсатора, имеющие емкости Ci — 10 мкФ и С2 = 50 мкФ, соединены последовательно. Каждый из них способен выдержать напряжение соответственно Ui и U2. Какое наибольшее напря- жение Umax может выдержать батарея из этих конденсаторов? Решите задачу при: a) Ui = U2 = 120 В; б) Ui = 120 В, U2= 6 В. 12.58. Пять точек попарно соединены через конденсаторы с емкостью С. Какова емкость Со между любыми двумя из этих точек? 12.59* . Определите разность потенциалов между точками А и В в схеме, представленной на рисунке. Какой станет эта разность потен- циалов и полная емкость С системы конденсаторов, если между точками А-и В включить резистор с сопротивлением R1 К задаче 12.59 К задаче 12.60 12.60. Определите разность потенциалов между точками А и В в схеме на рисунке. 12.61. Определите емкости показанных на рис. а-е систем. Все конден- саторы имеют емкость С.
95 Электричество и магнетизм Рис. а Рис. б Рис. в Рис. г Рис. д Рис. е К задаче 12.61 12.62*. Три источника ЭДС и три конденсатора соединены так, как показано на рисунке. Найдите напряжение U на каждом из конден- саторов, если (э! — 300 В, #2 = 150 В, = 100 В; Ci = 15 мкФ, С2 = 10 мкФ, С3 = 5 мкФ. К задаче 12.62 К задаче 12.63 12.63*. В схеме, изображенной на рисунке, емкость каждого конденсатора равна С. Вначале ключ разомкнут, конденсатор 1 заряжен до напряжения Uo, остальные конденсаторы не заряжены. Опре- делите напряжение на каждом из конденсаторов после замыкания ключа.
Задачи 96 12.64*. Две одинаковые металлические квадратные пластины размерами ах а находятся на расстоянии d « а друг от друга. Одна из пластин имеет заряд +3Q, а другая — заряд +Q. Определите напряжение U между пластинами. Как разместятся заряды на каждой из пластин? 12.65*. Найдите разность потенциа- лов U на выходе цепи (см. рисунок), если на вход пода- но напряжение Uo — 80 В. Емкости конденсаторов: Ci = 1 мкФ, С2 = 2 мкФ. 12.66*. Пространство между обкладками плоского конденсатора частично заполнено диэлектриком с диэлектрической проницаемостью е. Пло- щадь ыластин конденсатора равна 8. Определите емкость кон- денсатора С в каждом из случаев, показанных на рисунках а, б, в. hzzzzzzizzzm т Рис. а Рис. 6 Рис. в 12.67*. Решите задачу 12.66 для случаев б, в, заменив диэлектрик на проводник. 12.68*. В плоский конденсатор помещают две параллельные тонкие металлические пластины на одинаковом расстоянии друг от друга и от обкладок конденса- j-— — тора (см. рисунок). На обкладки кон- ’ денсатора подано напряжение U, об- кладка 1 заземлена. 1) Каковы потен- циалы пластин 2 и 3? 2) Как изменят- ся потенциалы пластин и напряжен- 12 3 4 ность поля во всех трех промежутках, если пластины 2 и 3 на короткое время замкнуть проволокой? 3) Во сколько раз изменя- ется емкость конденсатора при замыкании пластин 2 и 3? Изме- няются ли при этом заряды на обкладках 1 и 4?
97 Электричество и магнетизм Энергия электрического поля. Движение заряженных частиц в электрическом поле 12.69*. Конденсатор подключен к аккумулятору. Как изменится энергия конденсатора при раздвигании его пластин? Как согласуется это изменение с законом сохранения энергии? Каким будет ответ в случае, если заряженный конденсатор отключен от аккумулятора перед раздвиганием пластин? 12.70*. Заряженный конденсатор заполняют диэлектриком с диэлектри- ческой проницаемостью е. Как изменяется его заряд q, напря- жение на конденсаторе U, напряженность поля в конденсаторе Е, запасенная энергия W, если: а) конденсатор отключен от батареи; б) конденсатор подключен к батарее? 12.71*. Плоский конденсатор, заряженный и отключенный от источника ЭДС, помещен в сосуд с жидким диэлектриком. Если открыть кран у дна сосуда, диэлектрик вытечет и энергия конденсатора увеличится (ср. с задачей 12.70). Откуда возьмется при этом добавочная энергия? 12.72. Пластины плоского конденсатора раздвигают. В каком случае придется совершить большую работу: а) конденсатор все время подключен к источнику напряжения; б) конденсатор отключен от источника после зарядки? 12.73**. Какое количество теплоты Q выделится в ----------•f"1"" цепи при переводе ключа из положения 1 ] \ !> в положение 2 (см. рисунок)? Энергией zzC "^“С С-- электромагнитного излучения можно пре- | небречь. -------1------- 12.74*. Капля ртути, заряженная до потенциала <р0, распадается на N одинаковых капель с одинаковыми зарядами. Капли разлетаются на большое расстояние друг от друга. Определите потенциал <р каждой из образовавшихся Цапель. 12.76*. Во сколько раз изменяется энергия электрического поля при Распаде заряженной капли ртути на N одинаковых капель, разле- тающихся на большое расстояние друг от друга?
Задачи 98 12.76*. _____q Четыре одинаковых шарика с одинако- д— • выми одноименными зарядами q (см. ри- у ' / сунок) связаны одинаковыми нерастя- / \ / / жимыми нитями. Докажите, что равно- / *С / весне достигается, когда шарики распо- / ' \ / лагаются в вершинах квадрата. На ша- \ / рики действуют только кулоновские си- L' лы и силы натяжения нитей. Ч 12.77**. Внутри гладкой диэлектрической сферы радиуса R находится маленький шарик массы т с зарядом +q. Какой заряд Q нужно поместить в нижней точке сферы, чтобы шарик удерживался в верхней точке? Поляризацией сферы можно пренебречь. 12.78. Управляющие пластины в электронно-лучевой трубке образуют плоский конденсатор. Расстояние между пластинами d = 10 мм, длина пластин I = 50 мм. Электроны влетают в конденсатор посередине параллельно пластинам со скоростью v = 2,0 107 м/с. На пластины подают разность потенциалов U = 50 В. Какова форма траектории электронов внутри конденсатора? На какое расстояние h от первоначального направления сместятся электро- ны к моменту вылета из конденсатора? 12.79*. На какое расстояние Н от первоначального положения сместится светлая точка на экране электронно-лучевой трубки (см. задачу 12.78) после подачи напряжения на управляющие пластины? Расстояние от края конденсатора до экрана L = 20 см. 12.80. Электрон влетает со скоростью v0 в про- < « странство между пластинами плоского * конденсатора под углом а к плоскости пластин через отверстие в нижней плас- тине (см. рисунок). Расстояние между хП!”................ пластинами равно d, напряжение U. По ' + какой траектории будет двигаться электрон? Каково минимальное расстояние s между электроном и верхней пластиной? 12.81*. В плоский конденсатор длиной h = 50 мм влетает электрон под углом а = 15° к пластинам. Энергия электрона W = 1500 эВ, расстояние между пластинами d = 10 мм. При каком напряжений Ut на конденсаторе электрон вылетит параллельно пластинам
99 Электричество и магнетизм конденсатора? Каким будет ответ, если длину конденсатора уве- личить до 1г = 10 см? 12.82. Электрон влетает в плоский конденсатор параллельно его плас- тинам со скоростью Vo = 2,0*107 м/с. Напряженность поля в конденсаторе Е = 2,5 • 104 В/м, длина конденсатора Z = 80 мм. Определите величину v и направление скорости электрона в момент вылета из конденсатора. 12.83. Пучок электронов, разогнанных напряжением Ui = 5,0 кВ, влетает в плоский конденсатор посередине между пластинами и парал- лельно им. Длина конденсатора I = 10 см, расстояние между пластинами d = 10 мм. При каком наименьшем напряжении U2 на конденсаторе электроны не будут вылетать из него? 12.84**. Плоский воздушный конденсатор представляет собой две квадрат- ные металлические пластины размерами а х а, расположенные на расстоянии d друг от друга, причем d « а. Заряженный конденса- тор помещают в широкий сосуд с непроводящей жидкостью так, что пластины вертикальны и их нижние края находятся на уровне поверхности жидкости. Жидкость поднимается между пласти- нами и устанавливается на высоте h < а. Почему это происходит? Каким станет напряжение U между пластинами? Поверхностным натяжением можно пренебречь. Плотность жидкости р, диэлек- трическая проницаемость е. 13. ЗАКОНЫ ПОСТОЯННОГО ТОКА Закон Ома для участка цепи 13.1. Моток медной проволоки имеет массу т = 300 г и электрическое сопротивление R — 57 Ом. Определите длину проволоки I и площадь ее поперечного сечения S. 13.2. Плотностью тока j называют отношение силы тока в проводнике к площади сечения этого проводника: j = 4;. Выразите плотность О ^ока в проводнике через напряженность Е электрического поля в проводнике и удельное сопротивление р проводника. 4» ' - *
Задачи 100 13.3. Для нахождения сопротивления проводника Rx используют одну из двух схем (рис. а, б). Сопротивление проводника определяют по формуле Я» — U/I, где U — показание вольтметра, I — показание амперметра. Какая из схем дает меньшую погрешность при измерении больших сопротивлений? Малых сопротивлений? Выведите формулы, позволяющие с помощью этих схем измерить Rx как можно более точно, зная сопротивление амперметра ЛА и вольтметра Лу. 13.4. Для измерения сопротивления резистора Rx составлена схема, показанная на рис. б к задаче 13.3. Амперметр показал ток I = 2,0 А, вольтметр — напряжение 17 = 120 В. Определите Rx, если сопротивление вольтметра Ry = 3,0 кОм. К какой ошибке АЯХ приведет использование приближенной формулы Rx — U/I? 13.5* . Как с помощью вольтметра, микроамперметра и источника ЭДС с неизвестным внутренним сопротивлением измерить величину не- известного сопротивления Rx, сравнимого с сопротивлением воль- тметра Ry? 13.6* . Плоский конденсатор с квадратными пластинами 10 см х 10 см, находящи- мися на расстоянии d = 2,0 мм друг от друга, подключен к источнику постоян- ного напряжения 17 = 750 В. В прост- ранство между пластинами вдвигают (см. рисунок) стеклянную пластину тол- щиной 2,0 мм с постоянной скоростью v = 40 см/с. Какой ток i идет при этом по цепи? ! г
101 Электричество и магнетизм Последовательное и параллельное соединения. Вычисление сопротивления цепи 13.7. Из куска проволоки, имеющей сопротивление Ro = 32 Ом, сделано кольцо. В каких точках кольца следует подключить провода, чтобы получить сопротивление R = 6 Ом? 13.8. Какова максимально возможная величина сопротивления между двумя точками проволочного кольца (см. задачу 13.7)? 13.9. Имеются четыре одинаковых резистора с сопротивлением Ro = 6 Ом. Какие сопротивления R можно получить с их помощью? Начер- тите соответствующие схемы соединений. 13.10. Из одинаковых резисторов по 10 Ом требуется составить цепь сопротивлением 6 Ом. Какое наименьшее количество резисторов для этого потребуется? Начертите схему цепи. 13.11. Определите полное сопротивление показанной на рисунке цепи, если Ri = R2 = R& = Re = 3 Ом; R3 = 20 Ом; R4 = 24 Ом. Определите силу тока, идущего через каждый резистор, если к цепи прило- жено напряжение U = 36 В. К задаче 13.11 К задаче 13.12 13.12* . ♦Черный ящик» имеет три клеммы: 'А, В, С (см. рисунок). Известно, что он содержит только резисторы. Сопротивления «черного ящика» при подключении к различным парам клемм: Иав = 5 Ом, Яве = 8 Ом, Rac = 9 Ом. Предложите схему «черного ящика», содержащую минимально возможное число резисторов.
Задачи 102 18.13*. Обмотка реостата имеет сопротивление Яо. Для каждой из трех схем включения реостата (рис. а, б, в) постройте график зави- симости сопротивления цепи R от сопротивления г правой части реостата. Рис. в 13.14. Если на вход электрической цепи (см. рисунок) подано напряжение Ui = 100 В, то напряжение на вы- ходе U3 = 40 В; при этом через резистор Rz идет ток /2 = 1 А. Если на выход цепи подать напряжение U3 = 60 В, то напряжение на входе будет Ui' = 15 В. Определите величины сопротивлений Ri, R3, R3. 13.15*. Определите сопротивление R каждой из показанных на рис. а, б, в цепей. Сопротивление каждого из резисторов Ro; сопротивле- нием соединительных проводов можно пренебречь. 13.16*. Определите сопротивление R тетраэдра, изготовленного из шести одинаковых проволочек с сопротивлением Ro каждая. Тетраэдр включен в цепь двумя вершинами. 13.17*. Имеется л точек. Все точки соединены попарно резисторами с сопротивлением Ro. Каково сопротивление R между любыми дву- мя из этих точек? 13.18*. Определите сопротивление R между точками Аи В показанной на рисунке цепи. Сопротивление каждого из резисторов Ro.
103 Электричество и магнетизм К задаче 13.1S К задаче 13.15 13.19*. Определите сопротивление R проволочного куба (см. рисунок) при включении его в цепь точками Ai и С. Сопротивление каждого ребра Ro. 13.20*. Определите сопротивление R проволочного куба (см. задачу 13.19) при включении его в цепь точками Ai и D. 13.21*. Определите сопротивление R проволочного куба (см. задачу 13.19) при включении его в цепь точ- ками А и Ai. 13.22*. Определите сопротивление R цепи (см. рисунок) л<ежду точками А и В, если сопротивление каждого звена Ro.
Задачи 104 13.23*. Определите сопротивление R показанной на рисунке цепи, если сопротивление каждого звена Во. 13.24**. Определите сопротивление R бесконечной це- пи, показанной на рисунке. 13.25*. Сопротивление показанной на схеме (см. ри- сунок) цепи измеряется между точками А и В. Какое сопротивление Ях необходимо включить между точками С и D, чтобы сопротивление всей цепи не зависело от числа ячеек в ней? К задаче 13.23 13.26*. Цепь (см. задачу 13.25) содержит N ячеек (схема одной из ячеек показана на рисунке). Между точками С и D включено сопро- тивление Rx =С>/3 - 1)Я. Во сколько раз напряжение на выхрде цепи (между точками С и В) меньше напряжения на входе (между точками А и В)?. „ , .
105 Электричество и магнетизм К задаче 13.26 R, 13.27. К гальванометру, сопротивление, которого НТ = 330 Ом, присоединили шунт, понижа- ющий чувствительность гальванометра в п = 10 раз. Какое сопротивление R надо подключить теперь последовательно, чтобы общее сопротивление цепи не изменилось? 13.28. К миллиамперметру, рассчитанному на максимальный ток I = 100 мА, присое- диняют добавочное сопротивление, чтобы получить вольтметр, которым можно из- мерять напряжение до U = 220 В. Какой должна быть величина Яд этого сопротивления, если известно, что при шунтировании миллиамперметра сопротивлением Яш = 0,2 Ом цена его деления возрастает в п = 10 раз? 13.29. Гальванометр с шунтом соединен после- довательно с резистором, сопротивле- ние которого Яд (см. рисунок), и исполь- зуется как вольтметр. Как нужно из- менить Яд, чтобы увеличить цену дел- ения вольтметра в п раз? Сопротив- ление гальванометра Яг, шунта Яш- 13.30*. Какой заряд пройдет через ключ замыкания? к (см. рисунок) после его К задаче 13.32 13.31. Определите разность потенциалов между точками А и В (см. задачу 13.30) до замыкания ключа К. 13.32*. В цепи, представленной на рисунке, гальванометр показывает отсутствие тока. Выразите сопротивление Ях через Я1, Яг, Яз.
Задачи 106 13.33*. На рисунке показана схема мос- тика Уитстона для измерения сопротивлений. Здесь Во — эта- лонное сопротивление, Я, — не- известное сопротивление. Сколь- зящий контакт D, соединенный с гальванометром G, перемеща- ется по проводу АВ, имеющему большое сопротивление. Дока- жите, что ток через гальванометр не проходит, если выполнено Я /1 „ условие . . Сопротивлением соединительных проводов можно пренебречь. 13.34*. При сборке мостика Уитстона (см. задачу 13.33) ошиблись и поменяли местами гальванометр G и ключ К. Как можно измерить неизвестное сопротивление Ях с помощью такой схемы? 13.35*. Определите силу тока через каждый из резисторов (см. рисунок), если к цепи приложено напряжение U = 84 В. Сопротивления резисторов: К, = /?5 = Rg= 12 Ом; В2 = R» = Ri = 6 Ом; Я4 = 24 Ом; .В, = 3 Ом. К задаче 13.35 13.36*. В цепи (см. рисунок) сопротивления резисторов подобраны так, что токи через проводники AiA2 и BiB2 не идут. Возникнут ли токи в этих участках цепи, если соединить проводником точки А3 и Вя? Ёак изменятся при этом потенциалы точек Ai, А2, Bii~B2?
107 Электричество и магнетизм 13.37*. Какой Фок идет через амперметр (см. рисунок), если Ri = Я4 = R, а Я2 = Яз = ЗЯ? К цепи приложено напряжение U. Сопротивление амперметра можно считать пренебрежимо малым. К задаче 13.37 К задаче 13.3S 13.38. Какую силу тока I покажет амперметр в изображенной на рисунке цепи? Сопротивления резисторов: Ri — 6 Ом; R2 = 8 Ом; Яз = 12 Ом) Я4 = 24 Ом. ЭДС источника Я = 36 В, его внутреннее сопротивление г = 1 Ом. 13.39. Найдите силу тока I через источник и напряжение U на источнике (см. рисунок), если его ЭДС й = 15 В, а внутреннее сопротивление г = 4 Ом. Сопротивления всех резисторов- одинаковы: Я = 68 Ом. 13.40*. Определите силу тока /1 че- рез резистор с сопротивле- нием Я1 (см. рисунок). Соп- ротивления резисторов: Я1 = 5 Ом; Яз = 7 Ом; Я = 2 Ом. ЭДС источника 6 = 30 В, его внутреннее сопротивление г = 2 Ом. 13.41*. Определите силу тока 1л через амперметр (см. рисунок), если сопротивления резисторов: Я1 = 20 Ом; R2 = Я4 — 8 Ом; Яз = 1 Ом. ЭДС источника & = 50 В, его внутреннее сопротивление г = 1 Ом. Сопрот^ле^ие амперметра можно считать пренебрежимо малым.
Задачи 108 К задаче 13.40 13.42. Батарея аккумуляторов замкнута на лампу. При этом напряжение на зажимах батареи Ui = 20 В. При параллельном подключении еще одной такой же лампы напряжение падает до U2 = 15 В. Определите сопротивление R каждой лампы. Считайте, что соп- ротивление лампы не зависит от ее накала. Внутреннее сопро- тивление батареи г = 1 Ом. 13.43*. Два вольтметра, подключенные последовательно к ненагруженной батарее, показывают соответственно C7i = 5 В и U2 = 15 В. Если подключить к батарее только первый вольтметр, он покажет £71' = 19 В. Определите ЭДС батареи. 13.44*. Определите показание ампермет- ра в схеме (см. рисунок), если 8 = 15 В, Я1 = 4,2 Ом; Я2 = 8 Ом; Яз = 12 Ом. Каким станет это показание, если поменять мес- тами амперметр и источник ЭДС? Внутренние сопротивления ис- точника и амперметра малы по сравнению с сопротивлениями резисторов. 13.45. К аккумулятору с внутренним сопротивлением г = 0,01 Ом под- ключен резистор с сопротивлением Я = 10 Ом. Вольтметр дает одинаковые показания при последовательном и параллельном под- ключении к резистору. Определите сопротивление вольтметра Яу.
109 Электричество и магнетизм 13.46*. Два источника ЭДС соединены, как показано на рисунке. Опре- делите разность потенциалов между точками А и В. Какой станет разность потенциалов, если изменить полярность включения вто- рого источника? К задаче 13.46 К задаче 13.47 13.47*. В цепи, показанной на рисунке, ЭДС каждого элемента 6, внут- реннее сопротивление г. Какова разность потенциалов между точками Ai и А2? Между точками Ai и А*? Сопротивлением соединительных проводов можно пренебречь. 13.48*. Как изменится ответ в задаче 13.47, если все элементы будут обращены друг к другу одноимен- ными полюсами (разумеется, пол- ное Число элементов N должно быть четным)? 13.49**. N одинаковых элементов соеди- нены в батарею. Внутреннее сопро- тивление каждого элемента г. При каких значениях т и п (см. ри- сунок) сила тока через резистор с сопротивлением R, подключенный к батарее, будет наибольшей? Ре- шите задачу при N = 100, г = 1 Ом, R = 2 Ом.
Задачи 110 13.50*. В цепи с внешним сопротивлением R = 2 Ом необходимо обес- печить силу тока I — 2 А. Какое наименьшее число N элементов потребуется для этого, и как они должны быть соединены в батарею? ЭДС каждого элемента 5 = 2 В, внутреннее сопро- тивление г = 1 Ом. 13.51*. В конце зарядки аккумулятора сила тока Л = 3,0 А, а напряжение на клеммах ГЛ = 8,85 В. В начале разрядки того же аккумулятора сила тока 12 = 4,0 А, а напряжение L72 = 8,5 В. Определите ЭДС & и внутреннее сопротивление г аккумулятора. 13.52*. Генератор с ЭДС = 24 В и внутренним сопротивлением и = 0,10 Ом заряжает батарею аккумуляторов с ЭДС <5’2 = 20 В и внутренним сопротивлением г2 = 0,30 Ом. Параллельно батарее включена лампа с сопротивлением R = 1,5 Ом. Какие токи h, Г2,13 протекают через генератор, батарею и лампу? 13.53. Имеются два последовательно соединенных элемента с ЭДС и $2 и внутренними сопротивлениями п и г2. При каком внешнем сопротивлении R напряжение на зажимах одного из элементов равно нулю? На зажимах какого именно элемента это возможно? 13.54*. В цепи (см. рисунок) гальванометр показывает отсутствие тока. Эталонный элемент имеет ЭДС 60 = 1,5 В, внутреннее сопротивление г0 = 1,5 Ом. Сопротивления резисторов: Ri = 4 Ом; R2 = 4,5 Ом. Определите ЭДС <5 аккумулятора. К задаче 13.54 К задаче 13.56 13.55. Реостат подключен к источнику тока. При изменении сопротивления реостата от Ri = 4,0 Ом до R2 = 9,5 Ом сила тока в
Ill Электричество и магнетизм цепи изменяется от /1 = 8,0 А до 12 = 3,6 А. Определите ЭДС <5 источника тока и его внутреннее сопротивление г. 13.66. Резистор с сопротивлением R подключают к клеммам А и В (см. рисунок). Определите силу тока I через этот резистор. 13.57**. Если к точкам А и В показанной па рисунке цепи подключить идеаль- ный вольтметр (имеющий бесконеч- но большое сопротивление), он по- кажет напряжение Uo. Если к тем же точкам подключить идеальный ампер- метр (имеющий нулевое сопротивле- ние), он покажет силу тока /о. Опре- делите силу тока I, текущего по включенному между точками А и В резистору с сопротивлением R. Цепи с резисторами и конденсаторами 13.58. Определите напряжения Ui и U2 на конденсаторах с емкостями С\ и С2 (см. рисунок). К задаче 13.58 13.59. Определите напряжения Ui, U2, U3 на каждом из конденсаторов (см. рисунок). 13.60. Определите ЭДС источника (см. рисунок), если заряд конденсатора
Задачи 112 q = 1,08 мКл. Сопротивления резисторов: Ri - 90 Ом; R2 = 30 Ом; Ra = 60 Ом; = 40 Ом. Внутреннее сопротивление источника г = 1 Ом. Емкость конденсатора С = 5 мкФ. 13.61*. Какой заряд q пройдет через резистор R2 после размыкания ключа К (см. рисунок)? Сопротивления резисторов одинаковы: Ri = R2 = — Ra — R4 ~ R. 13.62*. Определите заряд q на конденса- торе емкости С (см. рисунок). Внутреннее сопротивление источ- ника пренебрежимо мало. Работа и мощность тока 13.63*. В комнате горит электрическая лампа мощностью Pi — 100 Вт, подключенная к сети с напряжением U = 220 В. Сопротивление проводов, подводящих к квартире электроэнергию, составляет Во = 4 Ом. Как изменится напряжение на лампе, если включить1 электрокамин мощностью Р2 = 500 Вт?
113 Электричество и магнетизм 13.64 . Электрический нагреватель питается от N одинаковых аккумуля- торов, каждый из которых имеет внутреннее сопротивление г. Нагреватель потребляет одинаковую мощность при последователь- ном и параллельном соединения аккумуляторов. Определите сопротивление нагревателя Я. 13.65 . Аккумуляторная батарея имеет ЭДС & = 12 В и внутреннее сопротивление г = 0,10 Ом. Сколько лампочек мощностью Ро ~ 25 Вт каждая, рассчитанных на напряжение U = 10 В, можно под- ключить к этому источнику ЭДС, чтобы они горели нормальным накалом? 13.66 . Определите напряжение U на полюсах источника ЭДС (см. рисунок) и общую мощность Р, потребляемую резисторами. Какие токи h и Iz текут через резисторы? Сопротивления резисторов Ri = 8 Ом, Rz = 24 Ом; ЭДС источника <5 — 40 В, его внутреннее сопротивление г = 2 Ом. 13.67 . На участке АВ в цепи (см. рисунок) выделяется одинаковая мощность при разомкнурэм и замкнутом ключе. Определите сопротивление Вх, если Ro = 20 Ом. Напряжение U считайте неизменным. 13.68 . К источнику ЭДС подключаются поочередно резисторы с сопро- тивлениями Ri n Rz. В обоих случаях на резисторах выделяется одинаковая мощность. Определите внутреннее сопротивление г источника.
Задачи 114 13.69 . Имеются два резистора с сопротивлениями Ri = 2,0 Ом ийз = 4,5 Ом. Их подключают к источнику тока сначала параллельно, а потом последовательно. При какой величине внутреннего сопротивления г источника тока в обоих случаях во внешней цепи выделяется одинаковая мощность? 13.70 *. При поочередном подключении к источнику ЭДС двух электри- ческих нагревателей с сопротивлениями Ri = 3 Ом и R2 = 48 Ом в них выделяется одинаковая мощность Р = 1,2 кВт. Определите силу тока 1КЛ при коротком замыкании источника. 13.71 . На одной лампочке написано «220 В, 60 Вт»; на другой «220 В, 40 Вт». Лампочки соединяют последовательно й включают в сеть с напряжением 220 В. Определите полную потребляемую мощность и мощность каждой из лампочек при таком включении. Считайте сопротивление ламп не зависящим от температуры. 13.72 . Имеется пять электрических лампочек, рассчитанных на напря- жение 9 В каждая. Три из них имеют расчетную мощность по 4 Вт, две — по 6 Вт. Как следует включить их в сеть с напряжением 18 В, чтобы все они горели нормальным накалом? 13.73*. Определите ток короткого замыкания /к.< аккумуляторной батареи, если при токе h = 5 А нагрузка потребляет мощность Pi = 30 Вт, а при токе ?2 = Ю А — мощность Р2 = 40 Вт. 13.74. Электрический чайник имеет две обмотки. Йри включении одной из них вода закипает через ti = 12 мин, при включении другой — через t2 = 24 мин. Через сколько времени закипит вода в чайнике, если включить обе обмотки параллельно? Последовательно? Теп- лообменом с воздухом пренебречь. 13.75*. Поселок, потребляющий электрическую мощность Р = 1200 кВт, находится на расстоянии I = 5 км от электростанции. Передача электроэнергии производится при напряжении U = 60 кВ. До- пустимая относительная потеря напряжения (и мощности) в про- водах k = 1%. Какой минимальный диаметр d могут иметь медные провода линии электропередачи? 13.76. Во сколько раз следует повысить напряжение U на линии элект- ропередачи, чтобы при передаче той же мощности потери в линии
115 Электричество и магнетизм уменьшились в 400 раз? Потери во всех случаях считать малыми по сравнению с передаваемой мощностью. 13.77*. Электропоезд метро идет по горизонтальному пути со скоростью а затем со скоростью v2 преодолевает подъем с уклоном k — 0,04. Потребляемая сила тока на горизонтальном участке h = 240 А, а на подъеме 12 = 450 А. Коэффициент сопротивления движению ц = 0,02. Определите отношение скоростей щ/ог. 13.78. Какую силу тока I надо пропустить через железную проволоку диаметром D = 0,5 мм, чтобы через т = 1 с проволока начала плавиться? Начальная температура проволоки to = 0°С; теплопе- редачу в окружающую среду и зависимость сопротивления от температуры не учитывать. 13.79*. Три тонких проволоки одинакового диаметра — железная, медная и алюминиевая — соединены последовательно. Их подключают к источнику высокого напряжения, и одна иэ проволок перегорает. Какая? Начальная температура to = 0°С. 13.80*. Три тонких проволоки одинаковых размеров — железная, медная и алюминиевая — соединены параллельно. Какая из них пере- горит первой после подключения к источнику высокого напря- жения? Начальная температура to = 0°С. 13.81*. Предохранитель изготовлен из свинцовой проволоки сечением Si — 0,2 мм2. При коротком замыкании сила тока достигла величины I = 20 А. Через какое время т после короткого замыкания начнет плавиться предохранитель? На сколько за это время нагреются подводящие медные провода сечением S2 = 2,0 мм2? Начальная температура предохранителя to — 27°С. Считайте, что сопротивление свинцовой проволоки не зависит от температуры. 13.82*. Источник тока с ЭДС 3 и внутренним сопротивлением г замкнут на реостат. Постройте графики зависимости от сопротивления R реостата следующих величин: силЬг тока I, напряжения U, мощ- ности Р во внешней цепи, полной мощности Ро и КПД цепи ц. При каком R достигается максимальная мощность во внешней цепи? Каков при -этом КПД цепи?
Задачи 116 13.83**. Оцените время т разрядки конденсатора (см. рисунок) после замыкания ключа. Емкость конденсатора С, сопротивление цепи R. Какой вид имеет график зависимости напряжения U на кон- денсаторе от времени? 13.84**. Сопротивление реостата (см. рисунок) R = 16 Ом. ЭДС источника тока <э = 12 В, его внутреннее сопротивление г = 2 Ом. Выразите I через отношение х = -у следующие величины: силу тока I через источник; напряжение U на полюсах источника тока; мощность Р, выделяющуюся в реостате. Постройте соответствующие графики. 14. ЭЛЕКТРИЧЕСКИЙ ТОК В РАЗЛИЧНЫХ СРЕДАХ 14.1. По медному проводнику течет ток. Плотность тока j = 6 А/мм2. Определите среднюю скорость v упорядоченного движения элект- ронов. Можно считать, что на каждый атом меди приходится один свободный электрон. 14.2* . На цоколе электрической лампы накаливания с вольфрамовой нитью написано: 120 В, 500 Вт. Если пропускать через эту лампу ток h = 8-мА, то падение напряжения на ней составляет Ui = 20 мВ; при этом нить накала практически не нагревается (температуру ti = 20°С). Какова температура t нити накала в рабочем состо- янии?
117 Электричество и магнетизм 14.3. Подключенная к сети спираль электроплитки раскалилась. Как изменится накал, если на часть спирали попадет вода? 14.4. Угольный стержень соединен последовательно с железным такого же сечения. При каком отношении длин стержней сопротивление системы не будет зависеть от температуры? 14.5. Почему при включении в квартире мощного электронагревателя горящие лампочки заметно меркнут, но затем их яркость опять возрастает (хотя и не достигает первоначальной)? 14.6* . Какую длину I имеет вольфрамовая нить накала лампочки, рассчитан* ной на напряжение U — 220 В и мощность Р = 200 Вт? Температура накаленной нити Т = 2700 К, диаметр нити d = 0,03 мм. Считайте, что удельное сопротивление вольфрама прямо пропорционально абсолютной температуре. 14.7. Какое количество теплоты q выделяется в единице объема про- водника за единицу времени при плотности тока у? Удельное сопротивление проводника равно р. 14.8* . Вольфрамовая нить диаметром di = 0,10 мм и длиной I = 1,0 м натянута в вакууме. К концам нити подводят напряжение и медленно его повышают. При каком напряжении tA нить пере- горит? Считайте, что сопротивление вольфрама прямо пропор- ционально абсолютной температуре Т. Мощность теплового излу- чения с единицы площади поверхности нити можно считать равной аТ* (здесь о = 5,67 НТ® Вт/(м2*К4) — постоянная Стефа- на-Больцмана). Каким будет ответ, если диаметр нити da = 1,6 мм? 14.9. Никелирование металлической пластинки с площадью поверх- ности S = 48 см2 продолжалось t = 4,0 ч при силе тока I = 0,15 А. Определите толщину Л слоя никеля. Валентность никеля п = 2. 14.10. Каков расход электроэнергии W на получение m = 1,0 кг алю- миния, если электролиз ведется при напряжении U = 10 В, а КПД установки т] = 0,80? 14.11. При электролизе подкисленной воды через ванну прошел заряд Q • 2500 Кл. Выделившийся кислород находится в объеме V = 0,50 л под давлением р = 101 кПа. Какова его абсолютная температура Т?
Задачи 118 14.12. Две одинаковые электролитические ванны заполнены раствором медного купороса; в первой ванне концентрация раствора выше. Сравните количество выделившейся на их катодах меди, если ванны соединены: а) последовательно; б) параллельно. 14.13. Две одинаковые электролитические ванны соединены между со- бой последовательно. В одной из них — раствор CuCl, в другой — СиС12. В какой из ванн на катоде выделится больше меди? 14.14* . При нанесении металлических покрытий с помощью электролиза иногда в конце процесса изменяют направление тока на противо- положное. В результате поверхность становится более гладкой. Почему? 14.15* . Конденсатор емкостью С = 10 пФ соединен последовательно с резистором, имеющим сопротивление R = 1,0 кОм. Расстояние между пластинами конденсатора d = 3,0. мм. Воздух между пластинами конденсатора ионизируется рентгеновским излуче- нием: каждую секунду в 1 см3 воздуха образуется п = 5 104 пар ионов с зарядом е, равным элементарному. Определите силу тока I в цепи и падение напряжения U на резисторе, если к цепи подключен источник высокого напряжения. 14.16. Что произойдет с горящей электрической дугой, если сильно охладить отрицательный электрод? Положительный? 14.17. Какой наименьшей скоростью v должен обладать электрон, чтобы он мог ионизировать неподвижный атом неона? Потенциал иони- зации атомов неона (р = 21,5 В. 14.18*. Какой наименьшей кинетической энергией W\ и скоростью v должен обладать ион неона, чтобы при столкновении его с не- подвижным нейтральным атомом неона могла произойти иони- зация атома? Потенциал ионизации атомов неона <р = 21,5 В. 14.19 При какой абсолютной температуре Т средняя кинетическая энер- гия движения атомов неона равна энергии, необходимой для иони- зации этих атомов? Потенциал ионизации атомов неона <р = 21,5 В. 14.20**. Электрическое поле действует на свободные электроны в газе и на однозарядные ионы с одинаковой по величине силой-^Кдзалось бы,
119 Электричество и магнетизм вследствие этого электроны и ионы должны одинаково часто приобретать кинетические энергии, достаточные для ударной иони- зации молекул газа. Однако в действительности главную роль в этом процессе играют электроны. Почему? 14.21*. Легирование германия акцепторной примесью (например, ин- дием) намного увеличивает концентрацию дырок. Как при этом изменяется концентрация свободных электронов? 14.22*. Один из концов полупроводника нагревают. Почему при этом возникает разность потенциалов между нагретым и холодным концами? Потенциал какого из них выше? 15. МАГНИТНОЕ ПОЛЕ. СИЛА ЛОРЕНЦА 15.1. По двум одинаковым круглым металлическим обручам идут оди- наковые токи. Один из обручей расположен вертикально, другой — горизонтально (см. рисунок). Определите направление вектора магнитной индукции В в общем центре обручей. 15.2. К двум точкам проволочного кольца подведен ток (см. рисунок). Протекающие по кольцу токи создают магнитное поле. Куда направлен вектор магнитной индукции В этого поля в центре кольца? 15.3. Из одинаковых кусков проволоки спаян куб. Источник тока, подключение противоположным (лежащим на одной пространст-
Задачи 120 венной диагонали) вершинам куба. Докажите, что магнитное поле в центре куба отсутствует. 15.4. Объясните поведение прямоугольной проволочной рамки с током в магнитном поле. Имеет ли значение, чем создается поле — Землей или стальным магнитом? 15.5. Мягкая спиральная пружина висит, погрузившись нижним кон- цом на небольшую глубину в ртуть (см. рисунок). Что произойдет после замыкания ключа? К задаче 15.6 15.6. Между полюсами электромагнита в горизонтальном однородном магнитном поле с индукцией В = 0,05 Тл находится прямолиней- ный проводник массой т — 5,0 г и длиной I = 50 см, подвешенный горизонтально на гибких проводах под прямым углом к маг- нитному полю (см. рисунок). Через проводник пропускают ток. При какой силе тока I исчезает натяжение проводов, поддержи- вающих проводник? В какую сторону должен при этом идти ток? 15.7. Горизонтальные рельсы находятся в вертикальном однородном магнитном поле на расстоянии I = 15 см друг от друга. На них лежит стальной стержень массой т = 300 г, перпендикулярный рельсам. Коэффициент трения между стержнем и рельсами ц = 0,20. Чтобы стержень сдвинулся с места, по нему необходимо пропус- тить ток силой I = 40 А. Какова индукция В магнитного поля? 15.8*. Проволока лежит в плоскости, перпендикулярной однородному магнитному полю. По проволоке идет ток. Докажите, что вели- чина действующей на проволоку силы Ампера определяется толь- ко силой тока и расстоянием между концами проволоки, но не зависит от ее формы и длины.
121 Электричество и магнетизм 15.9* . По жесткому кольцу из медной проволоки течет ток силой I = 5,0 А. Кольцо находится в перпендикулярном к его плоскости маг- нитном поле с индукцией В = 0,50 Тл. Сила Ампера стремится растянуть кольцо. Определите механическое напряжение ст в проволоке, если радиус кольца R = 5,0 см, а площадь сечения проволоки S = 3,0 мм2. 15.10. В вертикальном однородном магнитном поле с индукцией В = 0,50 Тл подвешен на двух тонких проволочках горизонтальный проводник массой т = 30 г и длиной I = 49 см. По проводнику пропускают ток силой I = 1,2 А. На какой угол а от вертикали отклонятся проволочки? 15.11. Почему два параллельных проводника, по которым текут токи в одном направлении, притягиваются, а два параллельных элект- ронных пучка отталкиваются? 15.12. Электрон влетает со скоростью v в однородное магнитное поле с индукцией В. Скорость электрона направлена перпендикулярно вектору В. По какой траектории будет двигаться электрон? 15.13. Как зависит период Т обращения электрона По окружности (см. задачу 15.12) от скорости электрона? 15.14* . Электрон влетает в однородное магнитное поле под углом а к направлению поля. По какой траектории будет двигаться элект- рон, если его скорость и, а магнитная индукция поля В? 15.15* . Опишите движение электрона в однородных параллельных элект- рическом и магнитном полях. Начальная скорость электрона v направлена под углом а к векторам Е и В. 15.16. Объясните действие «фильтра ско- ростей», показанного на рисунке. Внутри прибора созданы однород- ные поля: магнитное с индукцией В и электрическое с напряжен- ностью Е. Поля направлены пер- пендикулярно друг к другу и к начальной скорости частиц.
Задачи 122 15.17. Около сильного длинного полосового магнита расположен гибкий свободный провод (см. рисунок). Как расположится провод, если по нему пропустить ток? К задаче 15.18 К задаче 15.17 15.18. По двум жестким незакрепленным прямолинейным проводникам, скрещивающимся под прямым углом, проходят токи h и 1г (см. рисунок). Как будет меняться взаимное расположение провод- ников вследствие магнитного взаимодействия между ними? 15.19. Кювета с раствором медного купороса находится между полюсами сильного электромагнита (см. рисунок). Медный электрод в цен- тре кюветы соединен с положительным полюсом источника тока, а медное кольцо, лежащее в жидкости по периметру кюветы, — с отрицательным. Что произойдет при замыкании цепи? 15.20. Горизонтальное сверхпроводящее кольцо, по которому течет ток силой I = 2,0 А, «парит» в неоднородном магнитном поле (см. рисунок). Вектор магнитной индукции в точках, где находится
123 Электричество и магнетизм кольцо, образует угол а = 30° с осью кольца и равен по модулю В = 0,10 Тл. Определите массу т кольца, если его радиус В = 5,0 см. 15.21*. По металлической ленте течет ток силой I. Лента помещена в однород- ное магнитное поле с индукцией В (см. рисунок). При этом между точ- ками А и С возникает разность по- тенциалов (эффект Холла). Объясни- те это явление. Определите разность потенциалов ПАс, если АС = a, AD = = 5; концентрация свободных элект- ронов равна п. 15.22*. Сплошной металлический цилиндр радиуса R = 20 стл вращается с постоянной угловой скоростью а = 103 рад/с. Чему равна напряженность Е электрического поля внутри цилиндра на рас- стоянии г от оси? Какова разность потенциалов U между поверх- ностью цилиндра и осью вращения? Какова должна быть ин- дукция В магнитного поля, направленного вдоль оси цилиндра, чтобы электрическое поле не возникало? 15.23*. Пучок однозарядных ионов проходит «фильтр скоростей» (см. задачу 15.16), в котором Е — 500 В/м и В = 0,10Тл, и попадает затем в область однородного магнитного поля с индукцией Bi = 60 мТл. Ионы движутся под прямым углом к направлению вектора Bi. На каком расстоянии х друг от друга окажутся ионы двух различных изотопов неона с относительной атомной массой 20 и 22, пройдя половину окружности? 15.24**. Магнитное поле (см. рисунок) симметрично относительно оси г, причем проекция вектора магнитной индукции В на ось z z составляет В, = Во(1 + —). Опре- Ло делите угол а между вектором В и осью z в точке А, лежащей на расстоянии R от оси z и на рас- стоянии h от плоскости хОу. х
Задачи 124 16. ЭЛЕКТРОМАГНИТНАЯ ИНДУКЦИЯ ie.i. Какие явления происходят в кольце, когда в него вдвигают магнит? Рассмотрите случаи, когда кольцо сделано из: а) диэлект- рика, б) проводника. 16.2. Определите направление индукционного тока в металлическом кольце, к которому приближают южный полюс магнита (см. рисунок). К задаче 16.2 К задаче 16.3 16.3. Прямоугольная проволочная рамка равномерно вращается вокруг неподвижной оси. Параллельно этой оси расположен провод, по которому течет ток I (см. рисунок). Обозначим плоскость, в которой лежат провод и ось вращения рамки, буквой а. При каких положениях рамки в ней возникает наименьшая ЭДС индукции? Наибольшая? 16.4. Каркас для глобуса сделан из двух тонких изолированных друг от друга стальных обручей, расположенных во взаимно перпендикулярных f / )/i\ 1 плоскостях (см. рисунок). В одном из них течет I 1)1 изменяющийся электрический ток. Возникает ли во втором обруче ЭДС индукции? 16.5* . у Магнит падает в длинной вертикальной медной трубе, воздух из которой откачан. Опишите ха- рактер падения. Магнит с трубой не соприкаса- к задаче 16.4 ется.
125 Электричество и магнетизм 16.6. Маятник, состоящий из металлических нити, шарика и острия, погруженного в ртуть (см. рисунок), совершает малые колебания в однородном магнитном поле. Как изменится характер колебаний после замыкания ключа К? К задаче 16.6 К задаче 16.11 16.7* . Плоская проволочная рамка может свободно вращаться вокруг оси, перпендикулярной магнитной индукции однородного маг- нитного поля. Укажите положение устойчивого равновесия рам- ки, если магнитное поле убывает; если магнитное поле возрастает. 16.8*. Свинцовое кольцо радиуса г расположено перпендикулярно ли- ниям индукции однородного магнитного поля. Магнитная ин- дукция поля равна В. Охлаждая кольцо, его переводят в сверх- проводящее состояние, а затем магнитное поле выключают. Какой магнитный поток Ф будет пронизывать после этого плоскость кольца? 16.9. Какие явления возникают в сверхпроводящем кольце, когда в него вдвигают магнит? 16.10*. Сверхпроводящая катушка радиуса г состоит из N витков и имеет индуктивность L. Определите силу тока I, возникающего в катуш- ке при включении внешнего однородного магнитного поля с индукцией В, направленной вдоль оси катушки. 16.11*. По сверхпроводящему проводу, имеющему форму кольца радиуса г, идет ток. Индукция магнитного поля в центре кольца равна Во. Проводу придают другую форму (см. рисунок). Какова теперь индукция В магнитного поля в центре кольца?
Задачи 126 16.12*. Сверхпроводящее изолированное кольцо, по которому идет ток, деформируют, превращая его в «восьмерку» из двух одинаковых колец (см. рисунок). Затем «восьмерку» складывают вдвое так, что получается меньшее кольцо. Как изменится индукция маг- нитного поля в центре кольца? К задаче 16.12 16.13**. Как изменится ответ в задаче 16.12, если перед складыванием «восьмерки» не перекручивать ее (см. рисунок)? 16.14*. Катушку радиуса г = 3,0 см с числом витков п = 1000 помещают в однородное магнитное поле (ось катушки направлена вдоль линий поля). Индукция поля изменяется с постоянной скоростью дд — = 10 мТл/с. Какой заряд q будет на конденсаторе, подключен- М ном к концам катушки? Емкость конденсатора С — 20 мкФ. 16.15*. Катушка (см. задачу 16.14) замыкается накоротко. Определите выделяющуюся в ней тепловую мощность Р, если сопротивление катушки R = 16 Ом. 16.16*. В замкнутую накоротко катушку вводят магнит: один раз быстро, а другой — медленно. Одинаковый ли заряд проходит по цепи в обоих случаях? Одинаковое ли количество теплоты выделяется? 16.17*. Катушка радиуса г с числом витков ч и сопротивлением R на- ходится в однородном магнитном поле с индукцией В. Ось катушки направлена вдоль линий поля. Концы катушки замкнуты. Какой заряд q пройдет через катушку, если повернуть ее ось на угол а?
127 Электричество и магнетизм 16.18*. Кусок провода длиной I = 4,0 м складывают вдвое и его концы замыкают. Затем провод растягивают по периметру горизонталь- ной площадки. Какой максимальный заряд graax может при этом пройти через провод, если его сопротивление R = 2,0 Ом? Вер- тикальная составляющая магнитного поля Земли Вг = 50 мкТл. 16.19. Металлическое кольцо радиуса I имеет пренебрежимо малое сопротивление. Кольцо находится в однородном магнитном поле с индукцией В, перпендикулярной плоскости кольца. Две ме- таллические «стрелки» сопротивлением R каждая имеют контакт между собой и с кольцом (см. рисунок). Одна из них неподвижна, другая равномерно вращается с угловой скоростью со. Определите силу тока I через стержни. К задаче 16.19 К задаче 16.20 16.20*. Металлический стержень может скользить без трения по парал- лельным длинным «рельсам», находящимся на расстоянии I друг от друга и соединенным перемычкой с электрическим сопро- тивлением R (см. рисунок). Система находится в перпендикуляр- ном ее плоскости однородном магнитном поле с индукцией В. Как будет двигаться стержень, если к нему приложить постоянную силу F? Электрическим сопротивлением стержня и «рельсов» можно пренеберечь. 16.21**. Как изменится ответ в ххххххххххх задаче 16.20, если в X ‘ х г х X X X X X К X цепь включить источ- X X г X X X X X // К gK ник тока и идеальный L ri. . г диод, как показано на X х fr X X X X X К X рисунке? X, , X 3 X X X X X 6 К X X хххххххххх
Задачи 128 16.22*. На цилиндрический железный сердечник радиуса г надето изолированное проволочное кольцо того же радиуса, имеющее электрическое сопротивление А. В сердечнике создано однородное магнитное поле, индукция которого меняется по закону В = aBot. Как меняется со временем сила тока i в кольце? Разность потенциалов между диаметрально противоположными точками кольца? 16.23**. Виток провода изогнут в виде восьмерки (см. рисунок), так что = 20 мм и г2 = 60 мм. В течение времени At = 0,50 мс однородное маг- нитное поле, перпендикулярное плоскости витка, равномерно возрастает. Начальное значение индукции магнитного поля равно нулю, конечное В = 50 Тл. На какое напряжение U должна быть рассчитана изоляция между проводами, чтобы не произошел пробой? 16.24**. Катушка с индуктивностью L и электрическим сопротивлением R подключена через ключ к источнику тока с ЭДС <5. В момент t = 0 ключ замыкают. Как изменяется со временем сила тока I в цепи сразу же после замыкания ключа? Через длительное время после замыкания? Оцените характерное время т возрастания тока в такой цепи. Внутренним сопротивлением источника тока можно пренебречь. 16.25**. Через катушку (см. задачу 16.24) течет постоянный ток. В момент to источник тока отключают и катушку замыкают накоротко (см. рисунок). Какой вид имеет график зави- симости силы тока от времени? Ка- ково характерное время т убывания тока в цепи? 16.26*. На замкнутый ферромагнитный сердечник намотана катушка. Как зависит ее индуктивность L от числа витков N? Магнитную проницаемость сердечника считать неизменной. 16.27*. Два одинаковых сверхпроводящих кольца могут перемещаться поступате- льно вдоль одной прямой (см. рису- нок). Плоскости колец параллельны ДРУГ другу и перпендикулярны этой
129 Электричество и магнетизм прямой. В начальный момент расстояние между кольцами намно- го превышает их размеры; по кольцам текут в одном направлении токи Jo. Какие токи I установятся в кольцах после того, как они сблизятся вплотную? Как при этом изменится энергия магнитного поля? 16.28**. Опишите движение сверхпроводящих колец (см. задачу 16.27), если в начальный момент токи в кольцах различны по величине. 16.29*. В цилиндрическом сердечнике радиуса Я создано однородное магнитное поле, направленное вдоль оси цилиндра. Индукция магнитного поля изменяется со временем по закону В = kt. Определите напряженность Е вихревого электрического поля на расстоянии г от оси цилиндра. 16.30**. Индукция однородного магнитного поля в цилиндрическом сердечнике радиуса г (см. рисунок) возрастает со временем по закону В = kt. Проволоч- ное кольцо радиуса 2г имеет общую с сердечником ось. Какова разность по- тенциалов между точками А и В? Ка- кое напряжение покажет подключен- ный между точками А и В вольтметр? Сопротивление вольтметра велико по сравнению с сопротивлением кольца. 16.31**. Половина проволочного кольца изготовлена из меди, а другая половина — из латуни. Сечение проволоки всюду одинаково, радиус кольца г = 30 мм. Кольцо надето на цилиндрический сердечник того же радиуса. В сердечнике создано однородное магнитное поле, индукция которого возрастает с постоянной ско- д д ростью — = 500 Тл/с. Определите напряженность электрического поля в различных частях кольца. 16.32**. В проволочном кольце (см. задачу 16.30) разность потенциалов между точками А и В равна нулю. Значит ли это, что в проводнике, включенном между точками А и В, не возникнет электрический ток? Может ли включение такого проводника изменить разность потенциалов между точками А и В? 16.33**. •Определите разность потенциалов U между двумя точками со- единения медного и латунного полуколец (см. задачу 16.31).
Задачи 130 16.34**. Между точками А и В (см. задачу 16.30) включен сверхпроводник АСВ (см. рисунок). Определите силу тока на различных участках цепи и разность потенциалов U между точками А и В. Со- противление проволоки, из которой сделано кольцо, равно R. 16.35. При торможении поезда метро электродвигатели постоянного тока отключают от сети и подключают к специальным реостатам. Зачем это делают? 16.36*. Металлический диск радиуса г = 10 см расположен в однородном магнитном поле, перпендикулярном плоскости диска. Диск вра- щается с угловой скоростью со = 500 рад/с. Два скользящих контакта (один на оси диска, другой на его окружности) соединены с резистором сопротивлением R = 10 Ом. Какая мощность Р выде- ляется в резисторе, если индукция магнитного поля В = 0,50 Тл? 16.37**. Сверхпроводящее кольцо радиуса I помещено в однородное горизонтальное магнитное поле с индукццей В. Ось кольца парал- лельна линиям магнитной индукции поля (см. рисунок). Стержень массой т и длиной I, имеющий сопротивление R, закреплен одним концом в центре кольца и может без трения поворачиваться вокруг этой точки, сохраняя электрический контакт с кольцом. По какому закону должно изменяться напряжение U, приложен- ное между кольцом и его центром, чтобы стержень вращался с постоянной угловой скоростью <а? 16.38*. Сопротивление в цепи нагрузки генератора переменного тока уве- личилось. Как должна измениться мощность двигателя, вращаю- щего генератор, чтобы частота переменного тока не Изменилась?
131 Электричество и магнетизм 16.39*. В каком случае обмотка электромотора сильнее нагревается про- ходящим по ней током — когда мотор работает вхолостую или совершает некоторую работу? Напряжение в сети считать посто- янным. 16.40*. Электродвигатель включен в сеть постоянного тока напряжением U ~ 120 В. Сопротивление обмотки двигателя R — 12 Ом. Какую максимальную мощность М,1ах может развить этот двигатель? При какой силе тока 10 достигается эта мощность? Напряжение в сети считать постоянным. 16.41**. Магнитное поле имеет вертикальную ось симметрии (ось z). Про- екция вектора магнитной индукции В на эту ось Вг = Во (1 + flo С большой высоты падает медное кольцо диаметра d, имеющее электрическое сопротивление R; плоскость кольца все время гори- зонтальна, а центр движется Вдоль оси г. Определите установив- шуюся скорость падения о, если масса кольца равна т. 17. ЭЛЕКТРОМАГНИТНЫЕ КОЛЕБАНИЯ И ВОЛНЫ. ПЕРЕМЕННЫЙ ТОК Электромагнитные колебания и волны 17.1. Батарею из двух одинаковых конденсаторов емкостью С — 10 нФ каждый, заряженную от источника постоянного напряжения, подключают к катушке индуктивностью L = 8,0 мкГн. Определите период Т и частоту v возникающих в контуре электромагнитных колебаний, если конденсаторы в батарее соединены: а) последова- тельно; б) параллельно. 17.2. Какова максимальная сила тока 1м в контуре (см. задачу 17.1), если напряжение источника Um — 200 В? 17.3. Определите силу тока в колебательном контуре (см. задачу 17.2): а) через t\ = 0,31 мКс после подключения батареи конденсаторов к катушке; б) в момент, когда напряжение на батарее конденса- торов и = 100 В. Конденсаторы соединены параллельно.
Задачи 132 17.4. Что нужно делать для приема более коротких волн — сближать или раздвигать пластины конденсатора, включенного в колеба- тельный контур приемника? Увеличивать или уменьшать пло- щадь перекрытия пластин в конденсаторе переменной емкости? 17.5**. Конденсатор переменной емкости колебательного контура радио- приемника может изменять емкость от Ci до С2 > Ci. Какой комплект из N сменных катушек следует использовать, чтобы диапазон длин волн, на которые можно настраивать приемник, был как можно более широким и не содержал «просветов»? Какова верхняя граница этого диапазона, если нижняя гра- ница равна 17.6**. Конденсатор переменной емкости может изменять свою емкость от Ci = 56 пФ до С2 = 670 пФ. Сколько катушек надо иметь, чтобы колебательный контур радиоприемника можно было настраивать на любые радиостанции, работающие в диапазоне длин волн от Xi = 40 м до Хг — 2600 м? 17.7. Сколько электромагнитных колебаний, соответствующих длине волны X = 300 м, происходит в течение одного периода звуковых колебаний с частотой v = 500 Гц? 17.8. Колебательный контур радиоприемника настроен на частоту v = 6 МГц. Во сколько раз нужно изменить емкость конденсатора контура, чтобы настроиться на длину волны А. = 150 м? 17.9* *. Электрический контур состоит из конденсатора постоянной емкос- ти и катушки, в которую можно вдвигать сердечник. Один сер- дечник — ферритовый1’, другой изготовлен из меди. Как изме- нится частота собственных колебаний контура, если вдвинуть в катушку ферритовый сердечник? Медный? 17.10. Воздушная линия электропередачи переменного тока промышлен- ной частоты v = 50 Гц имеет длину I = 600 км. Определите сдвиг Феррит — магнитное соединение железа, являющееся изолятором.
133 Электричество и магнетизм по фазе Дф напряжений в начале и в конце этой линии. Скорость распространения сигналов по проводам равна скорости света в вакууме с. 17.11 Антенна корабельного радиолокатора находится на высоте Л == 25 м над уровнем моря. На каком максимальном расстоянии smax радиолокатор может обнаружить спасательный плот? С какой частотой п могут при ётом испускаться импульсы? 17.12. Радиолокатор работает на волне X = 5,0 см и испускает импульсы длительностью т = 1,5 мкс. Сколько колебаний содержится в каждом импульсе? Какова минимальная дальность Smin обнару- жения цели? 17.13*. Почему увеличение дальности радиосвязи с космическими кораб- лями в три раза требует увеличения мощности передатчика в 9 раз? Во сколько раз следует увеличить мощность передатчика для увеличения в три раза дальности радиолокации? В обоих случаях излучатель радиоволн можно считать точечным. Поглощением энергии при распространении радиоволн пренебречь. 17.14*. Антенна телевизора (точка С на ри- ^км сунке) принимает, наряду с волной от А^_ _ _ _ j В телецентра (точка А), волну, отражен- \ / ную от железной крыши (точка В). В \ / результате изображение на экране дво- \ / ится. На сколько сдвинуты получаемые 4км \ /4км изображения друг относительно друга? \ л Ширина экрана телевизора / = 50 см; Ч [ кадры на экране вменяются с частотой \ у П = 25 с"1,' изображение разлагается на \ ЛГ = 625 строк,. \ .. 17.1^* \./^^-;'-Ц.-?, ' ', .• КонД(!Йбр|ф«^1^®ЫЬ С1 До р/-<----* .„.1 . >> с, < Сг ]&: Ъмль!’ ’-Z Z - ’ : £ X после замыкания ключа? Конденсаторы, и ка- . тушку считайте идеальными,
Задачи 134 17.16**. Один из двух одинаковых конденсаторов (см. рисунок) заряжен до напряжения Uo, другой — не заряжен. Какое напряжение 17 установится на конденсаторах после замыкания ключа? Как сог- ласуется этот результат с законом сохранения энергии? Рас- смотрите, в частности, случай, когда соединительные провода являются сверхпроводящими. К задаче 17.16 К задаче 17.17 В цепи (см. рисунок) Ci = С2 = С. До замыкания ключа напряжение на первом конденсаторе равно Ui, второй конденсатор не заряжен. Определите максимальное значение 7м силы тока через катушку с индуктивностью L после замыкания ключа. Сопротивлением катушки можно пренебречь. Переменный ток 17.18. Определите частоту п вращения прямоугольной рамки в однород- ном магнитном поле с индукцией В = 0,50 Тл, если в рамке ин- дуцируется ЭДС с амплитудой &м = 20 В. Площадь рамки S = 200 см2, число витков N = 40. Ось вращения перпендикулярна вектору индукции магнитного поля. 17.19*. В цепь последовательно включены резистор с сопротивлением R, конденсатор с емкостью С и катушка с индуктивностью L. По цепи протекает переменный ток i = /mcosoC Определите амплитуды напряжения на каждом из элементов цепи и во всей цепи. По какому закону изменяется приложенное к цепи напряжение? 17.20*. В цепь последовательно включены резистор с сопротивлением R, конденсатор с емкостью С и катушка с индуктивностью L. К цепи прикладывают переменное напряжение и = t7Mcosa>t, где
135 Электричество и магнетизм Um — заданная постоянная величина, а и можно изменять в широких предедах произвольным образом. Какой вид имеют зависимости амплитуды силы тока 1м и амплитуды напряжения на резисторе Umr от циклической частоты со? 17.21. В цепь последовательно включены резистор с сопротивлением R = 1,0 кОм, конденсатор с емкостью С = 1,0 мкФ и катушка с индуктивностью L — 0,50 Гн. Найдите емкостное сопротивление Хс, индуктивное сопротивление Хъ и полное сопротивление Z цепи при частотах vi = 50 Гц и v2 = 10 кГц. При какой частоте v0 в цепи наблюдается резонанс? 17.22. В цепь переменцого тока включены последовательно резистор с сопротивлением JR, конденсатор с емкостью С и катушка с индук- тивностью L. Амплитуда силы тока в цепи равна /м- Определите среднюю мощность Р, потребляемую за период каждым из элемен- тов цепи. Конденсатор и катушку считайте идеальными. 17.23* . В цепь переменного тока включены последовательно резистор, конденсатор и катушка. Выразите среднюю мощность Р, потреб- ляемую всей цепью, через действующие значения силы тока I и напряжения U. Сдвиг фаз между колебаниями силы тока и напряжения равен ф. 17.24*. В цепи переменного тока (см. ри- сунок) показания первого и второго вольтметров Ui = 12 В и U2 = 9 В. Каково показание U2 третьего вольт- метра? 17.25. Проводник имеет активное сопро- тивление R = 30 Ом и индуктив- ность L = 0,127 Гн. Каковы полное сопротивление Z проводника и сдвиг фаз ф между током и напряжени- ем, если проводник включен в сеть переменного тока с частотой vi = 50 Гц? С частотой v2 = 400 Гц? 17.26. При подаче на катушку постоянного напряжения 30 В сила тока равнялась 1,0 А. При подаче переменного напряжения 30 В сила тока составила 0,60 А. Какова индуктивность L катушки? Какая мощность Р выделялась в катушке при прохождении постоянного и переменного тока? Частота переменного тока равна 50 Гц.
Задачи 136 17.27*. В сеть переменного тока частоты v — 50 Гц включены последова- тельно лампочка, конденсатор емкостью С = 20 мкФ и катушка. Индуктивность катушки без сердечника равна Li = 50 мГн, а при полностью введенном сердечнике L2 = 1,5 Гн. Как изменяется накал лампы по мере введения в катушку сердечника? 17.28*. В сеть переменного напряжения промышленной частоты v = 50 Гц включены последовательно лампочка, катушка с индуктивностью L = 0,50 Гн и конденсатор емкостью С = 10 мкФ. Как изменится накал лампы, если к конденсатору подключить параллельно вто- рой такой же конденсатор? Третий? 17.29*. Неоновая лампа с напряжением зажигания Ua — 156 В включена в сеть 220 В, 50 Гц. Определите частоту п вспышек лампы. В течение какой части периода лампа горит? Напряжение гашения лампы считайте равным напряжению зажигания. 17.30*. Найдите действующие значения I переменных токов (см. рисунки 17.31**. На рисунке показана схема простейшего выпрямителя, ис- пользуемого в школьных эк- спериментах. Емкость конден- сатора С = 500 мкФ, сопротив- ление резистора R = 1,0 кОм. Каково максимально возмож- ное значение напряжения t/вых на выходе выпрямителя? Какое обратное напряжение должен выдерживать диод? А D , R сеть - 36 В == С В ( К задаче 30
137 Электричество и магнетизм 17.32*. Разборный школьный трансформатор включен в сеть. Ко вто- ричной обмотке подключена нагрузка. Как изменится сила тока в первичной и вторичной обмотках при удалении верхней части сердечника (см. рисунок)? К задаче 17.32 К задаче 17.36 17.33. Почему для реостата замыкание одного-двух витков не опасно, а трансформатор может выйти из строя, если хотя бы один виток обмотки замкнется накоротко? 17.34. Через замкнутый кольцевой сердечник трансформатора, понижа- ющего напряжение с Ui = 220 В до t/2 = 42 В, пропущен провод, концы которого присоединены к вольтметру. Вольтметр показы- вает 17 = 0,50 В. Сколько витков имеют обмотки трансформатора? 17.35. Трансформатор понижает напряжение с 220 В до 42 В. а) В какой из обмоток провод должен быть толще? б) Можно ли подключить трансформатор к сети постоянного напряжения 100 В? в) Можно ли включить в сеть переменного тока с напряжением 220 В первичную катушку трансформатора, снятую с сердечника? 17.36**. Два одинаковых идеальных трансформатора имеют обмотки из ЛГ1 = 200 и N2 = 600 витков. Они соединены последовательно различными обмотками (см. рисунок) и подключены к источнику переменного напряжения 17 = 2(Х) В* Определите напряжение С7дс между точками А и С.
„ sina n2 Закон преломления света----= — sinp rti Оптическая сила линзы D = Л - 111 Формула тонкой линзы ~т + т = т; а / г Увеличение тонкой липзы Г = — = h Связь показателя преломления среды со скоростью света с п = — v Условие максимума при интерференции Ad = kX, k = О, ± 1, ± 2, ... Условие минимума при интерференции Ad = (2k + 1)^, k = О, ± 1, ± 2, ... Формула дифракционной решетки dsintp = kX, k = О, 1, 2, ...
ОПТИКА ЗАКОНЫ ГЕОМЕТРИЧЕСКОЙ ОПТИКИ ОПТИЧЕСКИЕ СИСТЕМЫ И ПРИБОРЫ
Задачи 140 18. ЗАКОНЫ ГЕОМЕТРИЧЕСКОЙ ОПТИКИ Световые лучи. Отражение света 18.1. Вертикальный шест высотой Л = 1,0 м, поставленный недалеко от уличного фонаря, отбрасывает тень длиной h = 80 см. Если расстояние между фонарным столбом и шестом увеличить на s = 1,5 м, то длина тени возрастет до 12 = 1,3 м. На какой высоте Н находится фонарь? 18.2*. Оцените размер области поверхности Земли, где одновременно наблюдается солнечное затмение (полное или частичное). Радиус Солнца гс = 7* 105 км, радиус Луны гл = 1700 км; расстояние от Земли до Солнца Re = 1,5 • 10s км, от Земли до Луны Ял — 3,8 • 105 км. 18.3*. В ясный безоблачный вечер лучи заходящего солнца попадают в затемненную комнату через узкую горизонтальную щель в ставне. Длина щели- а = 6 см, расстояние от окна до противоположной стены I — 3 м. Какова форма светового пятна на стене, если солнечные лучи падают на нее под прямым углом? Оцените размеры светового пятна. Что произойдет с пятном, если умень- шать ширину щели? Длину? 18.4*. Солнечные лучи, проходя сквозь маленькие отверстия в листве дерева, дают на земле светлые пятна в форме эллипсов одинаковой формы, но разных размеров. Большая ось самых крупных эл- липсов а = 16 см, а малая ось 5 = 12 см. Какова высота Н дерева? Под каким углом а к горизонту падают солнечные лучи? Угловой размер солнечного диска р = 9,3 10'3 рад. 18.5. Теплоход проходит мимо стоящей на якоре шхуны. В момент наибольшего сближения боцман шхуны вытягивает руку вперед и, глядя только правым глазом, заслоняет большим пальцем вытянутой руки нос теплохода. Открыв левый глаз и закрыв правый, он видит, что теперь его палец закрывает корму теплохо- да. Боцман мгновенно и довольно точно называет расстояние L до теплохода. Попробуйте и вы это сделать, если известно, что длина теплохода а = 100 м, длина вытянутой руки боцмана Z = 60 см, расстояние между зрачками Ь — 65 мм. 18.6. Высота солнца над горизонтом (угол между солнечными лучами и горизонтальной плоскостью) составляет а = 48°л .Под каким
141 Оптика углом Р к горизонту следует расположить зеркало, чтобы осветить солнечными лучами дно глубокого вертикального колодца? 18.7*. Луч света, идущий из точки А, при- ходит в точку В, отразившись от плос- кой пластины CD (Cnl. рисунок). До- кажите, что, «подчиняясь» закону от- ражения, луч «выбирает» кратчайший путь. 18.8. Два зеркала образуют двугранный прямой угол. На эту систему зеркал падает луч, перпендикулярный ребру угла. Как изменится направление распространения света после отражения от двух зеркал? 18.9. Два зеркала образуют двугранный угол <р. На одно из них падает под углом а луч, перпендикулярный ребру угла. На какой угол отклонится этот луч после двух отражений? 18.10*. Три зеркала образуют между собой двугранные прямые углы (см. рисунок). Докажите, что после трех отражений от этой системы зеркал направление любого луча света изменяется на противопо- ложное. К задаче 18.10 К задаче 18.11 18.11. В каких точках комнаты должен находиться человек, чтобы видеть в зеркале экран телевизора АВ (см. рисунок) Целиком? 18.12. Какова должна быть минимальная высота вертикального зеркала, чтобы человек ростом Н мог видеть в нем свое изображение во весь рост? На какой высоте должно висеть зеркало?
Задачи -142 18.13* . Ранним утром изображение Солнца в спокойной воде слепит глаза, а в полдень на это же изображение можно смотреть даже без темных очков. Почему? 18.14*. Толченое стекло теряет свою прозрачность, но в воде оно вновь становится прозрачным. Объясните эти явления. 18.15. Почему ночью лужа на неосвещенной дороге кажется водителю темным пятном на светлом фоне? 18.16. Свеча А находится между двумя зеркалами, образующими дву- гранный прямой угол (см. рисунок). Сколько изображений свечи дают зеркала и где эти изображения расположены? 18.17* . Сколько изображений предмета АВ дают зеркала (см. рисунок)? Где расположены эти изображения? Угол между зеркалами 60°. К задаче 18.17 К задаче 18.16 Преломление света 18.18. Луч света идет из стекла в воду и преломляется на плоской границе стекло-вода. При каком угле падения а отраженный и преломленный лучи перпендикулярны друг другу? 18.19**. Принцип Ферма. Свет идет из точки А в точку В, преломляясь на плоской грани- це раздела двух сред (см. рисунок). Дока- жите, что время прохождения света из точки А в точку В минимально как раз в том случае, когда свет «подчиняется» закону преломления.
rf3 Оптика 18.20. Свая вбита в дно реки и возвышается над водой на hi = 1,0 м. Глубина реки й2 = 2,0 м. Определите длину тени сваи на поверхности воды и на дне реки, когда высота солнца над горизонтом а = 30°. 18.21. Угол падения луча света на стеклянную плоскопараллельную плас- тинку а = 6О\ Выйдя из пластинки, луч сместился на а = 15 мм. Какова толщина Л пластинки? 18.22. Луч света направлен так, что. испытывает полное отражение на границе воды и воздуха. Сможет ли он выйти в воздух, если на поверхность воды налить подсолнечное масло? Масло с водой не смешивается. 18.23*. Показатель преломления жидкости плавно увеличивается от па у поверхности до пь У дна сосуда. Луч падает на поверхность жидкости под углом а. Определите угол 0 падения луча на дно сосуда. 18.24*. В жарких пустынях иногда наблюдается мираж: вдалеке «воз- никает» поверхность водоема. Какими физическими явлениями обусловлен такой мираж? 18.25*. В ясный солнечный день стоящий на дне озера водолаз видит в водном «зеркале» у себя над головой отражение всех участков дна, находящихся от него на расстоянии s = 10 м и более. Какова глубина Н озера? Рост водолаза h = 1,7 м. 18.26. Прямоугольный плот длиной а — 5 м и шириной b = 2,5 м плавает в открытом бассейне глубиной h = 1 м. Каковы размеры тени на дне бассейна в солнечный день? А когда все небо затянуто тучами? 18.27*. Могут ли солнечные лучи испытать полное отражение внутри дождевой капли? Каплю считайте шарообразной. 18.28**. Почему асфальт под дождем темнеет^ а полированный гранит — нет? 18.29*. Преломляющий угол призмы <р = 45°. Луч света выходит из призмы под тем же углом, под каким он в нее входит; при этом луч
Задачи 144 отклоняется от первоначального направления на угол 3 = 25°. Определите показатель преломления п материала призмы. 18.30 . д Преломляющий угол <р призмы (см. ри- /^\ сунок) равен 60°. Угол падения луча на / \ грань призмы а = 30 °. Определите угол \ 3 отклонения луча от первоначального / х \ направления после прохождения через / \ призму. Показатель преломления мате- / \ риала призмы п = 1,5. 18.31. Луч падает на боковую грань стеклянной призмы под прямым углом. Определите угол 8 отклонения луча от первоначального направ- ления, если преломляющий угол <р призмы равен: а) 30°; б) 60°. 18.32*. На боковую грань равнобедренной призмы падает луч, идущий параллельно основанию призмы. При каком условии луч, пройдя призму, не изменит своего направления? 18.33. Угол падения луча на боковую грань призмы равен а. Преломля- ющий угол призмы <р, показатель преломления материала призмы п. Определите угол 3 отклонения луча от первоначального направ- ления, если углы а и <р малы. 18.34. Нижняя поверхность плоскопараллельной стеклянной пластинки посеребрена. На пластинку сверху падает луч света; в результате от нее отражаются два параллельных луча, идущих на расстоянии а = 20 мм друг от друга. Определите толщину h пластинки, если угол падения луча а = 60°. 18.35*. Если смотреть сверху на неглубокий водоем с чистой водой, то дно хорошо видно, однако глубина водоема кажется меньшей, чем она есть в действительности. Во сколько раз? 18.36. Рыба, находящаяся на глубине hi — 1,0 м, смотрит вертикально вверх в глаза рыболову. Голова рыболова находится на высоте й2 = 1,5 м над водой. Каким покажется рыбе расстояние I до головы рыболова?
^45 Оптика 18.37*. Над водой на высоте hi = 1,0 м поместили горизонтально плоское зеркало. На какой высоте Л над водой увидит свое отражение рыба, находящаяся на глубине Лг = 0,50 м? 18.38**. На сферическую каплю воды падает луч света. Определите угол 8 отклонения луча от первоначального направления в результате двух преломлений и одного отражения на поверхности капли. Угол падения луча из воздуха на поверхность капли равен а. 19. ОПТИЧЕСКИЕ СИСТЕМЫ И ПРИБОРЫ Линзы 19.1. Определите построением (см. рис. а, б, в), где находятся оптичес- кий центр О тонкой линзы и ее фокусы F, если MN — главная оптическая ось линзы, А — светящаяся точка, Ai — ее изобра- жение. Определите также тип линзы (собирающая или рассеива- ющая) и тип изображения (действительное или мнимое). А М>------------------N •А, Рис. а .А, А • М-------------------N Рис. б А * А, м-------------------N Рис. в К задаче 19.1 Рис. б К задаче 19.2 19.2. Определите построением положение фокусов линзы (см. рис. а, б), если задана главная оптическая ось MN и ход произвольного луча.
Задачи 146 19.3*. Постройте изображение точки А (см. рисунок), лежащей на глав- ной оптической оси линзы. К задаче 19.3 К задаче 19.5 19.4. Как изменится изображение, полученное на экране при помо- щи собирающей линзы, если закрыть рукой верхнюю половину линзы? 19.6. Из стекла двух сортов с различными показателями преломления изготовлена слоистая линза (см. рисунок). Какое изображение точечного источника света даст эта лйнза? Считайте, что на границах между слоями свет полностью поглощается. 19.6. Всегда ли двояковыпуклая линза является собирающей? 19.7. Предмет находится на расстоянии ai = 8,0 см от переднего фокуса линзы, а его изображение — на экране на расстоянии а2 — 18 см от заднего фокуса линзы. Определите фокусное расстояние F линзы. 19.8* . Каково минимально возможное расстояние Zmin между предметом и его действительным изображением, получейным с помощью собирающей линзы? Какое увеличение Г дает линза в этом случае? Фокусное расстояние линзы равно F. 19.9* . Собирающая линза с фокусным расстоянием F = 40 мм исполь- зуется в качестве зажигательного стекла. Определите мини- мальный диаметр d полученного с ее помощью светлого пятна. Угловой диаметр солнечного диска при наблюдении с Земли Р = 9,3« 10~3 рад. »
147 Оптика 19.10. Постройте изображение предмета в собирающей линзе при d < F, т. е. когда предмет находится между линзой и ее фокусом. Докажите, что формула тонкой линзы применима и в этом случае, если считать f < 0. 19.11. Постройте изображение предмета в рассеивающей линзе. До- кажите, что формула тонкой линзы применима и в этом случае, если считать f < 0 и F < 0. 19.12*. На собирающую линзу с фокусным расстоянием Г = 20 см падает сходящийся пучок лучей (см. рисунок). Расстояние а от оптичес- кого центра линзы до точки А равно 30 см. На каком расстоянии /от линзы возникает изображение Ai? Действительное или мнимое это изображение? 19.13*. Решите предыдущую задачу, если линза является рассеивающей с фокусным расстоянием F — -20 см (см. рисунок). 19.14. Рассеивающая линза с фокусным расстоянием F = -10 см дает мнимое изображение предмета, уменьшенное в 2 раза. На каком расстоянии d от линзы находится предмет? Постройте ход лучей и изображение. 19.16*. Определите оптическую силу D рассеивающей линзы, если она дает изображение предмета на расстоянии I = 6,0 см от самого Предмета. Высота предмета Л = 8,0 см, высота изображения Н = 4,0 см. 19.16*. Фокусное расстояние собирающей линзы F = 60 мм. Точечный источник света находится на расстоянии d = 60 мм от линзы на
света, Задачи 148 ее главной оптической оси MN. Линзу разрезали по диаметру на две равные части, которые раздвинули на расстояние s = 10 мм симметрично относительно оси MN. Определите расстояние Н между двумя изображениями источника 19.17*. Самодельный осветитель представляет собой цилиндрическую трубку радиуса R = 10 мм с черной внутренней поверх- ностью. В трубку (см. рисунок) встав- лена собирающая линза, а на оси труб- ки на расстоянии d = 20 см от линзы находится точечный источник света. На перпендикулярно оси трубки на расстоянии а = 30 см от линзы, осветитель дает круглое светлое пятно радиуса г = 5,0 мм. Опре- делите фокусное расстояние F линзы. 19.18*. Экран Помещают перед осветителем (см. задачу 19.17) на таком расстоянии, что на экране получается четкое изображение точеч- ного источника света. Фокусное расстояние линзы F = 15 см. Каков будет радиус г светлого круга на экране, если вынуть из трубки линзу? 19.19. Какое фокусное расстояние F должен иметь объектив: а) для получения уменьшенной фотокопии текста в масштабе 1 : 10 (фотографирование производится с расстояния d = 1 м); б) для получения снимков местности в масштабе 1 : 5000 при топо- графической съемке с самолета, летящего на высоте h = 1 км? 19.20. Фотограф находится на расстоянии d = 10 м от дороги. Он фото- графирует проезжающий со скоростью v = 54 км/ч автомобиль, когда тот находится в ближайшей точке дороги. Размытость контуров изображения на пленке не должна превышать величины а = 50 мкм. Каково должно быть время экспозиции At? Фокусное расстояние объектива F = 40 мм. 19.21*. Со спутника, летящего на высоте 27 = 150 км, фотографируют ночной город. Разрешающая способность пленки (наименьшее расстояние между изображениями двух точек, когда изображения не сливаются), равна AZ = 0,01 мм. Фокусное расстояние объектива F = 10 см. Каким должно быть расстояние L между уличными фонарями, чтобы их изображения на снимке получились раздель- ными? Оцените время т экспозиции, при котором движение спут- ника не приводит к заметному размыванию изображения.
149 Оптика 19.22*. При съемке с расстояния di = 4,25 м изображение предмета имеет высоту Hi = 2,7 мм; при съемке с расстояния d2 = 1,0 м — высоту Н2 = 12 мм. Определите фокусное расстояние F объектива. 19.23*. Объектив с фокусным расстоянием F — 50 мм дает на пленке изображения точек, расположенных на расстоянии do = 2,0 м от объектива. Будем считать изображение на пленке достаточно резким, если его размытость (размер «изображения» каждой точки предмета) не превышает а = 0,10 мм. При каком расстоянии d предмета от фотоаппарата на пленке получается достаточно резкое изображение предмета? Диаметр объектива D = 30 мм. 19.24*. Каким должно быть фокусное расстояние F объектива (см. задачу 19.23), чтобы изображение оставалось достаточно резким при d -> <ю? 19.25*. Краб ползет по дну на глубине Hi = 2,0 м. Его фотографируют сверху. Объектив с фокусным расстоянием F = 50 мм находится на высоте Н2 = 1,0 м над поверхностью воды. Какое расстояние d должно быть установлено на шкале дальности объектива? Какова длина L изображения клешни, если длина самой клешни I = 10 см? 19.26. Лампа находится на расстоянии I — 90 см от стены. На каком расстоянии f от стены следует разместить собирающую линзу с фокусным расстоянием F = 20 см, чтобы получить на стене четкое изображение нити накала лампы? Главная оптическая ось линзы должна быть перпендикулярна стене. 19.27. Собирающая линза дает на экране четкое изображение предмета. Высота предмета й, высота изображения hi > й. Линзу передвигают ближе к экрану и опять получают на экране четкое изображение. Какова его высота йг? 19.28*. Предмет находится на расстоянии I = 45 см от экрана. С помощью линзы получают на экране уменьшенное изображение предмета. Перемещая линзу, получают на экране другое изображение, в п = 4 раза больше первого. Каково фокусное расстояние F линзы? 19.29. Свеча находится на расстоянии I = 2,5 м от экрана. Собирающая линза дает на экране четкое изображение свечи при двух поло- жениях, находящихся на расстоянии а = 50 см друг от друга. Определите оцтдческую сцду D линзы.
Задачи 150 19.30*. Расстояние между двумя точечными источниками света I — 32 см. Где следует поместить между ними собирающую линзу с фокус- ным расстоянием F = 12 см, чтобы изображения обоих источников оказались в одной точке? 19.31**. Постройте изображение наклон- 7 4 ной стрелки АВ (см. рисунок), проходящей через фокус собира- ющей линзы. _____________ 19.32*. уТ Середина стержня, имеющего дли- г ну I = 10 мм, находится на рас- А стоянии d = 18 см от собирающей Ф линзы на ее главной оптической оси. Определите длину L изобра- жения стержня в двух случаях: а) стержень расположен парал- лельно плоскости линзы; б) стержень расположен вдоль главной оптической оси линзы. Фокусное расстояние линзы F = 12 см. Оптические системы 19.33*. Систему линз называют телескопической, если падающие па нее параллельные лучи выходят из системы, по-прежнему оставаясь параллельными. Как надо расположить две линзы, чтобы они образовали телескопическую систему? 19.34*. Две тонкие линзы с оптическими силами Di и D2 имеют общую главную оптическую ось и расположены вплотную друг к другу. Докажите, Что их можно заменить одной линзой с оптической силой D = Di + Рз- 19.36*. За собирающей линзой с фокусным расстоянием Г = 30 см расположено на расстоянии а = 15 см плоское зеркало, пер- пендикулярное главной оптической оси линзы. Где находится изображение предмета, расположенного перед линзой на рассто- янии d = 15 см? Каким будет это изображение — действительным или мнимым?
151 Оптика 19.36*. Плоскую поверхность плосковыпуклой линзы (см. рисунок! С фокусным расстоянием F посеребрили. Где получится изобра- жение Ai точечного источника А, расположенного в фокальной плоскости линзы? К задаче 19.36 К задаче 19.37 19.37*. Постройте ход лучей и определите положение изображения пред- мета АВ (см. рисунок) в оптической системе, состоящей из со- бирающей линзы и плоского зеркала. 19.38*. Точечный источник света расположен на главной оптической оси собирающей линзы на расстоянии 1,5 F от нее. На каком рассто- янии нужно разместить за линзой перпендикулярное ее главной оптической оси плоское зеркало, чтобы отразившиеся от него и вторично прошедшие через линзу лучи образовали параллельный пучок? 19.39*. Система состоит из собирающей линзы с фокусным расстоянием Ft = 30 см и рассеивающей линзы с фокусным расстоянием Fz = -15 см. Линзы расположены на расстоянии а = 30 см друг от друга, их главные оптические оси совпадают. Где следует разместить точечный источник света, чтобы эта система давала пучок параллельных лучей? 19.40*. Две собирающие линзы с фокусными расстояниями Fi = 12 см и Fz = 15 см расположены друг за другом на расстоянии а = 36 см. Предмет находится на расстоянии d = 48 см от первой линзы. На каком расстоянии f от второй линЗы находится изображение предмета? 19.41*. '-В Трубку вставлены две собирающие линзы с фокусными рассто- яниями Fi — 80 мм и Fz = 50 мм. Главные оптические оси линз совпадают, расстояние между Линзами I = 16 см. Предмет высотой
Задачи 152 h = 20 мм помещен на расстоянии d = 40 см перед первой линзой. Где находится изображение? Какова его высота Н? 19.42*. Объектив состоит из двух тонких линз: собирающей с фокусным расстоянием Fi = 20 см и рассеивающей с фокусным расстоянием F2 — -10 см. Расстояние между линзами I = 15 см. С помощью объектива получают на экране изображение Солнца. Какую одну линзу надо взять, чтобы получить изображение Солнца такого же размера? 19.43*. Система состоит из двух линз (собирающей и рассеивающей) с одинаковыми по модулю фокусными расстояниями. Главные оп- тические оси линз совпадают. С помощью этой системы на экране получено изображение Солнца. Когда линэы поменяли местами, экран пришлось передвинуть на s = 30 см. Каково фокусное рассто- яние F собирающей линзы? 19.44*. Объектив фотоаппарата состоит из двух линз. Рассеивающая линза с фокусным расстоянием А = -50 мм расположена на расстоянии I = 45 см от пленки. Где должна находиться собирающая линза с фокусным расстоянием F2 — 80 мм, чтобы на пленке получались резкие изображения удаленных предметов? Зрение. Оптические приборы 19.45. Почему, ныряя с открытыми глазами, мы видим размытые очер- тания предметов? Почему маска для ныряния позволяет отчетливо видеть под водой? 19.46. В известном романе английского писателя-фантаста Герберта Уэллса описаны приключения человека-невидимки. Если ве- рить писателю, такой человек может безнаказанно совершать дерзкие преступления. В чем состоит главная физическая ошиб- ка фантаста? 19.47*. Близорукий человек читает без очков, держа книгу на расстоянии d — 10 см от глаз. Какова оптическая сила D необходимых ему очков для чтения1*? ** ** В этой и последующих задачах расстояние между линзой очков и глазом считать малым.
153 Оптика 19.48. Как изменится оптическая сила хрусталика глаза при переводе взгляда со страницы учебника на облака за окном? 19.49. Человек с нормальным зрением надел очки с оптической силой D — +3 дптр. На каком расстоянии d можно расположить предмет, чтобы человек рассматривал его без ощутимого напряжения? 19.50. Фотоаппарат дает достаточно четкие снимки предметов, располо- женных не ближе а — 1,0 м от объектива. На каком расстоянии d от объектива может находиться фотографируемый предмет, если использовать насадочную линзу с оптической силой D = 2,0 дптр? 19.51*. Ближний и дальний пределы аккомодации глаза близорукого человека di = 10 см и dz — 30 см. Человек носит очки, в которых хорошо видит удаленные предметы. На каком наименьшем рас- стоянии а он может держать книгу, читая в этих очках? 19.52*. Ближний и дальний пределы аккомодации глаза близорукого человека di = 8 см и dz = 17 см. Каковы будут эти пределы d'i и d'z, если человек наденет очки с оптической силой D = -4 дптр? 19.53**. Ближний и дальний пределы аккомодации глаза близорукого человека di = 9 см и dz = 14 см. Какие очки потребуются этому человеку, чтобы он мог хорошо видеть все предметы на любом расстоянии, превышающем а = 15 см? Удастся ли обойтись одной парой очков? 19.54*. Часто говорят, что лупа «приближает» к глазу рассматриваемый предмет. Правильно ли это утверждение? Как связано фокусное расстояние лупы с величиной, которую указывают на оправе (например, «х4»)? 19.55. Самодельный «мелкоскоп». Чтобы разглядеть очень мелкий пред- мет, можно поДнести его близко к глазу и разглядывать через маленькое (диаметром 0,5 мм — 1 мм) отверстие в листе картона или черной бумаги. Объясните принцип действия такого простого оптического прибора. 19.56*. Правильно ли утверждение: «телескоп дает увеличенные изобра- жения удаленных звезд»?
Задачи 154 19.67**. Зрительная труба, объектив которой имеет фокусное расстояние F = 30 см, «настроена» для наблюдения Луны. В какую сторону и на сколько следует передвинуть окуляр, чтобы рассматривать предмет, находящийся на расстоянии d = 10 м от объектива? 19.58*. Человек с нормальным зрением настраивает микроскоп так, чтобы глаз уставал как можно меньше. Определите увеличение Г мик- роскопа, если фокусное расстояние его объектива Fi = 3,0 мм, а окуляра Fz = 50 мм. Рассматриваемый предмет находится на расстоянии (/ = 3,1 мм от объектива. 19.59**. Фокусное расстояние объектива микроскопа Ft = 3,5 мм, а увели- чение микроскопа Г = 100. Определите фокусное расстояние Рг окуляра, если он расположен на расстоянии I = 12,4 см от объектива. 20. ВОЛНОВАЯ ОПТИКА 20.1. Опыт Физо по определению скорости света состоял в следующем. Световой пучок проходил через узкую прорезь между зубцами вращающегося колеса, отражался от зеркала, расположенного на расстоянии I = 8,7 км от колеса, и возвращался, опять проходя между зубцами колеса. При какой минимальной частоте v вра- щения колеса отраженный свет исчезал? Количество зубцов на колесе /V - 720. 20.2. При отражении от крыши, покрытой листовым железом, свет сильно рассеивается, а радиоволны от той же крыши отражаются зеркально. Почему? Свет и цвет. Дисперсия света 20.3. Как будет выглядеть белая надпись на красном фоне, если осве- тить ее зеленым светом? 20.4. Вода освещена красным светом с длиной волны А = 728 нм. Какова длина волны А' этого света в воде? Какой цвет увидит человек, открывший глаза под водой? Какой цвет зафиксирует положенная в воду цветная позитивная фотопленка?
155 Оптика 20.5*. Английский физик лорд Рэлей установил, что на мелких неодно- родностях среды короткие волны рассеиваются значительно силь- нее, чем длинные. Объясните, исходя из этого, голубой цвет неба. 20.6*. Почему когда Солнце или Луна находятся низко над горизонтом, они приобретают красный оттенок? 20.7*. Почему освещенный столб дыма на темном фоне кажется синева- тым, а на фоне светлого неба — желтым или красноватым? 20.8. Почему зимой в ясную погоду тени деревьев на снегу имеют голубоватый оттенок? 20.9*. Правильно ли часто встречающееся утверждение: «смешивая красные, синие и зеленые лучи в разных пропорциях, можно получить любой цвет»? 20.10*. Луч белого света падает под углом а = 30° на призму с преломляю- щим углом <р = 45°. Показатель преломления стекла призмы для красного света пк = 1,62, а для фиолетового иф = 1,67. На каком расстоянии L от призмы следует разместить экран шириной з = 10 см, чтобы получить на нем изображение всего видимого спектра? Интерференция света 20.11*. Две когерентные световые волны в результате интерференции взаимно погашаются в некоторой области. Куда девается их энергия? 20.12**. Лучи белого света падают под углом а = 60° на очень тонкую прозрачную пластинку. При этом пластинка в отраженном свете кажется зеленой. Как изменится цвет пластинки, при небольшом уменьшении угла падения лучей? При его увеличении? 20.13*. Цвета тонких пленок (например, пленки бензина на воде) заметно отличаются оттенками от цветов радуги. Почему? 20.14*. Когда монохроматический свет падает нормально на поверхность мыльной пленки, интенсивность отраженного света зависит от длины волны: она имеет максимум при Xi = 630 нм и ближайший
Задачи 156 к нему минимум при Х2 = 525 нм. Какова толщина пленки d? Показатель преломления пленки п = 1,33. 20.15**. Почему интерференционная окраска наблюдается только у доста- точно тонких пленок? 20.16*. Два когерентных источника мо- нохроматического света с длиной волны X = 600 нм находятся на расстоянии АА-2 = 1,0 мм друг от друга и на одинаковом расстоя- нии L = 3,0 м от экрана (см. рисунок). Каково расстояние х между ближайшими максиму- мами освещенности (серединами светлых полос) на экране? Будет ли наблюдаться в точке О макси- мум освещенности? 20.17*. Точечный источник монохрома- тического света находится на рас- стоянии S — 1,0 мм от большого плоского зеркала и на расстоянии L = 4,0 м от экрана, перпендику- лярного зеркалу (см. рисунок). Каково расстояние х между сосед- ними максимумами освещеннос- ти? Длина волны света А = 600 нм. 20.18*. Два плоских зеркала образуют двугранный угол а = 179,5° (см. рисунок). На одинаковых расстояниях d = 10 см от каждого из зеркал расположен точечный источник А монохроматического света с длиной волны X = 600 нм. Определите расстояние х между серединами соседних светлых интерференционных полос на экра- не, расположенном на расстоянии L = 3,0 м от линии пересечения зеркал. Щирма В не дозволяет попадать на экран свету непосред- ственно от источника А, 20.19*. Точечный источник А монохроматического света с длиной вол- ны X = 500 нм расположен на расстоянии Z = 50 см от экрана. На расстоянии 1,5 Z от экрана находится параллельное ему плоское
157 Оптика зеркало. Какой вид имеет интерференционная картина на экране? Темная или светлая интерференционная полоса проходит на рас- стоянии г = 2,0 мм от точки О (см. рисунок)? К задаче 20.18 К задаче 20.19 20.20*. Чтобы уменьшить коэффициент отражения света от поверхности стекла, на нее наносят тонкую прозрачную пленку с показателем преломления ип меньшим, чем у стекла (так называемое «просвет- ление оптики»). Считая па = 'In, где п — показатель преломления стекла, определите необходимую толщину пленки Л. Длина волны света X = 500 нм, свет падает на поверхность нормально. 20.21*. На поверхность объектива нанесена «просветляющая» пленка толщиной Л = 300 нм (см. задачу 20.20). На объектив падает нормально пучок белого света. Какой оттенок будет иметь свет, прошедший через объектив? Отразившийся от объектива? 20.22**. Собирающая линза с фокусным расстоянием F = 10 см разрезана пополам по диаметру, и половинки раздвинуты на расстояние Л = 0,50 мм. Перед линзой на расстоянии d = 15 см находится точечный источник монохроматического света с длиной волны X = 500 нм. Оцените число N светлых интерференционных полос на экране, расположенном за линзой на расстоянии L = 60 см. Промежуток между половинками линзы закрыт непрозрачным Экраном.
Задачи 158 20.23**. «Кольца Ньютона». Плоско-выпуклая лин- за с большим радиусом кривизны выпук- лой стороны (R — 1 м) лежит на плоской стеклянной пластине (см. рисунок). Систе- му освещают сверху монохроматическим светом с длиной волны X = 500 нм. При наблюдении сверху (в отраженном свете) видно круглое темное пятно, окруженное концентрическими свет- лыми и темными кольцами. Объясните явление. Определите ра- диус г.? третьего темного кольца. Дифракция света 20.24. На дифракционную решетку с периодом d = 14 мкм падает нормально монохроматическая световая волна. На экране, удален- ном от решетки на L - 2,0 м, расстояние между спектрами второго и третьего порядка з = 8,7 см. Какова длина волны X падающего света? 20.25. Монохроматический свет с длиной волны X = 520 нм падает нормально на дифракционную решетку, имеющую 500 штрихов на миллиметр. Определите наибольший порядок наблюдаемого спектра kmax* 20.26.. Почему при получении дисперсионного спектра с помощью приз- мы наименьшее отклонение испытывают красные лучи, а в диф- ракционном спектре — фиолетовые лучи? 20.27*. На дифракционную решетку с периодом d = 2,0 мкм падает нормально свет с длиной волны X = 500 нм. За решеткой располо- жена собирающая линза с фокусным расстоянием F = 50 см. Где нужно разместить экран, чтобы получить на нем четкий диф- ракционный спектр? Каково расстояние з на экране между спек- тром третьего порядка и центральным максимумом? 20.28*. Как изменится вид дифракционного спектра, если источник бело- го света, дифракционную решетку и экран переместить из воздуха в воду, не меняя расстояний между ними? Рассмотрите также случай, когда для получения дифракционного спектра использу- ется стоящая за дифракционной решеткой собирающая линза.
159 Оптика 20.29*. На дифракционную решетку с периодом d = 4,0 мкм падает нормально свет, пропущенный через светофильтр. Полоса пропус- кания светофильтра — от Xi = 500 нм до Хг = 550 нм. Будут ли спектры разных порядков перекрываться друг с другом? 20.30*. Свет с длиной волны X падает наклонно на дифракционную решетку с периодом d. Угол падения равен а. Выведите формулу, определяющую значения угла ф между дифракционными мак- симумами и нормалью к плоскости дифракционной решетки.
Релятивистское «сокращение длины» I = loVl - v2/c2 „ to Релятивистское «замедление времени» т = -т========- VI - о2/с2 „ - v' + V Релятивистский закон сложения скоростей v =---------- F 1 + v'V/c2 „ „ mov Релятивистский импульс р = 7= , = VI - и /<? _ „ „Ар Основной закон релятивистской динамики F = —— At , ТПоС2 Энергия релятивистской частицы Е = тс = . ....~ = VI - v/c2 Квант энергии электромагнитного излучения Е = hv тпи 2 Уравнение Эйнштейна для фотоэффекта hv = -—+ А Де-бройлевская длина волны X = — Р Дефект масс ядра Атп - ZmP + Nm„ - М. Энергия связи ядра Е^ = Лтс2 Закон радиоактивного распада N = No 2 ,/г
ТЕОРИЯ ОТНОСИТЕЛЬНОСТИ И АТОМНАЯ ФИЗИКА РЕЛЯТИВИСТСКАЯ МЕХАНИКА КВАНТЫ, АТОМЫ, ЯДРА, ЧАСТИЦЫ О
Задачи.J 62 21. РЕЛЯТИВИСТСКАЯ МЕХАНИКА 21.1. При какой скорости и движения тела релятивистское сокращение длины тела составляло бы 1 % ? 50% ? 21.2. Среднее время жизни мюона (нестабильной элементарной части- цы) согласно справочнику т0 = 2,2 мкс. Пучок мюонов движется со скоростью v = 0,95с. Какова средняя длина I их пробега в отсутствие столкновений? 21.3*. Космическая ракета удаляется от наблюдателя со скоростью о — 0,80 с. На ракете установлена пушка, которая может сообщать снаряду скорость и относительно ракеты, а) Какую скорость V относительно наблюдателя имеет снаряд, выпущенный вперед со скоростью и = п? б) При какой величине и снаряд, выпущенный назад, будет приближаться к наблюдателю со скоростью V — о? 21.4*. Релятивистская частица распадается на два одинаковых «оскол- ка». Скорость одного из них равна нулю. Определите скорость о частицы до распада и скорость щ второго «осколка». Известно, что при распаде неподвижной частицы оба «осколка» имеют скорость и. 21.5*. Одна из двух одинаковых частиц неподвижна, другая движется с релятивистской скоростью v. Пользуясь релятивистской формулой сложения скоростей, определите скорость и центра масс частиц. 21.6*. В системе отсчета К расстояние между точками, в которых прои- зошли два события, равно I, а промежуток времени между этими событиями равен т. Обозначим S = eV - I2. В системе отсчета К' соответствующая величина S’ = сгт'г - I'2. Исходя из постулатов теории относительности, докажите, что величины S и S' всегда имеют одинаковый знак и могут обращаться в нуль только одно- временно. 21.7*. Как известно, одновременность событий относительна. Существу- ет ли такая система отсчета К', в которой попадание мяча в окно происходит одновременно с ударом по этому мячу? 21.8**. Атомы водорода могут излучать характерные радиоволны с дли-
163 Теория относительности и атомная физика ной волны Хю = 21 см (в системе отсчета, связанной с излучающим атомом). Какую длину волны X имеет принимаемое на Земле излучение, если атомы водорода движутся со скоростью v = 0,60 с перпендикулярно направлению на Землю? 21.9*. Космический корабль летит со скоростью и = 0,60 с относительно Земли. В кабине корабля растет стебель лука со скоростью ио = 5,0 см/сут. Какова скорость и роста стебля с точки зрения земного наблюдателя? Стебель расположен под прямым углом к направлению движения корабля. 21.10. Лазер установлен на космической станции, вращающейся вокруг своей оси с угловой скоростью го. Лазер дает световой пучок, перпендикулярный оси вращения. Светлое пятно от этого пучка Перемещается по экрану, установленному нормально к пучку света на расстоянии I от космической станции, со скоростью о = го I. При I > с/го скорость движения светлого пятна на экране превысит скорость света. Не противоречит ли это постулатам теории отно- сительности? 21.11**. Тело с массой покоя т движется прямолинейно с ускорением а. Какая сила F действует на него в момент, когда скорость тела равна и? 21.12*. Электрон разгоняется в однородном электрическом поле с напря- женностью Е до релятивистской скорости. Запишите формулу зависимости скорости v электрона от времени и постройте график этой зависимости. Через какое время т скорость и достигнет половины скорости света? Начальную скорость электрона счи- тайте равной нулю. 21.13**. Какое расстояние s должен пройти электрон (см. задачу 21.12), чтобы приобрести скорость о? Запишите формулу зависимости s(t). 21.14. Тело с массой покоя т движется криволинейно с постоянной по величине скоростью V. Какая сила F действует на него в тот момент, когда ускорение тела равно а? 21.16*. Как зависит период Т обращения релятивистского протона в Циклотроне от скорости протона о? Величина индукции маг- нитного поля равна В. 6*
Задачи 164 21.16. Выразите полную энергию W релятивистской частицы и ее кине- тическую энергию W\ через массу покоя т и импульс р. 21.17*. На сколько меньше скорости света должна быть скорость элект- рона,’ чтобы его энергия была равна энергии покоя протона? 21.18. Масса покоящегося поезда т = 3000 т. На какую величину Ат увеличивается масса поезда при движении со скоростью v = 72 км/ч? 21.19. На сколько уменьшается масса т = 10 кг воды при замерзании? 21.20. При какой скорости v движения частицы ее кинетическая энергия равна энергии покоя? 21.21*. Какую скорость vv приобретет протон, пройдя ускоряющую раз- ность потенциалов U = 700 кВ? Какую скорость <л приобретет электрон, пройдя такую же разность потенциалов? Начальная скорость частиц равна нулю. 21.22**. На концах трубки со сжатой пружиной — удерживаются нитью одинаковые шарики ( т W'lTTTTTX-f т J с массой т (см. рисунок). При разрыве —л-Х нити шарики разлетаются со скоростью и. Определите скорости щ и v2 разлетающихся шариков, если в момент «выстрела» трубка поступательно движется в направлении своей оси со скоростью и > и. Определите также суммарный импульс р разлетающихся шариков. Не противоречит ли полученный результат закону со- хранения импульса? Скорости и, и считать релятивистскими. 21.23*. Масса покоя каждого из «осколков» (см. задачу 21.4) равна т. Какова была масса покоя М частицы? Определите импульс pi частицы до распада и суммарный импульс р2 «осколков». Согласуется ли полученный результат с законом сохранения импульса? 21.24*. Релятивистская частица распадается на два одинаковых «оскол- ка», каждый из которых имеет массу покоя т. Один из «осколков» неподвижен относительно лабораторной системы отсчета, а другой движется со скоростью v = 0,80 с. Какую скорость и и массу покоя М имела частица до распада?
165 Теория относите ъьности и атомная физика 21.26**. Какую энергию W нужно сообщить протону, чтобы при бом- бардировке неподвижных водородных мишеней стали возможны те же процессы, что и в случае столкновения двух протонов, дви- жущихся навстречу друг другу с энергией Wi = 70 ГэВ каждый? 21.26**. До какой энергии W потребуется разогнать встречные электроны, чтобы при их столкновении могли происходить те же процессы, что при электрон-электронных столкновениях в «обычном» уско- рителе, где пучок электронов с энергией Wi = 150 ГэВ падает на неподвижную мишень? 22. КВАНТЫ, АТОМЫ, ЯДРА, ЧАСТИЦЫ 22.1. Гелий-неоновый лазер работает в непрерывном режиме, развивая мощность Р = 2,0 мВт. Излучение лазера имеет длину волны А — 630 нм. Сколько фотонов излучает лазер за одну секунду? 22.2. Энергия каждого фотона в пучке монохроматического излучения Wo = 4,4 10 19 Дж. Какова длина волны А этого излучения в воде? 22.3. Под каким напряжением U работает рентгеновская трубка, если самое «жесткое» излучение в спектре этой трубки имеет длину волны А™,, = 3,0 1011 м? 22.4. Для калия красная граница фотоэффекта А^ах - 0,62 мкм. Какую максимальную скорость и могут иметь фотоэлектроны, вылета- ющие при облучении-калия фиолетовым светом с длиной волны А = 0,42 мкм? 22.5. Минимальная частота света, вырывающего электроны с поверх- ности металлического катода, v0 = 6,0 Ю14 Гц. При какой частоте v света вылетевшие электроны полностью задерживаются раз- ностью потенциалов U = 3,0 В? 22.6. При освещении поверхности некоторого металла фиолетовым све- том с длиной волны Ai = 0,40 мкм выбитые светом электроны полностью задерживаются разностью потенциалов (запирающим напряжением) Ui = 2,0 В. Чему равно запирающее напряжение Е72 при освещении того же металла красным светом с длиной волны А2 = 0,77 мкм?
Задачи 166 22.7*. На поверхность падает нормально электромагнитное излучение с интенсивностью I. Определите давление р излучения на поверх- ность в двух случаях: когда поверхность черная (абсолютно по- глощающая) и когда она зеркальная. 22.8*. Свет падает ца зеркальную поверхность. Определите давление р света на эту поверхность, если интенсивность излучения равна I, а угол падения а. 22.9*. Находящийся в вакууме легкий ци- линдр может с малым трением вра- щаться вокруг своей оси (см. ри- сунок). Половина поверхности ци- линдра окрашена в черный цвет, другая половина — зеркальная. Как повернется цилиндр под действием солнечного излучения, если солнеч- ные лучи направлены перпендику- лярно оси цилиндра? 22.10. Определите длину волны X электромагнитного излучения, если энергия одного кванта этого излучения равна энергии покоя электрона. С какой скоростью v должен двигаться электрон, чтобы его импульс сравнялся с импульсом такого фотона? Определите отношение энергии W движущегося электрона к энергии W, фотона. 22.11. Может ли один гамма-квант в вакууме превратиться в пару электрон-позитрон? 22.12. При какой частоте v излучение способно при взаимодействии с веществом вызвать рождение пары электрон-позитрон? 22.13* . Может ли свободный электрон поглотить фотон? 22.14**. Рентгеновское излучение с длиной врлны X, взаимодействуя с веществрмготклоняется от первоначального направления распрос- транения на угол 0. При этом длина волны рассеянного излучения увеличивается на ДХ (эффект Комптона). Выразите величину ДХ. через угол 0, рассматривая рассеяние как результат столкновений
167 Теория относительности и атомная физика рентгеновских фотонов с неподвижными свободными электро- нами. Учтите, что при этом взаимодействии электроны приобре- тают релятивистские скорости. 22.15**. Длина волны рентгеновского излучения после комптоновского рас- сеяния (см. задачу 22.14) увеличилась с Xi = 2,0 пм до Х2 — 2,4 пм. Определите кинетическую энергию Wk вылетающих электронов (выразите ее в МэВ) и их скорость V. Определите также угол рассеяния 0 рентгеновского излучения и угол а между направ- лением вылета электронов и направлением падающего излучения. 22.16**. Биметаллическая пластина изготовлена из двух тщательно отпо- лированных плоских пластин: серебряной и литиевой. Пластина размещена в вакууме, и на поверхность серебра падает нормально пучок монохроматического фиолетового света с длиной волны X = 0,40 мкм. Пластину развернули на 180°. Во сколько раз изменится действующая на нее сила? Считайте, что фотоэффект вызывается одним из 100 падающих на поверхность лития фото- нов и что фотоэлектроны вылетают нормально к поверхности с максимально возможной скоростью. 22.17. Движущаяся частица распадается на два фотона, которые летят в противоположных направлениях. Определите скорость о распав- шегося мезона, если энергии фотонов равны Wi и W2. 22.18*. Нейтральная частица распалась на два фотона, летящие под углами 01 и 02 к направлению движения частицы. Определите скорость v распавшейся частицы. 22.19*. Нейтральная частица, движущаяся со скоростью и, распадается на два фотона. Определите минимальный угол 0mm разлета этих фотонов. 22.20*. Покоящийся л-мезон распадается на мюон и нейтрино. Опре- делите скорость v образовавшегося мюона. Считайте, что нейтрино имеет нулевую массу покоя. Масса покоя тс-мезона mi = 273 те, а масса покоя мюона т2 ~ 207 тпе, где те — масса покоя электрона. 22.21. Определите де-бройлевскую длину волны Xi протона, прошедшего разность потенциалов U = 3,0 МВ; де-бройлевскую длину волны Х2 электрона, прошедшего ту же разность потенциалов.
Задачи 168 22.22. Пучок электронов, пройдя через узкую щель, дал на фотоплас- тинке такую же дифракционную картину, как монохромати- ческий свет с длиной волны X = 550 нм. Чему равна скорость v электронов? 22.23. Рассматривая электрон как классическую частицу, движущуюся в атоме водорода по круговой орбите вокруг неподвижного прото- на, выразите скорость о электрона и его механическую энергию W через радиус г орбиты. 22.24. Пользуясь полученными в задаче 22.23 результатами, определите радиус г орбиты электрона, при котором де-бройлевская длина волны электрона совпадает с длиной его траектории. Какую энер- гию Wj необходимо сообщить электрону в этом состоянии, чтобы ионизировать атом? Величину W\ выразите в электронвольтах. 22.25. Чему равен период Т полураспада изотопа, если за сутки распада- ется в среднем 750 атомов из 1000? 900 атомов из 1000? 1 атом из 1000? 22.26. Период полураспада радиоактивного йода-131 равен восьми сут- кам. За какое время t количество атомов йода-131 уменьшится в 1000 раз? 22.27**. В образцах урановой руды всегда содержится некоторое количес- тво атомов тория-234, образовавшихся в результате а-распада урана-238 (период полураспада Tv ~ 4,5*10® лет). Торий также радиоактивен (период полураспада ГТь = 24 сут). Сколько атомов тория содержится в образце урановой руды, в котором находится тп = 0,5 г урана-238? 22.28. Определите энергию связи £св и удельную энергию связи Есв/А для ядер Xi и 1.1А1. 22.29. Радиоактивный атом вГТЬ превратился в атом !J2Bi. Сколько при этом произошло а- и Р-распадов? 22.30. Какая энергия выделяется при реакции синтеза между дейтерием и тритием? Реакция сопровождается вылетом одного нейтрона. 22.31. Какую массу тп„ воды, взятой при 0°С, можно вскипятить, исполь- г.
169 Теория относительности и атомная физика зуя энергию термоядерного синтеза гелия из дейтерия и трития (см. задачу 22.30), если КПД преобразования энергии равен 10% ? Масса гелия т = 1 г. 22.32. При бомбардировке с помощью а-частиц бора Г В наблюдается вылет нейтронов. Напишите уравнение ядерной реакции, приво- дящей к вылету одного нейтрона. Определите ее энергетический выход W. 22.33. При бомбардировке нейтронами изотопа бора s°B образуются а-частицы. Напишите уравнение этой ядерной реакции и опре- делите ее энергетический выход W. 22.34. Найдите минимальную энергию W и частоту v гамма-кванта, способного «разбить» ядро дейтерия на протон и нейтрон. 22.35**. Найдите минимальную кинетическую энергию Wk протона, спо- собного «разбить» ядро дейтерия на протон и нейтрон. 22.36**. Какую минимальную кинетическую энергию Wo должна иметь а-час- тица для осуществления ядерной реакции ,,Li + *He -+ J°B + Jn? 22.37**. Ионы дейтерия (дейтроны), ускоренные до энергии Wi = 2,0 МэВ, направляют на тритиевую мишень. В результате реакции синтеза (см. задачу 22.30) из мишени вылетают нейтроны. Определите кинетическую энергию W2 нейтронов, вылетающих перпендику- лярно пучку дейтронов. 22.38**. Мишень из “Li подвергают бомбардировке нейтронами. В направ- лении движения нейтронов вылетают а-частицы с кинетической энергией Wa = 3,0 МэВ. Какова кинетическая энергия Wn нейтронов? 22.39*. Ядро iLi, захватывая протон с кинетической энергией Wi = 5,0 МэВ, распадается на две а-частицы с одинаковыми энергиями. Опре- делите кинетическую энергию W2 каждой из а-частиц и угол 0 разлета а-частиц. 22.40. Атомная электростанция мощностью Р = 1000 МВт имеет КПД 20%. Определите массу т расходуемого за сутки урана-235. Счи- тайте, что при каждом делении ядра урана выделяется энергия Wo = 200 МэВ.
Задачи 170 22.41. Сколько железнодорожных вагонов массой М = 60 т каждый можно было бы поднять на высоту h == 1 км за счет энергии, вырабатываемой АЭС (см. задачу 22.40) за сутки? На какую величину Azn уменьшается при этом масса ядерного горючего? 22.42* . Препарат S°Po массой т = 1,0 мг помещен в калориметр с теплоем- костью С = 8,0 Дж/К. В результате а-распада полоний превращается в свинец ™РЬ. На сколько поднимется температура в калориметре за время т = 1ч? Масса атома м°Ро равна 209,98287 а. е. м., масса атома 1ГРЬ равна 205,97447 а. е. м. Период полураспада полония Т = 138 сут.
ТЕСТЫ 6
Задачи 172 ТЕСТЫ С помощью предлагаемых ниже тестов вы сможете сами определить уровень своей подготовки. Выполняя тесты, можно набрать до 90 баллов. Мы заранее поздравляем тех, кому удастся набрать не менее трех четвер- тей этого количества: такой уровень соответствует, на наш взгляд, оценке «отлично». Тот, кто не сможет достичь этого уровня, но наберет более половины (45 баллов), заслуживает оценки «хорошо». Например, такой оценки заслуживает тот, кто ответил на вопросы всех тестов первого и второго уровня и хотя бы нескольких тестов третьего уровня. Наконец, для того чтобы получить оценку «удовлетворительно», надо на- брать не менее трети максимального количества баллов (для этого достаточно, например, выполнить все тесты первого уровня и половину тестов второго уровня). Сравнивая полученные вами ответы с приведенными вариан- тами ответов (стр. 187), учтите, что мы часто приводим округленные значения. В частности, чтобы не усложнять расче- тов, мы принимаем g = 10 м/с2. Предлагаемые тесты должны быть выполнены примерно за 6 часов. Если результат вас не удовлетворит, подумайте, к каким разделам курса физики вам стоит вернуться. Желаем успеха! Первый уровень (каждое задание оценивается в 1 балл) Т 1.1. Материальная точка движется вдоль оси х. Координата точки задается формулой х = 7 - 3/ + t2. Какой вид имеет зависимость скорости точки от времени? А. и» = -3t + t2. Б. и. = -3 + 2t. В. о, = 7 - 3t. Г. и, = -3 + t2- Д. Правильного ответа здесь нет. Т 1.2. Как изменится ускорение свободного падения при подъеме на высоту, равную диаметру Земли? Вращение Земли не учитывайте. А. Уменьшится в 2 раза. Б. Уменьшится в 3 раза. В. Уменьшится в 9 раз.
173 Тесты Г. Уменьшится в 4 раза. Д. Правильного ответа здесь нет. Т 1.3. Мяч массой 400 г плавает в воде, погрузившись в нее наполовину. Какая архимедова сила будет действовать на этот же мяч, если он будет плавать в керосине? А. 2 Н. Б. 3,2 Н. В. 4 Н. Г. 1,6 Н. Д. Правильного ответа здесь нет. Т 1.4. Определите ускорение, с которым движутся грузы ////// (см.рисунок). Массы грузов т — 500 г и М = 1,0 кг. _L Трением в блоке, массой блока и нити можно пре- небречь. А. 10 м/с2. m[j Б. 6,7 м/с2. В. 5 м/с2. Г. 3,3 м/с2. Д. Правильного ответа здесь нет. I—I Т 1.5. Велосипедист массой 60 кг проезжает со скоростью 36 км/ч середину подвесного мостика. Мостик под велосипедистом про- гнулся по дуге радиуса 20 м. Определите силу давления вело- сипедиста на мостик. А. 1200 Н. Б. 300 Н. В. 4,5 кН. Г. 600 Н. Д. Правильного ответа здесь нет. Т 1.6. Человек массой 80 кг бежит со скоростью 9 км/ч. Навстречу ему катится тележка массой 120 кг со скоростью 5 м/с. С какой скоростью будет двигаться тележка, если человек прыгнет в нее? А. 6,6 м/с. Б. 2 м/с, В. 0,6 м/с. Г. 4 м/с. Д. Правильного ответа здесь нет. Т 1.7. Какое количество теплоты необходимо, чтобы превратить 1 кг
Задачи 174 льда, взятого при температуре -10°С, в воду с температурой 10°С? А. 63 кДж. Б. 390 кДж. В. 42 кДж. Г. 370 кДж. Д. Правильного ответа здесь нет. Т 1.8. При изохорном нагревании на 6 К давление газа возросло на 2%. Какова была начальная температура газа? А. 200 К. Б. 250 К. В. 2.7°С. Г. 227°С. Д. Правильного ответа здесь нет. Т 1.9. Тепловая машина получила от нагревателя количество теплоты, равное 500 кДж, и передала холодильнику количество теплоты, равное 300 кДж. Каков КПД тепловой машины? А. 40%. Б. 67%. В. 25%. Г. 60%. Д. Правильного ответа здесь нет. Т 1.10. Металлической сфере радиусом 10 см сообщили заряд 2 нКл. Чему равна напряженность электрического поля на расстоянии 5 см от центра сферы? А. 1,8 кВ/м. Б. 3,6 кВ/м. В. 0. Г. 7,2 кВ/м. Д. Правильного ответа здесь нет. Т 1.11. На лампочке написано: 12 В, 0,3 А. Ее включают в сеть с напряжением 36 В последовательно с резистором. При каком сопротивлении резистора лампочка будет гореть нормальным на- калом? А. 40 Ом. Б. 120 Ом. В. 60 Ом. Г. 80 Ом. Д. Правильного ответа здесь нет.
175 Тесты Т 1.12. Электрон влетает в однородное магнитное поле, двигаясь парал- лельно линиям магнитной индукции. Как будет двигаться элект- рон? А. Прямолинейно, с положительным ускорением, скорость увели- чивается. Б. Равномерно прямолинейно. В. Прямолинейно, скорость уменьшается. Г. По окружности. Д. Правильного ответа здесь нет. Т 1.13. В колебательном контуре радиоприемника индуктивность ка- тушки 40 мкГи, а емкость конденсатора может изменяться от 25 до 300 пФ. На какую наименьшую длину волны можно настроить приемник? 600 м. Б. 300 м. В. 180 м. Г. 20 м. Д. Правильного ответа здесь нет. Т 1.14. Когда фотоны с энергией 4,0 эВ падают на поверхность металла, максимальная кинетическая энергия выбитых ими электронов равна 1,5 эВ. При какой минимальной энергии фотона возможен фотоэффект для этого металла? А. 1,5 эВ. Б. 2,5 эВ. В. 4,0 эВ. Г. 5,5 эВ. Д. Правильного ответа здесь нет. Т 1.15. При радиоактивном распаде ядра испускаются три а- и две Р-частицы. Какое ядро образуется в результате распада? a. rm. Б. 8TRa. В. ^Fr. Г. й’и. Д. Правильного ответа здесь нет.
Задачи 176 Второй уровень (каждое задание оценивается в 2 балла) Т 2.1. На рисунке приведен график ско- р рости прямолинейного неравно- мерного движения. В какой мо- мент времени тело имеет макси- XT's мальное ускорение? Максималь- / \ ную скорость? Максимальную ко- / \ ординату? /1 I \ A. tl, t2, <3. | I_____\ Б. <з, tz, ti. ° t, Т, t\\^/t5 * В. tl, t2, ti. Г. tz, ts, ti. Д. Правильного ответа здесь нет. Т 2.2. Массы показанных на рисунке грузов m = 1 кг и М = 2 кг. Угол наклона плоскости а = 30°, коэффициент трения между плоскостью и находящимся на ней грузом ц = 0,1. Какова сила натЯ"- жения нити? А. 10 Н. Б. 3,3 Н. В. 5 Н. Г. 13 Н. Д. Правильного ответа здесь нет. Т 2.3. Для растяжения пружины на 1 см потребовалось совершить работу 0,2 Дж. Какую работу надо совершить, чтобы растянуть пружину еще на 1 см? А. 0,2 Дж. Б. 0,4 Дж. В. 0,6 Дж. Г. 0,8 Дж-. Д. Правильного ответа здесь нет. Т 2.4. Как изменяется потенциальная энергия системы«шар-вода»при: 1) всплытии Пробкового шара; 2) погружении стального шара? А. Уменьшается в обоих случаях. Б. Увеличивается в первом случае, уменьшается во втором.
177 Тесты В. Остается неизменной в обоих случаях. Г. Увеличивается в обоих случаях. Д. Правильного ответа здесь нет. Т 2.5. По шнуру бежит вправо поперечная гар- моническая волна (см. рисунок). Как направлены скорости точек шнура А, В, С, D? А. Скорости всех точек направлены впра- во. Б. Скорости точек А и В направлены вниз, скорости точек С и В — вверх. В. Скорости точек Ви D равны нулю, скорость точки А направлена вниз, скорость точки С — вверх. Г. Скорости точек А и С равны нулю, скорость точки В направлена вверх, скорость точки D — вниз. Д. Правильного ответа здесь нет. Т 2.6. Плотность газа при нормальных условиях (0°С, давление 101 кПа) равна 1,96 кг/м3. Какой это может быть газ? А. Воздух. Б. Углекислый газ. В.- Кислород. Г. Гелий. Д. Правильного ответа здесь нет. Т 2.7. Как изменились бы абсолютная температура Т и давление р газа в герметично закрытом баллоне, если скорость каждой молекулы увеличилась бы вдвое? А. Т и р увеличатся вдвое. Б. Т увеличится в два раза, р — в четыре. В. Т увеличится в четыре раза, р — в два. Г. Т и р увеличатся в четыре раза. Д. Правильного ответа здесь нет. Т 2.8. Какое расстояние проедет автомобиль массой 4 т, израсходовав 5 л бензина? КПД двигателя 30%, коэффициент сопротивления дви- жению 0,05. А. 34,5 км. Б. 270 км. В. 42 км. Г. 80 км. Д, Правильного ответа здесь нет.
Задачи 178 Т 2.9. Как выглядит график данного процесса (см. рисунок) в координатах р, Т2 А. рис. а Б. рис. б В. рис. в Г. рис. г Д. Правильного ответа здесь нет. Т рис. а рис. в рис, б рис. г Р 4 3 1 2 Т Т 2.10. Четыре заряда расположены в вершинах квадрата со стороной 10 см (см. рисунок). Какова равнодействующая приложенных к каждому из зарядов кулоновских сил, если q = 5 нКл? А. 34 мкН. Б. 21 мкН. В. 43 мкН.
179 ТеСты Г. 56 мкН. Д. Правильного ответа здесь нет. +Q -Q -q +q К тесту 2.10 К тесту 2.12 Т 2.11. На одной лампе написано 120 В, 60 Вт; на другой 36 В, 40 Вт. Какую общую мощность будут потреблять эти лампы, если их соединить последовательно и подключить к сети с напряжением 36 В? Зависимостью сопротивления ламп от их накала пренебречь. А. 4,8 Вт. Б. 100 Вт. В. 24 Вт. Г. 45 Вт. Д. Правильного ответа здесь нет. Т 2.12. К показанной на рисунке цепи приложено напряжение 30 В. Чему равна сила тока через резистор Rs, если сопротивление каждого из резисторов 12 Ом? А. 0,5 А. Б. 1 А. В. 1,5 А. Г. 2,5 А. Д. Правильного ответа здесь нет. Т 2.13. На экране с помощью собирающей линзы получено увеличенное в 2 раза изображение предмета. Оптическая сила линзы 5 дптр. Каково расстояние от предмета до экрана? А. 20 см. Б. 40 см. В. 60 см. Г. 90 см. Д. Правильного ответа здесь нет.
Задачи 180 Т 2.14. Угол падения луча на поверхность плоскопараллельной плас- тинки равен 60°. Толщина пластинки 1,73 см, показатель прелом- ления 1,73. На сколько смещается вышедший из пластинки луч? А. На 3 см. Б. На 1,2 см. В. На 1 см. Г. На 0,87 см. Д. Правильного ответа здесь нет. Т 2.15. Период полураспада радиоактивного изотопа равен 4 ч. Какая часть атомов распадется за 12 ч? А.1. в4 в-1 г4 Д. Правильного ответа здесь нет. Третий уровень (каждое задание оценивается в 3 балла) Т 3.1. Две звезды, имеющие одинаковые массы М, вращаются вокруг общего центра, находясь на постоянном расстоянии R друг от друга. Каков период их обращения? А JGM' дз/2 <GM' Д. Правильного ответа здесь нет. Т 3.2. Протон после упругого лобового соударения с неподвижным ядром отлетел назад со скоростью, составляющей 60% от начальной. С каким ядром он столкнулся? *
181 Тесты А. ?Н. Б. 1Не. В. “Li. Г. Ше. Д. Правильного ответа здесь нет. т з-.з. Тело, брошенное вертикально вверх, побывало на высоте 50 м дважды с интервалом в 3 с. Каково полное время полета тела? А. 6 с. Б. 6,2 с. В. 7 с. Г. 9,3 с. Д. Правильного ответа здесь нет. Т 3.4. Какую минимальную работу необходимо совершить, чтобы пере- вернуть на другую грань однородный куб массой 3 т с длиной ребра 1 м? А. 30 кДж. Б. 6,2 кДж. В. 42 кДж. Г. 15 кДж. Д. Правильного ответа здесь нет. Т 3.5. В стоящий на столе цилиндрический сосуд налита вода до высоты Н. На какой высоте Л следует сделать маленькое отверстие в стенке сосуда, чтобы струя воды попадала на стол как можно дальше от сосуда? Каково при этом расстояние I от сосуда до точки попадания струи на стол? A. h = ~ I = V2 Н. £ > Б. h = ~ I = V2 Н. V2? В. h = 0, I = Н. Г. h = ^-,l = Н. Li Д. Правильного ответа здесь нет. Т 3.6. Два одинаковых маленьких шарика подвешены вплот- ную друг к другу на нитях длиной 1 м и 25 см (см. рисунок). Первый щарик отводят немного в сторону и отпускают без начальной скорости. Сколько столкно- вений шариков произойдет за 3,3 с? Все столкновения — лобовые и упругие. nil и 1Й 2
Задачи 182 А. 2. Б. 3. В. 4. Г. 5. Д. Правильного ответа здесь нет. Т 3.7. С помощью кипятильника мощностью 300 Вт не удается довести до кипения воду массой 1,2 кг из-за теплообмена с окружающей средой. Когда температура воды перестает увеличиваться, кипя- тильник выключают. На сколько понизится температура воды за следующую минуту? А. На 7°С. Б. На 5°С. В. На 2°С. Г. На 1°С. Д. Правильного ответа здесь нет. Т 3.8. На рисунке показано положение столбика ртути длиной 10 см в вертикально расположенной трубке длиной 90 см. Трубка герметично закрыта с обоих концов. Если ее расположить горизонтально, столбик ртути перемес- тится в середину трубки. Каким станет при этом дав- ление в трубке? Температуру считайте постоянной. А. 23 см рт. ст. Б. 38 см рт. ст. В. 10 см рт. ст. Г. 19 см рт. ст. Д. Правильного ответа здесь нет. Т 3.9. В калориметр, содержащий 300 г кипятка, бросили 100 г льда, имеющего температуру —20°С. Определите уста- новившуюся в калориметре температуру, если теплоем- костью калориметра можно пренебречь. А. 0°С. Б. 33°С. В. -2°С. Г. 72,5°С. Д. Правильного ответа здесь нет. Т 3.10. Какое максимальное напряжение можно приложить к показанной на рисунке батарее из одинаковых конденсаторов, если каждый конденсатор выдерживает напряжение 500 В?
183 Тесты. А. 1500 В. Б. 1000 В. В. 750 В. Г. 500 В. Д. Правильного ответа здесь нет. Т 3.11. Чему равна сила тока через ре- зистор Ri (см.рисунок)? Сопро- тивления всех резисторов оди- наковы и равны 16 Ом. ЭДС источника тока 3,6 В, внутрен- нее сопротивление 2 Ом. А. 0,19 А. Б. 0,225 А. В. 0,50 А. Г. 0,30 А. Д. Правильного ответа здесь нет. Т 3.12. Внутреннее сопротивление источника тока 3 Ом, его ЭДС равна 9 В. Как нужно соединить между собой четыре одинаковых резистора сопротивлениями по 5 Ом, чтобы при их подключении к этому источнику во внешней цепи выделялась максимально возможная мощность? А. рис. а Б. рис. б В. рис. в Г. рис. г Д. Правильного ответа здесь нет. рис, в рис, г
Задачи 184 Т 3.13. Колебательный контур с периодом колебаний 1 мкс имеет индук- тивность 0,2 мГн и активное сопротивление 2 Ом. На сколько процентов уменьшается энергия этого контура за время одного колебания? В течение этого времени ток можно считать синусо- идальным, потерями энергии на излучение пренебречь. А. На 0,001%. Б. На 0,01%. В. На 0,1%. Г. На 1%. Д. Правильного ответа здесь нет. Т 3.14. Дальнозоркий человек читает без очков, держа книгу на расстоя- нии 50 см от глаз. Какова оптическая сила необходимых ему очков для чтения? А. +2 дптр. Б. +6 дптр. В. +4 дптр. Г. -2 дптр. Д. Правильного ответа здесь нет. Т 3.15. Сколько энергии выделяется (или поглощается) при ядерной реакции Ше + "Be -> eT! + Jn? А. Поглощается 5,7 МэВ. Б. Выделяется 5,7 МэВ. В. Выделяется 14 МэВ. Г. Поглощается 14 МэВ. Д. Правильного ответа здесь нет.
ОТВЕТЫ УКАЗАНИЯ РЕШЕНИЯ
ИНТОР Научно-практический центр «Инновации в науке, технике, образовании». • Издаёт учебно-методическую литературу • Осуществляет подготовку учителей развивающего обучения (система Элъконина-Давыдова) • Проводит тематические семинары по психологическим проблемам обучения в системе Элъконина-Давыдова Адрес центра 121069, г.Москва, ул.Поварская, 14 8 (095) 290-51-84, (095) 290-35-58
ОТВЕТЫ НА ТЕСТЫ Т 1. 1. Б. Т 2. 1.А. Т 3. 1. Г. Т 1. 2. В. Т 2. 2. А. Т 3. 2. Б. Т 1. 3. В. Т 2. 3. В. Т 3. 3. В. Т 1. 4. Г. Т 2. 4. А. Т 3. 4. Б. Т 1. 5. Д. Т 2. 5. Г. Т 3. 5. Г. Т 1. 6. Б. Т 2. 6. Б. Т 3. 6. В. Т 1. 7. Б. Т 2. 7. Г. Т 3. 7. Д. Т 1. 8. В. Т 2. 8. Д. Т 3. 8. Г. Т 1. 9.А. Т 2. 9. В. Т 3. 9. Д. Т 1.10. В. Т 2.10. Б. Т 3.10. В. Т 1.11. Г. Т 2.11. А. Т 3.11. А. Т 1.12. Б. Т 2.12. Б. Т 3.12. Г. Т 1.13.Д. Т 2.13. Г. Т 3.13. Г. Т 1.14. Б. Т 2.14. В. Т 3.14. А. Т 1.15. Б. Т 2.15. Г. Т 3.15. Б.
Ответы, указания, решения 188 1.1. Ответ: а) 48 км/ч; б) 75 км/ч. Решение, а) По определению, средняя скорость где з — весь йуть, at — время всего движения. Время прохождения первой S S половины пути ti - , второй половины ta = 5—• Значит, t = ti+ &V1 4V2 s(Vi + O2) ,q . + tz = —, и v,v =-----------= 48 км/ч. 2uiV2 Vi + O2 t ,б) В этом случае ti = t> = На лошади барон проехал расстоя- Oit Uzt _ ние Si = —, а с лошадью на плечах он прошел Зг = -х~. Весь путь Л л 01 + V1 s = Si + 32 = —л—t, значит, Л Vi + V2 о.-,, = —г— = 75 км/ч. Почему же в случае а средняя скорость оказалась существенно меньше? Причина проста: в.этом случае барон нес лошадь на себе большую часть времени (7 = — = 4), ав случае б — лишь половину времени. 1.2. Ответ: в озере. Решение. Пусть расстояние з в стоячей воде мы проплываем со скоростью v, а скорость течения реки равна и. Тогда, чтобы 2з „ проплыть туда и обратно по озеру, потребуется время ti = —. По течению реки мы движемся со скоростью и + и, против течения со скоростью v - и. Поэтому, проплыв туда и обратно по реке, мы з з 2sv затратим время t2 =-------+------= —----- V + и V - и V - и Л ta V2 Очевидно, отношение — = —------- превышает единицу, т. е. tz > ti. tl v2 - и2 Итак, для движения по реке времени потребуется больше. Зна- чит, выигрыш во времени при движении по течению меньше, чем потеря времени при движении против течения. Так и должно быть: ведь против течения мы движемся большую часть времени (сравните с задачей 1.1). Полученный вывод особенно нагляден, когда v лишь ненамного превышает и: в этом случае движение против течения будет очень медленным и займет много времени. 1.3. Ответ: 40 км/ч. Решение. Средняя скорость автомобиля на второй половине пути Vz + Va _ составляет —~—. Полное время t прохождения пути з:
189 Механика s s 2vi + V2 + Va t — — +-------------— s--------------. 4V1 „(Пз + u-0 2ui(l>a + Оз) 2 2 s 2vi (v2 + Оз) . Тогда средняя скорость на всем пути vq, = - = —----------------= 40 км/ч. t 2vi + иг + из 1.4. Ответ: и = 40 км/ч. Решение. Представим себя на месте пассажира одного из поездов (т. е. перейдем в его систему отсчета). С точки зрения этого пассажира оба поезда стоят на месте на расстоянии з = пт друг от друга, а барон бежит со скоростью и + v, преодолевая расстояние з за время t. x—t Значит, их = (и + v)t, откуда v - = 40 км/ч. Это ненамного больше мировых рекордов в спринте, но ведь барон всего лишь разминался! 1.5. Ответ: = 6 ч. Решение. Обозначим расстояние между пунктами А и В через s, скорость моторной лодки относительно воды V, скорость течения S S реки (т. е. скорость плота) и. Тогда t = —, ti ---. Отсюда. и v + и о = и(£-1), s = ut. Обратный путь займет время: s ut tti . ta = --- = ---------- = --~ = 6 Ч. v - и it \ #-2ti “(к-1)-" Заметим, что величины и, и, s остались неизвестными. Для решения задачи оказалось достаточным установить связи между этими величинами. Полученное решение имеет смысл лишь при t > 2ti. Советуем проверить, что это-просто означает v > и. 1.6. Ответ: 45 с. Решение. Обозначим длину эскалатора з, его скорость и, скорость идущего человека относительно эскалатора и. Получим: з з v = —. и = —. ti’ ta На движение по поднимающемуся эскалатору потребуется время s s № 3 <з-=-------------=-----= -т мин = 45 с. v + и — + — ti + ta 4 ti t2 Таким образом, можно сэкономить целых 15 с!
Ответы, указания, решения 190 1.7. Ответ: 1,5 мин. 1.8. Ответ: а ) Л = 6,5 ч; б ) t> = 6 ч. Решение, б ) Скорость самолета ui от- носительно Земли — это векторная сум- ма скоростей v и и. Согласно условию, скорость vi должна быть направлена V, вдоль отрезка АВ, и — под прямым углом к этому отрезку. Поэтому v должна быть направлена под углом к АВ, чтобы компенсировать боковой снос (см. рисунок). Скорость Vs = ^v2 - и2, она одинакова при движении туда и обратно. Полное время полета составит: 1.9. Ответ: и = 14 м/с. Решение. Перейдем в систему отсчета, связанную с движущейся каретой. В ней скорость капель, равная v - и, наклонена под углом а к горизонту (см. рисунок). Из треугольника скоростей полу- чаем: v = и • tga = 52 км/ч = 14 м/с. / 1.10. Ответ: в Л /1 -I—а раз.’ 1.11. Решение. Если лодка будет держать курс перпендикулярно берегам, то ее скорость относительно берега v + и будет направлена под углом к отрезку АВ (см. рис. а). Лодку снесет от точки В вниз по течению на расстояние I. Это расстояние легко определить I и из подобия треугольника скоростей и треугольника АВС: - = —, SU откуда I = — - 120 м. Течение реки в этом случае вообще не Рис. а Рис. б
191 Механика влияет на время переправы: t> = — - 40 с. Чтобы попасть в точку В, следует держать курс под углом а = arcsin ~ = 37 ° к отрезку АВ (вверх по течению, см. рис. б). g Время переправы в этом случае t2 = , ,— , = 50 с (см. решение NIT - U задачи 1.8). Переправа во втором случае занимает больше вре- мени, хотя и происходит по более короткому пути! 1.12. Ответ: 4 км/ч; на второй вопрос ответить нельзя. Решение. Наиболее простое решение получим, если перейдем в систему отсчета, связанную с плывущей шляпой. В этой системе отсчета вода неподвижна; лодка же сначала удаляется от шляпы, а затем с той же скоростью приближается к ней. Значит, с момента поворота до момента «поимки» шляпы лодка двигалась тоже полчаса. Всего же шляпа плыла по течению час. За это время она проплыла 4 км; следовательно, скорость течения реки 4 км/ч. Заметим, что скорость лодки нам неизвестна, но для ответа на поставленный вопрос знания этой скорости и не требуется. То же самое можно сказать о первоначальном направлении движения лодки. 1.13. Ответ: t = —; Z; s„„„ = . I. Ui + и2 yvi + v2 Решение 1. Определим расстояние между авто- мобилями через йремя t после прохождения пер- \ вым автомобилем перекрестка. Это расстояние у (см. рис. а) составит s = ^/(oit)2 + (I - иг()2. Оста- \|<^| ется определить минимальное значение функции — —X - s(t) при t > 0. Это можно сделать, например, А В вычислив производную s'(t). Приведем и другой *'' Рис. а способ — выделение полного квадрата в подко- ренном выражении: s = "^(ui2 + щ2)*2 - 2lv2t + 1г = ЛЛо/ + V22) (t -.~ + F—r1—; • V ' Vi + V2 ’ Vi + U2 , Л>2 Минимум этого выражения достигается, очевидно, при t = —---- Vl2 + V22 lVi и равен Sjnin — . NUl2 + V22 Решение 2. Заметим, что задачу усложняет одновременное движе- ние обоих автомобилей. Поэтому решение намного упростится,
Ответы, указания, решения 192 если связать систему отсчета с одним из них. Свяжем систему отсчета с первым автомобилем. В ней Vi = О, V2 = vz - vi (см. рис. б). Очевидно, траек- тория движения второго автомобиля в этой системе отсчета представляет со- бой прямую ВС, а минимальное рас- стояние Smin — длину перпендикуляра AD к этой прямой. Из подобия треу- гольника скоростей и треугольника л гыч 8„.|„ Vi BD 1>2 ABD следует: — = —; —р- = — . Отсюда I г 2 L г 2 _ Ivi _ Ivi _ BD _ Ivz _ Ivz ^2 + Vz ’ ^2 V]2 + Vz 1.14. Ответ: v = 8,3 м/с; pi = p2 = 11,5°. Решение. Скорость катера относительно берега равна V - и + и. Она направлена вдоль отрезка АВ. Проецируя равенство V = v + и на отрезок АВ и перпендикулярное ему направление, получаем: V, = vcosPi + ucosa; usina = vsinPi, где Vi — скорость катера при движении от А к В. При движении’катера обратно от В к А: Vz = vcosp2 - ucosa; usina = vsinp2. Отсюда видно, что Pi = р2 = р. Полное время движения t = ~ . Отсюда У1 vz y2cos Р - u cos a_____ 2s s I s%~ (vcos P)2 - — • vcos p - u2cos2a = 0; vcosp = - +‘\J^z + u2cos2a. Из системы двух уравнений {vsin Р - usin a, _ s + ^s2 + u2t2 cos2a vcos p =----——---------- . r, ut sin a находим tgp =----, ~ = , откуда p = ll,o°. Следователь- s + ns2 + u2t2 cos2a u sin a o „ ,. но, v =-------- 8,3 м/с. sin p 1.15. Ответ: река течет co скоростью 0,51 км/ч по направлению от В к А. Решение. Предположим, река течет со скоростью и от А к В. Учитывая, что время движения катера равно времени движения лодки, получаем —— = 2(—— + ~М, где s = АВ. Vi + U 'Uz + и Vz - и’
19& . -Механика О.тсюдаи22 - иг = 4uiVs + 4uv2, т. е. и2 + 4ии2 + 4viv2 - v22 = 0. Сле- довательно, и = - 2vs ± V5v22 - 4uiV2 = - 20 ± 19,49 (км/ч). Решение и = - 39,5 км/ч следует отбросить (тогда катер не смог бы подняться против течения); остается единственное решение: и = - 0,51 км/ч (река течет от В к А со скоростью 0,51 км/ч). 1.16. Ответ: спустя время l(di + d2 + ds) v(di d2) догорит вторая свеча, а Z(di + di + ds) спустя время-------з----- — и первая. Решение. Пусть за время А/ длина первой свечи уменьшилась на ДЛ1, а второй — на Дй2 (см. рисунок). Тень на правой стене опустилась за это время на иМ. Заштрихованная на рисунке фигура содержит три подоб- ных треугольника. Из их подобия следует: ДЛ1 _ ДЛ2 у М di di + ds di + d2 + ds Отсюда находим скорость укорачивания каждой из свечей: АЛ1 di \h2 di + d2 ---= v ----------,----= у -------— . At di + ds + ds At di + d2 + ds Время полного сгорания каждой из свечей: I М l(di + d2 + ds) I • At l(di + d2 + ds) ДЙ1 vdi ’ 2 дй2 y(di + ds) Естественно, ta < ti — первой сгорит правая свеча. , . _ п . l(di + d2 + ds) 1.17. Ответ: t, = —---------------7-; t2 usdi - vi(a2 + ds) l(di + ds + ds) v/di + ds) - Vi ds’ 1.18. Решение. Очевидно, движение по пути АВ (кратчайшему) может потребо- вать далеко не кратчайшего времени из-за медленного движения в воде. Вы- годнее может оказаться войти в воду в точке D (см. рисунок), сократив путь в воде даже за счет удлинения пути по берегу. Определим, при каком значе- нии DC — х время движения будет ми- нимальным. Это время:
Ответы, указания, решения 194 _ з - х + >Jd2 + х2 _ svi - xvr + v^d2 + х2 V2 Vi ViVi Очевидно, величина у = v2^d2 + x2 - viX должна быть мини- мальной (заметим, что эта величина уже не содержит s). Далее можно пойти различными путями. 1. Для умеющих дифференцировать: , xv2 rt dvi » = -SFT? " = °-отк,да * - • 2. Более длинный путь, но не требующий дифференцирования — выразим х через у’. v2^d2 + х2 = у + xvi; v2(d2 + х2) = у2 + 2xyu + х2 vi2; х2(и22 - и2) - О 2J2 2 П Л yvi ± V2 ^У2 - (l>22 - Vl2) d2 - 2xyvi + v2 d - у = О. Отсюда х = -------------------- v22 - Vi2 Минимальное значение у, при котором подкоренное выражение неотрицательно: утт = d4v22 - у/; при этом х - . У1>2 -V12 „ dvi , _ При s > г-~ ...... следует двигаться по берегу до точки D, а VO22 - О1 ; т. ~ , dvt потом плыть к В. При з < . —следует сразу плыть по прямой \1>2 - О1 к точке В. Читатель, знающий оптику, может воспользоваться аналогией с «движением» светового луча (принцип Ферма). Тогда угол а на рисунке определяется как предельный угол полного Vi ,, dvi отражения: sina = —, а х = atga = -=======. Vi . Уи22- - у/ 1.19. Ответ: 37° < 0 < 143° (см. рисунок); и2пЛп = ^- = 2,4 м/с. Следует бежать перпендикулярно направлению на автобус. Решение 1. Пусть автобус находится в точке А, человек — в точке В. Определим, под каким углом 0 к линии АВ может бежать человек (он должен попасть в точку С одновременно с автобусом АС или раньше его); время движения автобуса h = — , время дви- V1 v. ВС жения человека t2 = < ti. Отсюда АС и, 57Г > — • Применяя теорему синусов к Uv V2 треугольнику АВС(^ 5^'^) и учи'.: С
195 Механика d „ Vid _ тывая, что sina = —, получаем: sinp > Тогда arcsin < В < 180° - arcsin ; 37° < р < 143°. \v2S>. \ViS> Хотелось бы обратить внимание на то, что в этой задаче самым трудным является удачный выбор неизвестной величины р. тт • п Vi(l Поскольку sinp > условием разрешимости задачи является vid Pid Vvd „ . t • тт ---< 1 или иг >----. Значит, v^n =----= 2,4 м/с. При такой ско- V2S S S рости sin р = 1, р = 90° — т. е. бежать следует под прямым углом к направлению на автобус (а не к дороге!). Решение 2. Решение задачи становится проще и нагляднее, если перейти в систему отсчета, связанную с автобусом. В этой системе отсчета скорость человека V= v2 - Pi. Величина скорости v роли ие играет, а ее направление должно быть таким, чтобы человек пересек шоссе правее «стоящего» автобуса или вышел точно на автобус (см. рис. а). Конец вектора V лежит на окружности радиуса и2 (см. рис. б). Очевидно, угол р между отрезком АВ и направлением р2 должен лежать в пределах Pi < Р < Ра, где Pi и Ра . а U1 — два решения уравнения sinp = sina • —, удовлетворяющие усло- v2 •> вию 0 < Р < п. 1.20. Решение. Пусть автобус находится в точке А. Выберем произвольную точку В на шоссе впереди автобуса. Пусть АВ — I. I Автобус доедет до точки В За время t = — . Чтобы человек мог встретить автобус в этой точке, он должен находиться от нее на v2 расстоянии, не превышающем Я = v2t = I— (т. е. в предела& пока- У1 занного да, рисунке а круга).1 Чтобы рпредадит^4 кскбмоб 'геб^ 7» ' - -
Ответы, указания, решения 196 метрическое место точек, построим аналогичные круги для всех точек шос- се впереди точки А и на- йдем их объединение, т. е. множество точек, каждая из которых принадлежит хотя бы одному кругу. Это совсем не так сложно, как кажется на первый взгляд. Рассмотрим угол а между линией шоссе и касательной, проведенной из точки А к построен- ной окружности. Очевидно, Л Ог sma - -г - — , причем этот угол одинаков для всех построенных I Vi окружностей! Следовательно, все они вписаны в один и тот же угол 2a. Объединение всех построенных кругов представляет собой внутреннюю область угла 2a (рис. б). 1.21. Ответ: v = —-—'Ivi2 + v2 + 2oiV2Cos a . sm a Решение. Сравним положения прямых в начале и в конце проме- жутка времени At. Штриховой линией на рис. а показаны началь- ные положения, сплошной — конечные. За это время точка пересечения прямых переместилась из А в В. Перемещение з этой точки представляет собой диагональ изображенного на параллелограмма. Как видно из рис. б, перемещение определить, воспользовавшись теоремой косинусов: з = >/ui2 + о22 + 2V1V2COS a . sina рис. а можно
197 Механика 11 1 ----------------------------------------------------— Скорость точки пересечения прямых s a!vi2 + о? + 2uiV2COS а v = — =--------------------. А/ sin а Заметим, что эту скорость можно неограниченно увеличивать за счет уменьшения угла а. и 1.22. Ответ: а = 180° при v < и; а = arcsin — при v > и. Решение. При и < и угол а, очевидно, равен 180°: один из осколков может полететь назад со скоростью и - v. При v > и найти а сложнее. Перейдем в систему отсчета, движущуюся со скоростью V (так называемую систему центра масс). В ней осколки имеют одинаковую скорость, по модулю равную и. Направление же этой скорости может быть произвольным. На рис. а показаны возмож- ные положения конца вектора и. В «неподвижной» системе отсче- та скорость осколка V = и + и. Множество возможных положений конца вектора V заполняет окружность радиусом и, причем вектор v заканчивается в центре этой окружности (см. рис. б). Начало вектора v окажется вне окружности при v > и. Какой же отрезок, соединяющий точку А с точкой окружности, образует наибольший угол с вектором п? Очевидно, касательная к окружности. Значит, и а = arcsin —. V — и 1.23. Ответ: 0 < р < arccos —------ при и < и; Р = 90° при и = v; v + и2 , V2 — и1 arccos ------- < р < 180° при и > v. и + U2 Решение. Если в системе центра масс скорости осколков и и -и, то в неподвижной системе отсчета их скорос^иир1+ и и v - и (см. рис. а). Угол Р между векторами v + и и v - и можно определить
Ответы, указания, решения 198 (и + и) (и — и) из,соотношения созВ = :—*----г- Воспользуемся тем, что для |v + и| |v - и| любого вектора а его модуль а = ^аа. Тогда и2 — и2 cosB = “ — - и2 ~ц2 COS V(v2 + и2 +2и • v)(v2 + и2 - 2и • и) ^v2 + и2 )2 - 4 (и • v)2 ’ Посмотрим теперь, в каких пределах может изменяться вели- чина cos р. Удобно рассмотреть отдельно случаи: 1) v > и, 2) и = = и и 3) v < и. 1. Если и> и, то cosP > О, т. е. угол р меньше 90 ° (как раз такому случаю соответствует рис. а). Поскольку cosp является убывающей функцией Р, наименьшему значению р соответствует наибольшее значение cosP, а наибольшему значе- нию Р — наименьшее значение cosp. Единственной величиной в выражении для cosp, зависящей от направления и, является (и • v), и благодаря этому легко найти наибольшее и наименьшее зна- чения cosp: наибольшее (cosp = 1) дос- тигается при и || V, когда (и • v) = uv, а / О2 — и2\ наименьшее cosp = —------I при и 1 V, ' V2 + и2' когда (и • и) — 0. Следовательно, угол р изменяется в пределах от 0 до iv2 -и2\ arccos I—----I . На рис. б показан слу- 'V2 + и2’ чай, когда достигается наибольшее зна- чение угла разлета осколков. 2. Если v = и, то cosp = 0, т. е. Р = 90° независимо от направления и. Отметим только частный случай u || v: при этом один из осколков останавливается, и поэтому вопрос об угле разлета оскол- ков лишается смысла. 3. Если и < и, то cosp < 0, т. е. угол Р больше 90° (см. рис. в). Однако cosp является убывающей функцией р и при 90° < р < 180°, поэтому мы можем вос- пользоваться теми же соображениями, что при разборе случая 1. Результат, однако, получится совсем другим: •РивЧ в
199 Механики ’ Ц? — IXs наибольшее значение cpsp =? --- (оно отрицательно!) достига- ' V + иг ется при и 1 V, а наименьшее (cos0 = -1) при и || v (один осколок летит вперед, а другой — назад). Следовательно, угод (3 измена- / и2 — цау; ется в пределах от arccos —-тг до 180°. 'u2 + lT'f Интересно заметить, что «симметричный» случай (w 1 и) при v > и соответствует максимальному углу разлета, а при v < и — минимальному. 1.24. Решение 1. Обозначим ускорение а, длительность каждого из промежутков времени т. Совместим начало координат с началь- ной точкой движения И ось х с направлением движения. Тогда координата теда в конце n-го промежутка времени в(пт)2 •• .Хп = —= .1» 2, 3... ). Изрисунка а видно: .Л ат2 - ат2/ 2 . . ат2 1, ' Si ~ *2"» з„ - х„ - хп-1. = -у-(п2 - (л - Д)) = - 1). «i s. St 8 О х. х Рис. а Отсюда 8i : S2 : ... : sn = 1 : 3 : ... : (2п - 1). Решение 2. График зависимости скорости от времени v(t) для рассматриваемого движения показан на рисунке б. Воспользуемся тем, что перемещение тела за очередной промежуток времени т численно равно площади фигуры Нод соответствующим участком
Ответы, указания, решения 200 графика. Эти фигуры (см. рис. в) можно разбить на треугольники с площадью si, причем каждая следующая фигура содержит на два треугольника больше предыдущей. Поэтому получаем: si : 8з : вз : 84 : ... = 1 : 3 : 5 : 7 : ... 1.25. Ответ: Н = 200 м. Решение. Пусть полное время падения тела t. Тогда Н = Н - s = & ~ Отсюда Z Z з = f(t2 - (t - г)2) = ((2/ • г - г2), t = | . Отсюда Н = = 200 м. 1.26. Ответ: Н = 57 м, t = 3,4 с. 1.27. Ответ: поезд пройдет вдвое большее расстояние. Решение. Пусть скорость поезда в момент отцепления вагона vo, время движения вагона t. Тогда поезд за это время пройдет путь зп = vot. Средняя скорость движения вагона за n Uo + 0 Vo это время ’ Оср = —g— = . поэтому пройденый вагоном путь Vot 8» •“ Vcp ' t ~ л • Отсюда 8п = 2зв. Поезд пройдет путь вдвое больший, чем вагон. Этот результат становится совершенно очевидным, если построить зависимость скорости от времени для поезда и вагона (см. рисунок) и вспомнить, что перемещение численно равно площади под графиком v(t). 1.28. Ответ: Vo = 45 см/с; в = 30 см/с2. Решение 1. Используем формулу зависимости перемещения от времени при равноускоренном движении: , х ati2 , х at22 I -- foil-«у- ; I - votz —5- . z z Приравнивая правые части обоих выражений, получим a(tz + ti) v°~ 2 • ” Средняя скорость прямолинейного равноускоренного движения равна среднему арафметинескНму.нвфгатймконеннейюкарост^й.э^яЛ^тпэу эн
201 Механика Подставляем это выражение для vo в первое уравнение системы: ati(t2 + ti) _ ati2 _ atita 2 ” 2 “ 2 * _ 2/ 2 i(ti +e , Отсюда a = —— = 30 см/с ; v0 = ———- = 45 см/с. tit: Решение 2. Зависимость координаты тела от времени выражается формулой х = Vot - Отсюда t2 - ^-~t + = 0. Так как ti и t2 — корни этого уравнения при х = I, то, согласно теореме Виета, 2<л> . 21 ti + tz = — И titz — —. а а Из полученной системы уравнений находим ио и а: 2Z _ 2 (ti + /2)0 . _ а = —— - 30 см/с , Vo =-о = 45 см/с. *1*2 Z 1.29. Ответ: первый метр тело проходит за 0,45 с, последний — за 0,023 с; за первую секунду тело проходит 4,9 м, за последнюю — 39 м. 1.30. Ответ: 1,0 с. 1.31. Ответ: Л = Используя известную формулу равноускоренного дви- У2 -Vo2 _ —2^— и учитывая, что в верхней точке v = 0, получаем Решение. жения s - Если отсчитывать время от момента бросания второго h 2g' тела, то уравнения движения тел следует записать в виде: gt2 gt2 hi — hm»x 2 f ^2 = Уо* 2 • Приравнивая hi = Л2 = h (в момент встречи), получаем /и, Vo , _ 3 уо2 3 vo ~ 2g’8 g ~ 4ftm“’ 1.32. Ответ: t = у - j = 1,75 c, h = ~ = 19,3 m. 1.33. Ответ: vo = (H - h)= 7,0 м/с. 1.34. Ответ: t = 0,645 с; болт перемещается относительно Земли на 0,49 м вниз. Решение. Болт падает с ускорением g = 9,8 м/с2 относительно Земли и с ускорением 'gа отЯосительно лифта. ; Его* начальная
Ответы, указания, решения 202 скорость относительно лифта равна нулю. Поэтому время Падения „ (g + а) f определяется из уравнения Н =---~----и составляет _____ & =0,645 с. ’ g + а Чтобы определить перемещение болта относительно Земли, надо просто учесть, что в начальный момент падения болта его скорость направлена вверх и равна и. Поэтому за время падения болт переместится относительно Земли на s = vt —= - 0,49 м. а Поскольку s < 0, перемещение направлено вниз. 1.35. Ответ: а > gctga. Решение. Начав свободно падать, тело А за время t сместится по щ2 вертикали на si = За это же £ время клин должен сместиться в горизонтальном направлении, как заштрихованный треугольник на рисунке). ami„t2 s2 = —g—, получаем amin = gctga. 1.36. Решение. На каждом из участков движение будем считать равноускоренным. Воспользуемся тем, что сред- няя скорость равноускорен-, ного движения равна полу- сумме начальной и конечной скоростей. Средняя скорость на первом участке пути (на . О+1/i склоне горы) равна —~; на 7/777/7/7777 минимум, на 82 = sictga (см. Учитывая, что горизонтальном участке средняя скорость та же: . т, 81 01 82 V1 Итак, — где tz — время прохождения горизонталь- Г1 J 12 Л ного участка. Отсюда Oi = — - 8 м/с, t2 = — = 7,5 с. График o(t) ti Vi приведен на риеунке. Ускорение на горизонтальном участке ,16 , . , , ,, аг = —*.— = *- тё м/с « - 1.1 м/с . , т-rt t2 15 ’ ' • .
203 Механика 1.37. Ответ: о = 68 км/ч. Решение 1. Решение задачи затрудняется из-за того, что нам неизвестно время разгона поезда tP (а, значит, и время торможения tT). Остается надеяться, что ответ не зависит от величины tP. Удобно воспользоваться Тем, что средняя скорость на этапах разгона и торможения одинакова и равна 0,5и (при равноускорен- ном движении средняя скорость равна полусумме начальной и конечной скоростей). Поэтому пройденный за время разгона и V V , Vti торможения путь равен g • t„ + t, = -g~. Он действительно не зависит от tp! Значит, при равномерном Vti движении поезд прошел расстояние s —z-, затратив на это время а 8 ti _ 8 ----?г. Поскольку полное время движения равно —, получаем I? ш Рср Is Ж s л 2SU4, со / гч I--+ ti = — . Отсюда v = —-—--------= 68 км/ч. Однако реше- 'v 2' Un 2s-Upti ние этой задачи может быть более коротким и красивым. Решение 2. График v(t) для дви- жения поезда имеет вид, пока- занный на рисунке. Пройден- ное расстояние s численно равно площади трапеции, т. е. t + (t - ti) s =--------- v . Учитывая, что 8 t = — , получаем ответ, приве- денный в решении 1. 1.38. Ответ: см. рис. а, б. Ось у направлена вертика- График на рис. б состоит из участков парабол. ^2gH Рис. а
Ответы, указания, решения 204 h Рис. б к задаче 1.38 1.39. Ответ: см. рис. а, б, в. Решение. Первые 3 с движение происходило с постоянным уско- рением ай = -2 м/с2, следующие 2 с — без ускорения и последние 2 с — с ускорением а& — 1 м/с2. После этого тело остается непод- вижным. График а40 показан на рис. а. Перемещение тела в течение первых трех секунд оп- ределяется по формуле 8х = = Voxt + —z— = 4t - t (здесь и далее численные значения, вхо- дящие в формулы, приводятся без указания единиц измере- ния; все величины измеряются в единицах СИ). График sx(t) в этом интервале (см. рис. б) представляет собой параболу с вершиной при t = 2 с (в этот момент скорость тела обраща- ется В НУЛЬ). ПОСКОЛЬКУ 8х = Зм при t = Зс, в течение следу- ющих двух секунд 8х = 3 - 2(t - - 3). Величина t — 3 представ- ляет собой время равномерного движения. Отсюда зх = 9 — 2t; при t = 5 с (к моменту, когда равномерное движение закан- чивается) находйМ 8х '=4 -1м.-
205 Механика Аналогично для третьего этапа движения sx = -1 - 2(t - 5) + а Отметим, что при t = 3 с и t = 5 с график sx{t) не испытывает изломов; раз- личные его участки плавно переходят один в другой. Это обусловлено непрерыв- ностью графика L>x(t): мгновенных изменений vx, т. е. изменений угла на- клона касательной к графику Sx(t), не происходит. Для получения графика Z(t) достаточно заметить, что путь увеличивается при любом изменении sx. Поэтому убывающие участки графика s*(t) необходимо симметрично «отразить» вверх, сохраняя непрерыв- ность графика (см. рис. в). 1.40. Ответ: v = 300 м/с; а = 2,2 см/с2. Решение. Разумеется, нам потребуется толь- ко одна координата Харькова — широта <р = = 50°. Точка земной поверхности на этой широте (см. рисунок) описывает относи- тельно центра Земли окружность радиуса г = = 7?coscp. Период обращения Т = 1сут = 86400 с. Линейная скорость вращения 2nr 2-itR coscp v = == 300 м/с, а центростремительное ускорение _ c°gP _ 2 2 CM/C2 •— •— _<> £i • £i VJjI / v • r T ' 1.41. Решение. В отсутствие сопро- тивления воздуха тело движется с постоянным ускорением g (хотя дви- жение и криволинейное). Выберем систему координат с началом в точке броска, как показано на рисунке. Поскольку gx = 0, проекция тела на ось х совершает равномерное дви-
Ответы, указания, решения 206 жение. Тело как бы участвует одновременно в двух движениях — в равномерном вдоль оси х и в равноускоренном вдоль оси у. gt2 Координаты тела изменяются по закону х = voxt; у = v<>y t — & где Оох = v0cosa и Vo„ = Vosina. Время полета определим из условия у = 0, откуда _ 2v0„ _ 2 Vosina 1 ~ g g Дальность полета I — это координата х в момент падения: 2v<>2sina cosa Vo2sin2a / Vo., t = ---------- = -------. g g Очевидно, максимальная дальность достигается при sin2a = 1, т. е. о 45 . Найти уравнение траектории тела — значит связать непосредственно х и у, исключив t. Выразив t через х с помощью соотношения t - ' и подставив значение t в формулу для у, Со. получим: V»,, gXl g 2 и - х - -—-, - xtga - -— -------—- • х . Vo. 2vox 2vo cos a Это уравнение параболы. 1.42. Решение. Время движения пули такое же, , как у свободно падающего - /2Л~, тела: t = \ — (t не зависит от vo, по- V g скольку проекция vo на ось у равна 0). Проекции скорости пули на леи »о- ординат: vx = vox = vo, vv = vo<, + gyt =* = - gt. Поэтому (см. рисунок): v = "iv2 + v„2 = Vvo2 + (gt)2 - ^v02 + 2gh, tgq = — - — = ^gh Vx Vo Vo 1.43. Ответ: a = 76°. _ _ _ r 2vo2 • sinq • cosa , Решение. Дальность «полета» L ----------------- (см. задачу 1.41) . Высоту подъема можно определить из формулы: „ Vo,2 Vo2 . . я= 2i‘2i:sVia- Приравнивая Н и L, получаем tga — 4, т. е. a = arctg4 «= ;Г;44.’Ответ: ti*i>102re?-'W'»42,7 КМ{С ='вв^4“Нм: юлээш'
207 ' гханика 1.45. Ответ: pe = совалЛ—-. у 2sma Указание. Удобно выбрать начало координат на вершине горы, ось х направить горизонтально, а ось у — вертикально вниз. Тогда координаты точки падения х = Lcosa, у = Xsina должны удовлет- ворять уравнению траектории горизонтально брошенного тела. , ... _ /Га 2Ло2 . . Г. 2v‘ 1.46. Ответ: s =\lh +-= h\l 1 + —г-. Ng у gh Указание, з выражается через горизонтальную дальность sx полета груза: s = Nh? + Зх2. В свою очередь, sx = vt, где t — время полета. 1.4,7. Ответ: тело 2 упадет позже, чем тело 1; конечные скорости одинаковы. Время движения тела 2 будет максимальным, если площадку разместить на высоте 0,5Н. Решение. После отражения (см. рису- нок) скорость тела 2 направлена гори- зонтально (модуль ее не изменился), т. е. Vy = О. Очевидно, в результате соударения время падения увели- чится (гасится вертикальная скорость П;/). Насколько же оно увеличится? Пусть площадка расположена на вы- соте h < Н. Тогда для первого тела время и конечная скорость падения составят: ti = о, = N2gH. Для второго тела к моменту удара о площадку ti' = Vi' = N2g[H-h). Время падения после удара о площадку — это просто время свободного падения без начальной скорости с той же высоты: „ _ /2Л „ ti = \rzr- Полное время Падения второго тела ti = t2' + ti' Тогда -Л/1 - 77 покажем, что это отношение больше ti у л УЛ единицы. Возведем его в квадрат:__ ^-1 + 2 tt2 12 При Л -* О или h->H получаем tz -> ti. Наибольшая «задержка» падения происходит при h = 0,5 Н, когда t2 = tiN2 (см. мате- матическое приложение). Вернемся к вопросу о конечной скорости
Ответы, указания, решения 208 падения. Для второго тела этот вопрос теперь звучит так: с какой скоростью V2 упадет тело, если оно брошено с высоты Л горизон- тально со скоростью и-/? Ответ (см. задачу 1.42): 02 = ^v/2+2gh = J2gH. Итак, V2 = vi, то есть оба тела упадут с одинаковой по модулю скоростью. В начале изучения механики этот результат не может не удивлять! Но мы советуем вернуться к этой задаче после изучения закона сохранения энергии в механике. 1.48. Ответ: ti = 0,75 с; tz = 2,8 с. Решение. Начертим примерный вид траек- тории тела (см. рисунок) и сразу же увидим, что таких моментов будет два. Они определяются из условия |оу| = = OxtgP; учитывая, что vu = vosina - gt и vx — voces а (горизонтальная составляю- щая скорости неизменна), получаем: voSina -gt = ± vocosatgP, откуда t = —(sina ± tgP cosa). 1.49. Ответ: m = 7,2 кг. Решение. Определим сначала время полета каждой из капель воды. Это время (см. решение задачи 1.41) t =----. В воздухе находится вода, вытекающая из шланга именно за время t. Массу этой воды определить нетрудно: это масса воды в цилиндре высотой vot и площадью основания S. „ „ 2v02 • sina 2pva2S • sina _ „ m = pSvot = pS--------= —-----------= 7,2 кг. 1.50. Ответ: ^g(H + >/Я2 + s2) . Решение. Нужно рассматривать только траектории движения мяча, почти касающиеся вершины стены, т. е. проходящие через точку с координатами s и Н (см. рис. а). Следовательно, значения х = s и у = Н удовлетворяют уравнению траектории (см. задачу 1.41): Рис. а Рис. б г.
209 Механика Н = stga — gs2 2v02 • cos2a' (1) Остается выяснить, какое минимальное значение vo удовлетво- ряет этому соотношению. Перепишем формулу (1) в виде gs2 Н 1 = s sina cosa - Hcos2a = - -х- + ~-(s sin2a - H cos2a). aVq a a Максимально возможное значение выражения, стоящего в скобках, равно ^Н2 + s2 (см. математическое приложение). Следо- gs2 ^Н2 + S2 - Н gs2 ----------2 тт вательно, —? <--------S---и----2 = 'Н + s - Н. 2v0 2 Vanin Отсюда vomin2 = • —— = g(H + 'Ш2 + s2) . Итак, УН + s-Н VOmin — \jg(H + Vh^Ts2). Советуем проверить справедливость этого ответа в предельных случаях (Н -* 0, s -* О); обратите также внимание, что траектория мяча касается вершины стены отнюдь не в вершине параболы (как могло бы показаться), а на нисходящей ветви траектории. Почему именно на нисходящей, а не на восходящей? Предположим обрат- ное: стена расположена в точке А, а оптимальная траектория имеет вид, показанный на рис. б. Тогда стену можно «пере- двинуть» в любую точку отрезка АВ, и мяч все равно перелетит через нее! Но ведь с увеличением s необходимое значение Vomin должно возрастать — это следует из полученной нами формулы (да и без формулы достаточно очевидно). Таким образом, наше предположение привело к противоречию, а, значит, траектория мяча касается вершины стены действительно на нисходящей ветви траектории. ч 1.51. Решение. Траектория шарика — дуга окружности. Но это- еще не означает, что ускорение шарика обязательно направлено к центру окружности: так будет только при равномерном дви- жении по окружности! В таком случае центростремительное (или нормальное) ускорение an действительно направлено по радиусу и равно по величине v2/R. Но в данном случае движение отнюдь не
• Ответы, указания, решения 210 равномерное! Значит, должна быть и Составляющая ускорения, параллельная скорости, т. е. направленная по касательной к окружности. Эту составляющую называют тангенциальным уско- рением (а,). Если а„ «отвечает» за искривление траектории, то есть изменение направления скорости, то а, — за изменение ее модуля (разгон или торможение). Полное ускорение а = ап + От. Рас- смотрим ускорение шарика в каждой из точек. Точка А (см. рис. а). В этой точке Шарик на мгновение оста- навливается, о = 0. Значит, ап = 0, и остается только От, направ- ленное по касательной вниз. Точка В (см. рис. б). Здесь ап * О, а, * О. Примерное направление полного ускорения а показано на рисунке. Точка С (см. рис. в). В этой точке скорость максимальна (уже не растет, еще не убывает). Поэтому а, = 0, остается лишь ап, направленное по радиусу к центру окружности. 1.52. Ответ: v = ----- . sina Решение. Рассмотрим два положения лодки, разделенные малым промежут- ком времени At (см. рисунок). Оче- видно, А1А2 = t> At. За время At человек выбирает отрезок веревки, равный voAt. Значит, OAi - ОАг = foAt. Отложим на отрезке OAi отрезок ОВ = ОАг. Тогда AiB = voAt. При малых At отрезок АяВ можно считать перпендикулярным AiO. Поэтому из треугольника AiAzB получаем AiB = AiAasina и и = . sina Стоит обратить внимание на то, что v > vo, и скорость лодки растет по мере приближения к берегу. Типичной ошибкой явля- ется попытка «направить» вектор Vo вдоль веревки и приравнять его горизонтальную проекцию скорости V. На самом деле все обстоит наоборот: скорость вытягивания веревки равна проекции скорости лодки на направление веревки. 1.53. Ответ: и =---. сова Указание. См. задачу 1.52. 1.54. Ответ: 3*5 м/с.. Решение. Скорость нижнего края лестцицьх относительно верхне-
211 Механика го края <>дм =* ил - ив. Если бы точки Л Ат В (cwj, рисунок) не были жестко связаны между собой, скорость Worn могла бы быть направлена как угод- 9вЛ а но. Однако жесткая связь приводит к л уА тому, что отрезок АВ (точки А и В j В* связаны с лестницей) может лишь А х90°-а поворачиваться и сдвигаться как целое, а не укорачиваться или удлиняться (в системе отсчета, связанной t В, точка А описывает окружность). Значит, Готя перпендикулярна АВ и проекция Готн на АВ равна нулю: (vA - Vb)ab = О, т. е. (va)ab = (Vb)ab. Проекции скоростей обеих точек на соединяющий их отрезок одинаковы! Разумеется, этот вывод справедлив для любых двух точек недеформируемого твердого тела. В данном случае это при- водит к уравнению fAsina = vecosa, откуда l>b = fAtga — 3,5 м/с. 1.55. Ответ: а) О; б) 20 м/с; в) 14 м/с. Решение. Перейдем в систему отсчета, связанную с трактором. В этой системе отсчета скорости всех точек гусениц имеют одинако- вый модуль Uryc и различаются лишь направлением. Как связаны скорости v и Огуе? Учтем, что скорость точек нижней части гусениц относительно Земли равна нулю (нижние звенья гусениц некото- рое время просто лежат на земле). Эту скорость можно пред- ставить как сумму скоростей v + Ггусл (скорость Ггусл направлена Значит, Пгуе = о. Тогда для точки В на верхней части назад). гусеницы l>b = 2и = 20 м/с, а Vc = 1.56. Решение, а) Слова «катит- ся без проскальзывания» означа- ют, что точка диска, соприкаса- ющаяся в данный момент с доро- гой, имеет нулевую скорость (не случайно мокрое колесо остав- ляет на асфальте столь четкий след). Эта ситуация очень похо- жа на описанную в задаче 1.55. В системе отсчета, связанной с Центром1’/УгЙсН, йёё;‘тУ4'йй оббдк
Ответы, указания, решения 212 движутся ПО окружности С одинаковой ПО модулю скоростью Овр. Скорость V точки диска относительно неподвижного наблюдателе является суммой скоростей: V = v + и»Р. На рис. а показаны направления и»Р для различных точек обода. Очевидно, Va = О только при Пвр = и. Тогда Vb = Vb = ; Vc = 2v. Для определения ускорений точек тоже удобно «разбиение» движения на поступательное и вращательное. Однако поступа- тельное движение с постоянной, скоростью и не дает вклада в ускорение точек. Значит, надо учитывать только вращательное движение с постоянной линейной скоростью v по окружности радиусом R. Таким образом, все точки обода имеют одинаковое по v2 моДУлю ускорение а = направленное к центру диска. Скорости гС и ускорения точек А, В, С, D показаны на рис. б (Va = 0). б) Поскольку Va = 0, движение диска можно представить как вращение вокруг так называемой мгновенной оси вращения, про- ходящей через точку А. Тогда модуль скорости любой точки прямо пропорционален расстоянию от точки А. Интересующие нас точки лежат на окружности радиусом R с центром в точке А (рис. в). Однако попытка воспользоваться мгновенной осью вращения для определения ускорений точек диска привела бы к ошибке (можете убедиться в этом сами!). Дело в том, что эта ос’ не связана «жестко» с какой-либо из точек диска. _ _' . Rv Rv 1.57. Ответ: а) и = —-; б) и = —-. R-r R+r Решение 1. В обоих случаях скорость v совпадает со скоростью той точки катушки, где начинается прямолинейный участск нити. Для определения скорости и оси катушки удобно воспольз зваться понятием мгновенной оси вращения (см. решение задачи 1.56).
$19 Механика Очевидно, для случая а и _ R _ Rv Й v~ R-г ~R-г Для случая б \ и R Ry , .j, — =------------------; и —----. v R + г R + г В обоих случаях катушка движется в ту же сторону, в которую тянут нить. В случае а нить наматывается на катушку (и > и); в случае б нить сматывается д катушки (и < и). В случае а очень часто предсказывают движение катушки в противоположном на- правлении, ссылаясь при этом на собственный опыт. Как ни странно, Сделанный при решении задачи вывод действительно может не подтвердиться при проведении опыта. Дело в том, что если конец нити потянуть достаточно «резко», то проскальзы- вания катушки избежать трудно. _ Решение 2. а) Скорость любой точки катушки складывается (векторно) из скорости поступательного движения ка- тушки и и скорости вращения, равной произведению угловой скорости <о ка- тушки на расстояние точки от оси ка- тушки (см. рисунок). Воспользуемся тем, что нам известны скорости точек А И В: , л , " ч ' • • _ _ _ пи Ра = и - ©г = и, ив = и - (OR = 0. Отсюда и =-. • R - г Аналогично мо#сно рассмотреть и случай б. 1.58. РерхёнАе. Осколки разлетаются по различным параболи- ческим! Траекториям. Важно, что все они движутся с одинаковым и постоянным ускорением g. При отсутствии тяготения они, очеввдйо, двигались бы равномерно и прямолинейно и заполняли бы расширяющуюся сферу радиуса not с центром в точке взрыва. Но ведь тяготение легко «убрать» — именно потому, что уско- рения у всех осколков одинаковы! Перейдем в систему отсчета, связанную с телом, которое начало бы свободно падать в момент разрыва снаряда (так двигался бы сам снаряд, останься он целым). В этой системе отсчета ускорения у всех осколков отсутствуют, т. е. «тяготения нет». Значит, «облако осколков» — сфера радиуса foi, центр которой опускается с ускорением g без начальной ско- рости от места разрыва снаряда (вспомните праздничные фейер- верки!).ч ,.j ,./• • , J,.-,.) ОПЗ')Я> 1 •
Ответы, указания, решения 214 1.59. Решение. К моменту пер- вого удара скорость мяча ш = = ^2gH. Выберем систему ко- ординат, как. показано на ри- сунке. При каждом ударе vx не изменяется, а иу меняет знак, сохраняя величину; поэтому к моменту следующего удара о нак- лонную плоскость vu принимает то же значение, т. е. после каждого из * ударов Vy = viy = ^2gH cos а. По- же: этому время полета т после каждого отскока одно и то т = = 2лЖ= 2то, gcosa » g где то —-время падения шарика из начальной точки. Таким образом, координата п-го соударения шарика с плос- костью f»t»2 g sina (2п - I)2 to2 х"~ 2 ~ { 2 Здесь tn = (2п - 1) то — время от начала движения до n-го удара. Расстояние между ближайшим^ точками соударений s„ = Хп, 1 - Хн = 4hg sina То2 = 8пН sina. Поскольку sn прямо пропорционально п, получае*i: si : 8а : за : = 1 : 2 : 3 : ... 1.60. Ответ: I = 41 м. Решение. Эту задачу удобней всего решать в сие еме отсчета, связанной с движущимися автомобилями. Тогда м ясно считать, что сами автомобили неподвижны, а колеса равномс Жо вращают- ся. Наиболее удаленные от оси колеса точки имеют -сорость и (см. решение задачи 1.56). Такую Же начальную скорость Имеет в момент отрыва от колеса и камешек. Наибольшее расстояние он пролетит, если его начальная скорость образует с горизонтальной плоскостью угол a = 45° (см. решение задачи 1.41). Это рассто- y2sin2a о2 „ , и2 „ яние составит —-— = —. Итак, I = При скорости движения v = 72 км/ч = 20 м/с получаем I — 41 м. При решении задачи мы не учитывали, что «точка броска» расположена несколько выше уровня земли. т^рузового автомобиля с большими колесами зто может дйазаться весьма существенным.
215 Механик^ 1.61. Ответ: а) 5,0 м/с; б) 10 м/с. Решение. Два соседних кадра разделе- ны промежутком времени At = 1/24 с. чг 2л Угол между соседними спицами <р = . Почему может е вникнуть иллюзия «не- правильного» bi ащения колеса? Пусть, например, на деух соседних кадрах спицы располо; сены, как показано на рисунке: 1, 2, ... 6 — начальные поло- жения спиц, 1 , 2', ... 6' — конечные. Но спицы на экране не пронумерованы 4 3' и неотличимы! Поэтому мы неосознанно воспринимаем спицу 1' как новое положение ближайшей к ней «старой» спицы 2, а 2' — как новое положение спицы 3, и т. д. Так возникает кажущееся обратное вращение. Это возможно, если угол поворота колеса за время At не менее 0,5<р. Тогда угловая скорость вращения колеса 2 At’ скорость вращения крайних точек колеса (совпадающая со скоростью движения повозки, если нет проскальзывания) v > v„,i„ = ©Я = = 5,0 м/с. N • М Минимальная скорость, при которой колесо покажется невра- щающимся, будет соответствовать углу поворота <р за время At и СОСТаВИТ 2Vmin = 10 м/с. 1.62. Ответ: для четырех черепах t - —; Решение. Из соображений симметрии очевидно, что в любой момент черепахи будут находиться в вершинах квадрата. Этот квадрат поворачивается и «съе- живается». Встреча всех черепах прои- зойдет в центре исходного квадрата. Скорость приближения к этой точке можно определить, спроецировав ско- рость ui на отрезок ОА (см. рисунок): это v cos45 = -f= = const. V2 x 2e для трех t = Поскольку начальное значение отрезка ОА равно -j=,To
Ответы, указания, решения 216 а t = — = - . у v 12 Такой результат можно получить и иначе, определив скорость уменьшения стороны квадрата. Эта скорость определяется про- екциями на нее скоростей черепах: Д(АВ) х а - — 1?2АВ — V1AB — — V, t — — . At v Аналогично (любым из приведенных способов) полунаем, что 2а для трех черепах t = 1.63. Ответ: I = Решение. Пусть в на- чальный момент заяц был в точке А, а лиса — в точке В. Лиса бе- жит по достаточно сложной криво., (см. рисунок), однако х. т, подобно барону, не б> дем теряться и попро- буем для начала опре- делить, с какой ско- ростью лиса прибли- жается к зайцу. Пусть в некоторый скорость направлена под углом а к скорости зайца. Проецируя обе скорости на направление скорости лисы, получаём, что скорость сближения лисы и зайца их =*= t? - ocosa. К сожалению, в эту формулу входит угол а, меняющийся со Временем. Попробуем найти еще величину, куда входит угол а. Спроецируем положение лисы на линию тропинки (точка В"). В рассматриваемый момент точка В" движется как раз со скоростью vcosa и, значит, заяц удаляется от точкй В" со скоростью иг — и - vcosa. Скорости их и иг совпали! Это означает, что расстояние В"А' растет точно с такой же скоростью, с какой уменьшается расстояние В'А’, т. е. сумма этик расстояний остаемся неизменной! Мы нашли в нашей задаче своеобразныЙ)«закон,ррхран^ния»: В"^..+. #Д',= соцз^.Црркольку
217 Механика в начальный момент 1<"А' = 0, а В'А' = L, получаем: В"А' + В'А' = L. (1) Через достатг „но большой промежуток времени (что нас и интересует) лг га будет бежать практически по самой тропинке, т. е. точкт" В' и В" совпадут. Тогда В"А' = В'А'. Из формулы (1) следует, что В"А' = В'А' — L/2. Итак, через достаточно большое время (t » L/v) расстояние между лисой и зайцем будет вдвое меньше начального. Стоит, наверно, обратить внимание на то, что мы не можем найти вид траектории движения лисы в этой задаче: ее движение является слишком сложным. Но это не помешало нам ответить на поставленные вопросы, потому что из переменных величин удалось составить постоянную величину, имеющую важ- ный смысл. (1) 1.64. Ответ: t = , . и2 - V2 Решение. Повторив рассуждения, приведенные в решении задачи 1.63, получим систему уравнений: {Vi = и - ucosa, Vz - v - ucosa. (заметим, что vz вначале положительна, но через некоторое время становится отрицательной, когда точка В" начнет приближаться к точке А’). Исключая cos а из уравнений (1), получим: vvz - uvi = 2 2 , ЬА'В" = v - и , или (учитывая, что v, ------; v2 = —---, а величины At At v и и постоянны): А(и • А'В" + и А'В') . п —*----------------- = v2 - и1 < 0. At Итак, величина v • А'В" + и • А'В' убывает с постоянной ско- ростью и2 — и2. Начальное значение этой величины uL, конечное (в трагический для зайца момент) равно нулю. Промежуток вре- и • L мени между этими моментами t = —------- . и2 - V2 1.65. Ответ: о = 1 рад/с, аА = 0,20 м/с2. Решение. Груз опускается равноускоренно. Поэтому v = ^2as. Скорость v груза совпадает с линейной скоростью точек На ободе шкива, v ^2as , . значит, о = -= = —=— = 1 рад/с. /I гС Ускорение точки на ободе шкива склаДЫ- вается‘1йРн65ЕЙйальн6тё СцейтроёДрёмй^ейййЫ-13411
Ответы, указания, решения 218 го) а» и тангенциального щ (см. рисунок). Ускорение ах приводит к увеличению линейной скорости вращения точек обода шкива. Ускорение ап одинаково по модулю для всех этих точек: - — - %as Qn ~ R ~ R ' _______ /4^2 поэтому аА = У а,2 + а2 = + 1 = 0,20 м/с2. 1.66. Ответ: удар происходит при подъеме мяча; Л = 2,1 м; v = 7,7 м/с; мяч упадет на расстоянии 7,2 м от стены. Решение. Определим дальность L полета цяча в отсутствие стены (см. решение задачи 1.41): vo2 sin2a L =----------= 10,2 м. g Поскольку I < 0,5L, удар происходит при подъеме мяча. Для определения высоты h проще всего воспользоваться уравнением траектории полета (см. решение задачи 1.41), заменив х на I, у на hl gl2 h = Itga - ——— = 2,1 m. 2 Vo cos a Скорость v ч точке А можно выразить через ее горизонтальную и вертикальную лооекции: vA = '+ Va*2. Если учесть, чти Мх =vox, а vav2 - vov2 = -2gh, получим: Va = ^va«2 + . 2 - 2gh - >lvo2 - 2gh - 7,7 м/с. (Эта формула может был, получена сразу из закона сохранения механической энергии.) При упругом ударе о стену проекция скорости vy не изменяется, а Vx просто меняет знак. Значит, участок траектории АВ’ сим- метричен продолжению первоначальной параболической траек- тории АВ относительно плоскости стены (см. рисунок). Если стену «убрать», мяч ударится о землю на таком же расстоянии от
219 Механика основания стены, но по другую сторону. Очевидно, это расстояние равно L - I = 7,2 м. ____________ 1.67. о™»: 1 - • л/ф+л/Н^)’). Решение. Муха не может двигаться к капле меда по кратчайшему пути — сначала придется «погасить* начальную скорость ио, а на это нужно время. Траектория явно будет криволинейной. Однако легко указать систему отсчета, в которой траектория мухи пря- молинейна, — это система отсчета, движущаяся со скоростью ио относительно стола. В этой системе отсчета муха А вначале неподвижна и может начинать движение в любом направлении, а капля меда В убегает от нее по столу со скоростью, по модулю равной vo. Теперь даже муха сообразит: двигаться нужно по прямой на некую «упрежденную» точку С, и двигаться нужно с максимально возможным ускорением а (см. рис. а) Тогда at* АС= ~, вс = Vat. А Из прямоугольного треугольника АВС: = (v»t)2 + №, откуда Рис. а Рис. б В неподвижной системе отсчета ускорение мухи то же (так как ио = const); значит, движение будет равноускоренным. Одна- ко за счет начальной скорости траектория превращается в пара- болу, ось которой параллельна а (т. е. отрезку АС). Примерный вид траектории показан на рис. б. 1.68. Решение. Эту задачу естественно решать в системе отсчета, связанной с автомобилем. В этой системе отсчета точки обода колеса движутся по окружности со скоростью v. Значит, камешек начнет двигаться вертикально вверх именно с такой скоростью.
Ответы, указания, решения 220 Полное время его полета (до падения на колесо) t = —. Согласно условию, за это время колесо должно совершить целое число k оборотов: , 2nR t = k •--, v откуда v = 'ik itgR = 2,5 м/с • 'Ik. Итак, подходящих значений скорости достаточно много. Даже при k = 100 получаем вполне реальное значение скорости (около 90 км/ч). 2.1. Ответ: Р = 640 Н. Решение. Прежде всего заметим, что само по себе движение лифта в любом направлении и с любой (но постоянной!) скоростью не может влиять на вес нахо- дящихся в нем тел: это противоречило бы первому закону Ньютона. Чтобы оп- ределить, как влияет на вес тела в лифте ускорение лифта а, запишем для этого тела второй закон Ньютона: та = mg + N. Здесь N — си. - реакции со стороны пола лифта (см. рисунок). Проецируя это вектор ни. равенство на ось у, получаем: - та = - mg + N, откуда N = m(g - а). Остается лишь заметить, что найденная величина равна как раз весу тела: ведь согласно третьему закону Ньютона сила, с которой тело давит на пол (вес тела Р), равна по модулю и противоположна по направлению силе, с которой пол действует на тело (силе реакции опоры N): Р = - N. Итак, Р = N = m(g - а) — 640 Н. Вес тела уменьшается, когда ускорение тела направлено вниз (совсем не обязательно, чтобы само тело при этом двигалось вниз!). Аналогично, когда ускорение направлено вверх, Р = m(g + а) > mg, т. е. вес увеличивается. В заключение хочется пожелать барону найти столь же простое решение проблемы лишней массы тела. 2.2. Ответ: 22 кН, 42 кН. Решение. Фактически речь идет об определении веса Р движуще- гося автомобиля. Вес автомобиля при движении по криволиней- ной траектории не совпадает с весом mg = 32 кН неподвижного автомобиля', йоёй£Ы^к?у 'автоМо5иЛь двй^кЙся с уско^Ьйем. В
221 Механика v2 середине выпуклого моста это ускорение равно -= и направлено JK V3 вертикально вниз. Следовательно (см. задачу 2.1), Р = m(g - -х) = = 22 кН. В середине вогнутого моста центростремительное уско- у2 рение автомобиля направлено вверх, поэтому Р - m(g + -=г) = 42 кН. л 2.3. Ог.«т: < V ak при а < g, t = 0 при а > g. Решение. До момента отрыва предмет движется вниз с ускорением а под дей- ствием силы тяжести mg, силы упру- гости пружины Гущ, и силы реакции опоры N (см. рисунок). В момент отрыва сила N обращается в нуль. Проецируем уравнение второго закона Ньютона на вертикальную ось: та = mg - Fynp. Ис- пользуя закон Гука Fynp = kx и соотно- at2 , шение х - для удлинения пружины х, получаем _____________ ,mg /2zn(g -а) V У ak Время t уменьшается при увеличении а, при а = g оно обраща- ется в нуль. Очевидно, при а > g предмет не поспевает за подставкой и сразу отрывается от нее, т. е. t = 0. 2.4. Решение. На этот вопрос часто дают неверный ответ: когда туловище движется вниз, показание весов уменьшается, при движении вверх — увеличивается. Напомним, что на самом деле вес тела зависит не от скорости его движения, а от ускорения (см. задачу 2.1). На начальной стадии приседания ускорение туловища направлено вниз, и вес тела (показания весов) уменьшается. В конце приседания движение вниз замедляется; ускорение направ- лено вверх, и вес увеличивается. При выпрямлении вес человека сначала увеличивается, а затем уменьшается. 2.5. Ответ: а = 2,0 м/с2, Т = 1,2 Н, F = 2,4 Н. Решение. Благодаря нерастяжимости шнура величины переме- щения обоих грузов (а, значит, и их ускорения ai и аг) всегда одинаковы: ш = аг = а. Пренебрегая массами шнура и блока, а также трением в блоке, мы можем считать силу Т ндтяжения шнура одинаковой- На рис. а npK^apfJ, действующие на
Ответы, указания, решения 222 грузы силы. Запишем уравнения второго каждого из грузов: miai = mig + Т, maaa = mag + Т. В проекции на ось у уравнения при- нимают вид: | пиа - -mig + Т, '-/ига = -mag + Т. Вычитая второе уравнение системы из первого, получаем а =-------g = 2,0 м/с . Ша + 7П1 m „ 2mimag , „ „ _ Тогда Т =-------= 1,2 Н. Для нахождения 7П1 + ТП2 силы F упругости пружины динамометра достаточно рассмотреть силы, действую- щие на блок (см. рис. б). Условие равно- весия блока имеет вид F + 2Т = 0, откуда F = 2Т - 2,4 Н. Как правило, обязательной после решения задачи считается проверка размерности полученного резуль- тата и правдоподобности (реальности) полученного численного значения. Однако такие проверки не всег- да обнаруживают допущенную ошибку. На примере данной задачи покажем еще два эффективных метода проверки. Первый из них — проверка на симметрию. Очевидно, что если поменять грузы местами, то натя- жение шнура и величина ускорения грузов не должны измениться. Полученные для а и Т выражения удов- летворяют этому требованию (при замене mi <-> ma значение Т не изменяется, а ускорение а лишь меняет знак). Второй метод — проверка на частные и предель- ные случаи. Ведь даже до решения задачи ясно, что Рис. б при mi = ma = m ускорение равно нулю, а Т = mg; при mi -> О должно быть а -> g, Т -> 0 (груз тяг при свободном падении находится в состоянии невесомости). Полученные для а и Т формулы удовлет- воряют этому требованию. Обратим внимание на то, что вес обоих грузов одинаков, как бы сильно ни отличались их массы: ведь оба груза действуют на нить с одинаковыми силами. Вес большего груза меньше mzg, поскольку ускорение этого груза направлено вниз; меньший груз, ускорение которого'направлено вверх, испытывает «перегрузку».
223 Механика „ _ ____ znjsinp - misina 77iizn^(sina + sinp) 4.0. Отчет: a =-----------------g, 1 =--------------------- mi + mz Решение. На рисунке по- казаны силы, действую- щие на каждый из грузов. Напомним, что Ti = Tz = T (см. задачу 2.5). Считая, как обычно, нить нерас- тяжимой, получаем at = = в 2 = а (направления ус- корений выбраны предпо- ложительно, они могут оказаться и противополож- ными истинным направле- ниям). Уравнения второго закона Ньютона в векторной форме для каждого из грузор имеют вид: imiai = mig + Ni + Ti, [mzaz — mzg + Nz + Tz- Спроецируем каж^ре из этих уравнений на «свою» ось, параллель- ную соответствующей наклонной плоскости: > [хГ| пил - -znigsina + Т, щ И mza = zn2#sinp - Т. Складывая оба уравнения, получаем misinB - znisina Н =------с---------g ' mi + ГПг (при znisina znzsinp векторы ai и az направлены противоположно направлениям осей xi и xz соответственно). Подстановка этого выражения в любое из уравнений системы (1) приводит к резуль- тату: У _ mimzgjsina + sinp) mi + m.2 _ _ „ 2(2?П1 - тпа) 2mi - mz _ 3mimzg 2.7. Ответ: at = аг = --------g, Т = ----— . 4mi + тг 4mi + mz 4mi + mz Указание. Из нерастяжимости нити следует, что перемещения грузов связаны соотношением Si = 2sz, поэтому ai = 2az. Уравнения движения грузов имедет вид; nuai = mig - Т, mzaz 2Т - mzg (щ > О, аз > 0 соответствует цацравлрнию д» днцв»>р.Н2 т—. вверх).
Ответы, указания, решения 224 2.8. Ответ: F = + Fz*. mg та _ „ тх Решение. Левая часть стержня имеет массу —, а правая •— т(1 - х) л, ----1--. Обе части движутся с одинаковым ускоренней! а. под действием приложенных к ним сил. Значит, »«, = F, _F, = F_F,. Fi — Fi Отсюда a =------(как и следовало ожидать), F = Fi m Если x -> 0, то F Ft; если х -> I, то F -> F2. 2.9. Ответ: Р = 49,25 Н. Решение. Груз вместе с вагоном движется на и2 повороте с ускорением а = направленным л горизонтально (к центру поворота). Это уско- рение сообщает грузу равнодействующая си- лы тяжести mg и силы F упругости пружины (см. рисунок): та = mg + F. Из рисунка видно, что F - ^(mg)2 + (та)2. Согласно третьему закону Ньютона, Р = F, откуда Р = т = 49,25 Н, что на 0,25 Н превышает вес груза при движении без ускорения. 2.10. Ответ: Р - nry]+ ^j2 = 4,4 кН; в 6,4 раза. Указание. См. задачу 2.9. 2.11. Ответ: а) Т = —^—F = 1,2 Н. б) Т = ———F = 0,8 Н. 7П1 + Шг mi + Шг Указание. Обратите внимание на то, что сила Т сообщает в случаях 1 и 2 одно и то же ускорение телам разной массы. 2.12. Ответ: F < Т + pAfig при Т < \iMig-, F<T(\+ при Т > \iMig. ’ Ди 2' Решение. Если сила F совсем мала, шнур даже не натянется: для того, чтобы он натянулся, надо сначала сдвинуть с места первый груз. Минимальная сила, необходимая для этого, равна pAfig. Если F > [iMtg, шнур натянется; при дальнейшем увеличении F
225 Механика натяжение будет увеличиваться до тех пор, пока либо шнур порвется, либо сдвинется второй груз. Если шнур непрочный, то Т < р. Mig, a Fmax = pAfig + Т. Если шнур достаточно прочный, Т > iiMsg, и тогда силу F можно увеличивать до тех пор, пока второй груз не начнет двигаться с ускорением вшах, определяемым из условия: Мгвтах = Т - \lMig, т. е. Отах =------ (ббльшёе ускорение второму грузу придать уже невозможно — шнур разорвется). С таким же ускорением будет двигаться и первый груз, значит, (Mi + Мг)втах — -Ртах ц(М 1 + Mi)g. Подставляя сюда выражение для вшах, получаем: FmMI = (Mi + M2)(e„,.x + gg) = Т(1 + Л,о rrh-iim2 mimig(l + ц) . Л 2.13. Ответ: а =--------g, Т =------------при mi > щп2; а = О, mi + тг mi + тг Т = mig при mi < [тг. При подъеме лифта следует во всех формулах заменить g на g1 = g + во. Указание. Пока масса mi невелика, второй груз покоится от- носительно стола и на него действует со стороны стола сила трения покоя. Для нее выполняется соотношение FTV < pJV, где N — сила нормального давления на поверхность. Когда сила натяжения нити превышает максимальную силу трения покоя, система при- ходит в движение. При этом силу трения покоя сменяет сила трения скольжения FTV = pJV. См. также задачу 2.1. 2.14. Ответ: F = Зтп^1 - , где рв — плотность воды, р — плотность чугуна; ответ не зависит от v. Указание. В обоих случаях (движение вниз и вверх) ядро дви- жется равномерно, т. е. равнодействующая всех сил, действующих на него, равна нулю. 2.15. Ответ: время не зависит от утла наклона желоба. Решение. Пусть желоб образует угол а с го- ризонтом. Грузик соскальзывает под действием силы тяжести mg и силы реакции опоры N (см. рисунок). Проецируя уравнение второго закона Ньютона ma = mg + N на наклонную плоскость, получаем та = mgsina. Итак, на гладкой
Ответы, указания, решения 226 наклонной плоскости а = gsina (этот результат полезно запом- нить). Длина желоба I — 2Bcosp = 2Bsina. Таким образом, при изменении угла а отношение пути к ускорению остается неизмен- ным. А это значит, что и время t, определяемое из условия at2 ЦТ п I - -g-, тоже не зависит от угла a: t = 2 . Эта красивая задача принадлежит Галилею. 2.16. Ответ: а = 2,4 м/с2, t = 1,3 с. Решение. На рисунке показаны действующие на тело силы. Со- гласно второму закону Ньютона, та = mg.+ N + Гт₽. В проекциях на оси координат получаем та = mg sin a - FTp, (1) О = - mg cos a + N. (2) Сила трения скольжения FtV = = pTV. Из уравнения (2) N = = mgcosa, так что FtV = p mgcosa Подставляя это выражение в урав- нение (1), получаем а = g(sina - - psina ). Из этой формулы сле- дует, что соскальзывание может происходить при р < tga (в дан- ном случае это условие выполне- но, и a = 2,4 м/с2 > 0). При р > > tga получаем a < 0. Это озна- чает, что если бы тело двигалось вниз по наклонной плоскости с некоторой начальной скоростью, „ .. , я*2 его движение замедлялось бы. Из соотношения I = получаем Л Уа V g(sina - pcosa) = 1,3 с. 2.17. Ответ: а = 0,42 м/с2, Т = 9,4 И. При увеличении коэф- фициента трения а = 0, Т = 9,8 Н. Решение. Прежде чем расставлять действую- щие на грузы силы, сле- дует определить возмож- ное направление движе- ния (иначе куда же на- править силу трения Гтр?). В этом состоит типичная сложность задач с учетом трения. Про-
227 Механика ведем предварительный анализ условия. Проекция силы mg на направление прикрепленной к грузу mi нити равна znigsina; для силы mzg эта проекция равна m.2g. Очевидно, rmg > migsina. Значит, если система вообще движется, то груз m2 опускается, а груз mi поднимается. Теперь можно «расставить силы и уско- рения (см. рисунок). Напомним, что ai = a2 = а. Спроецируем уравнения второго закона Ньютона: для груза m2 — на ось у, для груза mi — на оси xi, yi. Получаем: т^а = mzg - Т, mia = Т - Ftp - /nigsina, . N - znigcosa = О. Добавим к этой системе уравнение для силы трения сколь- жения Ftp = p/V. Тогда FTp = (xmig’cosa, и из первых двух уравнений системы получаем: mz - mifsina + ц cosa) m mimzg а =----------------------g, Т --------(1 + sma + ц cosa). 7П1 + тпг mi + m2 Если ц = Ц1, то а = 0,42 м/с2, Т = 9,4 Н. Если ц = цг, то a < 0; однако мы уже выяснили, что двигаться в обратную сторону при данных mi, mz и а система не может. Поэтому результат a < 0 смысла не имеет. Почему же он возник? Из-за незаконного использования формулы Ftp = 11N: ведь при столь большом трении грузы будут неподвижны, и Ftp представ- ляет собой силу трения покоя. Про нее же можно сказать лишь то, что Ftp < p/V. Итак, если ц = цг ~ 0,3 (и вообще если ц > ——— - tga), то a = 0. При этом Т = mzg = 9,8 Н. znicosa _ „ „ Л I, mz - mi(sina + ucosa) mz 2.18. Ответ: |a|.= g------------------при — > sina + цсоза; mi -1- mz 1 1 mifsina - ucosa) - m2 mz |a| =--------------------при — < sina - jicosa; mi + mz _ m2 . a = 0 при sina - цсоза < — < sina -1- цсоза. nti Решение. Будем считать положительным такое направление дви- жения, когда груз т2 опускается, и запишем уравнения второго закона Ньютона для обоих грузов: imia = Т - znigsina + Ftp, [mza = mzg-T. (1) В этой системе двух уравнений есть три неизвестных: ускорение грузов а, сила Т натяжения нити и сила трения Ftp. Это обычная
Ответы, указания, решения 228 ситуация в задачах, где надо учитывать силу трения, и сейчас мы покажем, как она разрешается. Сложим уравнения (1) и получим одно уравнение: (mi + тпг)а = mig - znigsina + Ftp. (2) Формальное решение этого уравнения mig - migsina + К,, a--------------------- (3) 7П1 + Шг является на самом деле только началом решения нашей задачи! Какие могут быть виды движения грузов? Их три: первый — грузы движутся в «положительном» направлении, а > 0 (висящий груз опускается ); второй — грузы движутся в «отрицательном» направлении, а < О (висящий груз поднимается ); третий — грузы покоятся, а — О. Заранее мы не знаем, каким будет вид движения: это зависит от соотношения между величинами, входящими в условие. Значит, наша задача состоит в том, чтобы не только определить ускорение грузов, но и найти условия, при которых это ускорение будет иметь место. Начнем с первого вида движения. В этом случае Ftp = - ц migoos а и _ mig - mig(sina + pcosa) П — , V*) mi + mi но для того, чтобы это решение было правильным, необходимо выполнение условия а > О, то есть Иг — > sina + pcosa. (5) Если реализуется второй вид движения, Ftp = p/nigcosa и mig - znig(sina - pcosa) a = -------------------------------------- (b) m\ + mi Такое движение возможно при условии a < О, то есть — < sina - pcosa . (7) fill Интересно заметить здесь, что если правая часть уравнения (7) отрицательна, то такой вид движения вообще невозможен! Этого и следовало ожидать: при tga < ц второй груз не поедет вниз, даже если первого груза нет вообще. Наконец, в третьем случае a = О, и уравнение (3) позволяет определить силу трения покоя: Ftp = migsina - mig. (8) Эта сила трения должна удовлетворять условию |Ft₽| < u migcos a,
229 Механика то есть - pmigcosa < migsina -mzg < р migcosa, или для отношения масс: sina - pcosa < — < sina + pcosa . (9) TTl\ Итак, решение имеет вид: тг 1) если — > sina + pcosa, то первый груз опускается с уско- m-ig - mig(sina + ucosa) рением а -----------------------; rtli + Шг тг 2) если — < sina - pcosa, то первый груз поднимается с уско- ТП\ . । migfsina - pcosa) - mzg рением |a| =--------------------; mi + m2 3) Если sina - pcosa < — < sina + pcosa, то грузы остаются в покое. 2.19. Ответ: а) 400 м; б) 408 м. Решение. Случай а уже рассмотрен в задаче 1.27. Случай б намного сложнее. Он отличается от случая а тем, что после отрыва вагона поезд движется с ускорением, поскольку сила тяги оста- лась неизменной, а масса укороченного поезда уменьшилась; следовательно, уменьшилась и сила сопротивления. Ускорение поезда аа определяется из второго закона Ньютона для «остатка» поезда: (М - т)аа = Ft - р(М - m)g, (1) где Ft — сила тяги, ар — коэффициент сопротивления. Заметим теперь, что при равномерном движении целого поезда сила тяги как раз уравновешивалась силой сопротивления всего поезда, то есть Ft = Ц-Mg. Подставляя это выражение в уравнение (1), т получаем ап = \tg-----. М - т В то же время отцепленный вагон движется с отрицательным ускорением = -pg. Поезд проедет за время t торможения вагона a„t2 расстояние s„ = vot + -g-. Для его определения воспользуемся тем, Vot |a.|t2 что s. = -g- = —g—. Из этих соотношении следует, что первое ела- , г* Ли 1 гаемое в формуле для зп равно 2sB, а второе зв г—т.
Ответы, указания, решения 230 Поэтому s„ = sJ2 + гЦ) = з.(2 + ———1 = 408 м. ' |в»|/ ' М - тг 2.20. Ответ: а = arctg^; Т = nvJg2 + а2. Решение. На шарик действуют две силы: сила тяжести mg и натяжение Т нити (см. рисунок). Их равнодействующая F сообщает шарику такое же ускорение, какое имеет тележка, т. е. F = та. а Следовательно, tga = —. Величину силы Т нахо- дим из теоремы Пифагора: Т = nr/g2 + а2. а 2.21. Ответ: поверхность воды плоская, образует угол а = arctg— с горизонтальной плоскостью. Решение. Пусть ускорение тележки направлено впра- во (см. рисунок). Перейдем в неинерциальную систему отсчета, связанную с те- лежкой. В этой системе к «обычным» силам взаимо- действия добавляется сила инерции Гин = та (см. зада- чу 2.20), которую можно рассматривать как «допол- нительную» силу тяжести. Жидкость, таким образом, находится в однородном поле «сил тяготения» с новым ускорением свободного падения g = g — а. Поверхность жидкости будет плоской, причем плоскость поверх- ности перпендикулярна к g. С горизонтальной плоскостью она _ . . а образует (см. рисунок) угол а = arctg—. 2.22. Ответ: р = а; Т = zngcos а. Решение. На шарик действуют две силы: сила тяжести и сила Т натяжения нити (см. рисунок). Их равнодействующая F сообщает шарику такое же ускорение а, какое имеет тележка, т. е. F = та. Заметим теперь, что тележка движется по наклонной плоскости без трения с ускорением, равным по величине gsina. Значит, та — = mgsina; эТо как раз проекция силы тяжести mg на направление движения тележки. Следовательно, проекция силы Т на это же
231 Механика направление равна нулю, т. е. Т на- правлена перпендикулярно наклон- ной плоскости. Поскольку сила Т направлена вдоль нити, получаем, что нить перпендикулярна наклон- ной плоскости, т. е. Р = а. Отсюда Т = mgcosa. 2.23. Ответ: поверхность воды будет плоской, параллельной наклонной плоскости. Указание. См. задачи 2.21, 2.22. Замечание к задачам 2.22, 2.23. Если бы мы жили в вагоне без окон, скатывающемся без трения с наклонной плоскости, нам казалось бы, что наш вагон покоится относительно Земли: поверхность воды в любом сосуде была бы «горизонтальной», а все отвесы — «вертикальными». Правда, все предметы стали бы легче:«сила тяжести» в нашем вагоне стала бы равной zngcosa. Это можно представлять себе так, будто уменьшилось ускорение свободного падения: g заменяется на gcosa. 2.24. Решение, а) Кирпич остается неподвижным (а = 0), пока F не превысит максимальной величины силы трения покоя, равной [kmg. При этом сила трения покоя в точности уравновешивает силу F, т. е. Утр =- -У. При дальнейшем увеличении У кирпич сдвинется, и мы будем иметь дело с силой трения скольжения, равной [img и не за- висящей от У. При этом та = У - itmg. Итак, Г У„> - У, У < \kmg-, (У,,, = \kmg, F > \krng, а = 0, У < iimg; F а = ~- [kg, I > [kmg. Графики приведены на рис. а и б. Рис. а Рис. б
Ответы, указания, решения232 б) Поскольку в этом случае сила F направлена под углом к горизонту, величина силы нормального давления изменяется; это, в свою очередь, сказывается на силе трения. Проецируя уравнение второго закона Ньютона на оси координат (см. рис. в), получаем pic] та = Fcosa - Ftp, [у] О = Fsina + N - mg. Отсюда N = mg - Fsina (очевидно, кирпич не отрывается от стола при условии Fsina < mg). Пока кирпич неподвижен (при небольших F): а = О, Ftp = Fcosa. Это возможно, если FTP < pN, т. е. Fcosa < < H(mg - Fsina). Последнее неравенство можно записать в виде V-mg cosa + psina Обозначим правую часть этого нера- венства Fo. При F > Fo начинается скольжение и Ftp = pN = ^(mg - Fsina); Fcosa - p(mg' - Fsina) F(cosa + psina) - pmg CL ~~~ — nt m _ (cosa + psina)(F - Fo) m Таким образом, a = 0, FT₽ = Fcosa при F < Fo; (cosa + psina)(F - Fo) , „ . „ _ mg a =--------------, Fq, = р(т£ - Fsina) при Fo < F <-. m sina Соответствующие графики приведены на рис. г, д. Этот результат нельзя распространить на случай a < О (сила наклонена вниз, прижимая тело к поверхности). Во-первых, при этом отсутствует _ mg . ограничение F <----(тело не может оторваться от стола); во-вто- sma
233 Механика Рис. ж рых, при cos|a| - ц sin|a| < О (ptg|a| > 1) кирпич не сдвинется с места вообще, при сколь угодно большой F (его «заклинивает»). Гра- фики см. на рис. е, ж. 2.25. Ответ: F™* = ~г=^== (см. рис. б). VI + ц Решение. Следует иметь в виду, что при малых углах а ящик будет неподвижен, а при больших станет соскальзывать. В первом случае действует сила трения покоя, во втором — сила трения скольжения. Запишем второй закон Ньютона (см. рис. а): та = mg + N + ГТР. В проекциях на выбранные оси координат: ре] та = mgsina - Ftp, р7] О = N - mgcosa. Пока тело неподвижно, а = О, FTP — mgsina. Этот случай F реализуется, если -т? < ц, т. е. tga < Ц (при tga = ц возможно Рис. а Рис. б
Ответы, указания, решения 234 движение с постоянной скоростью, т. к. а = О). При tga > ц сила трения скольжения задается формулой Ftp ~ \1N = ц/n^cosa (при этом а = g(sina - pcosa) > О). Таким образом, FTP = mgsina при a < arctgp и FT₽ = pmgcosa при a > arctgp. Заметим, что при a = arctgp оба выражения для Ftp дают одно и то же значение: функция FTp(a) получилась непрерывной. ,,, tga График ее приведен на рис. б. Из соотношения sina = г-=_^==- N1 + tg а тл следует, что максимальное значение силы трения г т», = ~т==. VI + щ „ „ „ / 2/cosa 2.26. Ответ: t = ----------. ” gsin(p - a) Указание. Воспользуйтесь тем, что в первом случае сила трения как раз уравновешивает равнодействующую силы тяжести и силы нормальной реакции опоры, — это позволяет найти коэффициент трения ц. (1) 2.27. Ответ: ц = 0,10. Решение. При подъеме и спуске санки движутся с разными ускорениями, поскольку сила трения скольжения направлена противоположно скорости: }аа = g(sina + pcosa), a. — g(sina — pcosa). При движении вверх равна нулю конечная скорость, а при движении вниз — начальная; поэтому в обоих случаях применима _ at^ — , . 2 Пн tc 2 формула s = -х~. Следовательно, a„t„ = ad‘, то есть — = —rt = п . л а. Гц ускорений, имеет вид: Oil то есть — = а< Подставляя в это соотношение выражения для получаем линейное уравнение для ц. Его решение ^2 ____ | ц = —---tga. В данном случае ц = 0,10. п + 1 2.28. Ответ: человек должен бежать по доске с ускорением М\ а = 1 + — |sma, направленным вниз вдоль склона; направление ЛГ его скорости не имеет значения; ц > —tga. Решение. Поскольку доска находится в покое, сумма сил, дейст- вующих на нее, равна нулю. На доску действуют четыре силы (см.
235 Механика рис. а): сила тяжести Mg, сила нормальной реакции со стороны склона N, сила нормального давления человека Q и сила трения со стороны человека Ft. На человека действуют три силы (см. рис. б): сила тяжести mg, сила нормальной реакции со стороны доски Ni = ~Q и сила трения со стороны доски Fz = -Ft (мы дважды воспользовались третьим законом Ньютона). Запишем уравнения второго закона Ньютона для движения доски и человека в проекциях: [х] - Fi + Mgsina = О, [у~| N - Q - Mgcosa = X); |~х] та = mgsina + Fz, [у\ Nt - mgcosa == О (здесь а — величина ускорения человека). Складывая первое и третье уравнения, получаем (с учетом того, что Fi = Fz) a = ^1 + m)sina- С таким ускорением должен бежать человек по доске, чтобы она оставалась в покое; заметим, что это ускорение больше, чем gsina, т. е. больше того ускорения, с каким любое тело соскаль- зывало бы со склона без трения: человек отталкивается от доски, толкая ее вверх, й поэтому сам приобретает дополнительное ускорение, направленное вниз. Кроме того, отметим, что человек не обязательно должен бежать вниз: важно только, чтобы его ускорение было направлено вниз; значит, человек может либо ускоренно бежать вниз, либо замедленно — вверх! Второй вариант не очевиден и часто упускается из виду. Найдем теперь, при каком коэффициенте трения возможно такое движение. Между подош- вами человека и доской действует сила трения покоя — это еще один не очевидный факт: подошвы, отталкиваясь от доски, поко- ятся относительно нее! Значит, сила трения Ftp = Ft = Fz связана F-п, с силой нормального давления Nt = Q соотношением —? < ц. Левая
Ответы, указания, решения 236 часть этого неравенства, как следует из первого и четвертого . ,,, М уравнении системы (1), равна —tga. Следовательно, для коэф- , АГ фициента трения ц получаем условие: ц > -«—tga. Htf нg 2.29. Ответ: Ftp = 4л2тгп2 при г < г—- , Ftp = umg при г > „ „ 4л п2 4л п2 . Решение. Прежде всего надо выяснить, будет ли монета покоиться относительно диска или будет скользить по нему. В первом случае сила трения — это сила трения покоя, она сообщает монете центростремительное ускорение а — 4л2т2 и равна по величине та = 4п2тгп2. Для силы трения покоя должно выполняться условие Ft,, < [kN. В данном случае сила нормального давления равна mg, поэтому получаем неравенство 4л2тт2 < p.mg, откуда " 4л2п2 При выполнении этого неравенства монета будет по- коиться относительно диска и FT₽ = 4л2/пт2 (трение покоя ). Если 2’ pg указанное неравенство не выполняется, т. е. г > - монета 4л2п2 будет скользить по диску; при этом Ftp = pmg. Таким образом, Ц 0* Ftp = 4л2тт2 при г < 2 2 (трение покоя), Ftp = p.mg при pg 4л2п2 (трение скольжения). 2.30. Ответ: Т = 2л /Zcosa V g г > Указание. Согласно второму закону Ньютона, та = mg + F. Отсю- да (см. рисунок) а = gtga. С другой сторо- и2 / gl ны, а =-----. Отсюда v = sina \---- и Zsina V cosa 2xZsina л _ /Zcosa Т =-------=2л*\/----. v Ng При малых а величина Т совпадает с периодом малых колебаний математичес- кого маятника: ведь движение по окруж- ности можно представить как результат одновременных гармонических колебаний маятника в двух взаимно перпендикуляр- ных плоскостях.
237 Механика 231. Ответ: со = Л/- — = 4,7 рад/с, п = — = 0,75 с-1 = 45 об/мин. N г + Zsnict 2л Указание. Шарик движется по окружности радиусом R = г + Zsina; его центростремительное ускорение а = a>2R. Согласно второму закону Ньютона та — mg + Т, откуда а = gtga (см. задачу 2.30). 2.32. Ответ: R = = 5800 м. g Указание. Подъемная сила, действующая на крыло со стороны воздуха, перпендикулярна плоскости крыла; центростремитель- ное ускорение создается равнодействующей подъемной силы и силы тяжести. 2.33. Ответ: щ = 1,4 с 1, <р2 = агссоз/——-) =77°. '4л 1Пг ' Решение. Рассмотрим движение кони- ческого маятника (см. рисунок). Цент- V2 ростремительное ускорение а = — ему придает равнодействующая сил натя- жения нити Т и тяжести mg: та = mg + + Т. Из параллелограмма сил получаем та - mgtgtp. Если равнодействующая сил mg и Т недостаточно велика, чтобы сообщить шарику необходимое центрос- тремительное ускорение, он будет дви- гаться по расширяющейся спирали (это произойдет, например, при увеличении частоты вращения). Если же равнодействующая «чрезмерно» ве- лика, шарик будет двигаться по сужающейся спирали (например, при уменьшении частоты вращения). Учитывая, что г = Zsin<p и v — 2лт, приходим к уравнению относительно <р: sin<p(4n2Zn2--—) = 0 . coscp По крайней мере одно решение (<pi = 0 ) это уравнение имеет при всех п. Второе решение <р2 = arccosf—~~-) появляется при '4л Zn 7 , f, , < 1. т. е. при п > п 4п1п Какой смысл следует приписать наличию двух решений при
Ответы, указания, решения 238 достаточно быстром вращении (п > щ)? Оба они соответствуют состояниям равновесия (в том смысле, что значения <р остаются неизменными). Что же произойдет, если нить случайно откло- нится от вертикали на малый угол а? Для ответа на этот вопрос удобнее всего перейти во вращающуюся (неинерциальную) сис- тему отсчета. В ней при отклонении возникнет сила инерции F„„ = та = mor/sina, направленная от оси вращения, и «возвраща- ющая» сила Ft (равнодействующая сил Т и mg), равная по величине zngtga. Поскольку при малых углах sina » tga, отношение этих сил F,,, _ ыЧ _ 4пЧгЧ F„ ~ g ~ g ' При п > m это отношение больше 1, т. е. > F». Это означает, что случайно возникшее малое отклонение будет нарастать и угол а увеличится до <рг. Таким образом, равновесие при q>i = О является неустойчивым. При желании можно проверить, что второе поло- жение равновесия будет устойчивым. Итак, при п < т угол <pi = g = О. При п > т угол фг = arccos—-—- (реальная система не будет 4пЧпг сколько-нибудь долго находиться в состоянии неустойчивого рав- новесия). Если п = П2 = 3 с'1, то получаем ера = 77°. 2.34. Ответ: Т = 90 Н. Решение. Цепочку удержи- вает на ободе диска сила трения. В момент соска- кивания эта сила, очевид- но, практически исчезает. Это может произойти толь- ко тогда, когда уменьшится до нуля сила давления це- почки на диск (и, соответ- ственно, сила реакции со стороны обода диска). На рисунке показаны силы, действующие в этом слу- чае на малый элемент це- почки (занимающий дугу окружности диска с центральным углом a). Проецируя уравнения движения этого элемента массой Дт на ось х, получаем:
239 Механика * у. Тогда ^2пЧ _ а 2п 2’ Ат • а ~ 2Tsin^, где Т = Ti = Т2. Учтем теперь, что Ат = —т, а - 4n2n2R — центростремитель- 2л _ I .а ное ускорение, R - sin-g ат 2п откуда Т = mln2 = 90 Н, Тонкая цепочка при таком натяжении может порваться. 3.1. Ответ: 2,0 103° кг. Указание. Центростремительное ускорение Земли при ее дви- жении вокруг Солнца а - = 6 • 10 3 м/с2, где Т = 1 год. 3.2. Ответ: в 1600 раз. 3.3. Ответ: h — 3,2 м. Решение. Равноплечие весы находятся в равновесии, когда груз и гири притягиваются к Земле с одинаковой силой. Поскольку гири находятся дальше от центра Земли, их масса должна быть на Ат больше, чем у груза. Условие равновесия имеет вид Mm М(т + Ат) с = G ' (1> Здесь М — масса Земли; R — расстояние от груза до центра Земли, практически совпадающее с радиусом Земли. Из урав- a h(2R + h) „ t „ нения (1) следует, что Ат = т--51---. Поскольку h « R, полу- Л V2 = 3,2 2т . 2Л л чаем Ат = т Отсюда h К, 3.4. Ответ: F = GMm Указание. См. рисунок. Решение. Закон всемирного тяготения можно применить для
Ответы, указания, решения 240 вычисления силы притяжения между материальной точкой и сферически симметричным телом, считая его также материаль- ной точкой, расположенной в центре тела. Попробуем именно к этому случаю и свести нашу задачу. Выясним, на сколько увеличится сила притяжения, если заполнить полость в боль- шом шаре (сделать шар сплошным и однородным). Поскольку объем шара пропорционален кубу его радиуса, объем полости составляет 1/8 всего объема шара. Значит, для заполнения , М полости потребуется шар массы центр которого расположен на расстоянии d - от шарика массы т. Сила притяжения в результате увеличится на Fi = G Мт После заполнения полости притяжение, однако, снова описы- вается формулой закона всемирного ных точек! Следует только учесть, 8М „ _ _ _ 8Мт равна -у-. Итак, F + Fi = G . тяготения для материаль- что масса большого шара Отсюда Мт / 8 3.5. Решение. Если мысленно раз- бить поверхность сферы на мно- жество малых элементов (см. рис. а), то каждый из них «тянет» материальную точку массы т «к себе». Если барон прав, все эти силы должны компенсировать друг друга. Для точки в центре сферы это достаточно очевидно. Для произвольного положения А внутри сферы нужно удачно вы- брать способ разбиения сферы на участки. Проведем через точку А произвольную прямую В1В2. Рис. а Малые элементы сферы в окрестностях Bi и Вг вырежем узкими конусами с вершинами в точке А, полученными вращением обра-
241 Механика зующей DE вокруг оси BtBz (см. рис. б). Полученные малые эле- менты сферы можно считать плос- кими. Они образуют одинаковые углы с осью конусов BtBz (в рав- нобедренном треугольнике OBtBz углы OBtBz и OBzBt равны), и потому их линейные размеры про- порциональны соответственно от- резкам ABi и АВг, а площади — квадратам этих отрезков: S. АД2 „ т;- = • Так же относятся и S2 ав2 Ьт, АВг „ их массы: ----= . Эти участки сферы притягивают точку Дт2 массы т в противоположные стороны с силами Ft й F2: тЛт, тЛт2 F' G ABt2 ’ F* G AB2 ’ Учитывая полученную выше пропорцию, находим Ft = Fz. Эти силы компенсируют друг друга. Но ведь на такие пары элементов можно разбить всю поверхность сферы! Поэтому полная сила тяготения, действующая со стороны сферы на помещенное внутри нее тело, действительно равна нулю. Обратите внимание, что этот результат непосредственно связан с видом закона все- мирного тяготения, согласно которому F ~ г*2. Поэтому были поставлены опыты, в которых закон всемирного тяготения прове- рялся как раз по отсутствию тяготения внутри сферы. Откло- нений от закона пока не обнаружено... 3.6. Ответ: см. рис. б. Решение. Пусть М — масса Земли, R — радиус Земли. Тогда при Мт r> R согласно закону всемирного тяготения F = G . Однако при r<R эта формула уже неверна. Проведем мысленно сферу радиусом г с центром в центре Земли и рассмотрим отдельно силу притяжения к части Земли, ограниченной этой сферой (пусть масса этой части АГ), и к сферическому слою, заштрихованному на рис. а. Однако этот слой можно разбить на множество тонких сферических оболочек. Согласно результату, полученному при решении задачи 3.5, сила притяжения к этим оболочкам (а,
Ответы, указания, решения 242 Рис. а значит, и ко всему слою) равна нулю. Тело в шахте испытывает притяжение «не всей» Земли, а только слоев, лежащих глубже него. Объем сферы пропорционален кубу ее радиуса, поэтому М' г’ „ . Применяя закон всемирного тяготения, получаем FsGM^,=GMn,r Учитывая, что G -=$ = g , запишем К R2 при г < R; F = mg при г > R. при г < R результат в виде: F = mg— it График зависимости F(r) показан на рис. б. Таким образом, наибольшая сила тяготения — на поверхности планеты! Следует, правда, сделать одну оговорку: на самом деле плотность Земли не всюду одинакова, и поэтому приведенный результат для г < R не вполне точен. 3,7. Ответ: v = R\l~^— , Т = . yR + h R V g 3.8. Ответ: г = = 6,67? = 42 • 10я км, Т = 24 ч, v = = '4тг.В' 1 = Ry/^ = 3,1 км/с. Траектория полета должна лежать в экваториальной плос- кости. Указание. Спутник должен делать один оборот за то же самое время, что и Земля при суточном вращении, т. е. за 24 ч. Направление его вращения должно совпадать с направлением суточного вращения Земли. Значения г, v определяются из соот- ношений, полученных при решении задачи 3.7. 3.9. Ответ: 1000 м/с, 27,3 сут.
243 Механика 3.10. Ответ: сила притяжения к Солнцу больше в 2,2 раза. 3.11. Ответ: вес должен уменьшаться на 0,35%; на запад, v = 460 м/с. Решение. Легче всего ответить на второй вопрос. Чтобы вращение Земли не влияло на вес тел в самолете, самолет не должен учас- твовать в суточном вращении Земли. Для этого он должен лететь в направлении, об- ратном суточному вращению (т. е. с востока на запад). Его скорость относительно Земли должна совпадать со скоростью суточного вращения точек земного экватора: и = - 460 м/с, где R — радиус Земли, Т — продолжительность суток. Определим теперь относительное уменьшение веса тел на экваторе, считая Землю однородным шаром. Обозначим через g ускорение свободно пада- ющего тела относительно центра Земли. На тело, покоящееся на экваторе Земли, действует сила тяготения mg и сила N опоры (см. рисунок). Их равнодействующая mg - N придает телу центрост- и2 4n2R ремительное ускорение а = — = , поскольку это тело участ- IX 1 вует в суточном вращении Земли. Отсюда N = m(g- а) = mg(l - . Согласно третьему закону Ньютона, вес Р (т. е. сила давления тела на опору) равен по модулю силе N реакции опоры. На полюсе он равен просто mg (тело не участвует в суточном вращении Земли). Относительное уменьшение веса на экваторе: E = = = 3 5 . 10-з mg gT Итак, вращение Земли приводит к уменьшению веса тел на экваторе на 0,35% по сравнению с полюсом. На самом деле различие в весе около 0,5%. Это объясняется отклонением формы земной поверхности (или, скорее, формы поверхности Мирового океана) от сферической: поверхность «сплюснута» у полюсов. Однако в конечном счете и этот эффект обусловлен суточным вращением Земли. 3.12. Ответ: р = 19 500 кг/м3. „ „ „ „ Мт Указание. Вес тел на полюсе Pi = G - , а на экваторе Рг = Pi - - та (здесь а — центростремительное ускорение точек экватора при суточно’м вращении). По условию, Pi = ЗРг. Учитывая, что
Ответы, указания, решения 244 4 _3 4л2Д М = р • 3ЛД3, a = -yj- получаем р - = 19,5 103 кг/м3. 3.13. Ответ: в 17 раз. Решение. Невесомость на экваторе будет наблюдаться при а = g, где а — центростремительное ускорение точек на экваторе. Имен- но при таком ускорении тела на экваторе будут фактически на- с тт 4л2Д ходиться в состоянии свободного падения. Поскольку а - (здесь R — радиус Земли, Т — продолжительность суток „ Л То То А = 2лЛ/—, то есть сутки следует укоротить в п = -=- = —\ - "g 1 2л Т = раз (здесь То = 24 ч). Подстановка дает Т = 5080 с = 1 ч 25 мин (что, конечно, совпадает с периодом обращения спутника по близкой к Земле орбите) и п. = 17. «Побочные эффекты» будут ужасающие! Молекулы атмосферы из-за трения о Землю будут разгоняться, и даже при гораздо меньшем увеличении скорости вращения Земли значительная их часть получит возможность преодолеть земное тяготение и улететь в окружающее космическое пространство. Земля начнет быстро терять атмосферу. При наступлении на экваторе невесомости этот процесс станет катастрофически быст- рым — возникнут могучие воздушные потоки от полярных облас- тей к экватору, а от экватора — в открытый космос. Остается надеяться, что этот подвиг барону не удастся! 3.14. Решение. Сила Архимеда обусловлена увеличением давления с увеличением глубины (за счет веса жидкости или газа), и потому в невесомости сила Архимеда просто отсутствует. Закон Паскаля утверждает, что внешнее давление передается в любую точку жидкости или газа. Эта способность к передаче давления обуслов- лена подвижностью жидкости и газа и не имеет отношения к весу. Следовательно, закон Паскаля остается справедливым и в невесо- мости. 3.15. Ответ: да, достаточно; р = (здесь G — постоянная все- мирного тяготения). 3.16. Ответ: t = 2 ч. Решение. Читатель, конечйо, догадался: как бы ни был могуч Мюнхаузен, даже он не может изменить скорость спутника, оставаясь внутри него (это противоречило бы закону сохранения
245 Механика ^7 . Здесь Та — период обращения по орбите А; Ьв. постоянная. Учитывая, что импульса). Однако такое изменение скорости действительно мо- жет быть получено за счет кратковременного включения тормоз- ных реактивных двигателей. Очевидно, время t представляет собой половину периода обращения Тв спутника по орбите В: 1 2 • Согласно третьему закону Кеплера (он изучается в курсе астро- х Тв2 номии), -=г^ 1 А Ьа — большие полуоси соответствующих орбит. Очевидно, Ьа = 3/?; Ьв = 2R (здесь R — радиус Земли). Период Та движения по круговой орбите можно найти, исполь- зуя закон всемирного тяготения и второй закон Ньютона: _ тМ та = G------ , (З/?)2 где т — масса спутника, М — масса Земли, G — гравитационная 4л2 • 3/? „М а = уГг—, g = G , получим _2 108 л2/? _ _2 32л2/? Та =--------. Тогда Тв =------- g g 4.1. Ответ: для мяча KP = 32 Н, в случае наклонной поверхности FcP = 28 Н; для пластилинового шара Fcp = 18 Н в обоих случаях. Решение. Судя по высоте подскока мяча, его удар об асфальт можно считать абсолютно упругим: и = -vo (ио — скорость мяча до удара, и — непосредственно после удара). Эти скорости равны по величине v = и0 = ^2gH. Из второго закона Ньютона следует, (Fe„ + mg)t = mv - mvo. ч. что (1) Отсюда „ 2mJ2gH Гец— Удар пластилинового шара не- упругий и v = 0. Поэтому Fcp = m^2gH =----— + mg - 18 Н. Этот ответ для пластилиново- го шара остается в силе и при ударе о наклонную поверхность: ведь конечная скорость шара равна нулю независимо от на- клона поверхности. В случае же отскока мяча от наклонной по- + mg - 32 Н. и t =
Ответы, указания, решения 246 верхности меняется направление скорости v (см. рисунок). При упругом ударе действующая на мяч со стороны площадки сила (сила упругости) направлена нормально к поверхности площадки. Проекция скорости на ось у при ударе меняет знак: щ = -щ,. Проецируя векторное равенство (1) на ось у, получаем (Еср - i2^2gH \ - mgcosa)t = 2nwcosa, откуда FCI, = zncosa|—- + gj = 28 H. Заметим, что при H » gt2 вторым слагаемым в скобках можно было бы пренебречь (сила упругости намного превышает действу- ющую на мяч силу тяжести). 4.2. Ответ: F = 2р Sv2cosa = 135 Н. Указание. См. задачу 4.1. За время At на стену попадает вода массой т = р SuAt (здесь р — плотность воды). 4.3. Ответ: 150 м/с (барон неплохо «толкнул» ядро). Указание. Ядро должно «унести» весь импульс, поскольку импульс барона после соскока с ядра равен нулю. Масса ядра без барона втрое меньше, чем масса ядра с бароном; поэтому скорость ядра должна увеличиться втрое, чтобы произведение mv сохранилось. 4.4. Ответ: а) увеличить в 4 раза; б) увеличить в 3 раза. Решение. При неупругом столкновении с ракетой каждая пы- линка приобретает ее скорость и (скорость ракеты из-за ее большой массы при одном соударении изменяется пренебрежимо мало). Если бы не работа двигателя, ракета теряла бы импульс, передавая его пылинкам. Импульс силы тяги двигателя должен компен- сировать передачу импульса пылинкам за любой промежуток времени At: FAt = Nmv (здесь т — масса пылинки; W — число частиц, столкнувшихся с ракетой за время At). Очевидно, W зависит от скорости v и «густоты» облака частиц. Обозначим через п концентрацию частиц в облаке. Если площадь поперечного сечения ракеты S, то за время At она столкнется со всеми час- тицами в объеме Sv At. Значит, W = nSvAt и F — nmv2S. Мы видим, что F пропорциональна концентрации частиц п и квадрату ско- рости и. Итак: чтобы двигаться вдвое быстрее, силу тяги надо увеличить в 4 раза; в более плотной части облака силу тяги надо увеличить в 3 раза. 4.5. Ответ: на расстояние Hsin2 Л Решение. На бревно при падении действуют только сила тяжести и сила реакции опоры. Обе эти силы направлены вертикально, поэтому они не могут изменить горизонтальной проекции импуль-
247 Механика са бревна; следовательно, эта проекция равна нулю в любой момент времени. Это, однако, означает, что равна нулю и горизонтальная проекция скорости центра масс бревна О. Поэтому при падении точка О (совпадаю- щая с серединой бревна) движется по верти- кальной прямой. Из рисунка видно, что нижний конец бревна сместится на величину АА = А'О' - АО' - -^-(1 - cosa) = Hsin2^. и 4.6. Ответ: лодка сместится на s =----1 = 1 м. М + т Указание. См. задачу 4.5. 4.7. Ответ: лодка сместится на s = (т. - т,н=0 71 м М + /П1 + ni2 Указание. См. задачу 4.5. . Q ~ /TWoCOSa 4.8. Ответ: и --— = 1,25 М . „ „ m^2gH cosa 4.9. Ответ: и =--------. М + т м/с. 4.10. Ответ: по окружностям с радиусами М + т tnR и------ М + т Решение. На гладкой горизонтальной поверхности на систему «жук — обруч» не действуют внеЩние горизонтальные силы. Значит, центр масс всей системы О (см. рис. а) должен оставаться неподвижным, а жук А и центр обруча В будут как-то двигаться «вокруг него». По каким траекториям? Какие бы новые поло- жения А и В' (см. рис. б) ни занимали жук и центр обруча, А'В' - А'О + В'О = R. Отрезки А'О и В'О обратно пропорциональны А'О М „ массам тел:---= —. Отсюда В'О т А'О М + т М + т Рис. а Рис. б
Ответы, указания, решения 248 Поскольку длины этих отрезков неизменны, а точка О не- подвижна, точки А и В перемещаются по окружностям с общим центром в точке О и с радиусами, равными соответственно MR mR ------и--------. М + т М + т 4.11. Ответ: ц = "f—’— . Mgfcosa Решение. Поверхность жидкости устанавливается параллельно склону, если ускорение бака а = gsina (см. задачу 2.23). Как раз с таким ускорением соскальзывал бы бак без трения. Значит, «реактивная» сила тяги за счет вытекания струи воды как раз уравновешивает силу трения FTP = pMgcosa. Эта сила равна импульсу, уносимому вытекающей водой за единицу времени: „ А/п v pSvAt v „ , F =------= --------= pSu , At At где A/n — масса воды, вытекающей за время At; р — плотность pSv2 воды. Отсюда ц =------. Mgcosa . _ /11 1 \ 4.12. Ответ: и„ - mv + —------+ ... + —------——I . ^М М-т М - {п. - 1)т' Решение. Барон описал типичный случай реактивного движения: тело разгоняется, отбрасывая назад некоторую часть своей перво- начальной массы. Полный импульс системы при этом остается неизменным. Важно учесть, что с каждой отброшенной косточкой ракета становится легче и при очередном «толчке» ее скорость увеличивается сильнее. Найдем увеличение скорости Ап* ракеты при выплевывании бароном /г-й косточки. Это удобнее всего сде- лать в той системе отсчета, в которой послё этого ракета не- подвижна (и* = .0): (М - (k - l)m)uk-i = -то, откуда Auk = /по = Uk - Uk-i =-----------. Изменение скорости, разумеется, одно М - (k - 1)/п и то же во всех системах отсчета. Поэтому п и„ = Aut + Au2 +...+ Aun = £ Aut = k=i V"1 1 /11 1 \ ~ M- (k - l)m VM M-m M-(n- l)m> К сожалению, для такой суммы нет простого выражения, но для больших п можно заменить сумму интегралом (если считать, что косточки выплевываются
249 Механика fdt , т —----— (здесь ц = — — секундный иМ - цТ at М расход топлива; Т — время разгона). Отсюда и = uln- — знаменитая М - цТ формула Циолковского. Величина М - ц Т представляет собой массу ракеты к моменту Т. 4.13. Ответ: А = 1,2 кДж. Решение. При подъеме приходится пре- у одолевать силу тяжести только той час- ти веревки, которая все еще свешивает- ^8 ся вниз. Если длина уже вытянутой /////^к части веревки х, то требуется прикла- '////'///Ъъ^ mg(l — х) > / А /// дывать силу F =-----------. Работа при х/////////////Ъ>^ изменении х от О до I численно равна х площади под графиком функции F(x) (см. рисунок). Очевидно, А = —х—. Иначе (и проще) то же можно получить, вычисляя работу по подъему тела как А = mg&h, где Ай — изменение высоты центра тяжести тела. Центр тяжести висящей веревки находится в ее I mgl середине, поэтому Дй - -х и А = —= 1,2 кДж. 4.14. Ответ: А = ^—^(щ + щ). Указание. В ходе движения сила трения изменяется от щ/ng до fizmg, причем ее зависимость от пройденного пути линейная. 4.15. Ответ: в обоих случаях А = -fimgL. Решение. В случае а ответ получается сразу: А = -Ft^L = -\imgL (мы учитываем, что сила трения Етр направлена противоположно перемещению ящика относительно поезда). Рассмотрим случай б'. относительно Земли перемещение ящика равно s - L, где s — перемещение поезда за время скольжения ящика по крыше. Сила трения совершает над ящиком работу Ai = FTp(s — L), но на вагон действует такая же по величине сила трения со стороны ящика, направленная против движения поезда. Она совершает работу Аг = = - Утрв. Полная работа силы трения составит: А = Ai + Аг = -Ft₽L = -\mgL. В любой инерциальной системе отсчета эта работа одинакова, что неудивительно: она показывает, какое количество теплоты выделяется в результате трения ящика о крышу вагона. Эта величина определяется перемещением одного из трущихся тел относительно другого .
Ответы, указания, решения 250 4.16. Ответ: Рср = 120 кВт. Решение 1. Напомним, что под укло- ном дороги понимают синус угла ее наклона к горизонту. Обычно этот угол невелик, что позволяет считать sina = tga = a; cosa = 1. Работа двигателя А, равная Pcpt, сообщает автомобилю кинетическую энергию Ми2 Wk = , потенциальную энергию увеличивает внутреннюю энергию автомобиля, дороги и окружаю- щего воздуха на величину работы против силы трения: Q = F„,s = pMgs • cosa » pMgs. s Итак, Pq, t = —+ Mgs sina + pMgs. Учитывая, что t = — = WP = Mgh = Mgs- sina и 2s = —, получаем: _ Mvi v2 " 2s V 2 sina + pgs) = 120 кВт. v2 Решение 2. Из условия следует, что ускорение автомобиля а = V поскольку Р = Fv и Р..„ = Fv,v = F-^, необходимо найти силу тяги * л F. Согласно второму закону Ньютона Ma = F + Mg + N + FTP (см. рисунок). В проекциях на оси х и у. Ma = F - Mgsina - Ртр; 0 = = N - Mgcosa. Учитывая, что Ртр = Ц/V, находим 2 2 F - м(—~ + gsina + pgcosal ~ м(^~ + gsina + pg) . Отсюда получаем результат, приведенный в решении 1. 4.17. Ответ: Решение. Совершаемая двигателем за время At работа А равна кинетической энергии выбрасываемых за это время газов (ведь Д/пи2 энергия самой ракеты не изменяется): А = —~, где Лтп — масса выброшенных за время At газов. Поэтому мощность двигателя А Атиг _ N = — = . Остается определить скорость расхода топлива МП п „ ---. ото нетрудно: сила F, с которой газ действует на ракету, At
251 Механика уравновешивает силу тяжести, т. е. F — Mg. Такая же по величине сила действует со стороны ракеты на газ, сообщая ему за время At импульс FAt. Этот импульс уносится струей вытекающего газа: л гх А/П т, - „ MgU Mg At = A/nu. Отсюда---------. Таким образом, N = —5— . At и 2 4.18. Ответ: А = 2,0 кДж; АЖР = 0,98 кДж. Решение. Совершаемая работа А — Fh = 2,0 кДж. Приобретаемая ведром потенциальная энергия AWV = Mgh = 0,98 кДж. Итак, А > > АЖР. Казалось бы, на подъем груза израсходовано больше энергии, чем груз приобрел. Обратим, однако, внимание, что F > F - Mg > Mg, поэтому груз будет подниматься с ускорением а - ——. К концу подъема он приобретет скорость v = ^2ah Mv2 а, значит, и кинетическую энергию Wk = = h(F - Mg), Изме- ни нение механической энергии ведра АЖ = АЖР + Wk = Fh равняется совершенной работе. Возможен, конечно, и случай АЖ *А — если на ведро действует какая-то сила сопротивления. Тогда А = ДЖ + + Q, где Q — увеличение внутренней энергии взаимодействующих тел. 4.19. Ответ: А = = 612,5 Дж, АЖ„ = mgh = 490 Дж, F - та тгвЬ.а Wk = —= 122,5 Дж. F - та о2 4.20. Ответ: s =---- = 35 м. 2gtga 4.21. Ответ: А = 650 кДж. Решение. На рис. а показано начальное положение воды в бассей- не, на рис. б — конечное. Конечные высоты уровня воды в половинках бассейна определены из условия неизменности объема '2h(F - Mg) М Рис. а Рис. б
Ответы, указания, решения 252 (Sfl\ rr воды в каждой половине Поскольку перемещение перего- родки было медленным, вода не приобретала при этом кинетичес- кой энергии. Значит, совершаемая работа А идет на увеличение потенциальной энергии системы. Для каждой «половины > бассей- на эта энергия равна mgH, где т — масса воды в одной половине - „ / Sh\ бассейна (т = p-g-j, Н — высота центра тяжести, равная половине высоты уровня воды. Итак, до передвижения перегородки: Л W„ = 2mg 7, = mgh. После передвижения: 2. 2Л 3 4 L W„ = mg~Y + mg-^ = -gngh. Следовательно, A = = W„2 - W,,i = = 650 кДж. D О 4.22. Ответ: P = 33 кВт. Решение. Прежде всего заметим, что при подъеме и спуске на автомобиль действовала одинаковая по величине сила сопротивления Копр- Найдем Fc<>np. Так как автомобиль спускался равно- мерно, Fconp = mgsina. На рисунке по- казаны силы, действующие на авто- мобиль при подъеме. F — сила тяги, возникающая благодаря работе двигателя вследствие трения по- коя ведущих колес о дорогу. Поскольку ускорение отсутствует, F + + mg + N +Гсопр = 0. Проецируем это векторное равенство на ось х: F - mgsina - Fconp = 0, откуда F = 2mgsina. Тогда развиваемая мотором мощность Р = Fv = 2mgusina = 33 кВт (по условию, sina = 0,05). На самом деле приведенное решение справедливо только в том случае, когда спуск и подъем происходят с одинако- вой скоростью. Ведь при такой скорости важную роль играет сила сопротивления воздуха, а ее величина заметно растет с увели- чением скорости. 4.23. Ответ: р. = ctg = 2,4. Решение. Поскольку скорость движения v и мощность двигателя N на обоих этапах движения одинаковы, должна быть одинаковой
253 Механика и сила тяги F |F = — j. Кажется странным, что на подъеме и горизонтальном участке сила тяги может быть одной и той же, однако такое возможно: на наклонном участке уменьшается сила дарления ящика на дорогу, а, значит, и сила трения. На горизон- тальной дороге Fi = ц mg, где т — масса ящика. При равномерном движении на подъеме сила тяги F2 = mgfsina + ц cosa). Поскольку п п sina . a „ . Fi = F2, получаем ц =-----= ctg»- = 2,4 . 1 - cosa 2 Итак, в принципе описанная бароном ситуация возможна. Однако такое большое значение коэффициента трения полностью на совести рассказчика. 4.24. Ответ: v = 7,2 м/с. Решение. При малом угле а наклона дороги сила трения Ftp = = nmgcosa » pmg, т. е. практически та же, что и на горизонтальной дороге. Значит, сила тяги на подъеме равна Ftp + mgsina, на спуске Ftp - mgsina, а на горизонтальном участке — Ftp. Из условия постоянства мощности двигателя получаем N = mg + mgsina) = иг(ц mg - mgsina) = vpmg. NN 2N 2viVz Отсюда — + — =-----, следовательно, v =-------= 7,2 м/с. Ui V2 V Pi + p2 4.25. Ответ: N = ndn\iF = 38 Вт. 4.26. Ответ: 0 = arccos (2cosa — 1) Решение. Прежде всего обра- тим внимание, что в точках А и В (см. рисунок) обезьяна на- ходилась на одинаковой высо- те: поскольку скорость (а вмес- те с ней и кинетическая энер- гия) в этих точках обращается в нуль, потенциальная энергия обезьяны в них должна быть одинаковой. Если обозначить длину лианы I, то высота h по- дъема обезьяны в крайних поло- жениях относительно нижнего положения С составит: h — ОС - OF = 1-1 cos а ил! h = DC - DF = 4 - 4cos0. & и Отсюда p = arccos(2cosa - 1). При a < 60° получаем p < 90°,
Ответы, указания, решения 254 при этом лиана все время остается натянутой. При больших же значениях угла а угол р превысит 90° и в какой-то момент при Р = Ро > 90° натяжение лианы упадет до нуля, она провиснет, а обезьяна начнет двигаться по параболе. 4.27. Ответ: ц = 0,05. Решение. Кинетическая энергия санок равна нулю как в начале, так и в конце пути. Значит, начальная потенциальная энергия санок mgH целиком расходуется на работу против силы трения (т. е. потенциальная энергия переходит во внутреннюю). Посколь- ку сила трения на склоне горы с углом наклона а равна p.mg'cosa, а на горизонтальном участке \\rng, получаем: mgH = = [imgcosa^ L + [img'S, или mgH = [img(a + s). Здесь L — длина н склона горы. Мы учли, что Lcosa = а. Отсюда ц =----------= 0,05. а + s Заметим, что ответ не зависит от того, в какой именно точке заканчивается склон (формула для ц содержит лишь сумму а + s). Можно сказать, что полное горизонтальное перемещение санок не зависит от крутизны горы. Оно определяется лишь ее высотой и величиной ц. Для чего в условии упомянута плавность перехода наклонной поверхности в горизонтальную? Дело в том, что при движении ио криволинейной траектории меняется вес тела (см. задачу 2.2), а вследствие этого — и сила трения. Если радиус о2 кривизны траектории равен к, то центростремительное ускорение тела равно -=, rf „ ти2 _ а изменение веса не превышает величины ДР = . Поскольку для санок на rf др 27/ 7/ выезде с горы и2 < 2gH, получаем — < . При „ «1 изменением веса и силы mg а а трения можно пренебречь. Следовательно, полученный результат действительно справедлив при достаточно плавном переходе (при R > Н). 4.28. Ответ: v = 2и = 20 м/с. Решение. Прежде всего заме- тим, что соударение ноги и мя- ча можно считать упругим и центральным. Кроме того, мас- са ноги во много раз превыша- ет массу мяча. Поэтому ско- рость ноги при ударе практи- чески не меняется. Упругое столкновение легкого и массивного тел удобнее всего рассмат- ривать в системе отсчета массивного тела (в данном случае — ноги). Й этой системе отсчета скорость легкого тела при ударе
255 Механика меняет направление, оставаясь неизменной по модулю (иначе изменилась бы после удара ки- после удара нетическая энергия). В нашем случае до удара скорость мяча относительно ноги равна и; значит, после удара она остает- ся такой же (см. рисунок). Тог- да скорость мяча относительно земли после удара v = 2и согласно правилу сложения скоростей при переходе из одной системы отсчета в другую. 4.29. Решение. С какими телами взаимодействует шар, т. е. откуда он может получать энергию? На шар действует сила тяжести со стороны Земли и сила Архимеда со стороны воздуха. Значит, нужно рассмотреть процессы в замкнутой системе Земля — шар — воздух. При подъеме «освободившееся» место шара заполняется окружающим воздухом. Можно сказать, что шар «меняется местами» с воздухом, объем которого равен полному объему шара: шар поднимается, а воздух опускается. Вес этого воздуха больше веса шара, иначе ведь шар не будет подниматься! Итак, более легкий груз (хпар) поднимается, а более тяжелый — воздух — опускается на такое же расстояние. Разумеется, по- тенциальная энергия шара растет за счет убыли потенциальной энергии воздуха. Полная же потенциальная энергия системы убывает. т М 4.30. Ответ: v = 2 , скорость бу- 771\ it Ш \2г 2 м)+11 + а дет направлена под углом р = arctg [(1 + ^)tga] к горизонту. Решение. При закрепленном клине тело приобрело бы скорость Vi = ^2gh. При незакрепленном — скорость наверняка будет мень- ше: часть начальной потенциальной энергии тела перейдет теперь в кинетическую энергию клина. Обозначим через V (см. рис. а) скорость, приобретенную клином. Мы можем воспользоваться , AjV2 ти1 законом сохранения энергии mgn - —5— Н—и законом сохра- & и нения импульса в проекции на горизонтальную ось (поскольку отсутствует сила трения): -MV + mv* = 0. Учтем также, что скорость тела относительно клина v - V направлена под углрм a
Ответы, указания, решения 256 Рис. б Vy. к горизонту (см. рис. б): (vx + V)tga =• Итак, мы получили систему четырех уравнений: , MV2 mif mgh = -^- + -j-, -MV + mvx = О, V2 = Vx2 + v„2, , (Vx + V)tga = vy. Решая эту систему, находим т —5---------------1, =<1 + м)+ + мУ^ Угол Р наклона скорости v к горизонту определяется из условия =й=(1 + S)tga- Если т « М, получаем v ® ^2gh, р » а. Если т > М, то v ® ^2gh, р » 5 (тяжелое тело «не чувствует» легкой горки и & падает почти отвесно). v2 = 2gh I 1 - . „ пМ + т гт . a 4.31. Ответ: Vo = 2--ygl sin^r. т 2 Решение. Это знаменитая задача о так называемом баллистичес- ком маятнике. Советуем обратить на нее особое внимание. Наи- более распространенная ошибка здесь — попытка применения закона сохранения механической энергии: «кинетическая энергия Wk пули переходит целиком в потенциальную энергию Wv от- клонившегося груза». Это неверно, потому что механическая энергия не сохраняется: при попадании пули в брусок большая часть Wk переходит не в потенциальную энергию, а во внутрен- нюю: оба тела нагреваются. Для решения задачи весь процесс следует разбить на два этапа: первый, очень короткий — соуда- рение пули с бруском. При этом брусок практически не успевает
257 Механика сдвинуться с места, но приобретает скорость и. Закон сохранения механической энергии не выполняется, но выполняется (как при всех столкновениях) закон сохранения импульса:1’ mvo = (М + + т)и. На втором этапе брусок с застрявшей пулей за счет полученной скорости и отклоняется на угол а и при этом при- поднимается на высоту h = Z(1 - cosa). На этом этапе не происходит перехода механической энергии во внутреннюю, потому меха- (М + т)и2 ... . , _ ническая энергия сохраняется:-%----= (М + m)gn . Отсюда ТП21Л)2 2gl(l - cosa) = —-- , (М + ту М + т • a 1Л> =----V2gZ(l - cosa) = 2---ygl 8Шт . m m2 4.32. Ответ: M = 1 кг. Решение. Почему во второй раз пуля вошла в сыр глубже? При втором выстреле кинетическая энергия пули перешла только во внутреннюю энергию, тогда как при первом выстреле часть энер- гии пули перешла в кинетическую энергию головки сыра (вспом- ним, что в момент выстрела головка покоилась!), да еще и сама пуля, застрявшая в сыре, тоже сохранила часть кинетической энергии. Таким образом, в первом случае закон сохранения энер- mv2 (М + т)и2 „ ~ гии принимает вид —г— = ——=------+ Fsi . Здесь vo — скорость 25 £ пули; и — скорость головки сыра после удара пули; Fsi (работа против силы сопротивления) показывает, сколько механической энергии перешло во внутреннюю. Во втором случае закон сохра- нения энергии имеет вид —~0~ = Fs2 . Согласно закону сохранения импульса mv<> = (М + m)u. Решая полученную систему уравнений, находим „ mvn m2v02 Mmvo2 М „ Fsi = п -----------=----------= ------Fsz, 2 2(М + т) 2(М + т) М + т msi , откуда М --------- 1 кг. «2 - S1 Если бы головка сыра была легче, разность - si оказалась бы большей (попробуйте вбить гвоздь в подвешенный на веревке деревянный брусок!). ** Мы считаем нить достаточно длинной, так что небольшое перемещение бруска за время соударения практически не отклоняет ее от вертикального направления, и проекция силы натяжения нити на горизонтальную ось равна нулю.
Ответы, указания, решения 258 4.33. Ответ: Н = 100 м. Решение 1. При полете в воздухе человек приобретает скорость и = "42gh. В воде эта скорость уменьшается до нуля. В отсутствие сил сопротивления на человека действуют только постоянные силы (сила тяжести и сила Архимеда), поэтому движение можно „ Sh считать равноускоренным; тогда Н - , где а — ускорение человека при движении в воде. Согласно второму закону Ньютона, ш|а| = Fa - mg, где Fa = pVg — сила Архимеда, ар — плотность воды. Отсюда Я = —-—Л = ЮЛ = 100 м. pV - т Решение 2. В отсутствие сил сопротивления механическая энергия замкнутой системы тел сохраняется. Однако не забудем включить в замкнутую систему воду и Землю! В начале прыжка и в момент наибольшего погружения скорость человека равна нулю, т.е. кинетическая энергия отсутствует. Значит, потенциальная энер- гия WP системы в эти моменты одинакова: AWp = 0. Но ведь человек потерял высоту Я + Л, а, значит, ДИЛ, = -mg(H + Л). У какой же части системы энергия при этом увеличилась? Ответ прост: у воды! Погружаясь на глубину Я, человек вытеснил и заставил подняться ца такую же высоту Я воду массой pV, увеличив ее потенциальную энергию на pVgH. Итак, -mg(H + Л) + pVgH = 0. Отсюда Я = ——Л = 100 м. pV - т Решение 3. Воспользуемся тем, что полная работа всех приложен- ных к телу сил при перемещении из верхней точки в нижнюю равна изменению его кинетической энергии (в данном случае — нулю). Это приводит к соотношению mgh - (Fa ~ mg )Н = 0 откуда г г mSh т , .пп Н =--------=-------h = 100 м. Ft, - mg pV - т Глубина погружения получилась неправдоподобно большой (на самом деле при таком прыжке человек погружается в воду всего на несколько метров). Дело в том, что силы сопротивления (осо- бенно при движении в воде) велики и пренебрегать ими нельзя. 4.34. Решение. Найдем соотношение между скоростями vi и иг частей вагона после взрыва и их скоростями ui и иг после первого столкновения. Общий импульс системы после взрыва должен остаться равным нулю, поэтому miVi = mzVz. При дальнейшем движении импульсы частей вагона изменяются только по направ- лению (за счет взаимодействия с рельсами). Однако кинетическая
259 Механика энергия частей вагона в отсутствие трения измениться не может (в силу закона сохранения энергии), поэтому модули скоростей Vi и Vt постоянны. Перед первым столкновением импульсы частей вагона опять направлены противоположно, и общий импульс равен нулю. Значит, и niiui = тпгПг, т. е. отношение скоростей двух частей вагона не изменяется в результате их столкновения: Hi _ Vi _ ГП> и> v> mi' Так же относятся и пройденные ими от взрыва до первого столновения пути si и «2 (см. рисунок). Учитывая, что si + 82 == L и Si Vi Шг — = — - — получаем: Зз V2 1111 mi . пи т Si =------L, S2 =-------L. nil + m> mi + пи Но от первого столкновения до второго части вагона пройдут в обратном направ- А лении такие же точно расстояния (ведь У г отношение скоростей осталось прежним)! / / \ \ Значит, второе столкновение произойдет / \ как раз в точке Л. Вагон не только «срас- I I тается», но и останавливается на прежнем \s, / месте. Никаких следов от взрыва (если не X. считать, конечно, исчезновения мешка с ' порохом)! Заметим, что для пас несущественно, было ли первое столкновение упругим или нет. «Сработал» лишь закон сохра- нения импульса. А вот отсутствие трения при движении очень важно! 4.35. Ответ: 11 куропаток. Решение. Разумеется, всех куропаток нужно рассматривать как одинаковые препятствия для пули. Ответ, который напрашивает- ся, — «добычей станут все 20 куропаток» — будет опрометчивым. Ведь скорость пули не уменьшается на одну и ту же величину при последовательном пробивании каждого из одинаковых препятст- вий (просто потому, что каждое следующее препятствие преодоле- вается за большее время). Какая же «потеря» одинакова в каждом из пробитых препятствий? Пуля преодолевает в каждом из них одинаковую силу сопротивления и проходит одинаковый путь, следовательно, теряет каждый раз одну и ту же энергию AW на совершение работы по преодолению силы сопротивления. Если масса пули т, а начальная скорость Vo, то ЛИ- = = 0.0975 ар. л л л 9*
Ответы, указания, решения 260 Начальная энергия нули —составляет 10,3 таких « порций ». Значит, добыча барона составит 11 куропаток: 10 из них будут пробиты насквозь, а в одиннадцатой пуля застрянет. 4.36. Ответ: ui = вз, из = wi (шары «обмениваются» скоростями). Указание. При упругом ударе не происходит перехода кинетичес- кой энергии в другие формы энергии. Воспользуйтесь законами сохранения импульса и энергии. 4.37. Ответ: прямой угол. Решение. Пусть скорость налетающего шара Во, скорости шаров после удара Vi и из, масса каждого из шаров т. Запишем законы сохранения энергии и импульса: mvo2 mvi2 mv-2 —+ —5— , mvo = mvi + mvi . Л Л Л Отсюда vo = Vi + Уз, (1) vo2 = ui2 + V22. (2) Возводя первое уравнение в квадрат и вы- читая из него второе, получаем V1V2 = 0, т. е. шозсова = 0, где a — угол разлета шаров. Ж Поскольку при нецентральном ударе Vi * 0 и 1,1 иЛ оз * 0, получаем cosa = 0, т. е. a = 90°. Уравнения (1) и (2) допускают и простую геометрическую интер- претацию. Уравнение (1) означает, что векторы во, Vi и вз образуют треугольник (см. рисунок). А,уравнение (2) справедливо только в том случае, когда этот треугольник — прямоугольный. 4.38. * Ответ: см. рисунок. Решение. Пусть до удара скорость первого шара была во, а после удара Vi. Скорость второго шара после удара обозначим из.11 Тогда законы сохранения импульса и энергии принимают вид rrnvo = mivi + тпз1>з; miVo2 _ niiVi2 mzVi2 2 ~ 2 + 2 * 4 Точнее, ui и из — проекции соответствующих скоростей иа направление скорости но. Как показано ниже, щ < 0 при mi < m2. Это означает, что налетающий шар отлетает назад при столкновении с более массивным шаром.
261 Механика Отсюда получаем систему уравнений: (mi(v0 - vi) = m-iVi, lmi(t?o2 - ui2) = m2V2. Разделив второе уравнение почленно на первое, получим vo + + vi = V2. Подставляя V2 в первое уравнение системы, получим mi - тг 2mi ui =-------Vo . Тогда Vz =------Vo . mt + m2 mt + т2 тт тгг т^2 Начальная энергия первого шара Wo = —~—, а энергия, передан- & m2V2 ная второму шару, W2 = —5—• Поэтому доля переданной энергии _ W2 _ 4тП)7П2 _ 4k Wa (пи + m2)2 (1 + k)2 График зависимости a(k) приведен на рисунке. Обратите вни- мание, что наиболее эффективная передача энергии происходит при значениях k, близких к единице, а при k — 1 вся энергия налетающего шара передается другому шару. Итак, энергия луч- ше всего передается при упругих столкновениях частиц равной массы. Этот результат очень важен, например, при выборе вещес- тва — замедлителя нейтронов. Приходится его учитывать и фи- зикам-ядерщикам при попытках получить горячую плазму, необ- ходимую для осуществления термоядерных реакций. Плазма (иони- зированный газ) состоит из частиц двух типов: легких электронов и тяжелых ионов, причем отношение их масс составляет десятки и сотни тысяч. Разогревая плазму, увеличивают энергию элект- ронов. Однако передача энергии от электронов к ионам при столкновениях ничтожно мала. Переход к тепловому равновесию между электронами и ионами — процесс очень медленный. В результате возникает так называемая двухтемпературная плазма: «горячие» электроны.среди «холодных» ионов. 4 on ru. TIZ Ш1Ш2(Р) - V2)2 4.39. Ответ: W =------------. 2(/П1 + m2) Решение 1. В какой момент энергия упругой деформации пру- жины (а, значит, и сама деформация) максимальна? Очевидно, когда вагоны на мгновение оказались неподвижными один отно сителъно другого, т. е. в тот момент, когда они движутся с одинаковой скоростью и относительно Земли. Тогда законы сох- ранения импульса и энергии принимают вид: mivi + тя>2 - (mi + m2 )и, ” ’ nwi2 тгОг2 (mi + m2)u2 -2~ + -2- =--------2----+ W-
Ответы, указания, решения 262 Выражая и из первой формулы и подставляя это значение во вторую, получаем mimz . ,2 W = „----------(v> - • 2(ГП1 + mJ Если бы вагоны двигались навстречу друг другу, разность величин скоростей, естественно, следовало бы заменить на сумму: ведь зто просто начальная скорость одного вагона относительно другого. Интересно, что при абсолютно неупругом ударе тех же вагонов переход кинетической энергии в другие формы измеряет- ся той же величиной W. Это неудивительно: если в момент наибольшего сжатия пружины буфера сработает какая-нибудь защелка и не даст пружине выпрямиться, то удар тем самым превратится в абсолютно неуцругий. Решение 2. Можно решить эту задачу и в системе отсчета, связанной с центром масс системы. Скорость этой системы отсчета относитель- „ тал + miVt но Земли 1>Пм ----------, а скорости вагонов в этой системе отсчета mi + т>. , mAvi - i>a) , - Vi) V1' = 1>1 - =----------И 1>2 = 1>2 - 1>т - ------- • nit 4- т- mi 4- т> Суммарный импульс вагонов в системе центра масс равен нулю, а в момент максимального сжатия пружины оба вагона оста- навливаются. При этом вся их начальная кинетическая энергия переходит в потенциальную энергию деформированной пружины: jy _ т >У2,г _ mtmdvi - и>)' 2 2 2(/П) 4 т>) 4.40. Ответ: Больший шар будет двигаться со скоростью , 2и меньший — со скоростью --?= под углом 30° к начальной скорости \3 большего шара. 4.41. Решение. Поскольку между биллиардными шарами имеются промежутки, в первый момент соударения происходит взаимо- действие только между шарами 1 и 2. В результате (см. задачи 4.36, 4.38) шар 1 останавливается, полностью передав свой им- пульс и энергию шару 2 (столкновение биллиардных шаров можно считать упругим). В следующий момент происходит аналогичная передача энергии и импульса шару 3 и далее по цепочке. Пос- ледний шар 6, получив толчок, отклонится на тот же угол, на который первоначально был отклонен шар 1. Движение «проме- жуточных » шаров незаметно для глаза, поскольку длится ничтож-
263 Механика но малые промежутки времени. Когда шар 6 «вернется» после отскока, мы увидим отскок шара 1 и т. д. Если мы отводим первоначально шары 2 и 1, то этот процесс происходит дважды: сначала по цепочке пробежит «толчок» от удара шара 2, а затем, спустя ничтожное время, «толчок» от шара 1. В результате мы увидим, что с другой стороны цепочки отклонились шары 5 и 6, потом снова шары 1 и 2 и т. д. 4.42. Ответ: AWi = - 89 МДж, NW, = 290 МДж. Решение. В первом случае барон, по существу, пеупруго сталкива- ется с тем ядром, на которое пересаживается; при этом, конечно, часть кинетической энергии переходит во внутреннюю энергию барона и ядра. Значит, в этом случае изменение кинетической энергии &W 1 -- 0. Во втором случае барон должен оттолкнуться от ядра, на котором сидел, чтобы приобрести скорость встречного ядра. При этом ядро с бароном можно рассматривать как разрыв- ной снаряд, причем скорость после разрыва одного из «осколков» (барона) известна — она равна скорости встречного ядра (только при таком условии «мягкой» пересадки скорость этого ядра не изменится). Таким образом, во втором случае суммарная ки- нетическая энергия барона и ядер увеличивается (за счет расхода химической энергии в могучих мышцах барона при его толчке) и AWz > 0. Произведем подсчеты. В первом случае скорость ui ядра, на которое пересаживается барон, определяется из закона сохра- нения импульса (т + M)i>i Mvo~ ти0, а изменение кинетической энергии системы (т + М)и2 Ми2 ти,2 2Мти<2 пп ,1ТТ =-----9------ - -у-- - - -- = - ---= - 89 МДж. z 2 М + т Заметим, что при прямом попадании ядра в неподвижно стоя- щего барона энергии освободилось бы значительно меньше! Во втором случае закон сохранения импульса позволяет найти ско- рость t>2 оставленного бароном ядра: Mvi - тио = (М + т)ио. Изменение кинетической энергии в этом случае равно ж . + S* -"В* . , а» МДж. Z Z & м. Такая энергия выделяется при взрыве большого порохового заряда, так что барон и на этот раз неплохо «толкнул» ядро! т\ тело соскользнет с горки, не дойдя до ее вершины и передав горке часть своего импульса и своей энергии; при v > ио тело преодолеет горку и продолжит движение со скоростью о, а горка сместится вправо от начального 4.43. Ответ: при v < v<>
Ответы, указания, решения 264 положения и остановится; при v = vo тело может некоторое время двигаться вместе с горкой, находясь на ее вершине. Решение. Результат столкновения зависит от того, преодолеет ли тело вершину горки. Определим ту минимальную начальную скорость vo тела, при которой это возможно. Она, очевидно, позволяет телу подняться на вершину горки и там остановиться относительно горки, т. е. двигаться вместе с ней с некоторой скоростью и. Применив к этому движению законы сохранения энергии и импульса, получаем mvoi (т + М)и2 ТТ , —— = ----=—-— + mgH; mvo = (m + М)и . Z & Отсюда i>o =л/М1 + . Заметим, что чем легче горка, тем большая требуется скорость для ее преодоления. Когда v — vo, тело может некоторое время находиться в неустойчивом равновесии на вершине горки. Если бы вершина горки была плоской, тело могло бы там остаться, и столкновение можно было бы считать «неуп- ругим» (кинетическая энергия не сохраняется, однако в данном случае она переходит не во внутреннюю, а в потенциальную энергию). Если и < vo, тело съезжает с горки, не достигнув вершины, а сама горка начинает двигаться вправо. Конечные скорости ш тела и иг горки можно определить из законов сохранения энергии и импульса (см. задачу 4.38 ), как и для упругих лобовых столкно- вений частиц. Рассмотрим, наконец, случай v > vo. В этом случае тело «пере- валивает» через вершину и обгоняет горку, т. е. ui > иг. Единствен- ное решение, удовлетворяющее одновременно этому условию и законам сохранения энергии и импульса, имеет вид ui = v, иг = О, т. е. в конечном счете скорости тела и горки не изменились! Горка разгонялась, пока тело поднималось по ее левому склону, и уменьшала скорость, когда оно соскальзывало по правому склону. В итоге горка остановилась, сместившись вправо от начального положения и «вернув» налетевшему телу всю его начальную кинетическую энергию. 4.44. Решение. Пусть пиншг — массы тел; vi и vz — их скорости до соударения относительно системы отсчета Ki ui и иг — скорости после соударения в той же системе отсчета. Тогда изменение кинетической энергии составит: zniUi2 maii2 miVi2 mzVz2 Д1Рк = ^- + -2--------2-----
265 Механика Если система отсчета К движется со скоростью V относительно системы отсчета К', то в системе К' ко всем скоростям частиц следует добавить V. В этой системе отсчета . , /П1(И1 + V)2 т^цг + V)2 _ /П1(В1 4 V)2 _ пг^р2 + V)2 ~2 + 2~2 ~2~ rriiUi2 mzUz2 miVi2 пыг = —5— + —-------а-----5— + V(miui + т2и2 - miVi - zn2v2) - & & & = AWk 4 V(miUi 4 m2U2 - miVi - m^Vi) . Согласно закону сохранения импульса miUi 4 т2и2 - miVi - - m2v2 = 0. Значит, действительно AWk = AWV. Другой результат был бы странным: ведь чаще всего убыль кинетической энергии при столкновениях связана с изменением температуры Ai, одина- ковым во всех системах отсчета: AWk 4- crnAt = 0. 4.45. Ответ: hi = 2,0 см; й2 = 32 см. Решение. Прежде всего необходимо найти скорость v0 первого шара непосредственно перед ударом. Согласно закону сохранения niiVo2 Г7ГУ7 -Г. е. Vo = v2gH. Рассматривая упругий т. энергии migH = „ лобовой удар (см. задачу Удара: 4.38 ), находим скорости шаров после nti Vi = — 2/П11>о 1>2 =--------- mi 4 zn2 шаров определяется также из - т2 -------Vo; mi 4 zn2 Высота подъема каждого из закона сохранения энергии: . fi2 ТГ/пъ - т2\2 _ _ Ai = — = Я--------2 = 2,0 см; УГП1 4 тп2' . V22 7И1 \2 „„ й2 = „ = 4HI------1 = 32 см. &g К/П1 4 т2' Полученные формулы показывают, что при любых значениях масс шаров hi < Н, а вот йз может превышать Н почти вчетверо! Этот случай реализуется при mi > m2 (ср. с задачей 4.28). Ми2 4.46. Ответ: ц =---------. 2gL(M 4 т) Указание. Закон сохранения импульса позволяет найти конечную Mi? , скорость платформы и =-------. Работа силы трения А — -Ft^L М 4 т (см. задачу 4.15) совпадает с изменением кинетической энергии системы. 4.47. Решение. Обозначим Vi скорость i-той материальной точки,
Ответы, указания, решения 266 а и> — скорость той же точки в системе отсчета центра масс. Эти скорости связаны соотношением Vi — vc + и,. Кинетическая энергия?/ точек составит: N Н ( N Л' W ~ \т,и,~/2 = ^Гт,(о, + и,)~/2 = + v^tniUi + i-l >-1 <1 il 1 = 1 N N N Учтем теперь, что ^Гт, = М, 2^lm‘u'i = а '£гп'и“ = ® (СУМ- >=1 /=-1 ы марный импульс всех точек в системе отсчета их центра масс равен нулю), — и доказательство завершено. Укажем на особо важный частный случай: систему материаль- ных точек, составляющих твердое тело. В системе отсчета центра масс твердое тело может только вращаться вокруг этого центра, поэтому кинетическая энергия твердого тела равна сумме кине- тической энергии его поступательного движения (со скоростью и<) и кинетической энергии вращения относительно центра масс. 4.48. Ответ: частицы должны двигаться с одинаковыми по вели- чине и направлению скоростями. Решение. Часто встречающийся ответ «все частицы должны оста- новиться», вообще говоря, неверен: если до столкновения суммар- ный импульс всех частиц пе равнялся нулю, то все частицы никак не могут остановиться — это противоречило бы закону сохранения импульса. Заметим, однако, что скорссть центра масс при столк- новении частиц, образующих замкнутую систему, не изменяется. Следовательно, связанная с центром масс система отсчета являет- ся инерциальной. В этой системе отсчета суммарный импульс частиц равен нулю, поэтому все частицы после столкновения действительно могут остановиться; при этом вся их кинетическая энергия перейдет во внутреннюю. Согласно доказанному при ре- шении задачи 4.44 результату, «потеря» кинетической энергии будет такой же относительно любой другой системы отсчета. Очевидно, что большей потери кинетической энергии быть не может. В системах отсчета, не совпадающих с системой центра масс, частицы после столкновения должны двигаться с одинако- выми по величине и направлению скоростями. Такие столкно- вения Называют абсолютно неупругими (сталкиваются и слипаются два куска пластилина, пуля застревает в движущемся шаре и т. д.). 4.49. Решение. При изменении расстояния между точками на Дг потенциальная энергия изменится на величину с с С\г ДЖ,, =---------------------. ' : г 4- Дг г г(г + Дг)
267 Механика С/^г* __ Если Аг « г, получаем AW,, = —р- . Поскольку изменение потенциальной энергии равно работе силы, взятой со знаком Дг С «минус», можно записать А ~ F • Дг = С откуда F = -р, что и требовалось доказать. Определим теперь, притягиваются тела или отталкиваются. С Заметим сначала, что, согласно формуле W,, = нулевому уровню потенциальной энергии соответствует г > оо, т. е. бесконечно большое расстояние между телами. При приближении тел друг к ДРУГУ Wp увеличивается, если С > О, и уменьшается, если С < 0. Когда WP увеличивается при уменьшении г, работа силы взаимо- действия отрицательна (напомним еще раз, что работа равна изменению потенциальной энергии, взятому с противоположным знаком), а это означает, что сила направлена противоположно перемещению; т. е. С > 0 соответствует отталкиванию. Аналогично рассуждая, приходим к выводу, что С < 0 соответствует притя- жению. Вспомним теперь о законе всемирного тяготения F = G —— это как раз тот вид зависимости силы от расстояния, который мы получили при решении нашей задачи. И, действительно, потен- циальная энергия взаимодействия материальных точек, притя- гивающихся друг к другу по закону всемирного тяготения, равна т,гп2 . WP = - G — (зггак «минус» указывает па то, что это — притя- жение!). Точно т„кой ке вид имеет и потенциальная энергия взаимодействия двух сферически симметричных тел (в таком случае г — расстояние между их центрами). Рассмотрим, как изменяется потенциальная энергия при подъ- еме небольшого тела массой т на высоту h над поверхностью Земли (й < R, где R — радиус Земли). Получаем „ тМ тМ mMh „ тМ, AW,, = -G —— + G = G------« G -^-h . R + ft R R(R + h) R „ „тМ . „ Величина G —представляет собой силу притяжения тела к гС Земле, т. е. равна mg, и поэтому AWP = mgh, чего, конечно, и следовало ожидать. 4.50. Ответ: v = ^2gR =11 км/с. Указание. См. задачу 4.49. Вблизи поверхности Земли W =
Ответы, указания, решения 268 тМ - п = Wt + W), = —7. G —а на очень большом удалении от Земли 2 /С Wk и WP обращаются в нуль. Следовательно, W = 0 и вблизи поверхности Земли. Заметим, что скорость, о которой идет речь в условии задачи, называется второй космической. 4.51. Ответ: Wk = mv2. Указание. См. задачу 4.47. 4.52. Ответ: при ц < —5— волоком, при ц > Указание. При перемещении кантованием на длину ребра куба работа совершается для перевода куба в показанное на рисунке положение (дальше он поворачивается сам). Эта работа равна увеличению потенциаль- ной энергии при переходе центра тяжести куба из точки Oi в точку Ог. <2 - 1 5.1. Ответ: Н > 5,5 м. Решение. Равнодействующая трех сил, приложенных в точке А (см. рисунок), равна нулю. Это условие приводит к урав- нению 2Tsina - mg — О, где Ti — Тг = Т. Заметим, что при малых а сила на- тяжения каната может во много раз превышать вес груза. Минимально до- пустимое значение а определяется макси- мально допустимой величиной силы натя- жения каната. Минимально допустимая величина Н определяется I из условия Я,*. - h - ^tga. Выражая tga через sina, получаем: —i , .......== = 5,5 м 2^4Т - (/ng)2 5.2. Ответ: F =----------- ИОН. cosa + p.sina Указание. Согласно второму закону Нью- тона при движении без ускорения mg + + N + F + Ftp = O. В проекциях на оси координат (см. рисунок) получаем: Fcosa - Ftp = О, N + Fsina - mg = 0. Следует учесть, что Ftp = цУ (а не \irng, как часто ошибочно записывают). х
269 Механика 5.3. Ответ: F =--при ^tga < j. при ^tga > 1 сдвинуть cosa - jisma ящик невозможно. Решение. Запишем второй закон Нью- тона в проекциях на горизонтальную и вертикальную оси и используем фор- мулу FTp = [iN для силы трения сколь- жения. Получим (см. рисунок) Fcosa - Fmp - Ftp = О, N - Fsina - mg = О. Отсюда //z находим выражение для F: F =. cosa - pisina Анализ этого выражения показывает, что при достаточно боль- ших а и ц (когда ptga >1) сдвинуть ящик с места вообще не- возможно, его «заклинивает». Дело в том, что при увеличении F возрастает N, а вследствие этого — и Ftp. Сравните полученный результат с ответом задачи 5.2: толкая ящик, мы должны прикла- дывать большую силу, чем когда тянем его (при одинаковых т, ц, а). |img 5.4. Ответ: а = arctgu; Fml„ - --7 . N1 + (Г Решение. Очевидно, выгоднее направить силу под некоторым углом к горизонтальной плоскости, чтобы уменьшить силу дав- ления груза на плоскость, вследствие чего уменьшится и сила „ _ „ u.mg трения. Значения г и а связаны соотношением г =------------- cosa + pisina (см. задачу 5.2). Остается найти минимальное значение F как функции а. Для этого необходимо исследовать на максимум выражение /(a) = cosa -I- jxsina. 1-й способ (для умеющих дифференцировать): f'(a) = 0. Отсюда -sina 4- цсоза = 0; tga = ц; = VF+ ц2. 2-й способ изложен в математическом приложении. Подставляя в формулу для F полученное значение /™х, находим: с Р-'п8 , . . F,,i" = -^===== (при a = arctgjx). 5.5. Ответ: ц > 0,27. Решение. Действующие на клин силы показаны на рисунке (силой тяжести самого клина пренебрегаем). Приравнивая нулю проекцию равнодействующей этих сил на вертикальную ось, получаем хг . a , хг . a а „ a „ AiSin-^ + АгЗШ-х- ~ F„,iCOS-^ - FrpzCOS^ = 0. & £ &
Ответы, указания, решения 270 Учитывая, что Ni= N-г (в силу симметрии) и Frv < pJV, имеем Ц > tg^ - 0,27. 5.6. Ответ: при параллельном сое- динении k = ki + k>, при последо- вательном 1г =----. К\ + ki Решение. Коэффициент жесткос- ти упругой системы определяется из соотношения F = kx, где F — создаваемая системой сила упру- гости, ах — модуль общего удлинения системы пружин. а) При параллельном соединении х = Xi = хг, F = Fi + Fz. Здесь Xi, хг — удлинение каждой из пружин, a Fi, Fz — создаваемые ими силы упругости. Отсюда k = F- F, Fz , - + — = ki + ki . б) При последовательном соединении каждая из пружин рас- тягивается силой, равной F (это следует из третьего закона Ньюто- F на). Поэтому удлинения пружин равны соответственно Xi = и k\ F 1 1 1 Хг = т~ , а полное удлшюние составит х = xi + хг. Отсюда т = 4- — , П2 К К'2 , klki т. е. k ------. Таким образом, при параллельном соединении ki + ki система пружин имеет больший коэффициент жесткости, чем любая из пружин, а при последовательном —меньший. 5.7. Ответ: равнодействующая равна нулю. Решение. Предположим, что равнодействующая R всех сил отлич- на от нуля. Как изменится R, если каждую из сил F повернуть 2 л на угол — (например, по часовой стрелке)? При этом повернется вся система сил, а вместе с Ней и равнодействующая. Однако система сил осталась той же: каждый вектор силы просто занял место соседнего вектора. Значит, и R не должна измениться. Однако единственный вектор, не изменяющийся при повороте, — нулевой; так что Я = 0. 5.8. Ответ: стержень АВ растянут силой 17 Н, а стержень ВС сжат силой 34 Н. Решение. Условие шарнирного закрепления означает, что оба
271 Механика стержня могут быть только растянуты или сжаты, т. е. силы Fab и Fbc, действующие на точку В со стороны этих стержней, могут быть направлены только вдоль стержней (см. рисунок). Упоминание о шарнирном закреп- лении концов стержней весьма существен- но1’: если, например, стержень АВ жестко Закреплен в точке А, то без стержня ВС вообще можно обойтись, так как стержень АВ и сам удержит фонарь! Для окончательного выбора направлений сил следует учесть, что под действием подвешенного груза стер- жень АВ растягивается, а стержень ВС сжимается. Равнодейству- ющая всех трех сил, приложенных в точке В, равна нулю. Записывая это условие равновесия в проекциях на горизонтальную и вертикальную оси, получаем Fwcosa - Fab = 0; Firsina - mg = 0. mg Отсюда Fbc =----= 34 H; f\B = mgetga - 17 H. sina 5.9. Ответ: FA, = 0,98 кН; Fa. = 1,3 кН. Решение. Расставим силы, приложенные к точке С со стороны стержней и подвешенного груза (см. рис. а). Условие равновесия: mg + Fbc + Fai = 0. В данном случае записывать его в проекциях на какие-либо оси нецелесообразно. Проще изобразить сложение трех данных сил по правилу треугольника и использовать подобие треугольника АВС и треугольника сил (см. рис. б): 1 Во многих сборниках задач характер закрепления стержней (и даже их невесо- мость), к сожалению, не оговаривается. Указанное направление сил упругости стержней считается само собой разумеющимся. Советуем ознакомиться с зада- чами б.26 и 5.26, чтобы убедиться, что это не всегда так!
Ответы, указания, решения 272 Рас _ АС Рве _ ВС mg ~ АВ' mg АВ Из этих соотношений получаем приведенный выше ответ. I 5.10. Ответ: h - -j= . V3 Указание. Силы натяжения обеих нитей (см. рисунок) Ti = Т2 = = mg, а равнодействующая (R = Ti + Т2 + mg) равна нулю. Отсюда а = 60°. 5.11. Ответ: а) тз = 4 кг; б) тз = 'Im2 + тп22 + 2mim2cosa =4,4 кг. Решение. Сила натяжения каждой нити равна весу неподвижно висящего на ней груза (см. рис. a): Ti — mig, Т2 = rri2g. Равно- действующая приложенных в точке А сил равна нулю: Ti + Т2 + + m,3g — 0. В данном случае проецировать силы на различные оси нецелесообразно. Вместо этого изобразим треугольник сил, выра- жающий условие равновесия (см. рис. б), и применим к нему теорему косинусов: (msg)2 = Ti2 + Т22 - 2TtT2Cos (л - а), откуда Шз = Vmiz + тг + 2т1тгсоза. Разумеется, при mi = m2 задачу мож- но решить и проще. 5.12. Ответ: znafsina - gcosa ) < mi < m^sina + gcosa). Указание. См. задачу 2.18.
273 Механика 1 V5 5.13. Ответ: а = arctg-д = 27°; Т = -^~mg ~ 1,1 mg. & и Решение. Рассмотрим силы, действующие на половину цепочки (например, правую). Этих сил три: сила реакции подвеса N (равная силе натяжения цепочки в точке подвеса), сила натяжения цепочки в нижней точке Т' и сила тяжести mg/2. Сведем эти силы в одну точку (см. рисунок). Из условия равновесия следует, что векторная сумма этих сил равна нулю: Т' + N + mg/2 = О . Проецируя это равенство на верти- кальную и горизонтальную оси, получаем Nsina = Ncosa = Т'. Ct Отсюда N = —, Т' = —mgctgct . Естественно, сила натяжения Т’ 2sina равна нулю при a = л/2 и возрастает при уменьшении угла а, т. е. при увеличении расстояния между точками подвеса. Если Т' — mg (по условию), то ctga = 2, откуда a = arctg— = 27°. Для силы N Ct получаем: 7*' ,_______ N =-------- ТЧ1 + tg2a = mg-x- ~ 1,1 mg . cosa <5 Согласно третьему закону Ньютона Т = N - -^-mg& 1,1 mg . Ct 5.14. Ответ: гусь в 20 раз тяжелее одной куропатки, левое плечо весов вдвое длиннее правого. Решение. Обозначим вес гуся через Рг, вес куропатки — Рк, а длины левого и правого плеча самодельных весов — 1Я и In. Из условия равновесия в двух взвешиваниях находим: PrL = 40PKZn, Рг/п = ЮРк/л. Мы получили два уравнения для четырех неизвестных, но, к счастью, нас интересуют не сами неизвестные, а только их отно- Рг In шения у и . Обозначая эти отношения, соответственно, х и у, Л к 1и получим систему двух уравнений для двух неизвестных: ху = 40, - = 10. У Отсюда легко находим х = 20, у = 2. Это означает, что гусь в 20 раз тяжелее куропатки, а левое плечо весов вдвое длиннее правого. Заметим, что истинный вес гуся (если измерять его «в куропатках») равен не среднему арифметическому результатов
Ответы, указания, решения 274 „ /40 + 10 „л двух взвешивании |-------= 25J, а их среднему геометрическому (20 = V40 10). 5.15. Ответ: Гз = 6 мН; сила Fa направлена противоположно силам Fi и Fz; линия действия силы Fa должна отстоять на 1,2 мм от линии действия силы Ft и на 0,6 мм от линии действия силы Fz. 5.16. Ответ: Fa = 2 мН; сила Fa направлена в ту же сторону, что сила Fi; линия ее действия отстоит на 1,8 мм от линии действия силы Fz и на 3,6 мм от линии действия силы Ft. 5.17. Ответ: на расстоянии 78 см. 5.18. Ответ: 6,9 кН и 3,9 кН. Решение. На рисунке показаны действующие на балку силы (Ni nNz — силы реакции опор). Два условия равновесия1’ балки имеют вид: Ni + Nz~ mig - mag = 0, Моменты сил вычислены относительно оси, проходящей через точку О (левый конец балки). Из этих двух уравнений получаем 1 I - а 1а М = -^mig Ч-.—mag = 6,9 кН; Na = -^mig + .mag = 3,9 кН. м I ы I Согласно третьему закону Ньютона, силы Fi и Fz давления балки на опоры равны по модулю соответствующим силам реак- ции: Fi = Ni, Fz = Na. Как и следовало ожидать, вес самой балки «разделился» между опорами поровну, а вес груза — обратно пропорционально расстояниям до опор. 5.19. Ответ: 250 г, на 1/3 длины линейки. ‘’Мы называем первым, условием равновесия равенство нулю равнодействующей всех приложенных к телу сил (£F = 0), а вторым — равенство нулю алгебраичес- кой суммы моментов всех сил (£М * 0).
275 Механика 5.20. Ответ: а > л/3. 5.21. Ответ: Fa = 9,8 Н (сила направлена вверх); Fb = 350 Н (сила направлена вниз). 5.22. Ответ: Fi = 300 Н или Fz = 233 Н (если сила направлена вверх); Fi' = 700 И или Fi =2,1 кН (если сила направлена вниз). Решение. Рассмотрим сначала ситуа- цию, изображенную на рисунке. Если &i поднимается левый конец, доска по- ворачивается вокруг оси Оз (при этом сила реакции опоры Oi равна нулю). Момент силы Fi должен уравнове- шивать момент силы mg, поэтому rng(~ - а>) - Fi(Z - сь) = 0. Отсюда Fi =----‘——mg = 300 H. 2(1 - az) Если сила F2 приложена вертикально вверх у другого конца, получаем A - -^-^-mg = 233 Н. 2(Z - а>) Однако задача имеет не только эти решения. Можно припод- нять один конец доски, прилагая к другому ее концу силу, направленную вниз. В этом случае I — 2ai / — 2<Iz F,' = „ mg = 700 И; F/ = —mg = 2,1 кН. На первый взгляд, прикладывать силу вниз труднее (величина необходимой силы в этом случае больше). Однако надавить на доску вниз нетрудно: в нашем распоряжении собственный вес! Для «получения» силы F/ = 700 Н взрослому человеку достаточно сесть на край доски. 5.23. Ответ: ,Va = 400 Н, NB = 500 Н. Решение. Очевидно, стержень да- вит на стенку в точке А вверх, а в точке В — вниз. Соответствующие силы реакции опоры Na и 2Vb сог- ласно третьему закону Ньютона направлены противоположно (см. рисунок). Первое условие равно- весия приводит к уравнению Nb = = Na + mg, а второе (моменты
Ответы, указания, решения 276 берутся относительно точки А) — к уравнению NvAB — mg'АС. Отсюда „ АС АС ^ = т^АВ=РАВ= 500 Н’ хт АС-АВ ВС „ВС Na ~ mg АВ ~mg АВ ~Р АВ ~ 400 Н' Обратите внимание: обе силы реакции существенно превышают вес груза, так как АВ значительно меньше длины стержня. 5.24. Ответ: 350 Н; 430 Н; 50 Н. Решение. На рисунке показаны действующие на стержень силы: 2Va, Nb — силы реакции боковых стенок отверстия; N — сила реак- ции дна отверстия (ее модуль как раз и показывает, с какой силой стержень сжат). Первое условие равновесия в проекциях на оси х, у имеет вид: N - Psina = 0, Nb - — Na — Pcosa = 0. Второе условие равновесия (мо- менты сил берутся относительно точки A): Nb'AB - P-ACcosa = 0. Отсюда АС N = Psina = 50 Н, Лгв = Pcosa . „ 430 Н, Na = Pcosa AC-АВ AB ВС = Pcosa • = 350 AB H. 5.25. Ответ: N = ^s^V4 + ctg £ 2a = 11 H; сила N образует с верти- калью угол p = arctg (C^a) - 27° . ' £ * Решение. На рисунке показаны действу- ющие на стержень силы (направление силы 2V указано пока ориентировочно). Условием равновесия является равенство нулю алгеб- раической суммы моментов сил относитель- I но точки A: TZsina - mgg00801 = гДе I — длина стержня. Отсюда сила натяжения 1 нити Т = ^mgctga. и
277 Механика Поскольку равнодействующая сил N, Т и mg равна нулю, находим проекции N на горизонтальную и вертикальную оси: NX = T = —mgetga, N, - mg. Ct Тогда N = + M2 = —'ll + ctg2a = 11 H. Z! Угол p, образованный силой N с вертикалью, определяется из х „ N, ctga „ , /ctga\ _ условия tgP = тг = —x~; отсюда Р = arctg I—I = 2Т\ Получен- 2 V у & \ Ct ' ный результат показывает, что N (а, значит, и сила упругости стержня в нижней точке) для весомого стержня не обязательно направлена вдоль оси стержня (сравните с задачей 5.8). Обратите внимание, что линии действия сил Т, mg и N пересекаются в одной точке: это необходимое условие равновесия твердого тела. 5.26. Ответ: FA = FB = mg 2sina’ Решение. Сила упругости сжатого под дей- ствием подвешенного груза стержня ВС на- правлена вдоль отрезка ВС (см. рисунок). Аналогичное утверждение относительно си- лы упругости стержня АВ несправедливо, поскольку груз прикреплен к его середине и стержень «работает» на изгиб. На рисунке показаны три силы, действующие на стер- жень АВ (Na — реакция шарнира А). При- меняем второе условие равновесия (моменты „ лх АВ сил берутся относительно точки A): mg ~ = £ nig = Fbc ABsina, откуда Fbc = —;—. Записываем первое условие 2 sina равновесия в проекциях на горизонтальную и вертикальную оси: Nax + Feccosa = О, Na, - mg + Fecsina = О. Отсюда XT mS . V mS V I XT 2 x7' 2 mS NAx = Qctga, Na, — ~X-; Na — WAxJ + Na,2 = —-- . 2sina Согласно третьему закону Ньютона, Fa = Na. Fb = Fbc. Можно предложить и более короткий путь решения этой за- дачи. Достаточно заметить, что необходимым условием равно- весия является пересечение линий действия всех трех сил в одной точке. Очевидно, в данном случае этой точкой является середина стержня ВС: здесь пересекаются линии действия сил Fbc и mg.
Ответы, указания, решения 278 Отсюда следует, что сила Ла наклонена к горизонту под углом а и равна по модулю силе Гвс. Для нахождения величины силы Na надо теперь применить первое условие равновесия (в проекции на вертикальную ось). 5.27. Ответ: Fmi„ в случае перемещения оси рычага F’„„„ = F. Указание. Сила будет минимальной при максимальной длине плеча силы. е по ^h(2R - h) . п . „ 5.28. Ответ: F > mg —-—-- при h < В; при h> R решения нет. R - Л Указание. Для подъема на ступеньку момент силы mg относитель- но оси А не должен превышать момент силы F (см. рисунок). Максимальная сила F потребуется именно в начале подъема, когда плечо силы F минимально, а плечо силы mg максимально. К задаче 5.28 К задаче 5.29 а. пп т? ^h(2R - h) 5.29. Ответ: F = mg —------- „ ; направление F указано на рисунке. Указание. См. задачу 5.27. 5.30. Ответ: ц > 0,27. Решение. На рисунке пока- заны силы, действующие па одно из нижних бревен (по- казаны только те силы, ко- торые имеют ненулевой вра- щающий момент относитель- но мгновенной оси вращения О). Условием равновесия яв- ляется равенство величин вращающих моментов сил N и Frp относительно оси О. Учитывая, что N образует с вертикалью угол 30°, а Гтр — угол 60°, получаем:
279 Механика N • OB- F^.-AB; OB = Bsin30°; AB = Я(1 + cos30°). p 1 Отсюда ц„„„ = = 0,27 . __mgR <1(1 + 2B) ‘ 5.31. Ответ: T = V/(Z i- 2R) Решение. Прежде всего необходимо определить, ка- ково взаимное расположение веревки и шара (см. рисунок). Для этого проще всего воспользоваться вторым условием равновесия твердого тела: алгеб- раическая сумма моментов всех приложенных к телу сил относительно любой точки должна равнять- ся нулю. Выберем в качестве такой точки центр шара. Силы N и mg направлены вдоль радиусов шара, поэтому их моменты равны нулю. Следова- тельно, сила Т также должна быть направлена по m R радиусу шара. Таким образом, sina =-----и cosa = I + R п---г—г~ а/Д/ + 2R) „ - VI - sin'a = -------. Первое условие равновесия I + R (равенство нулю равнодействующей всех приложен- ных, сил) в проекциях на горизонтальное и вертикальное направ- ления дает N — Tsina = 0, Tcosa - mg = 0. Остается учесть, что, согласно третьему закону Ньютона, F = -N и F ~ N. Интересно отметить, что при таком подвесе Т > mg, т. е. прочность веревки должна быть больше, чем при обычном вертикальном подвесе. 5.32. Ответ: ц > 1 независимо от значений В и I. Решение. Удобнее всего применить правило мо- ментов (см. рисунок). Если вычислять моменты сил относительно оси, проходящей через точку А, получаем уравнение "N • R - F4. • R = 0, где R — F радиус шара. Так как Ftp < то ц> —=1. Обратите внимание, что ответ не зависит от ра- диуса шара и длины нити, что совсем не очевидно до решения задачи. 5.33. Ответ: 5,0 кг и 25 кг. Решение. Запишем условия равновесия шкафа (на рисунке Na и Ув — силы реакции опоры, действующие на каждую из Ножек; Fa и Ев — соответствующие силы трения). При этом огра- ничимся рассмотрением проекций сил на верти- те задаче 5.32
Ответы, указания, решения 280 кальную ось; моменты сил вычисляем относительно точки В. Получаем mg = = Na+Nb; mg~ - Na • AB - F • AD = 0. Ct Отсюда Na~^ ~F AB'Nb~ 2 +F AB' Если весы, как обычно, градуирова- ны в килограммах, то они покажут тпа = — = 5,0 кг, 7ПВ = — - 25 кг. g g Интересно, что перераспределение нагрузки на ножки вызвано действием горизонтальной силы F! 5.34. Ответ: ц„„„ = 0,5; Fmi„ = ^mg. Ct Решение. Чтобы куб начал поворачиваться относительно оси О (см. рисунок), должно а выполняться условие mg - Fa = 0, пос- Ct кольку сила N имеет нулевой момент от- носительно оси О. Проскальзывание в точке О отсутствует, если выполняется первое условие равновесия. В проекциях на оси координат F - /\Р = 0; N - mg = 0. Отсюда *4 1 М N ~ 2' При дальнейшем опрокидывании куба значения F и FTp могут уменьшаться по сравнению с начальным моментом. 5.35. Ответ: Е„,(„ _ mg ~ 2<2’ 1 Ципи — з * Решение. В момент начала опрокиды- вания куба сила N приложена на оси вращения О и не имеет вращающего момента (см. рисунок). Опрокидываю- щая сила F будет минимальной, если максимальным будет плечо этой силы. Для этого сила F должна быть приложе- на в точке А и направлена перпендику- лярно диагонали боковой грани куба. Из второго условия равновесия (правила
281 Механика моментов): mg - FmuO^ = О, F„„„ = . Чтобы куб не проскальзывал, должно выполняться первое условие равновесия. В проекциях на горизонтальную и верти- кальную оси: Fmincos45° - Ftp = 0; FminSin45° + N - mg = 0. Отсюда _ mg 3mg , , Ftp = —r~, a N = —. Следовательно, минимальный коэффициент 4 4 Ftp 1 трения |i„,u> = = g. 5.36. Ответ: а,,,,» = arctg(2p). Решение. Пусть масса лестницы т, а длина I (см. рисунок). Запишем усло- вия равновесия, причем моменты сил будем определять относительно оси, про- ходящей через нижнюю точку лестницы (при этом моменты двух сил обращаются в нуль, что облегчает решение): Ftp = N2, Nt = mg, mg~sina - A^Zcosa - 0. Ct Учитывая, что FTP < pZVi, получаем tga < 2ц. 22? 22? 5.37. Ответ: a = arctg -^-, ц > Решение. Равновесие нарушается, когда вер- тикаль, проведенная через центр тяжести О цилиндра, не пересекает площадь опоры, т. е. основание цилиндра (см. рисунок). Поэто- 22? му угол а определяется из условия tga = Однако равновесие может нарушиться и до опрокидывания цилиндра из-за того, что он начнет скользить по наклонной плоскости. Чтобы этого не произошло, должно выпол- 22? няться условие sina - pcos a < 0. Отсюда ц > tga = 5.38. Решение. Если приложить горизонтальную силу к боковой поверх-ности бруска вблизи его основания и постепенно эту силу увеличивать, то при достижении ею значения F = [img брусок начнет поступательно перемещаться по поверхности. Увеличивая высоту h точки приложения горизонтальной силы, можно до-
Ответы, указания, решения 282 биться того, что опрокидывание бруска начнется раньше его скольжения. Минимальная высота при которой это проис- а ходит, определяется из условия mg ~ = Fh,„„, , где а — сторона основания бруска. Отсюда ц ~ — . Разумеется, такой способ применим, если высота бруска превышает Л,„in. успений приводит к резуль- 5.39. Ответ: правые ножки — с си ножки — с силой Fi = --- (cosa - Решение. Запишем условия рав- новесия — равенство нулю рав- нодействующей всех сил (в про- екциях на оси координат, см. ри- сунок): mgsina - F^i - FtP2 = О, Ni + Na - mgcosa = О, а также равенство нулю алгебраической суммы моментов сил относитель- но центра тяжести: N> I - N= 4 - (F^ + = 0 • £ £ £ Решение полученной системы у тату _г 1 h . . .. 1 h . ч Ni = ;}mg(cosa + ysina), Nt = ~mg(cosa - . sma). & I & I Согласно третьему закону Ньютона, силы нормального дав- ления ножек на наклонную плоскость равны по модулю соответ- ствующим силам нормальной реакции опоры: Fi = Ni, F> = N->. I При tga = у сила F> обращается в нуль, т. к. шкаф опирается п только на правые ножки. При этом центр тяжести шкафа нахо- дится на одной вертикали с точкой опоры, равновесие неус- / тойчиво. При tga > fa шкаф опрокидывается. 5.40. Ответ: Fi = mgcosa^ + ; F2 = mgcosa(4 - gj) • Решение. В этом случае шкаф будет соскальзывать с наклонной плоскости с ускорением а. Проецируем уравнение второго закона Ньютона на оси координат (см. рисунок к решению задачи 5.39):
283 Механика mgsina-F^.i - F^i = та, (1) М + Лг2 - mgcosa = 0. (2) Теперь мы можем использовать формулу для силы трения скольжения: (5) Fti,i = рМ, (3) = pNi. (4) Поскольку шкаф движется поступательно (без вращения), мож- но приравнять нулю алгебраическую сумму моментов всех сил относительно центра масс: (К„> + К,,.,)”' = °- & £ А Полученная система из пяти уравнений легко сводится к системе двух уравнений (заметим, что уравнение (1) мы не используем): № -н = mgcosa, M(Z - цА) = А2(/ + цА). Отсюда (сравните с задачей 5.39): „ /1 „ ,, /1 пА\ Fi = /Vi = mgcosal^ + -=у), Fi = ЛГ2 = /ngcosal _ - -^г) . Интересно, что при ц = 0, т. е. при соскальзывании без трения, Fi = F>. Ничего удивительного: в неинерциальной системе отсчета, связанной со шкафом, наклонная плоскость становится как бы горизонтальной (см. задачу 2.23). „ Л 25/ 5.41. Ответ: -=-г. 24 Решение. Условимся считать кирпичи сверху вниз (т. е. верхний — первый). Центр тяжести каждого кирпича отс- тоит от его края на 1/2. Поэтому первый кирпич может выступать над краем второго не более, чем на 1/2. Тогда общий центр тяжести Са двух верхних кирпичей расположен (см. рисунок) на расстоянии //4 по горизонтали от края второго кирпича. Именно на это расстояние и может выступать второй кирпич над третьим. Центр тяжести трех верхних кирпичей Сз определяется из условия mg(-^ - х) = 2mgx , откуда х = т. е. третий кирпич может вы- 1 ступать над четвертым на -х своей длины. Аналогично доказыва- о 1 ется, что четвертый кирпич может выступать над пятым на О своей длины. Полное смещение верхнего кирпича относительно
Ответы, указания, решения 284 I I I I 251 нижнего составит 77 + t + z' + k = 2 4 6 8 24 Верхний кирпич, оказывается, может целиком выйти за пре- делы площади опоры! (См. задачу 5.42). 5.42. Решение. Обозначим через х„ величину максимального сме- щения n-того сверху кирпича над нижележащим. Согласно полу- ченному в задаче 5.41 результату Xi = 1/2, ха = 1/4., хз = 1/8, X-i = = 1/8. Можно предположить, что общая формула имеет вид I „ х„ = . Доказать это предположение можно методом математи- 2п ческой индукции. Пусть оно справедливо для п = k, причем й-й кирпич выдвинут относительно нижележащего на максимально возможную величину х. Найдем расстояние по горизонтади от центра тяжести С системы из (й + 1) кирпича до правого края нижнего кирпича. На рис. а заштриховано условное изображение j(k+l)mg Рис. а Рис. б системы из й верхних кирпичей. Искомое расстояние, представ- ляющее собой как раз Xk-н, легко определить из правила моментов: - Xk.i) = kmg Xk.i , I I t. e. Xhi = ~~——. Таким образом, если формула х„ = спра- ведлива для п — й, то она справедлива и для п = й + 1. Тем самым формулу для хп можно считать доказанной. Максимальное сме- щение верхнего кирпича относительно нижнего: I I I I /„111 2 + 4 + 6 + 8 +-2 1 + 2 + 3 + 4 + ’ При неограниченном увеличении числа кирпичей эта сумма стремится к бесконечности. Действительно, сумма ряда , 1111111 1 + 2 + 3 + 4 + 5 + 6 + 7 + 8 + ”’
285 Механика больше суммы ряда, который явно стремится к бесконечности: , 1 /1 1ч /1 1 1 Ь ,111 1 + 2 + '4 + 4) + 18 + 8 + 8 + 8) +-1 + 2 + 2 + 2 + ‘"‘ Со стороны такая стопка кирпичей выглядит как плавно изог- нутая колонна, показанная на рис. б. Таким образом, у нас снова нет оснований сомневаться в исключительной правдивости барона! Если не считать, конечно, того, что, во-первых, равновесие выложенной им стопки кирпичей неустойчиво (центр тяжести всей стопки расположен в точности над краем самого нижнего кирпича), а во-вторых, для того, чтобы край верхнего кирпича выступал над краем самого нижнего на целую милю, барону придется выстроить настолько высокую «гнутую башню», что он должен будет учесть неоднородность поля тяготения Земли. 5.43. Ответ: центр тяжести находится в точке пересечения медиан треугольника. Решение. Центр тяжести шариков 1 и 2 находится в середине соединяющей их стороны треугольника (см. рисунок). Задача сводится к следующему: нужно найти центр тяжести системы, состоя- щей из шарика массой т в вершине треугольника и шарика массой 2т — в середине противолежащей стороны. Центр тяжести О лежит на отрезке, соединяющем эти шарики (т. е. на ме- диане), деля его в отношении 2:1, считая от вершины треугольника. Очевидно, точка О должна принадлежать и двум другим медианам треугольника и делить их в таком же отношении (таким образом мы «физически» доказали хорошо известную математическую теорему). 5.44. Ответ: центр масс лежит на медиане, проведенной из третьей вершины, и делит ее в отношении 1 : 2 (считая от вершины). 5.45. Ответ: в алюминиевой части. 5.46. Ответ: Центр масс системы совпадает с центром шарика массой 700 г. Решение. Центр масс системы в однородном поле тяготения сов- падает с центром тяжести. Поэтому будем искать точку, в которой нужно разместить под стержнем опору для достижения равно- весия (см. рисунок).
Ответы, указания, решения 286 Будем отсчитывать координаты шариков от левого конца стер- жня и рассматривать моменты сил относительно этой же точки. Тогда условия равновесия принимают вид: ю ю N - 'frn.g = 0, Nx. - '^jn.gXi = 0 . i-i <=i ^jniX, Отсюда1’ х. =------. Подставляя численные значения, получаем хс = 60 см. 5.47. Указание. См. рис. а, б. Разбивая пластинку на два прямо- ” Можно доказать, что эта формула справедлива при любом расположении ма- териальных точек (не обязательно вдоль прямой); разумеется, так же находятся и две другие координаты центра масс. В векторном виде можно записать: Sm<r< гс =---- , где п — радиус-вектор i-ои точки, а п— радиус-вектор центра масс. Ет' У всех радиус-векторов начале общее (это начало координат), а концы совпадают с соответствующими точками.
287 Механика центр тяжести пластинки лежит на этом отрезке. Однако указан- ное разбиение можно проделать двумя способами. Таким образом мы получаем два отрезка, на пересечении которых и находится центр тяжести С пластинки. 5.48. Решение. Разрежем треуголь- ник на тонкие полоски, параллель- ные стороне АВ (см. рисунок). Центр тяжести каждой из таких полосок лежит в ее середине, т. е. принад- лежит медиане, проведенной из вер- шины С. Значит, и центр тяжести всего треугольника, состоящего из таких полосок, тоже лежит на этой медиане. Совершенно ана- логично доказывается, что центр тяжести принадлежит и двум другим медианам, а, следовательно, совпадает с точкой их пере- сечения. Заметим, что попутно оказалось доказанным и утверж- дение о пересечении всех медиан треугольника в одной точке (ср. с задачей 5.43). 5.49. Ответ: на расстоянии R/Q от центра в сторону, противопо- ложную отверстию. Решение. Расстояние от центра тяжес- ти пластины до центра круга обозначим х (см. рисунок). Из соображений сим- метрии очевидно, что центр тяжести лежит на диаметре, проходящем через центр отверстия. Если заполнить отвер- стие, то есть добавить к данному телу круг радиусом R/2, то центр тяг' сти о*:жжется в геометрическом центр ^ру- га. Точка О делит отрезок COi в отно- шении, обратном отношению масс пластины с вырезом и «добав- ленного» круга: х _ ГП> 0,5В т.\ Поскольку пластина однородна, масса фигуры пропорциональ- на ее площади; поэтому получаем тг _ тг(В/2)2 _ 1 пй ~ nI¥-n(R/2y ~ 3’ Отсюда х = О
Ответы, указания, решения 288 5.50. Ответ: центр тяжести находится на оси симметрии на рас- стоянии 2г/п от центра полуокружности. Решение. Вследствие симметрии фигуры центр тяжести С лежит на прямой MN (см. рис. а). Обозначим расстояние от центра окружности до центра тяжести через х. Разобьем дугу на малые элементы длиной Д/. Тогда по общей формуле для положения ]ГрД/х, центра тяжести (см. задачу 5.46) получим: х =----, где р — £рД/ линейная плотность (масса единицы длины) полукольца; х, — расстояние от i-ro элемента полукольца до диаметра АВ. Заметим теперь, что треугольник со сторонами х,, г подобен малому треу- гольнику со сторонами &U, Al, где \1, — проекция i-ro элемента на диаметр АВ. Из этого подобия следует равенство Д/х< = Д/,г, и для ]Гд/,г г^ДД положения центра тяжести получаем х = —---=------. Но ведь — длина полукольца, a — длина его диаметра. Значит, г • 2r 2r х --------------------= — ® 0,64г. лг л Другое, очень красивое решение этой задачи принадлежит И. Алешину (Квант, 1991, №10). Рассмотрим целое кольцо, вра- щающееся вокруг своего центра с угловой скоростью со. В этом
289 Механика О) случае сила упругости в нем Т = parr2 (см. задачу 2.34; мы учли, что пг = 2лрг, I = 2пг, п = со/2л). Равнодействующая приложен- ных к левому полукольцу сил равна 2Т (см. рис. б), а ускорение его центра масс а = ы~х. Масса левого полукольца m = рпг. Как известно, центр масс любой системы движется как материальная точка, в которой сосредоточена вся масса этой системы и к которой приложены все действующие на систему внешние .силы. Поэтому можно записать ma ~ 2Т, или рлгогг = Зр&гг2. Отсюда х = 2г/п. 6.1. Ответ: F3 > Fi > F>; Fi = P, F2 < P, F3 > P. Решение. Сила F давления жидкости на дно сосуда определяется давлением р жидкости у дна сосуда и площадью S дна: F = pS. Давление жидкости, в свою очередь, определяется плотностью р жидкости и высотой h ее столба: р — pgh. Мы не учитываем здесь вклада атмосферного давления, одинакового для всех трех сосудов. Очевидно, при одинаковом ко- личестве воды во всех сосудах уровень воды в треть- ем будет наивысшим, а во втором — самым низким. Соответственно получаем Ft > Fi > Fi. Сравним величины Ft, Ft, F?. с весом Р воды в каждом из сосудов. Начнем с первого сосуда: F1 = pgh • S = pgV = mg = Р (здесь V = hS — объем налитой воды). Как и следовало ожидать, Fi = Р. Значит, А < P,F3> Р. Мы столкнулись с разновидностью знаменитого гидростатического парадокса. Объяс- нение неравенства F> < Р просто: наклонные стенки принимают на себя часть веса воды, выполняя в этом смысле функцию дна. В третьем сосуде ситуация обратная: наклонные стенки увеличивают силу Давления жидкости на дно. Согласно закону Паскаля вода давит
Ответы, указания, решения 290 на стенки, «прогибая» их наружу. Стенки согласно третьему закону Ньютона действуют на воду с силой, направленной внутрь жидкости (см. рисунок). Очевидно, при этом сила реакции дна Ns больше mg. Согласно третьему закону Ньютона Ns = F3, так что действительно Ft > mg. 6.2. Решение. Для отрыва дна во всех случаях требуется, чтобы сила давления на него сверху плюс сила тяжести самого дна ненамного превысили силу давления воды снизу. Из решения задачи 6.1 следует, что нужно отдельно рассмотреть два возмож- ных случая. Рис. а Рис. б 1) Конус расширяется кверху (см. рис. а). Сила давления на дно налитой в сосуд воды F ~ pghS. Эта сила меньше полного веса налитой воды на величину веса .\mg заштрихованной на рисунке части жидкости (можно считать, что вес \mg принимают на себя наклонные стенки). Поэтому гиря, имеющая тот же вес, что и вся налитая вода, давит на дно сильнее воды и дно отпадет; то же можно сказать о налитой ртути: из-за большей плотности и меньшего объема в заштрихованную боковую область попадет значительно меньшая по сравнению с водой доля ртути. Бензин же имеет плотность меньшую, чем вода; поэтому сила его дав- ления на дно меньше — дно не отпадет. 2) В случае, если конус сужается кверху (см. рис. б), сила давления жидкости на дно превышает ее вес на величину веса жидкости, которая могла бы поместиться в заштрихованном объе- ме. Поэтому под действием гири или налитой ртути дно не отпадет, а при наливании бензина — отпадет. 6.3. Ответ: сила давления на дно Fi = pga3 (здесь р — плотность воды), на каждую боковую стенку F2 = Fi/2. Решение. Обозначив плотность воды через р, запишем силу дав- ления воды на дно: Fi = р • а2 = pga а2 = pga2, где р — давление жидкости у дна аквариума. Давление на боковую
291 Механика стенку линейно возрастает от О до р с увеличением глубины, поэтому для вычисления полной силы Fz давления воды на боко- вую стенку можно воспользоваться средним давлением pef = О + р pga „ _ pga3 = —»— = ~5~ • Получаем F3 = р,„ а* - Lu Л 6.4. Ответ: h = а/3 = 20 см. Указание. Сила давления на боковую стенку равна (см. Л задачу 6.3). 6.5. Ответ: hz = 50 см; жидкость потечет справа налево. Решение. J3 состоянии равновесия давление должно быть одина- ковым в точках, лежащих на одном уровне в одной и той же жидкости. Значит, давление р на границе ртуть-спирт и ртуть- вода одинаково. Оно складывается из атмосферного давления рд и давления pgh столба жидкости, находящейся сверху. Поэтому р = = рд + р1#Л1 == рд + pzghz. Здесь pi и pz — плотность воды и спирта соответственно. Отсюда Лз - - 50 см. Разумеется, закон сооб- р2 щающихся сосудов здесь неприменим: если в сосуде находятся разные жидкости, следует исходить из равенства давлений в «общей» части сосудов. Рассмотрим теперь, что произойдет, если открыть кран. Это зависит от соотношения давлений pi и pz слева и справа от закрытого крана. Очевидно, pi = р - pigho, pz = р - — рг^Ло; тогда рг - pi = gh<i(pi - рг) > 0. Значит, давление в спирте на уровне трубки выше, и поэтому после открывания кране, жидкость потечет справа налево: спирт будет перетекать в левое колено и смешиваться с водой. В результате часть ртути будет «выдавливаться» из левого колена в правое. Однако в конце концов вода и спирт смешаются и уровни ртути снова сравняются. 6.6. Решение. Вначале керосин будет перетекать из правого колена трубки в левое (см. решение задачи 6.5) и всплывать над водой. Однако в отсутствие капиллярных явлений граница раздела между водой и керосином может быть только горизонтальной. Значит, часть воды перетечет из левого колена в правое. Равно- весие будет достигнуто, когда поверхность воды в левом колене установится на уровне нижнего края горизонтальной трубки. Поскольку давление в керосине должно быть одинаковым у обоих концов горизонтальной трубки, уровень керосина будет одинаков
Ответы, указания, решения 292 в обоих коленах трубки (см. ри- сунок; штриховой линией показан уровень ртути до открывания кра- на). Изменение уровня ртути Н мало из-за большой плотности ртути. Поэтому высота столба во- ды в правом колене приближенно равна hi-ho, т. е. 16 см. Для более точного определения уровней жидкостей воспользуемся равен- ством давлений в обоих коленах внутри ртути: ptg(ho + Н) = pig(/h - h0 - Н) + + р^(2Л0 - hi) + png 2Н (здесь pi — плотность ртути). Отсюда н = (р. - р-)(2/г0 - hi) 2(p.i - pi) = 0,63 мм. Эта величина действительно мала по сравнению с ho. Посколь- ку разница уровней воды справа и слева составляет около 8 см, высота столба керосина в левом колене 21 см, а в правом 29 см. 6.7. Ответ: не изменится; не изменится; понизится. Решение. Часто утверждают, что уровень воды повысится — ведь часть льда возвышалась над поверхностью воды. При этом забы- вают, что плотность при превращении льда в воду увеличивается, поэтому объем уменьшается. Лучше подойти к вопросу иначе. Когда лед плавает, на него действует выталкивающая сила, равная весу льда. Согласно закону Архимеда эта сила равна весу вытес- ненной льдом воды. Когда лед растает, он превратится в воду такого же веса (а, значит, и такого же объема), как и вытесняемая им при плавании вода. Следовательно, уровень воды при таянии льда не изменится. Наличие вмороженной в лед пробки не изме- нит ответ: ведь пробка, освободившись ото льда, будет плавать, вытесняя опять же воду с весом, равным весу пробки. А вот стальная гайка после таяния льда утонет. Вес вытесненной гайкой воды меньше веса самой гайки, а до таяния льда на «долю» гайки приходилась вытесненная вода равного с гайкой веса. Значит, уровень воды понизится. Эту задачу можно решить и другим способом (см. задачу 6.8).
293 Механика 6.8. Ответ: уровень воды понизится. Решение. По существу этот вопрос уже разобран при решении задачи 6.7 (случай со стальной гайкой). Однако полезно привести и другие решения. 1. Вначале, как вытекает из условия равновесия Fa = (М + m)g , г. , .. „ М + т и формулы Fa = Рв£к1, объем вытесненной воды V, --------, где Р» М и т — массы лодки и камня соответственно. После выбрасы- вания камня лодка вытесняет объем М/р,„ а камень — собствен- ный объем т/р„ (здесь р, — плотность камня ). Полный вытеснен- „ М т „ ныи объем теперь составит V2 = — + — Поскольку Р« > р» (камни Р» Рв тонут!), получаем Уг < Vi, т. е. уровень воды понизится. 2. Полная сила F давления на дно бассейна равна весу всего содержимого бассейна и потому остается неизменной. Однако вначале дна касалась только вода, давившая на него с силой F = p„ghiS. После падения камня на дно F = p,ghiS + f, где f — сила давления камня на дно. Из соотношения p.ghiS + f = p.ghiS следу- ет, что hi <- hi, то есть конечная глубина воды в бассейне меньше, чем начальная. 6.9. Ответ: в бассейне; не изменился. Решение. В результате вычерпывания воды осадка лодки умень- шается (она всплывает). При этом объем вытесненной воды умень- шается на величину, равную объему V ведра, а осадка лодки — V на hi - 5- , где Si — площадь внешнего поперечного сечения лодки на уровне воды. Уровень воды в лодке (относительно ее дна) V уменьшается на hi - (здесь S> — площадь поверхности воды 02 внутри лодки). Итак, сама лодка поднимается на hi относительно уровня воды в бассейне, а толщина слоя воды в ней уменьшается на hi. Значит, изменение уровня воды в лодке относительно воды V в бассейне составит АЛ = hi - hi = ~^~zr(Si - Si). Поскольку Si > Si 0102 (за счет толщины стенок лодки), получаем АЛ < 0: уровень воды в лодке становится ниже, чем в бассейне. В бассейне уровень воды, конечно, останется прежним: ведь суммарная сила давления на дно не изменилась (см. задачу 6.8). 6.10. Ответ: потонет; всплывет. Решение. Нужно сравнить силу тяжести (лг* + m&)g, действующую на наполненную бутылку, с силой Архимеда Fa, равной весу
Ответы, указания, решения 294 вытесненной жидкости p«gK Здесь т» = ржУж — масса жидкости в бутылке объемом У»; ms = ргУв — масса самой бутылки; рж, рс — плотности соответственно жидкости и стекла; Vs — объем стенок и дна бутылки; V = Уж + Vs — внешний объем бутылки. Найдем разность R силы тяжести и силы Архимеда: R = (тж + + ms}g — Fa — (р< - pxjVsg. Если бутылка с водой находится в воде, то рг > рж, Л > 0; следовательно, бутылка утонет.. Если бутылка со ртутью находится в ртути, Рс < рж, R < 0, и бутылка всплывет. Этот ответ можно получить и гораздо проще: нужно только сообразить, что бутылка, доверху наполненная жидкостью, «пове- дет себя* так же, как сплошной кусок стекла в этой жидкости. 6.11. Ответ: 61 Дж. Решение. Определим сначала, на сколько придется опустить льди- ну. Высота йо выступающей над водой части льдины определяется из условия равновесия Fa = mg, где Fa = pBgS(H - йо) — сила Архимеда; т = pxgSH — масса льдины (здесь р» и р., — плотность воды и льда соответственно). Получаем йо - ———Н. В начальный р, момент силы Fa и mg уравновешивают друг друга. По мере уменьшения высоты й выступающей над водой части льдины (см. рис. а) от йо до 0 необходимо прикладывать сверху вниз все Рис. а большую силу F. Ее максимальное значение Г.Мх = gSH(pB - рл). Работа А при погружении льдины численно равна площади под графиком F(h) (см. рис. б): А = = = дж. 4 2р. 6.12. Ответ: т = 170,06 г; im = 170,03 г. Указание. Вес гирь в воздухе равен mtg(l - р./рл), а вес шара mg(l - р./р.). Здесь р», рл, р<- — плотность воздуха, латуни и стекла соответственно. \ .
295 Механика 6.13. Ответ: 97 кг. Решение. Обозначив через т начальную массу воздушного шара, а через рв — плотность воздуха у поверхности Земли при 0°С, запишем условия равновесия шара до и после подъема: wi£=p,.£7; (m-Am)g-^g 1,5V. Вычитая второе уравнение из первого, получим: Лт = 0,25рвК = = 97 кг. 6.14. Ответ: N = —[p(d2/ii + D2h) - p»D2h + p.d\H - ЛО] . Решение. Обозначим плотность бетона через р, плотность воды — рв. Обратим внимание: обычного «уменьшения веса» за счет силы Архимеда здесь не происходит, так как вода не давит на нижнюю часть конструкции снизу. Действующие на конструкцию силы указаны на рисунке (N — сила реакции дна; Fi и F-> — силы давления воды соответственно на верхнюю и нижнюю горизон- тальные поверхности конструкции). Из условия равновесия сле- дует N = mg + Fi - Fz, причем т = ^(d2/i> + О2Л); „ nD2 ,rr , ,, „ n(D~ - d2) Fi = ——pug(H — hi — Л); Fi - —'—p»g(H - hi) . Отсюда N = ~[p(d2hi + P2h) - pJD2h + p.d2(H - Л1)]. Согласно третьему закону Ньютона эта величина совпадает с величиной силы давления конструкции на дно. 6.15. Ответ: уменьшится. Решение. На первый взгляд может показаться, что из-за давления * масла на верхнюю грань бруска он опустится. Однако заметим, что еще более толстый слой масла давит на поверхность воды. А
Ответы, указания, решения 296 согласно закону Паскаля это давление полностью передается на нижнюю грань бруска. Таким образом, разность между силами давления на брусок снизу и сверху возрастает, поэтому брусок будет «всплывать» из воды, т. е. глубина его погружения в воду уменьшится. 6.16. Решение. Представим, что тело исчезло; тогда объем 71 (см. рисунок к условию задачи) заполнится жидкостью 1, а объем V-з — жидкостью 2. Выделенные объемы жидкостей будут, конечно, находиться в равновесии. Значит, сумма действующих на них сил тяжести уравновешивается равнодействующей сил давления со стороны окружающих слоев жидкостей. А ведь эта равнодейству- ющая как раз и представляет собой силу Архимеда! Поскольку сила давления не изменяется вследствие замены твердого тела на «жидкое» с той же поверхностью, получаем J’a = pt Vig + p’Vzg, т. e. сила Архимеда равна общему весу вытесненной телом жид- кости. 6.17. Ответ: ———. Р> - Р- Решение. Из решения задачи 6.16 мы уже знаем, что законом Архимеда можно пользоваться и в этом случае. Обозначим через Vi и V-z части объема тела, находящиеся в нижней и верхней жидкостях. Тогда условие равновесия тела имеет вид: piVi^ + р>Угё = — pVg, где V — объем всего тела. Учитывая, что V-z = V - Vi, получаем: Р ~ Р2 V pi - р/ V, Интересно проследить предельные случаи: если р ра, то — -► О, т. е. тело почти полностью оказывается в верхней, более легкой жидкости; если р > pi, то уг -> 1, т. е. тело почти полностью погружается в нижнюю, более тяжелую жидкость. 6.18. Ответ: на 1,3 см. Указание. См. задачи 6.16, 6.17. 6.19. Ответ: Н = а——— = 4,6 см. Здесь р — плотность стали, р,„ — р[>т — рн плотность ртути, рв — плотность воды.
297 Механика 6.20. Ответ: р = ———р« - 2500 кг/м3. Pi — Рг 6.21. Ответ: не изменится в обоих случаях. Решение- При плавании мяча в воде в неподвижном лифте выпол- няется условие равновесия mg = PngVB, где т — масса мяча, V» — объем вытесненной воды, рв — ее плотность. Когда лифт движется с ускорением, удобнее всего перейти в связанную с ним неинер- циальную систему отсчета, в которой вес покоящегося тела равен m(g ± а). Знак «плюс» соответствует ускорению, направленному вверх, знак «минус» — ускорению, направленному вниз. Можно сказать, что просто происходит замена ‘g на g ± а. Однако в связанной с лифтом неинерциальной системе отсчета сила Архи- меда рв^Ув изменяется во столько же раз, во сколько раз изменя- ется вес любого тела! Значит, равновесие сохранится без изме- нения объема вытесненной воды Иа. 6.22. Ответ: сосуд перевесит гири. Указание. Из-за повышения уровня воды увеличится ее давление на дно сосуда. 6.23. Ответ: А/п = 240 г. Решение. Чтобы определить, на сколько «потяжелеет» сосуд, т. е. на сколько увеличится сила его давления на весы, воспользуемся третьим законом Ньютона. На брусок со стороны воды действует сила Архимеда Га, направленная вертикально вверх. Следователь- но, со стороны бруска на воду (а в конечном счете — на чашку весов) действует такая же по величине сила, направленная верти- кально вниз. Чтобы уравновесить весы, на другую чашку надо добавить груз Ат, удовлетворяющий условию Amg = Fa — p^gV. Отсюда Am = p»V = 240 г (масса дополнительного груза должна равняться массе вытесненной.бруском воды). 6.24. Решение. Ответ становится .очевидным, если рассмотреть действующие на сосуд с грузом внешние силы: сила реакции опоры (чашки весов) должна уравновесить силы тяжести, дейст- вующие на сосуд и брусок. Значит, сила давления сосуда на чашку весов равна сумме этих сил тяжести. Поскольку вес гирь равен весу только сосуда с водой, весы выйдут из равновесия — левая чашка перевесит. Для восстановления равновесия на другую чаш- ку весов надо добавить груз, масса которого равна массе бруска: т = pV = 1,87 кг (здесь р — плотность стали). Можно получить этот результат и иначе. Брусок действует на перекладину с силой Pi, направленной вниз: Pi = mg - Fa. Кроме
Ответы, указания, решения 298 трго, при погружении бруска в воду сила давления воды на дно увеличивается на Рг - Fa (см. задачу 8.23). Полное увеличение веса левой чашки весов Р = Pl + Р2 = (mg - Ел) + Fa = mg, как уже получено выше. Распространенная ошибка при решении подобных задач — замена Р на Pi (ведь брусок в Воде «легче»). Мы видим, что к этому привычному утверждению следует отно- ситься достаточно осторожно. На наш взгляд, правильнее было бы говорить не об уменьшении веса тела при погружении его в жидкость, а о перераспределении веса тела между двумя «опо- рами», роль одной из которых играет жидкость. 6.25. Решение. На примере этой задачи хорошо видно, что выра- жение «тело, погруженное в воду, уменьшает свой вес» неточно (скорее, просто неправильно!): если тело покоится, его вес всегда равен силе тяжести. Напомним, что вес -— это сила, действующая со стороны тела на опору или подвес. Опорой для плавающего в воде тела является воде • эта опора мягче перины, но это все-таки самая настоящая опора. Эт<> справедливо, конечно, и для тел, плавающих на поверхности (с чем сразу согласится тот. кто умеет лежать на воде). Все, что находится внутри плавающего тела, никакой невесомости ощущать не будет: представим самих себя Внутри батискафа (и даже на обычном корабле — ведь для него вес тоже «полностью исчезает»!). Поэтому гидроневесомость — состояние подводного пловца — только отчасти моделирует насто- ящую невесомость: например, органы тела пловца по-прежнеМу давят друг на друга под действием силы тяжести. А вот органы тела космонавта в условиях настоящей невесомости также неве- сомы. 6.26. Решение. Ответ зависит от того, является ли полый шар более сжимаемым, чем вода, или менее. Если шар более сжимаем, чем вода, его плотность будет увеличиваться при погружений быстрее, чем плотность самой воды (вода с увеличением глубины, хоть и ненамного, но все-таки сжимается: если бы она была абсолютно несжимаемой, уровень мирового океана повысился бы примерно йа 30 м!). Поэтому тело станет тяжелее воды и пойдет на дно. Если же, наоборот, шар менее сжимаем, то при погружении в более плотные слои воды выталкивающая сила увеличится и шар вер- нется на прежнюю глубину. Таким образом, полностью погружен- ное в жидкость тело устойчиво плавает на определенной глубине только в том случае, если оио менее сжимаемо, чем эта жидкость. Аналогично объясняется устойчивое плавание дирижабля или шара-зонда на определенной высоте.
299 Механика 6.27. Решение, а) В атом случае рычаг, очевидно, равноплечий. Поскольку плотность алюминиевого шара меньше, то его объем больше, и потому на него в' воде подействует большая сила Архимеда. Следовательно, при погружении шаров в воду «пере- тянет* железный шар. б) При одинаковом объеме шаров масса железного шара боль- ше. Значит, он находится на коротком плече рычага (ведь рычаг уравновешен). В воде на оба шара подействует одинаковая сила Архимеда. Но момент силы Архимеда, действующей на алюми- ниевый Шар, больше (у этой силы больше плечо). Поэтому и в этом случае в воде «перетянет» железный шар. Совпадение обоих ответов не случайно: относительное «уменьшение веса» тела при погружении в жидкость тем больше, чем меньше плотность тела. 6.28. Ответ: примерно 5 • 10й кг. Решение. Поскольку речь идет лишь об оценке, мы вправе не гнаться за слишком большой точностью. Нам не потребуются данные о толщине, плотности, составе атмосферы. Предлагаем ее просто взвесить. Вес атмосферы равен полной силе давления атмосферы на поверхность Земли площадью S = 4tlR2. Получаем , а> 4nR2pA , mg = 4тгЯ • рд, откуда т = —g~’ где Р* — атмосферное давление. Принимая R — 6400 км, а рд = 105 Па, получим т ® 5 • 1018 кг. Эта величина составляет менее одной миллионной части полной массы нашей планеты. Такая простая оценка возможна потому, что основная часть земной атмосферы сосредоточена на высотах, весь- ма малых по сравнению с радиусом Земли. Значит, можно счи- тать, что на атмосферу действует «обычная» сила тяжести mg. 6.29. Ответ: Q = mgh - 1), где р» и р — плотность воды и пробки Р соответственно. Решение. Поплавок поднимается вверх под действием равнодей- ствующей R двух сил: силы Архимеда Fa и силы тяжести mg. Поскольку R = Fa - mg, то совершенная над поплавком при всплытии работа А = Rh = (Fa - mg)h. Эта работа является мерой перехода механической энергии во внутреннюю: Q = А (ведь поплавок в начале и в конце покоится). Сила Архимеда выража- ется через объем поплавка V = —: Р Fk=p^V=^mg. Р
Ответы, указания, решения.300 Отсюда Q - mgt^- - 1), где р, и р — плотность воды и пробки соответственно. К этому результату можно прийти и исходя из закона сохранения энергии (см. задачу 6.30). 6.30. Ответ: А - 180 Дж; не равна. Решение. Чтобы поднимать тело в воде с малой скоростью и без ускорения, к нему следует приложить силу Fi = mg - Fa = mg - - pegV, где Fa — сила Архимеда, рв — плотность воды. Для дальнейшего подъема в воздухе необходима сила Fz = mg. Полная работа при подъеме А = Fih + РгН = mg(h + Н) - p»gVh. = 180 Дж. Изменение потенциальной энергии тела AWT = mg(h + Н ). Мы видим, что А меньше AWT на величину png'V/z. Что же представляет собой эта величина? Заметим, что тв = pBV — масса вытесненной телом воды. Именно такая масса воды перемещается с поверх- ности на «освободившееся» место на глубине h. Тогда величина AW. = - nugh = -pB<gV/i представляет собой изменение потенциаль- ной энергии воды. Таким образом, работа A =AWT + AWв равна изменению потенциальной энергии всей системы (включающей тело и воду). 6.31. Ответ: р = —- ~—— . и nh(R - г2) Решение. Вода начнет вытекать тогда, когда жидкость припод- нимет кастрюлю. Для этого необходимо, чтобы сила давления жидкости па дно кастрюли F = pS = pgh(pR2 - л г2) уравнялась с действующей на кастрюлю силой тяжести. Здесь S — площадь дна кастрюли (с учетом отверстия); р — давление столба жидкости высотой Л. Отсюда р = - . Т--. и nh(R2 - г1) 7.1. Ответ: 0,50 с; 0,17 с; 0,33 с. Решение. Прежде всего заметим, что амплитуда А мала по срав- нению с длиной I маятника и поэтому колебания можно считать гармоническими. Следовательно, период колебаний Т = 2лА^= — 2,0 с. За время Т маятник совершит одно полное колебание, то есть пройдет путь, в четыре раза превышающий амплитуду его колебаний. Если маятник начинает движение из положения рав- новесия и проходит путь, равный амплитуде, значит, он как раз доходит до своего крайнего положения, и на это уходит время ti - = 0,50 с. 4
301 Механика Для ответа на два последних вопроса необходимо использовать уравнение гармонических колебаний. В случае, когда х = 0 при t = О, . (2nt\ „ это уравнение имеет вид: х = Asinco£ = Asin^-y-J. Если маятник А прошел путь, равный половине амплитуды, то х = значит, Л (27lt\ 1 2nt п . Т - %, то есть -у = , или Z = jg = 0,17 с. На вторую т т т половину пути маятнику понадобится время 7'^9 ~ ®«33 с. 4 J. <£ О Итак, на вторую половину пути требуется больше времени (это можно было предвидеть заранее: ведь движение маятника при удалении от положения равновесия замедляется). 7.2. Ответ: О 7.3. Решение. При движении маятни- ка нить составляет постоянный угол а с вертикалью, а центростремитель- ной силой является равнодействую- щая силы тяжести mg и силы натя- жения нити N. Из рисунка видно, что эта сила F = mgtga. При движении по окружности F — ma2r (циклическая 2л ч _ частота и = у ). Отсюда получаем: m Л - / г Л /feina Т = 2п\------= 2л Л/----- * gtga > gtga При малых углах а можно считать, что cosa = 1, то есть Т = как и для обычного математического маятника. При этом зависимость периода от угла а практически исчезает. 7.4. Ответ: 8,9 км. Решение. Отставание маятниковых часов на горе связано с тем, /г что период колебаний маятника Т ~ 2лЛ/~ зависит от ускорения свободного падения. При подъеме g уменьшается, поэтому Т увеличивается, а частота колебаний уменьшается. Значит, за то же самое время маятник часов на горе совершит меньшее число колебаний и поэтому часы покажут, что прошло меньше времени. Показания часов т прямо пропорциональны числу колебаний
Ответы, указания, решения 302 т То маятника, т. е. частоте колебании, поэтому — = где то и То — те Т показания часов и период колебаний маятника на уровне моря. Значит, когда маятниковые часы внизу покажут время то, часы То на горе покажут время т = То~^, то есть отстанут на Ат = т0 - т = = То|1 - . С другой стороны, из формулы для периода коле- То банин следует уг g R1 следует — =------где R — радиус Земли, а Л — высота горы. go (R + Л) Подставив это выражение в формулу для Ат, получаем . I-, R \ Л ” R + h' R + h h что примерно равно то -=, поскольку h «к R (высота даже самой К высокой горы намного меньше радиуса Земли). Отсюда получаем а из закона всемирного тяготения Л = R— - 8,9 км. Это немного больше, чем высота Джомолунгмы То (8848 м) — самой высокой горной вершины на Земле. Однако читатели, знакомые е правилами приближенных вычислений, понимают: произведя расчет с точностью до двух значащих цифр, мы не имеем оснований усомниться в правдивости барона. 7.5. Ответ: — = 1 - В2 (В + А)2 ® 2— ® 0,002 , т. е. маятник надо уко- ротить на 0,2% его длины. 7.6. Ответ: v = = 12,5 м/с = 45 км/ч. Указание. Маятник раскачивается от толчков на стыках рельсов; наиболее сильное раскачивание (резонанс) наблюдается при сов- падении частоты «стука колес» с собственной частотой маятника. 7.7.Ответ: а)Т = 2л; б) Т = 2п\ ——; в) Т= 2лЛ/г-,---- 7; 4g + a 4g - а 4ylg2 + а2 г) Т = глЛ/——. ’geos а Период колебаний пружинного маятника во всех случаях оста- _ Г. Irn ется равным Т = 2лЛЛ-^- . Указание. Во всех случаях удобно перейти в систему отсчета, в которой математический маятник неподвижен. В этой неинер-
303 Механика циальной системе отсчета ускорение свободного падения отлича- ется от обычного (см. задачи 2.5, 2.20, 2.23), что и приводит к изменению периода Т колебаний математического маятника. 7.8. Решение. В начале движения сопротивлением воздуха для падающего лифта можно пренебречь. Рассмотрим движение маят- ника в начале падения лифта, когда лифт движется с ускорением свободного падения. При этом в лифте возникает состояние неве- сомости, что равносильно «выключению» силы тяжести. Исчеза- ет, конечно, и сила, возвращающая маятник в положение равно- весия. Значит, линейная скорость маятника будет сохраняться, и движение его определяется тем, в каком начальном состоянии «застигло» маятник состояние невесомости. Например, если он в начальный момент находился в крайнем положении (скорость равнялась нулю), он так и «застынет» в этом положении. При дальнейшем падении лифта начнет сказываться сопротивление воздуха. Это приведет к постепенному «включению» силы тя- жести: маятник снова начнет совершать колебания, частота кото- рых будет увеличиваться до тех пор, пока ускорение лифта не станет равным нулю, т. е. пока движение лифта не станет равно- мерным (это вполне возможно, поскольку речь идет о падении лифта в небоскребе!). При равномерном движении на маятник в лифте будет действовать «обычная» сила тяжести и он будет колебаться с той же частотой, что и в покоящемся лифте. 7.9. Ответ: 0,80 с. Т = 2л Решение. На первый взгляд кажется, что в условии не хватает данных — не известны ни масса груза ти, ни коэффициент жест- кости пружины А, а период колебаний Т как раз от них и зависит: m Однако отношение -г можно легко узнать, потому что п для неподвижно висящего на пружине груза mg = АД/, то есть т А/ т /дГ , и поэтому Т = 2л А/—, так что период (и частота) коле- Н ё ’ ё баний груза на пружине однозначно определяется величиной удлинения пружины, когда к ней подвешен груз. В данном случае период колебаний равен 0,80 с. _________ _ „ . . _ „ _ I т . Im(ki + Аз) 7.10. Ответ: а) Т = 2ЛЛ/-; б) Т = 2л*\/- . 'ki + kz V kikz в) т = 2лл/~--. ’ Ai + kt Указание. В случае в независимо от расстояния между стенками существует Положение равновесия, в котором действующие на
Ответы, указания, решения 304 груз силы упругости со стороны обеих пружин взаимно ком- пенсируются. При отклонении от этого положения возникают «добавки» ДГ1 и ДГг к обеим силам упругости, направленные всегда в одну сторону (когда одна пружина растягивается, другая на столько же сжимается). Значит, в случае в мы фактически имеем дело с таким же параллельным соединением пружин, как и в случае а. Вычисление коэффициента жесткости системы пружин см. в задаче 5.6. 7.11. Ответ: доска будет совершать гармонические колебания с [L периодом Т = 2лЛ/---; доска соскочит с валиков. > 2pg Решение. Очевидно, если центр дос- ки расположен на равных рассто- яниях от обоих валиков, то доска будет оставаться в равновесии. В любом другом случае доска придет в движение. Поскольку в условии говорится о быстром вращении ва- ликов, между ними и доской происходит проскальзывание, и сила трения скольжения, действующая на доску со стороны каждого из валиков, направлена внутрь (см. рисунок). Из второго закона Ньютона и формулы для силы трения скольжения следует, что /па* = Ft,>i - - p(Ni - Ni). Здесь Ni и Ni — силы давления доски на первый и второй валики соответственно. Если доска сместилась от положения равновесия на расстояние х, то из правила моментов следует: L N, 2 Х L 2+Х Учитывая, что JVi + Nz = mg, получаем: Ni = Ms = . Тогда уравнение движения имеет вид: 2 pg ах ---р^х . Это уравнение описывает гармонические колебания с цикли- _ l2pg [~L ческой частотой хов = \/~~ и периодом Т = 2п\-—. Если каждый валик станет вращаться в обратную сторону, то обе силы трения будут направлены «наружу». Смещение доски от
305 Механика положения равновесия увеличит (за счет перераспределения на- грузки на валики) как раз ту силу трения, которая действует в сторону смещения (и уменьшит силу трения о другой валик). Поэтому вместо возвращающей силы возникает сила, уводящая доску еще дальше от положения равновесия. Равновесие в этом случае неустойчиво, колебаний вокруг него не возникает. Доска будет сброшена с валиков. 7.12. Ответ: Т = 2л -\/ -—* . " А(/П1 + т2) Решение. Для определения периода ко- лебаний пружинного маятника очень заманчиво было бы воспользоваться «готовой» формулой Т = 2л ко она относится к случаю, когда один конец пружины закреплен. Может быть, у нашей пружины тоже есть неподвижная точка, которая может сыграть роль закрепленного конца? В той инер- циальной системе отсчет?., в которой полный импульс обоих тел равен нулю, такая точка действительно существует — это центр масс грузов С (см. рисунок). Если полная длина пружины I, то расстояние от центра масс до каждого из грузов задается соотно- rtii I2 , , , , m-i mi шениями — = —; Ц + l2 - I. Отсюда Ц =-----1; 12 =----1. По- тг И ТП1 + ГП2 Ш1 + ГП2 скольку при колебаниях все витки пружины деформируются одинаково (т. е. h и 1г изменяются в одинаковое число раз), неподвижный центр масс С как бы «привязан» к одной и той же точке пружины, то есть исходную систему можно рассматривать как два отдельных пружинных маятника. Поскольку коэффициент жесткости пружины обратно про- порционален ее длине, мы можем найти коэффициенты жесткости „ kt I k2 I частей пружины с длинами h и lz из соотношении ~г = —; — = 7-, R Zi к /2 , . mi + т2 , , mi + т2 следовательно, Ai = k-----, k2 = k-----. m2 mi Тогда период колебаний каждого из маятников составит Т = 2лЛ & = 2лЛ & = 2лл/ . V Ai V «г \ k(mi + т2) Естественно, периоды колебаний обеих «половинок» оказались одинаковыми (иначе центр масс не мог бы покоиться). Грузики колеблются в противофазе: направления их скоростей в любой момент противоположны друг другу.
Ответы, указания, решения 306 7.13. Решение. Правый грузик начнет двигаться вправо, а левый грузик будет прижат к стене до тех пор, пока пружина будет сжата. В тот момент, когда деформация пружины исчезнет, левый грузик перестанет давить на стену и оторвется от нее. После этого оба грузика будут двигаться по столу, совершая колебания друг относительно друга. Однако центр тяжести системы будет двигать- ся равномерно и прямолинейно: на систему грузов не действуют внешние горизонтальные силы. Колебания грузов происходят в противофазе. Таким образом, качественное решение задачи мы нашли. Что- бы получить количественный ответ, необходимо ответить на три вопроса: а) чему равен период Т колебаний грузиков, б) с какой скоростью Оцм движется центр масс системы, в) в каких пределах изменяется длина пружины при колебаниях. Ответ на первый вопрос уже дан в решении задачи 7.12: Т = 2к л/—- = 0,24 с. V k(mt + m2) Для ответа на второй вопрос найдем скорость центра масс. Отметим, что эта скорость меняется только до тех пор, пока левый грузик давит на стенку (а стенка на него!). Следовательно, надо определить скорость центра масс именно в момент отрыва левого грузика от стенки: в дальнейшем эта скорость остается неизмен- ной. В момент отрыва пружина не деформирована (этим и опре- деляется момент отрыва!), значит, вся потенциальная энергия деформации пружины перешла в кинетическую энергию правого k(Al)2 m2V2 „ грузика: —=— = —~—• Отсюда определим скорость правого грузи- Л и ка в момент отрыва левого грузика от стены: о2 = —. Левый m2 грузик в этот момент еще покоится (oi = 0), поэтому скорость , 7П1О1 + ТПаРг центра масс, вычисленная по формуле ип„ =-------------, составит zni + пи M^km2 _ , и.,» =------= 0,19 м/с. mi + т.2 Чтобы ответить на третий вопрос, заметим, что в моменты наибольшего растяжения или сжатия пружины грузы покоятся друг относительно друга и кинетическая энергия системы равна П1 + 7П2)02цм „ ——л-------. Потенциальная же энергия пружины в этот момент равна -у, где х — величина максимального удлинейия. Согласно
307 Механика закону сохранения энергии k{M)2 kX? (mi + 7П2)и2пм 2 ~ 2 + 2 Подставляя в это уравнение выражение для и,(И, находим х = AZ«\/ т'— = 1 см. V ГП1 + ГПг Длина недеформированной пружины равна 1л + AZ. Значит, при колебаниях длина пружины меняется от Zmm — lo + AZ - х = 7 см ДР 1тлх = Zo + AZ + x = 9cm. Таким образом, деформация пружины в ходе движения не достигает начального значения AZ. Дело в том» что „г часть начальной энергии деформированной пружины Wo = „ не- & реходит в кинетическую энергию поступательного движения сис- темы как целого: (mi + mi ft(AZ)2 пъ Tv k =----s-----= —— —„ =---------------TVo. A mi + ГПг & ГП1 + ГПг ГПг Эта часть тем больше, чем больше отношение —. Она заметно 7П1 изменится, если грузы поменять местами (от 75% до 25% в нашем случае). 7.14. Ответ: 0,26 м. Решение. Чтобы определить толщину d отколовшейся льдины, достаточно узнать ее массу М, поскольку М = piSd. Эту массу можно, в свою очередь, найти, зная период колебаний льдины, на которой находится барон: при изменении осадки на величину Ах возникает возвращающая сила, обусловленная изменением силы Архимеда: AFa = pBgAV = p»gSAx. Как видно, возвращающая сила пропорциональна величине отклонения льдины от положения равновесия, поэтому колебания будут гармоническими. Цикли- ~ ы ______ 2 Рр^^ ческая частота колебании определяется соотношением «г = —— . М + т т * ' P'SS Таким образом, М = —-— т, а толщина льдины «г М _ p.gS - та2 p„S <n2p.,S = 0,26 м. : Мы учли, что со = 2nv, где v = 1 Гц. Заметим, что на самом деле колебания льдины очень быстро затухнут из-за сопротивления Йоды!. '
Ответы, указания, решения 308 _ ml т ml Г1\ 7.15. Ответ: Ti = 2лЛ/----; Т2 = гаЛ/----- + *\/~). Vkl + mg ' kl + mg yg> Решение. Определить период колебаний системы К можно не только исходя из уравнения движения, I \ но и воспользовавшись выражением для полной | \ механической энергии. Пусть отклонение сис- । темы от положения равновесия описывается ве- . \ личиной х, а выражение для энергии \ W = Ах2 4 В(х')2. (1) __х__ \ Здесь А, В — положительные константы; х' — = dx/dt — скорость изменения величины х. L — Величина х совсем не обязательно совпадает с декартовой коор- динатой: это может быть, например, угол отклонения маятника от вертикали. Если в системе отсутствует трение, то полная механическая энергия остается неизменной. Значит, W = О. Отсюда получаем А А Ах+Вх"= О, или х" = - ~б* . Поскольку о > О, это уравнение сов- 25 25 падает с уравнением гармонических колебаний а* = -юо2х. Следо- вательно, колебания являются гармоническими тогда (и только тогда!), когда выражение для энергии имеет вид (1), причем циклическая частота со0 Попробуем применить этот подход к решению данной задачи. При малом отклонении х груза от положения равновесия пружина приобретает потенциальную энергию кроме того, груз приоб- & ретает потенциальную энергию mgh, где (см. рисунок) h I - - yl2 - х2 * -ху (см. математическое приложение). _ mu2 mix')2 Если учесть и кинетическую энергию —= —х—, то получим Л TV - /ь X mg\x2 ТУ-(Л + ~)2’ + ~2“- Это выражение соответствует формуле (1) при А — ~:(k + & mg о т „ , + —г-), В = —. Поэтому можно утверждать, что колебания — гар- I & ионические с периодом _ 2л Г, = — Фо = 2пу^=2пу[ ml kl + mg
309 Механика Если заменить пружину на полоску резины, то при смещении груза влево от положения равновесия резина свободно провисает и не влияет на движение груза (разумеется, если можно пренеб- речь ее весом). Поэтому период колебаний Тг такой системы складывается из половины Ti и половины периода обычного математического маятника: ' V Ы + mg V g> 7.16. Ответ: —- = *\/l + -j- . V1 V 4mg Решение. В отсутствие пружины мы имеем дело с обычным математическим маятником, частота колебаний которого Прикрепление пружины увеличивает возвращающую силу при отклонении маятника от равновесия, так что частота колебаний должна возрасти. Проще всего определить частоту v2 колебаний системы из выражения для полной механической энергии W системы. При отклонении груза ст положения равновесия на х « mgx2 «Z он приобретает потенциальную энергию mgh = - (см. реше- ЛЬ X ние задачи 7.15); кроме того, растяжение пружины на -% сообщает *© с ей потенциальную энергию —х—. С учетом кинетической энергии Л mv1 m. ,ч2 img k\x2 m(x')2 ~Г = T(x> получаем w = (— + l)У + ~2~- Отсюда (см. решение задачи 7.15) находим циклическую час- тоту и частоту колебаний: Гоо = А /4 + ; v2 = — . у I im 2л Следовательно, в результате прикрепления пружины частота - v2 . /, kl колебании увеличивается в — = 1 + раз. 7.17. Ответ: Т = 2л*^^, если маятники отклонены в одну сторону; Т = 2kl ПрИ o'11010™™11 их в противоположные стороны. Решение, а) Пружина не деформируется при колебаниях, так что сила упругости не возникает (связь между маятниками «не рабо-
Ответы, указания, решения 310 г ' ' тает»). Поэтому оба маятника колеблются независимо друг от „ „ [Г друга с одним и тем же периодом Т = 2л^- • б) Воспользуемся методом, описанным в решении задачи 7.15. Если каждый из маятников отклонился от положения равновесия на величину х, то деформация пружины равна 2х, а энергия k(2xY ^mgx1 „mv2 mg. , ,.2 ~ системы W = -~д ь 2—57- + 2—»- = (2k + -Л)х2 + zn(x'). Отсюда T = 2it^J------ = 2n*\/----—-. ^2k + —- y<mg+2kl 7.18. Ответ: t = 42 мин; v = 7,9 км/с. Решение. Поскольку эта задача предложена в разделе, где рас- сматриваются колебания, у читателя может возникнуть догадка, что движение «сквозь Землю» — гармоническое колебание. Чтобы подтвердить (или опровергнуть) эту догадку, вспомним, что на тело, находящееся на расстоянии х от центра Земли, действует х , сила тяготения mg— (т — масса тела, g — ускорение свободного rv падения у поверхности Земли, R — радиус Земли; см. задачу 3.6). Эта сила пропорциональна величине отклонения х от положения равновесия и является возвращающей, поэтому движение «сквозь Землю» — действительно гармоническое колебание с периодом Т = 2лЛ^ = 84 мин. Полученный период совпадает с периодом обращения искусст- венного спутника Земли на малой высоте, но этому обстоятельству не стоит придавать слишком большого значения: если учесть неоднородность Земли, то совпадение исчезнет. Пролет сквозь Землю в одну сторону составляет половину периода, т. е. 42 мин. Согласитесь, что барон действительно нашел довольно быстрый способ перемещения из Арктики в Антарктику! Читателю не составит труда убедиться, что через центр Земли барон пронесся со скоростью v = ^Rg - 7,9 км/с, равной как раз первой космиче- ской скорости! 7.19. Ответ: 0,71 с. Решение. Прежде всего необходимо разобраться в характере дви- жения. Заметим, что оно не является равноускоренным: чем дальше выезжают санки на асфальт, тем больше сила трения Ftp, а, значит, и ускорение санок. Найдем уравнение движения. Пусть т — масса санок, х — длина той части полозьев, которая в данный
311 Механика момент уже выехала на асфальт. Тогда на зту часть полозьев хг х г* *'*’ приходится только часть веса санок Ni = mgj-. Соответствующая X сила трения об асфальт = pNi = pmg-j-, а уравнение движения г, х санок имеет вид: max = -F^ - -pmgj, или а, = - ^х- Это хорошо известное уравнение гармонических колебаний, период которых Т = 2лл/—. Продолжительность движения от точки х = 0 до . Т п I п остановки соответствует четверти периода: t = , = — =0,71 с. « 2 Vpg 7.20. Ответ: Т = Решение 1. Пусть уровень жидкости в одном из колен сосуда увеличился на h относительно равновесного уровня (см. рисунок). Тогда в другом колене уровень я:идкости на столько же опустился. Разность давлений, создаваемых двумя вертикальными столбами жидкости, состав- ляет Ар = pg- 2h, а возвращающая сила F = = Ар • S = 2pgSA (р — плотность жидкости, S — площадь сечения сосуда). Поскольку масса жидкости m = 2pSH, уравнение движения столба жидкости имеет вид: 2pSHa = -2pgSh, g т. е. а - - ту А. Это — уравнение гармонических колебаний с И периодом Т = Решение 2. При изменении уровня жидкости на h потенциальная энергия жидкости растет. Ее прирост AWP такой же, как при подъеме столбика жидкости высотой Л (и массой Am = pSh) на высоту Л (см. рисунок): AWP = bmgh = pSgti1. mu1 Кинетическую энергию жидкости запишем в виде Wk = —5- = = pSH(h’)2. Если отсчитывать механическую энергию W системы от «нулевого уровня», соответствующего положению равновесия, 'го W = pSgH + pSH(A')2. Из этого выражения для энергии следует (см. решение задачи 7.15), что период колебаний Т = 2n\]^i
Ответы, указания, решения 312 7.21. Ответ: Т = 2л*\/--------. N gfmdi + mdz) Решение. При нахождении периода Т будем исходить из выра- жения для полной механической энергии W колебательной сис- темы (см. решение задачи 7.15). Если угол отклонения стержня от вертикали ср, то каждый из шаров поднимается от положения равновесия на высоту Л1,2 = coscp). При этом система приоб- ретает потенциальную энергию = mtghi + might = (mdi + mdz)g(l - coscp) = 2gsin2 ^(mdi + mdz) . Выражая скорости шаров через угловую скорость стержня со = ср', получаем щ,2 = cp'Zi.z. Тогда кинетическая энергия системы ... mtv,2 тмг (ср')2. ,, , Л Wk = —+ —s— - ^-(mdi2 + mdz2) . iU iU £ Поскольку мы рассматриваем малые колебания (ср « 1 рад), sin^ можно заменить на и выражение для полной энергии приобретает вид т„ g, , , ч 2 md2 + rnd-t2 ,.2 W = -^(mdi + znzZzJcp2 +-g-----(ср') • Эта формула позволяет сделать вывод: система совершает гар- - m I mil* + тт монические колебания с периодом Т = 2 л д/-----------—. Для ’ g(mdi + mdz) проверки можно рассмотреть предельные случаи Zi -* lz, Zi -> О или mi -*• О. При этом полученная формула переходит в хорошо извест- ную формулу периода колебаний математического маятника. _ Л А / mdi2 + mdz2 „ , , 7.22. Ответ: Т = 2лД/—---------,. При mdi -* milt период Т стреми- ’ ggndi - тД тся к бесконечности, при mdi — mdz колебания вообще не возни- кают (любое положение стержня соответствует безразличному равновесию). Указание. См. задачу 7.21. 7.23. Ответ: А > = 6,2 мм. 4л2 Указание. Тело движется вместе с подставкой, если максимально- го значения силы трения покоя достаточно для сообщения телу максимального ускорения при колебаниях. 1 [g 7.24. Ответ: v < —= 7,0 Гц. 2л Указание. В момент отрыва от подставки тело находится в состоя-
313 Механика нии невесомости. Это возможно, если максимальное ускорение подставки превышает ускорение свободного падения; максималь- ное ускорение, направленное вниз, достигается в верхней точке. 7.25.О™,:Л=л/(^)' + ^. Решение. Несмотря на то. что удар шарика о чашку неупругий, мы все-таки можем воспользоваться законом сохранения меха- нической энергии, поскольку в условии сказано, что чашка очень легкая. Действительно, в этом случае скорость чашки с шариком непосредственно после удара практически равна скорости шарика в момент удара, и поэтому кинетическая энергия чашки с шари- ком после удара тоже практически равна кинетической энергии шарика перед самым ударом. Пусть в результате удара максимальное отклонение чашки вниз равно х. Поскольку скорость чашки с шариком в этом положении равна нулю, потерянная шариком энергия mg(h + х) полностью переходит в потенциальную энергию деформированной /2 kx~ пружины т. е. mg(h + х) = —х~. Отсюда получаем два решения £ & 1,2 ~ k V k ) k ‘ Что означают эти два решения? Дело в том, что составленному нами уравнению удовлетворяет любое х, при котором скорость чашки с шариком равна нулю. А таких точек, конечно, две — они соответствуют верхнему и нижнему крайним положениям чашки - „ . Xi + хг mg при колебаниях. Среднее арифметическое —™— = -т~ соответст- Z R вует новому положению равновесия (когда на чашке лежит груз). „ л - 1(т^\г 2mgh Очевидно, Xi,2 = -г- ± А, т. е. А - Л/(-7-1 + —г—. Заметим, что при Л = 0 (шарик осторожно кладут на чашку) амплитуда колебаний А не обращается в нуль! Она, как и следо- mg вало ожидать, равна -г-, R т. е. равна расстоянию от начального положения чашки до нового положения равновесия. 7.26. Ответ: у = Acos(ot - kx), где k - Решение. Амплитуды колебаний всех точек по условию одинако- вы (волна незатухающая) и равны А, поэтому колебания в точке с координатой х будут отличаться от колебаний в точке О лишь начальной фазой. Это отличие обусловлено тем, что волна прихо-
Ответы, указания, решения 314 дит в точку с координатой х с запаздыванием на At = — по сравнению с точкой О. Следовательно, интересующее нас уравнение коле- баний имеет вид: у = Acosco(t - At) = Acos^cot - ~). „ 2л _ . _ . Учитывая, что й = у и иТ = л, где Т — период, а Л — длина . , 2п . волны, получаем у = Acos(tot ——х) — так называемое уравнение Л «-г» , 2л со бегущей волны. Величину k = — = — называют волновым числом. Если записать уравнение бегущей волны используя k, получим у - Acos(cot - kx), откуда видно, что волновое число играет ту же роль по отношению к координате, что циклическая частота по отношению ко времени. 7.27. Ответ: частота не изменяется; длина волны увеличивается в 4,4 раза. Решение. При падении из воздуха на поверхность воды звуковая волна частотой v вызовет колебания поверхности воды с такой же частотой. В свою очередь, колеблющиеся точки поверхности вы- зовут в толще воды распространение волн той же самой частоты. Следовательно, при переходе через границу раздела сред волна не изменяет своей частоты. Однако скорости звука в воздухе vi и воде тт , Vl иг различны. Поскольку длина волны в воздухе Ai = —, а в воде Ха = —, получаем ~ = — = 4,4. Таким образом, длина волны при v Xi Vi переходе из одной среды в другую изменяется пропорционально скорости волны в данной среде. 7.28. Ответ: v* = k • 50 Гц, где k — произвольное натуральное число. Решение. Собственные колебания воздушного столба представля- ют собой стоячие волны, причем у закрытых концов трубы нахо- дятся узлы стоячей волны. Поскольку расстояние между ближай- шими узлами равно Х/2, на длине трубы должно укладываться X 2Z целое число полуволн: I = ~rk (k *= 1, 2, 3, ...). Тогда X* = -г, и 1 - а к собственные частоты v* = в -^k, где и — скорость звука в воздухе. X* 41 Итак, v* = k • 50 Гц.
315 Механика 7.29. Ответ: при уровнях воды 25 и 75 см. Решение. Усиление звука происходит в случае резонанса, т. е. когда частота звука совпадает с собственной частотой колебаний воздушного столба в сосуде. Поскольку плотность воды намного превышает плотность воздуха, поверхность воды можно рассмат- ривать как твердую нижнюю «пробку» воздушного столба. Поэто- му у поверхности воды должен находиться узел, а у верхнего края сосуда — пучность стоячей волны. Расстояние между ближай- шими узлом и пучностью стоячей волны равно Х/4, а между бли- жайшими узлами к/2. Поэтому если обозначить высоту уровня воды через Л, то длина воздушного столба Н - Л должна удовлет- X Л. ворять соотношению Н - h = ^ + k yz, где k — целое число. Пос- 4 & кольку X = — = 1 м (о — скорость звука в воздухе), это соотношение выполняется при k = 0, Ао = 0,75 м и k ~ 1, hi = 0,25 м. Итак, звук усиливается при уровнях воды 25 см и 75 см. 7.30. Ответ: 19 см. Решение. Резонаторный ящик представляет собой «трубу», откры- тую с одного конца. Минимальная длина ее, при которой наблю- , X и . дается резонанс, I - -г = — = 19 см (здесь v — скорость звука в 4 4v воздухе). Никаких нарушений закона сохранения энергии, естественно, нет. Громкость звучания определяется (при заданной частоте) не энергией излученного звука, а его мощностью. Резонатор увели- чивает мощность излучаемого камертоном звука, т. е. расход энергии его колебаний в единицу времени. Поэтому камертон, установленный на резонаторе, быстрее израсходует свою энергию и затихнет. 7.31. Ответ: h = — (2k + 1) = 50(2Л + 1) м, где и — скорость звука 4м в воде; k — 0, 1, 2, 3, ... . 7.32. Ответ: 300 Гц; 150 Гц. 7.33. Решение. Почему, собственно, следует ожидать какого-либо изменения звука из-за изменения состава «атмосферы»? Дело в том, что при замене азота гелием значительно понижается средняя молярная масса газовой смеси, а потому заметно увеличивается скорость звука и. Звук голоса (как и любой звук) можно представить как резуль- тат сложения гармонических звуковых волн различных частот
Ответы, указания, решения 316 (набор этих частот называют спектром звука). Колебания различ- ных частот имеют разные амплитуды. Звучание голоса определя- ется частотами, для которых амплитуды звуковых колебаний велики. Гортань представляет собой «резонаторный ящик», уси- ливающий звуки с определенными длинами волн. Эти длины волн К зависят от формы и размеров гортани и поэтому не изменяются при замене воздуха на дыхательную смесь. Однако из-за увели- v чения v «резонансные» частоты гортани v - — изменятся (станут Л выше), и поэтому голос станет более высоким. То же самое произойдет, конечно, и с тоном духовых инструментов (которые также представляют собой резонаторы). Частота звучания камертона определяется его формой и разме- рами, а также плотностью и упругими свойствами материала (стали). Поэтому его звучание в кислородно-гелиевой дыхательной смеси не изменится. Если камертон установлен на резонаторе, то по указанным причинам его тон не изменится. Однако резонанс- ная частота столба «воздуха» в резонаторе станет другой, поэтому резонанс нарушится, и резонаторный ящик практически не будет усиливать звучание камертона. 7.34. Ответ: форму конуса с углом при вершине 2arcsin Решение. Фронт ударной волны пред- ставляет собой огибающую сферичес- д „ ких волн, испущенных пулей в каж- г- * + —• —W м v В J дым из предшествующих моментов вре- мени. При и > и пуля обгоняет испу- Рис’ “ щенный ею звук, так что фронт ударной волны может охватывать лишь область позади пули. Докажем, что фронт ударной волны представляет собой кони- ческую поверхность. Пусть пуля в данный момент находится в точке А. Рассмотрим звуковую волну, испущенную пулей в точке В (см. рис. а). Эта волна имеет к данному моменту «возраст» АВ Д£ = Значит, ее волновая поверхность — сфера радиуса R — v = v&t = АВ Построим конус с вершиной в точке А, касающийся этой сферы. Угол а между образующей конуса и его осью (направ- . R v лением полета пули) находим из соотношения sina = tv. = —. АВ и Оказывается, этот угол не зависит от длины отрезка АВ1 Значит, этот конус касается всех волновых поверхностей ранее испущен-
317 Механика ных звуковых волн, т. е. является для них огибающей поверхностью (ср. с задачей 1.20). Это — так называемый звуковой конус (см. рис. б). 7.35. Ответ: на расстоянии 10 км. Решение. Наблюдатель в точке А услышит звук (громкий хлопок), когда точки А достигнет поверхность звукового конуса (см. реше- ние задачи 7.34), который самолет «та- щит» за собой. К этому моменту само- лет окажется в точке В (см. рисунок). h Очевидно, АВ =-----= 2й = 10 км, где sina sina = ^ = 0,5. Обратите внимание: до- шедший до наблюдателя звук самолета «родился» не в точке С, а в точке 2J! 7.36. Ответ: более высокий звук слышит водитель лежащего автомобиля. Решение. В системе отсчета лежащего автомобиля изменяется длина волны звука от приближающегося источника: за один период Т колебаний источник приближается на расстояние иТ, поэтому расстояние между ближайшими «гребнями» волн (уплот- нениями воздуха) равно не v„T, а и„Т - иТ. Это расстояние и есть длина звуковой волны, воспринимаемой неподвижным относи- тельно Земли автомобилистом, т. е. X = (у.№ - и)Т. В то же время скорость звука в его системе осталась неизменной, поскольку она определяется упругими свойствами воздуха, который покоится относительно этого автомобиля. Следовательно, звук, слышимый пострадавшим водителем, имеет частоту 1Л<» Оз» Vo V1 = — = ------— =---------- 533 Гц. X (о». - и)Т и V:,, Для движущегося автомобилиста повышение звука «покоя- щейся» сирены происходит по другой причине: в его системе отсчета длина волны звука не изменяется (X - Хо), поскольку источник звука неподвижен относительно воздуха, однако изме- няется скорость звука. По отношению к едущему автомобилю воздух вместе со всеми сгущениями и разрежениями звуковых волн движется навстречу со скоростью и, поэтому скорость этого звука для движущегося автомобилиста v = и». + и. Следовательно,
Ответы, указания, решения 318 он слышит звук, частота которого V VM + и ' Р-' / и \ кпл _ va = — = —- =--------------= vol 1 + —I = 500 Гц. Ал Ал Ал ' Р’»* Таким образом, более высокий звук приближающейся сирены услышит пострадавший автомобилист. Эффект изменения часто- ты звука при движении источника или приемника обнаружил X. Доплер, поэтому он называется эффектом Доплера. Обратите внимание, что при медленном движении (— « 1) ' 1Л|В ' получаем Vi » v2, т. е. движение источника или приемника звука приводит к одинаковому изменению принимаемой частоты. _ _ /, usina \ , 7.37. Ответ: v = voll +-1, если катер удаляется от берега; V - Vo |1 - psinai , —j~|» если катер приближается к берегу. Решение. Если катер удаляется от берега со скоростью v, как показано на рисунке, то проек- ция его скорости на направление распространения волн равна -usina. Время At между двумя ударами гребней волн о катер определяется из условия и psinaAt + uAt = А = —. Vo Здесь X — длина волны. Учи- тывай, что At = —, получаем psinax V = 41 + ~V~)- _ i, psinai _ Если катер приближается к берегу, то v = Vo |1 —|. В этом случае катер может обгонять волну или отставать от нее. При osina = и катер «оседлает» волну и будет двигаться вместе с ней. При этом удары волн о катер прекратятся (v = 0). а а - 7.38. Ответ: во = -^=, аг. = 0; во = Решение. Из условия следует, что оба динамика излучают коге- рентные звуковые волны с длиной волны X = - = 0,5 м (здесь v — v а.
319 Механика скорость звука в воздухе). До точки С обе волны доходят за одинаковое время. Согласно условию, в этой точке они усиливают друг друга, значит, мембраны обоих динамиков колеблются в одной фазе. Кроме того, можно считать, что обе волны приходят в точку С (и в близкие к ней точки D, Е) с одинаковой амплитудой Разность хода обеих волн в точке D составит Дв = AD - BD - \ х = 2CD ~ -у; в точке Е разность хода Де = АЕ - BE = 2СЕ = . 4 2 Отсюда следует, что в точку D волны приходят с разностью фаз 9, 6а а в точку Е — с разностью фаз л, т. е. в противофазе. Очевидно, в точке Е обе волны должны почти «погасить» друг друга, и поэтому амплитуда колебаний в точке Е равна 0. В точке D „ а складываются два колебания одной частоты с амплитудами 9 и А сдвигом фаз Следовательно, А U U 71 U Cl 71 XD = -gCOSCOt + £COS((Ot + = gCcOSCO* - sincoO = COS((X)£ + Итак, амплитуда колебаний в точке D равна -f=. Если поменять \2 местами выводы одного из динамиков, то мембраны обоих дина- миков станут колебаться в противофазах. Теперь обе волны «га- сят» друг друга в точке С, зато усиливают в точке Е: амплитуда колебаний в точке Е станет равной а. В точке D изменение фазы одной из волн на противоположную не влияет на амплитуду результирующе- _ а го колебания — эта амплитуда по-прежнему равна 8.1. Ответ: г] = 88%. Указание, т] = mgh FL , где F — направленная вдоль наклонной плос- кости сила, необходимая для равномерного подъема тела массой т. 8.2. Ответ: h = 3,6 м. Решение. Механическая энергия тела при движении сохраняется. После отпускания веревки тело летит вертикально вверх и в верхней точке вся его кинетическая энергия переходит в потен- „ 1. mv‘ циальную. Значит, mgh = —5—, где v — скорость тела в нижнеи А точке. Скорость и находим, применяя второй закон Ньютона (см.
Ответы, указания, решения 320 рисунок): та = Т - mg, где а = — — центростремительное уско- рение при прохождении нижней точки окружности. Отсюда v2=- 4Л== 3>6 м- \mg > &\mg > К задаче 8.2. 8.3. Ответ: а = 60°; Т2 = & Решение. Максимальный угол отклонения а зависит от скорости шарика v в нижней точке: при отклонении от положения равно- весия происходит переход кинетической энергии в потенциаль- n mv2 ную. Согласно закону сохранения энергии —— = mgh (здесь Л = & = 1(1 - cosa) , где I — длина нити, см. рис. а). Скорость v можно определить из второго закона Ньютона (см. задачу 8.2): Таким образом, cosa = a = 60°. & Рис. а Рис. 6
321 Механика Силу Тг также можно найти из второго закона Ньютона. Для этого не требуется даже знать величину полного ускорения шари- ка. Достаточно учесть, что в крайней точке отсутствует центрост- ремительное ускорение, поскольку в этой точке скорость шарика обращается в нуль. Следовательно, сумма проекций всех сил на радиус окружности должна равняться нулю. Отсюда (см. рис. б) _ mS Тг = mgcosa = & 8.4. Ответ: a = arccos = 71°; Т„,« = 3mg. О ,, „ „ „ „ mv2 Указание. См. задачу 8.3. В произвольный момент времени - гС = Т - mgcosa, —— = mgBcosa. Отсюда Т = 3mgcosa. & 8.5. Ответ: минимальная прочность нити 6mg, а стержня 5mg. Решение. Очевидно, наибольшая сила натяжения достигается в нижней точке: здесь, во-первых, центростремительное ускорение шарика направлено вертикально вверх, а во-вторых, скорость, а с ней и центростремительное ускорение, максимальны по величи- не. Из второго закона Ньютона следует, что Т = m[g + где v„ — скорость шарика в нижней точке, I — длина нити. Согласно mv,,2 2 о закону сохранения энергии —= mo, + 2mgl, где о» — скорость Ct шарика в верхней точке. Поскольку нас интересует минимальная возможная прочность нити, возникает вопрос: при каком мини- мальном значении ов возможно движение шарика по окружности? Это такое значение ов, при котором шарик «еще» движется по окружности, но в верхней точке «уже» не растягивает нить. Если в верхней точке сила натяжения нити равна нулю, то центро- v.2 е стремительное ускорение — шарику сообщает только сила тя- жести: mg = откуда v2 = gL Тогда v„2 = 5gZ и T = 6mg. При у» < в какой-то точке траектории нить свободно провиснет и шарик начнет двигаться по параболе. Итак, минимальная проч- ность нити должна в шесть раз превышать силу тяжести, дейст- вующую на шарик. Если нить заменить на невесомый стержень, то шарик не сможет «сойти» с окружности и изменить траек- торию, Поэтому минимально возможное значение ов практически равно нулю. Тогда и,2 = 4gl и Т = 5mg. Требования к прочности нити оказываются более высокими, чем к прочности стержня!
Ответы, указания, решения 322 8.6. Ответ: a = arctgV2 = 55°; и2 = 2gfcosa = Т = 'JSmg. а„ - Указание. Из рисунков а, б видно, что нормальная составляющая ускорения а„ = asina = gsinatga. Используйте также соотношение s' 8.7. Ответ: v„ = 32н) 220 М/Ч:‘ Решение. Пусть масса снаряда т, ско- рость его в верхней точке v. После раз- рыва один из осколков получает ско- рость -v. Скорость и другого осколка определяется из закона сохранения им- пульса: т т mv = --jv + у и. Отсюда и - Зг. Начальные скорости обоих осколков направле- ны горизонтально (см. рисунок), поэтому время t их полета I2PJ одинаково и равно Л/---; следовательно, s = (и + u)t = 4vt. Отсюда V g ____ г S g скорость снаряда в верхней точке v = .Л/пЕУ • Согласно закону 4 \ ЛЛ mvo2 rr mv2 сохранения энергии —л = tngH + —5—, откуда щ = a Z = 220 м/с. gs3 = 32Н о о (т,- т2)2 8.8. Ответ: уменьшить на----—— ' zni + тг Указание. См. задачу 2.5.
323 Механика 8.9. Ответ: 4л11ТПз + 7П17П2 - Зтапз тг(гП1 + ma) - 4mizn3 :--------------g’ q2 =--------------------g', 4mtma + mXmi + m3) 4miffis + m^nti + m3) 47П17П3 + ГПлШа - 3011012 a3 =-----------------------g. 4т,т3 + oiXoii + пи) Решение. Сделанные в условии оговорки позволяют считать силу Т натяжения нити постоянной по всей ее длине. Поэтому урав- нения движения грузов имеют вид mig - Т = oiiai, (1) mzg - 2Т = mzaz, (2) msg - Т = тзаз. (3) Здесь ai, az, аз — проекции векторов ускорений грузов на ось, направленную вертикально вниз. Из нерастяжимости нити следу- ет, что сумма длин всех четырех вертикальных отрезков нити остается при движении неизменной. Значит, в ait2 aat2 a.3t2 времени t сумма + 2—= 0, откуда ai Подставив в это уравнение выражения для ai, az, из уравнений (1) — (3), находим: imimzmag любой момент + 2az + аз = 0. аз, полученные 7* =-----------------—. 4/niwis + т^т, + тз) ' ' Подставив выражение (4) в уравнения (1) — (3), получим выражения для ускорений грузов, приведенные в ответе. 8.10. Ответ: ускорение стержня ai = gsin2a; клина az - gsinacosa. Решение. На рис. а, б указаны силы, действующие на стержень и клин. Спроецируем уравнения второго закона Ньютона на оси коордийат (силы, действующие на стержень, удобно проецировать на вертикальную ось; силы, действующие на клин, — на горизон- тальную). При этом учтем, что согласно третьему закону Ньютона Ni <= Ns = N. Получаем mat = mg - Ncos a; (1) Puc. a Puc. 6 11*
Ответы, указания, решения 324 Чтобы при движении не нарушался контакт между стержнем и клином, должно выполняться условие кинематической связи (см. рис. в, на котором штриховой линией показано конечное . 1. х положение клина): h = itga или = —z~ tga, где t — про- Z и Должительность опускания стержня. Итак, а\ = aatga. Тогда, выражая через аг силу N и подставляя это значение в уравнение (1), получаем maztga = mg - mazctga. Используя соотношение tga + ctga =---------, приходим к ответу: аг = gsinacosa, а sina • cosa значит, ai — gsin2a. I 23 8.11. Ответ: па тг выше гвоздя; на vyZ выше гвоздя. 3 а4 Решение. Шарик будет двигаться по окруж- ности до тех пор, пока натяжение нити не обратится в нуль. После этого момента шарик будет двигаться уже только под действием силы тяжести, то есть по параболе (как тело, брошенное под углом к горизонту). Найдем сначала, в какой точке исчезнет натяжение нити. Пусть при этом нить образует угол а с вертикалью (см. рисунок), а скорость шарика равна v. Мы можем воспользоваться законом сохранения энергии (за нулевой уровень принята высота точки В): mv2 mgZcosa _ mgl . ~2~+ 2 ~ 2 ‘ ( '
325 Механика Второе уравнение, связывающее v и а, является следствием вто- рого закона Ньютона (в проекции на направление нити): и2 gcosa = o3z- (2) 2 1 Из уравнений (1) и (2) получаем cosa = х, и2 = ~^gl. Следователь- €> о I I но, точка С находится выше точки В на величину ^cosa = к- При О дальнейшем движении по параболе шарик поднимется еще н.а , i/siira 1 //, 4\ 5 высоту h = = 2 • з(1 - э) = 54^- Значит, верхняя точка траектории находится выше точки В на I 23 величину Н = J + h = 8.12. Ответ: N = zng<3cosa - 2). Указание. Закон сохранения энергии позволяет определить ско- mv2 ч „ __ рость v тела: —т- = mgR(l - cosa). где R — радиус шара. Прое- Л цируя уравнение второго закона Ньютона на радиус шара, полу- чаем (ср. с задачей 8.4) —— = mgcosa - N. /С на высоте к 8.13. Ответ: Н = ^Я. о Решение. Согласно полученному в задаче 8.12 результату, сила давления тела на шар по мере соскальзывания убывает и обраща- 2 ТЛ ется в нуль при cosa = -х. Именно в этот момент тело отделяется О от шара и далее движется только под действием силы тяжести. В момент отрыва от поверхности шара тело находится 5 Н - 2?(1 + cosa) = xl?. О о 5Я(4<2 + V5) 8.14. Ответ:------------= 1,462?. Решение. Выберем систему координат, как показано на рисунке. Согласно ре- зультату, полученному при решении за- дачи 8.13, координаты точки А отрыва тела от шара: _ . Я>/5 5 Хо = Яэта = —п—9 у о = ^Я. О <5
Ответы, указания, решения 326 . После прохож- Скорость ио тела в точке А определим из закона сохранения лги»2 энергии: = mgR(i - cosa), откуда va = Л дения точки А тело движется по параболе под действием, силы тяжести. Его координаты изменяются по закону X = Ха + VoJ - у уа + ua,t 2 3В 3 \ з t 2 (мы учли, что и<>« = Pocosa и Uov = - Vosina). Приравнивая у нулю находим время t полета тела: 10 [r~Ia 1 \ t = з _ Vhp И ко°РДипатУ х точки падения тела: Л ( 8.15. Ответ: Н - 2,5 R. Указание. В верхней точке «мертвой петли» центростремительное и‘ R ускорение тела, равное не меньше, чем ускорение свободного гС падения. 8.16. Ответ: v = 'Igh. Решение. Воспользуемся законом сохранения энергии. Учтем при этом, что кинетическая энергия скатывающегося без проскальзы- вания обруча равна ти2 (см. задачу 4.51). Следовательно, ти2 = = mgh, откуда v = 4gh. Тело, соскальзывающее без трения с той же наклонной плоскости, приобретает скорость в V2 раз большую. Может возникнуть вопрос: какая сила «притормаживает» ка- тящийся обруч? Оказывается, та же самая сила, которая не дает обручу проскальзывать, — сила трения покоя. Благодаря дей- ствию этой силы уменьшается кинетическая энергия поступатель- ного движения и на столько же возрастает кинетическая энергия вращения обруча вокруг своего центра. s 8.17. Ответ: g’ = к = 4,9 м/с". £ Решение. Прежде всего необходимо найти правильное значение ускорения а обруча. Воспользуемся известными нам значениями начальной (fo = 0) и конечной (u = Tgh, см. задачу 8.16) скорости, а также известным соотношением для равноускоренного дви- жения 2а1 ~ V2 - fo2. Здесь I — длина наклонной плоскости. Подставляя в это соотношение значение и и учитывая, что h = feina,
327 Механика gsma _ , получаем а = —а—. Скатывается обруч вдвое медленнее, чем он соскальзывал бы без трения! Использованная Галилеем ошибочная g > формула а = g'sina приводит тогда к результату g' = ~ = 4,9 м/с". Л 8.18. Ответ: F = 5mg; F = 2mg. Указание. См. задачи 8.15 и 8.16. „ Л imtniig „ 3(/П12 + /Па2) - 2/П1/П2 8.19. Ответ: F =------; г =-------------------g. Ш1 + Ш1 ГП\ + ГП> Решение. Как ни странно, ответить на второй вопрос легче, поэтому с него и начнем. Сила давления на ось зависит от скорости v движения шариков в момент прохождения положения равно- весия. Эту скорость найдем из закона сохранения энергии (за нулевой уровень принимаем уровень точки О): I I nw‘ niiv2 -m,g^ + zn^ + -у + "-у- = О, , ,/П1 - т> тт „ откуда и = gl-------. Найдем центростремительное ускорение каж- т, + т> V1 „ /П1 - ГП г „ дого из грузов: а = -.— = 2g-----. Величина mifli + niiai равна f/2) 7И1 + т>. равнодействующей всех внешних сил. Таких сил здесь три: силы тяжести zmg, mig и сила реакции оси ЛГ. Итак, midi + тм> = mig + + тig + N. Проецируя это уравнение на вертикальную ось (см. Рис. а Рис. б рис. а), получаем: mia - mia = -mig -mig + N, откуда хт л / x (m> + "lO2 + 2(/И1 - m2)2 N - mi(g + a) + m/g - a) = 1------'----1----—-g - mi + mi _ ^3(/И!2 + mi) - 2/И1/П2 mi +. mi
Ответы, указания, решения 328 Согласно третьему закону Ньютона F = N. Для ответа на второй вопрос учтем следующее: — ускорения шариков равны по величине; они направлены вертикально (противоположно друг другу, см. рис. б), так как скорости шариков равны нулю и центростремительное ускорение отсутствует; — равнодействующая R всех приложенных к стержню сил и сумма М моментов этих сил равны нулю: ведь масса стержня пренебрежимо мала. При R * 0 стержень приобрел бы бесконечно большое ускорение, а при М * О — бесконечно большое угловое ускорение. Из условия М = 0 следует, что Pi = Ра; из условия 2? = О следует N = Pi + Рз (здесь Pi,2 — вес соответствующего груза). Равенство Pi = Рг выполняется независимо от масс mi и тз и обусловлено изменением веса при ускоренном движении (см. задачу 2.5): zni(g--a)=zn2(£+a). _ ПЪ - ГП > Отсюда а =--------g; следовательно, mi + Шл Znumig 4m,m2g Pi = Рг =-----и F = N =--------- zni + zn2 zni + zn2 На второй вопрос задачи можно получить ответ и иначе. Достаточно заметить, что неподвижный блок, рассмотренный в задаче 2.5, и стержень, рассматриваемый в данной задаче, представляют собой одинаковые равноплечие рычаги. Благода- ря этому можно просто воспользоваться результатом, получен- ным в задаче 2.5. 8.20. Ответ: и = - ^)> ? = 4zng^^2 Х~. Решение. Для нахождения и используем закон сохранения энер- X гии. Масса более длинной части веревки zny, ее центр тяжести Lt находится на у ниже блока и потенциальная энергия равна ---(нулевой уровень выбран на высоте блока). Потенциальная &Li энергия всей веревки _ mg£ mg(L - х)2 " 2L 2L L В начальный момент (при х = х ) потенциальная энергия Wop = ; : ’ А
329 Механика = - —।, а кинетическая энергия у веревки отсутствует. Согласно закону сохранения энергии в любой момент выполняется соотно- шение mgL mgx? mg(L - х)2 mv2 ~ 4 2L ~ 2L + 2 • Отсюда v = ^^(L2 + 4х2 - 4Lx) ~ j)- Чтобы определить силу, действующую на блок, необходимо найти ускорение веревки. Сила F равна удвоенной силе Г натя- жения веревки у блока. Обратите внимание, что вес частей ве- X ревки одинаков и равен Т = my(g - а) (ср. с задачей 2.5) незави- Ху симо от соотношения длин обеих частей. Это связано с изменением веса при ускоренном движении. X Итак, F = 2Т - 2m—(g - а). Ускорение а веревки можно найти, Ху X например, так же, как в решении задачи 2.5, считая пи = ту, Ху т> = т—у—. В данном случае можно поступить и иначе — вое- Ху пользоваться полученной формулой скорости. Из нее следует, что у/ Следовательно, F = 4mg-----—. При х = L получим F = 0 (в Ху этот момент веревка уже свободно падает и потому находится в состоянии невесомости). 8.21. Ответ: о = y'lgL; а = Т = Л Л 4 Указание. См. задачу 8.20. Если длина соскользнувшей со стола части шнура равна х, то Т = m-zr(g - а) = mg-~^—. Ху Ху Эти выражения справедливы при условии х < у. Случай х > рассмотрен в статье И.Гельфгата («Квант», 1993, №1/2, с. 55). 8.22. Решение. На санки действуют сила тяжести и сила треиия скольжения. Пусть к моменту остановки в точке В координаты санок равны х и у (см. рисунок). Тогда сила тяжести совершила
Ответы, указания, решения 330 над санками работу At = mgy, а сила трения — работу Аз = -p/ngx (см. задачу 4.27). Поскольку кинетическая энергия санок отсут- ствует и в точке Л, и в точке В, полная работа внешних сил равна нулю: А = Ai + Аз = О. Отсюда у = цх. Последний этап решения должен быть графическим: точка В определяется как первая (не считая точки А) точка пересечения профиля склона е прямой у = — ух. Разумеется, если крутизна склона в точке А меньше, чем у прямой у = цх, санки вообще не сдвинутся с места. К задаче 8.23 8.23. Ответ: ц < 0,1. Указание. См. рисунок. Максимальное из возможных значений ц определяется как угловой коэффициент проведенной к траек- тории движения касательной (см. задачу 8.22). 8.24. Ответ: v = ^2gH. Указание. При вытекании жидкости происходит превращение потенциаль- ной энергии только в кинетическую, поскольку из-за отсутствия трения ме- ханическая энергия не превращается во внутреннюю. Таким образом, мож- но считать, что верхний слой жид- кости массой т (см. рисунок) как бы «проваливается» сквозь весь сосуд и падает из отверстия внизу. Естественно, падая с высоты Н, «порция» жидкости приобретает скорость v = ^2gH. 8.25. Решение. Согласно формуле и = = ^l2gh (см. задачу 8.24) по мере пони- жения уровня жидкости скорость вы- текания уменьшается. Очевидно, для поддержания постоянной скорости по- нижения уровня сечение сосуда долж- но уменьшаться книзу (см. рисунок). Понижение уровня АЛ за время At определим из условия ЯАЛ = о • uAt, где Я — площадь
331 Механика сечения сосуда на уровне поверхности жидкости; о — площадь ЛА и _ ДА отверстия. Отсюда — = тщ. Поскольку — не должно зависеть от Д/ ь Л/ Л, получаем S ~ v ~ VA. Если горизонтальное сечение сосуда имеет форму круга радиуса г, то S ~ г* и, следовательно, г1 ~ V/г, т. е. г~4^. 8.26. Решение. Часто, не задумываясь, отвечают: «Рекорд увели- чится в шесть раз». Давайте разберемся, правилен ли такой ответ. Если отталкивание от пола па Земле и на Луне происходит одинаково, то тело спортсмена в обоих случаях приобретает одина- ковую кинетическую энергию. При подъеме эта энергия переходит в потенциальную: AWP = mg&h. Значит, ЛАЛ = 6АЛ3. Очень важно, однако, учесть, что здесь АЛ — изменение высоты центра тя- жести тела. А ведь тело спортсмена в прыжке движется не поступательно! Будем для простоты считать, что в момент толчка тело спортсмена занимает вертикальное положение, а в момент прохождения над планкой — горизонтальное. Оценим также (весьма грубо) начальную высоту центра тяжести как ho — 1 м. Поскольку земной рекорд по прыжкам в высоту — около 2 м 40 см, находим ЛА;, — 1,4 ми ДАл = 6АЛ, = 8,4 м. Тогда полная высота рекордного прыжка на Луне составит ho + ДАЛ = 9,4 м, что всего в четыре раза превышает земной рекорд. Правда, это тоже непло- хо: можно прыгнуть на балкон четвертого этажа! 8.27. Ответ: R < 3 км. Решение. Оценим сначала скорость v движения после оттал- кивания — она одинакова и на Земле и на малой планете. При прыжке на Земле увеличение АЛ высоты центра тяжести состав- ляет около 1 м (см. задачу 8.26). Таким образом, “ = mg&h, Л откуда и = ^2gAh ® 4,5 м/с. Чтобы человек мог улететь с малой планеты, вторая космическая скорость для этой планеты не долж- на превышать и, т. е. (см. задачу 2GM „ —=— < и, где М и R — it масса и радиус планеты. Согласно условию, плотность малой планеты _ М Мо „ и Земли одинакова, поэтому где Мо и Ro — масса й радиус £1 £1о 2?3 Земли. Таким образом, М = Jno R R ~ где £Со
Ответы, указания, решения 332 и» ~ 11 км/с — вторая космическая скорость для Земли. Следова- тельно, R < Ro— « 3 км. Oil Не такая уж маленькая эта планета: ее площадь может оказаться больше 100 км2, т. е. на ней мог бы разместиться целый город! 8.28. Ответ: ио =Л/----- ’ (1 + m/M)sm2a Решение. Если бы доска лежала на земле, то величину (Л> можно , , Т Oo2sin2a было бы наити из условия L =-----------. В данном же случае g следует учесть «отдачу». Доска приобретает скорость «отдачи» величиной и, которую можно определить из закона сохранения импульса (в проекции на горизонтальное направление): Ми — = nwocos а. Лягушка попадает на другой конец доски; следовательно, „ Oo2sin2a L =---------------------------+ ut, g где t = 2oosina ------— продолжительность прыжка. „.„J SL - V (1 + zn/M)sin2a Отсюда 8.29. Ответ: p = 440 кг/м3. Решение. Пусть палочка отклонилась от вертикали на небольшой угол Р (см. ри- сунок), причем под водой находится ее часть длины h, а полная длина палочки I. Тогда дальнейшее движение палочки зависит от соотношения вращающих мо- ментов двух сил (относительно точки О): момента Mi силы тяжести mg и момента М2 силы Архимеда Fa. Сила mg, прило- женная в середине палочки, имеет плечо ^sinp. Сила Fa, приложенная в середине погруженной части палочки, — плечо (<-|)sinp. ; Разумеется, Mi = М2 — 0 при р = 0, т. е. вертикальное положение стержня соответствует равновесию (правда, пока неяс- MT'Uk- 'Н',’. ? ’ ' S’ .• ‘ х . . -. >
333 Механика но — устойчивому или неустойчивому). А при р * О Mi = mg2 = pSlg2 = pZ2 м* Fa(Z-|) p»S/ig(z-^) P^2l~h> Здесь рв — плотность воды, S — площадь сечения палочки. Очевидно, при достаточно малых h из формулы (1) следует Mi > > Мг, т. е. палочка после малого отклонения будет возвращаться к вертикальному положению — равновесие устойчиво. Однако при . , . . М, р h ® I (палочка почти полностью погружена в воду) -гт- « -с~. Пос- ла 2 р„ кольку р < рв, то Mz > Mi и малое отклонение от вертикали будет нарастать. Устойчивое равновесие теперь, очевидно, достигается при некотором наклонном положении палочки. Это и требовалось доказать. Мы можем найти и то значение ho глубины погружения, при котором происходит потеря устойчивости вертикального рав- новесия. Устойчивость теряется при Mi = Мг, поэтому из урав- нения (1) следует pZ2 = p,h0(2l - ho). (2) Отсюда ho = /(1 - "£) (второй корень квадратного урав- нения превышает Z и должен быть отброшен). Условие равновесия палочки в наклонном положении (при Р * * 0) также имеет вид Mi — Мг. Отсюда снова следует уравнение (2). Значит, после отклонения палочки от вертикали при дальней- шем подъеме сосуда величина h остается неизменной (равной ho), h При этом отношение а = у определяется формулой а = 1 — - А /1 - , откуда р = арв(2 - а). При а = 4 получаем р = 440 кг/м®. V р, 4 8.30. Ответ: скорости всех трех грузов равны 4gi. Решение. Сама постановка вопроса подсказывает, что по исте- чении достаточно большого промежутка времени скорость средне- го груза перестанет расти, т. е. он будет двигаться без ускорения. В самом деле, в начальный момент этот груз опускается с уско- рением g, но затем его ускорение уменьшается по мере умень- шения угла а (см. рисунок). Когда угол а станет очень малым, ускорение практически станет равным нулю, а силы натяжения обеих частей нити — равными mg. Скорости всех трех грузов при этом будут почти одинаковыми. Обозначим величину скорости
Ответы, указания, решения 334 грузов через и, изменение высоты средне- го и боковых грузов — соответственно через Huh. Согласно закону сохранения энергии „ „ „ . „ти2 (2т)и2 2mgH = 2mgh + 2—^— + —=— & А (средний груз, опускаясь, теряет энергию 2mgH‘, за счет этой энергии растет по- тенциальная энергия крайних грузов и кинетическая энергия всех трех грузов). Из нерастяжимости нити (что, безусловно, подразумевается в условии задачи) следует, что по истечении достаточно большого промежутка времени Л + I = Н. Отсюда v = xgl. 2 Тб 8.31. Ответ: а„„» = arccos^ = 48°; ц > на расстоянии 1,12/. о Л Решение. Ответ на первый вопрос фактически уже получен ранее 2 2 (см. решение задачи 8.13 ): cosa„,»x = т. е. a„,.« = arccos^ = 48°. «5 о Необходимое значение ц можно определить из условия равен- ства нулю в любой момент равнодейстйующей R всех приложен- ных к стержню сил (ведь масса т стержня пренебрежимо мала, а R = та): N + F„> + Fy„„ = О, где N — сила нормальной реакции со стороны пола, FTp — сила трения, Fyup — сила упругости, действую- щая со стороны шара на стержень. Отсюда (см. рис. a) N = Fynpcosa, F Ftp = Fynpsina. Тогда ц > = tga. Стержень не проскальзывает, _ Tl - cos2a,„„ Тб если ц tganiHX — ~ о * COSan)HX. " С того момента, когда N обращается в нуль (при со8а1)1нх шар начинает двигаться как тело, брошенное под углом к гори-
335 Механика зонту, то есть с ускорением g. В этот момент координаты шара (см. рис. б) таковы: , . <5. . , 2 ХО = /SinCXmax = ЛО = ZCOSOCmax = О о Скорость шара в этот момент составляет ио = \2g(Z - Ло) = = \/-=-и образует с горизонтом угол amSx. V о Время t свободного падения определяется из условия Ло = = Uosina,,,.^ + Отсюда t = з^^С^33 - V5). Горизонтальное перемещение шара при свободном падении 8х = = UocosotmaxZ; следовательно, расстояние от точки опоры до точки падения АВ = Хо + 8л= ^(5<5 + 4^23) « 1,12 I. £ ( 8.32. Ответ: см. рис. б. Решение. В положении равновесия вращающий момент (отно- сительно оси О) действующей на брусок силы Архимеда должен равняться нулю. Значит, центр тяжести вытесненной воды (т. е. точка приложения силы Архимеда, см. задачу 8.29) находится в одной вертикальной плоскости с осью О. Это условие выполняется при двух положениях бруска (см. рис. а, б). В случае а центр Рис. а а тяжести вытесненной воды находится на глубине где а — aV2 а сторона квадрата, в случае б — на глубине —g— < — (см. задачу 5.48). Объем вытесненной воды в обоих случаях одинаков — он равен половине объема бруска. Устойчивому равновесию соответ- ствует минимальная потенциальная энергия системы брусок-вода. Поскольку потенциальная энергия бруска при повороте не изме- няется (его центр тяжести остается на уровне воды), устойчивому равновесию соответствует минимальная глубина центра тяжести
Ответы, указания, решения 336 вытесненной воды. Итак, положение устойчивого равновесия бруска показано на рис. б. Отметим, что именно в таком положении и будет плавать брусок, если его плотность равна половине плот- ности воды. , F 8.33. Ответ: а = —. т Решение. Движение диска не является поступательным. Однако для определения ускорения центра диска нет необходимости в анализе этого движения. Заметим, что действие силы F можно заменить действием четырех сил (см. рис. а), причем Fi — Fa = F F = Fa = 5 ; Fi - - 7; . Нетрудно видеть, что пара сил Fi и F< не придает центру диска ускорения (эта пара сил вызывает только ускоренное вращение диска относительно точки О). Силы же F2 и F3 (см. рис. б) придают диску поступательное ускоренное дви- жение. Их равнодействующая, равная F, приложена в точке О, F Следовательно, ускорение центра диска а = —. т Можно доказать, что справедлив и более общий результат: центр масс любой системы движется как материальная точка, в которой сосредоточена вся масса системы и к которой приложены все внешние силы, действующие на систему. 8.34. Ответ: против часовой стрелки; ц = ~ctga; Р = Решение. Прежде всего нужно понять: почему цилиндр не «отра- жается» под углом а. Очевидно, все дело в том, что он вращается. Из-за проскальзывания при соприкосновении цилиндра с поверх- ностью возникает сила трения скольжения Ftp, действующая на цилиндр горизонтально в сторону, противоположную вращению агсМ-?)-
337 Механика изменение сила реак- его нижней точки (см. рис. а). Эта сила вызывает горизонтальной составляющей импульса цилиндра, а ции поверхности N — изменение вертикальной составляющей импульса. Направление силы Frp показано на рис. а, что позволяет определить и направление вращения. Второй закон Ньютона мож- но записать в виде Д(тпРх) = -ApAt, Д(тпр.у) = N&t, где At — любой малый промежуток времени при соударении, в течение которого Гтр и N можно считать Достоянными. Поскольку сила трения скольжения равна yiN (согласно условию цилиндр вращается быстро, поэтому можно считать, что проскальзывание не прекратится), получаем Д(тпРх) = -цД(?пр,,). Поскольку это соот- ношение выполняется для любого малого Д£, оно выполняется и для столкновения в целом. Пусть ио — скорость цилиндра до удара, и — после удара, т — масса цилиндра. Тогда mvocosa = цт(ии - pov). Поскольку дефор- мация при столкновении является упругой, pv = -vou — Posina. 1 X Отсюда ц = gCtga. Если бы цилиндр вращался в противоположную сторону, то изменилось бы и направление Гтр. В результате цилиндр отлетел бы под углом р к поверхности, причем р < а (см. рис. б). Второй закон Ньютона в этом случае дает mvx - mvox = p(znpy - znuoy), или тписовр - тпросовос = p.(znpsinp + znposina). Из упругости деформации следует Posina = psinp. Исключая из последних двух уравнений и и ро, получаем sina(cosp - (jsinP) = sinp (psina + cosa), gP 1 + 2ptga 2'
Ответы, указания, решения 338 Этот ответ можно получить и сразу, если учесть, что во втором случае горизонтальная проекция Импульса возрастает на столько же, на сколько в первом случае убывает (т. е. на тиох). 8.35. Ответ: скорость спутника увеличивается, но это не проти- воречит закону сохранения энергии. Решение. При движении по круговой орбите сила притяжения к Земле придает спутнику центростремительное ускорение, поэтому то2 ~ тМ -р- = G - где т — масса спутника, и — его скорость, М — масса Земли, г — расстояние от спутника до центра Земли. Из л М* этой формулы следует, что и = ^JG —, т. е. с уменьшением высоты скорость спутника увеличивается! Неужели трение может приво- дить к увеличению кинетической энергии? Чтобы разобраться в этом, запишем выражение для полной механической энергии спутника, которая является суммой кинетической и потенциа- „ . . то2 „тМ льнои энергии (см. задачу 4.49): W = W\ + W], = ~~~ - G-= & г 1 _ тМ = - ----. Отсюда следует, что полная механическая энергия а г при уменьшении г уменьшается, чего и следовало ожидать из-за наличия трения. Тогда возникает вопрос: какая же сила разгоняет спутник,когда он переходит на более низкую орбиту? Снижаясь, спутник движется не по круговой орбите, а по спирали (как бы спускаясь по спиральной «наклонной плоскости»). А при таком движении сила тяготения образует уже не прямой, а острый угол с направлением скорости спутника; поэтому сила тяготения совер- шает положительную работу. Из формулы для полной механичес- кой энергии следует, что только половина «потерянной» потен- циальной энергии «расходуется» на увеличение кинетической энергии — вторая же половина превращается из-за трения во внутреннюю энергию: спутник и окружающий его слой атмосферы нагреваются. 8.36. Ответ: на 5 107 м/с2. Решение. Барон, как всегда, сказал правду: он обнаружил явле- ние, обусловленное так называемой приливной силой. На невра- щающейся планете он, очевидно, измерял ускорение свободного падения относительно ее центра О (см. рис. а). Решая задачу 3.1, мы установили, что ускорение самой точки О при движении планеты вокруг Солнца До = 6 • 10 ’ м/с2. На любое тело у поверх- ности планеты действует сила тяготения самой планеты и Солнца.
339 Механика Сила притяжения планеты придает всем телам одинаковое по величине ускорение ga, направленное к центру планеты. А вот сила притяжения Солнца сообщает телам разные ускорения в точках А, В и С, удаленных от него на различные расстояния: ал > ао, ас < ао. Величину ав можно считать равной ао. В системе отсчета, связанной с центром планеты, тело в точке А приобретает за счет притяжения Солнца дополнительное уско- рение (см. рис. б), направленное от центра планеты и составляю- щее |яа — ао| = ал - ао; тела в точке С также приобретают ускорение ас — ао, направленное от центра планеты. А вот в точке В тела не приобретают из-за тяготения Солнца дополнительного ускорения относительно центра планеты. Таким образом, даже на шарооб- разной планете, не вращающейся вокруг своей оси, ускорение свободного падения оказывается не всюду одинаковым. Нетрудно видеть, что в точке В достигается максимальное значение g, а в точке А или С — минимальное. Обозначим R расстояние от точки О до центра Солнца. Тогда М „ ... ^*22?-г) 2GMr 2г “А а° ^-^ВХИ-гУ* R' ~ Ra°’ ii 2r Jac - Oo| = ao- a< » -g-a» (мы учли, что г « R). К Таким образом, ускорение свободного падения в точках А и С одинаково. Максимальное и минимальное значения g отличаются 2л* друг от друга на -g-ao» 5 • 107 м/с2. Интересно, что уменьшение ускорения свободного падения (а, значит, и измеряемой величины силы тяжести) наблюдается как на дальней от Солнца стороне планеты, так и на ближней. На Земле приливные эффекты обус- ловлены тяготением Солнца и Луны. Из-за вращения Земли образующиеся приливные волны обходят земной шар примерно
Ответы, указания, решения 340 за сутки. Поскольку приливных «горбов» — два, приливы наблю- даются примерно через 12 часов. 8.37. Решение. Пусть жесткость проволоки/г, масса груза т. Тогда mg ’ висящии неподвижно груз растягивает проволоку на х = -т-. Зна- R чит, проволока приобрела потенциальную энергию AWt = ~х- = £i = —хг—, а груз потерял энергию mgx, т. е. NWz =--т—. Итак, К лишь половина потерянной грузом энергии перешла в потенциаль- ную энергию растянутой проволоки. Куда же девалась вторая половина? Ответ прост: эта энергия перешла во внутреннюю. В данном случае нельзя пренебрегать силами трения, как бы малы они ни были. При отпускании груза в системе возникают коле- бания: груз по инерции «проскакивает» положение равновесия, двигаясь вниз, затем то же происходит при движении вверх. В отсутствие трения эти колебания вообще никогда бы не пре- кратились! Получается, что половиной энергии lAWal система «расплачивается» за возможность перейти в состояние равно- весия. Уменьшение трения не влияет на величину этой «платы», оно приводит только к более медленному затуханию колебаний при переходе к равновесию. Этот процесс становится более нагляд- ным, если заменить проволоку пружиной или полоской резины. 9.1. Ответ: 3,9 1018. Решение. Прежде всего определим общее количество N молекул тт - /ял- ВО ды в стакане. Поскольку масса одной молекулы т0 = — (М — Ал молярная масса воды, Na — постоянная Авогадро), то N = — = ТПо т _ Мт - Az А’ Тогда из пропорции - - следует, что за время т испа- .. N-x mN At п .. ,л18 рялось количество молекул М = —-— =-----= 3,9 10 . t Mt 9.2. Ответ: 1,0 109. Решение. Общее количество ионов натрия в кристаллике соли т No = М\™, где М — молярная масса NaCl. За длительное время ионы натрия равномерно распределятся по всему объему озера, равному hS. Значит, концентрация ионов натрия в стакане и в N /яг озере одинакова: -у = {N — искомое количество ионов натрия
341 Молекулярная физика ' л „ V mVNA в стакане). Отсюда N = ./Vo^-g = MhS = 1,0 10’. 9.3. Ответ: р = —. г а „ _ mi Решение. Плотность кристалла р = где mi — масса, приходя- щаяся на одну кубическую ячейку. Для определения mi можно представить каждый атом как маленький шарик. Каждой из восьми «сходящихся» в центре атома кубических ячеек принад- 1 т° тт , лежит зг этого шарика, т. е. масса ~б~. Поскольку у кубической О о ячейки восемь вершин, получаем т, = 8 • = т0, т. е. на каждую О кубическую ячейку кристалла приходится по одному атому. Итак, р = —з - Интересно, что такая простая кристаллическая решетка в природе не встречается. 9.4. Ответ: а = 2,8 10 10 м. Указание. См. задачу 9.3. В случае NaCl на одну кубическую ячейку объемом а3 приходится «пол-иона» натрия и «пол-иона» хлора, т. е. масса, равная полусумме масс ионов Na+ и СГ. 4М 9.5. Ответ: р = —3 (NA — постоянная Авогадро). Указание. См. задачу 9.3. В данном случае на каждую кубическую ячейку приходится по четыре иона. 9.6. Ответ: давление увеличилось бы. Решение. В газах среднее расстояние между молекулами превы- шает размеры молекул. При этом преобладает притяжение моле- кул (отталкивание проявляется фактически лишь при столкно- вениях). Как влияет взаимное притяжение молекул на величину давления? Давление газа на поверхность твердого тела обусловле- но столкновениями молекул газа с этой поверхностью. Оно воз- растает с увеличением средней скорости подлетающих к поверх- ности молекул. На подлетающую к поверхности молекулу силы притяжения со стороны других молекул газа действуют в основ- ном «сзади», приведя к уменьшению ее скорости. Значит, исчез- новение притяжения приведет к увеличению скорости молекул и, следовательно, давление на поверхность увеличится, 9.7. Ответ: давление не изменится. Решение. Часто приходится слышать ответ: «Поскольку масса газа в сосуде после сжигания угля увеличилась, то дЬл'Жно йоз-
Ответы, указания, решения 342 расти и давление». Однако согласно уравнению молекулярно- NkT кинетической теории р = nkT = у- (N — количество молекул газа, п — их концентрация, k — постоянная Больцмана), давление газа зависит не от массы газа, а от количества его молекул. Уравнение реакции горения С + О2 СО2, т. е. одну вступившую в реакцию молекулу кислорода заменяет одна образовавшаяся молекула углекислого газа. Полное количество молекул газа не изменяется; следовательно, конечное давление в сосуде совпадает с начальным. (Мы учли, что объем кусочка- угля пренебрежимо мал по сравнению с объемом сосуда.) 2 9.8. Ответ: -„ро. О Решение. Уравнение реакции 2Н2 + О2 > 2НгО показывает, что в данном случае кислород и водород прореагируют полностью, без остатка. При этом из двух молей водорода и одного моля кисло- рода получаются два моля водяных паров, т. е. количество моле- кул газа уменьшается в 1,5 раза. Значит (см. решение задачи 9.7), 2 в сосуде установится давление ~^ра. О 9.9. Ответ: р — 176 кПа. mi Решение. В смеси находилось Vi = = 0,25 моль водорода и Az 1 /Иг v2 = = 0,25 моль кислорода. В реакцию 2Н2 + О2 -+ 2Н2О вступит Az 2 весь водород и половина кислорода. В результате реакции образу- ется Va = 0,25 моль водяного пара и остается v< = 0,125 моль кислорода. Если объем баллона V, а первоначальная температура в нем Г, то piV = (vi + vz)RT и pV = (va + vt)RT. Отсюда p = = 1,76 10s Па. Vi + v2 9.10. Ответ: средние кинетические энергии одинаковы и равны 6,2 • 1(Г21 Дж; средняя квадратичная скорость молекул кислорода равна 480 м/с, водорода — 1900 м/с. Указание. Воспользуйтесь соотношениями -= 3 _ пил? М Йз них следует, что Е вообще не зависит от характеристик самих молекул; средняя квадратичная скорость выше у более легких молекул. ,
343 Молекулярная физика 9.11. Ответ: р = 140 МПа. Решение. В результате исчезновения сил взаимодействия между молекулами вода превратилась бы в идеальный газ с той же плотностью р = 1000 кг/м3. Используя уравнение Менделеева-Кла- „ тг т п— ,г RT нейрона pV = и соотношение m = pV, получаем р = р = = 1,4 • 10“ Па = 1400 атм. Здесь Т = 300 К, М — молярная масса воды, R — универсальная газовая постоянная. Огромная величина полученного давления показывает, сколь велики силы молекуляр- ного «сцепления». 9.12. Решение. Сначала жидкость вытечет из трубки, после чего давление во всей трубке будет равно атмосферному. Значит, и давление В жидкости на уровне нижнего конца трубки тоже будет равно атмосферному. Следовательно, жидкость из нижнего отвер- стия будет вытекать под действием избыточного давления pt = — pghi, где р — плотность жидкости. Это давление, конечно, остается неизменным, чем и объясняется постоянство скорости вытекания жидкости (по мере вытекания жидкости в сосуд через трубку будут проскакивать пузырьки наружного воздуха). Дав- ление воздуха в сосуде над жидкостью меньше атмосферного. Поэто- му, если открыть отверстие 2, через него в сосуд устремится наружный воздух. Теперь атмосферное давление установится на уровне от- верстия 2, и жидкость заполнит трубку до этого же уровня. В результате вытекание жидкости из нижнего отверстия ускорится: теперь избыточное давление рч = pgh-t > pi. Открывая же малое отверстие 1, мы ничего не изменим: давление в жидкости на этом уровне при наличии трубки и так равно атмосферному. Заметим, что «сосуд Мариотта» представляет собой идеальные водяные часы. 9.13. Ответ: ро = 3,7 кПа. Решение. Воздух в трубке испытывает изотермическое сжатие от m начального объема V до объема V-----, где р — плотность воды. Р Конечное давление в трубке равно р» + pgh. Согласно закону Бойля- Мариотта p«V = (р. + ряЛ)(^ - — )• Отсюда ра = (р, + р§Л)(1 - - == 3700 Па. Как видим, ро '< ря. Этого и следовало ожидать, поскольку вода заняла почти весь объем трубки. 9.14. Ответ: х — 25 см. Решение. Когда трубку вынимают из ртути, воздух в ней испы- тывает изотермическое расширение от объема ZS/2 до объема (Z - x)S, где S — площадь сечения трубки. Начальное давление в
Ответы, указания, решения 344 трубке равно атмосферному, т. е. pgH (р — плотность ртути), а конечное — меньше атмосферного на величину pgx. Запишем для воздуха в трубке уравнение закона Бойля-Мариотта: IS pgH -у = (pgH - pgx)(l - x)S. Hl Решая квадратное уравнение х2 - (Н + l)x + — - 0, получаем ы Н + I 1 ггг;---7; х = —5— - -хуН + I = 0,25 м 4 Л (второй корень уравнения не имеет смысла — он превышает Z). Обратим внимание, что нам не пришлось выражать давление в паскалях, поскольку оно измерялось высотой столба той же жид- кости (ртути), которая рассматривалась в задаче. , /(Я + а - О2 ~~ Н + а-1 9.15. Ответ: h ='\|--------+ HI------------= 48 см. Указание. Воспользуйтесь рисунком и уравнением Бойля-Мариот- та (см. задачу 9.14). 9.16. Ответ: й2 = |(>/Я2 + 4Hh, - Н) = 23 см. 9.17. Ответ: р = 560 мм рт. ст., уровень ртути понизится на 10 см. Решение. К моменту прекращения вытекания ртути давление у обоих концов сифона становится одинаковым (равным атмосфер- ному). Пусть уровень ртути понизится на величину х. У левого конца сифона давление складывается из давления воздуха р и , „ Й _ давления столба ртути высотой — - х. Если давление р измерять высотой столба ртути, получаем: ТТ Л Н = р + --х. (1) Воздух в сосуде при вытекании ртути испытывает изотермичес- - /й _ кое расширение от объема до объема + хр, где S — площадь, ^ечшвзля.' сосуда. Этот процесс описывается уравнением закона
345 Молекулярная физика Бойля-Мариотта: H^S = + x)s. (2) Определив р из уравнения (1) и подставив его в уравнение (2), А2 получим: х2 + хН - -г = 0. Отсюда х = 10 см, р = 560 мм рт. ст. 9.18. Ответ: ро — 50 кПа. Решение. Вначале объем воздуха в каждой из половинок трубки L - h составлял —„— • S. Если трубку поставить вертикально, воздух в верхней части испытает изотермическое расширение до объема iL - h ч I—-— + I IS и давления рг, воздух же в нижнеи части испытает (L - h \ изотермическое сжатие до объема —%— - l|S и давления р2. Согласно закону Бойля-Мариотта ' L-A_, /L - h \ <L - h \ р« g S - /Ц + tyS - Р\ g Условие равновесия столбика ртути имеет вид р2 - pi = pgh, где р — плотность ртути. Отсюда получаем (L - й)2 - 4Z2 _ р° = —iT- = 50 кПа- 4l(L - й) 9.19. Ответ: кислород занимает часть сосуда длиной 2,9 см. Решение. В равновесии давление р и температура Т (а также, по условию, и масса т) обоих газов должны быть одинаковыми. Запишем для каждого из газов уравнение Менделеева-Клапейрона: _ //I I II I * _ /Я _l_.ll рК = -^-RT, pV. = -^-RT. Мк м, Отсюда отношение объемов газов равно обратному отношению их К Мк „ молярных масс: Если обозначить через х длину части У к Л2 в Vu I — х сосуда, занятой кислородом, то ~ =--. Отсюда У к X х ~ I——— = 2,9 см. М. + М к О ОЛ PlVl -I- P2V2 9.20. Ответ: р = --. V1 + Г2 Решение 1. Можно считать, что газ из первого баллона изо- термически расширяется до объема Vi + V2. Тогда его давление
Ответы, указания, решения 346 после расширения согласно закону Бойля-Мариотта Р* - Р'Т, 77• Vi + Vz Аналогично рг = Рг- Согласно закону Дальтона полное давление в баллонах равно сумме парциальных давлений обоих газов: , , PiVi + pTVz p = Pl + Pz = --------• И И y V- + Vz Решение 2. Воспользуемся уравнением Менделеева-Клапейрона: pTVi = ViRT, pzVz = vzRT, p(Vi + Vz) = vRT. Здесь Vi и V2 — количество вещества соответственно в первом и втором баллонах, v — общее количество вещества. Поскольку v = Vi + V2, получаем: piVi +P2V2 = p(Vi + Vi), что приводит нас к тому же ответу. 9.21. Ответ: р* = р«(1 + ДГ^~). Решение. Фактически речь идет об изотермическом сжатии пор- ции воздуха, которая при атмосферном давлении занимала объем V + WAV, до объема V. Согласно закону Бойля-Мариотта pTV = = p»(V + NAV). Отсюда р* = р„(1 + N С каждым качанием W компрессора давление возрастает на одну и ту же величину р» -у-. 9.22. Ответ: р* = р.(———)N. Решение. Обозначим давление в сосуде перед k-м качанием р* 1. Очередное качание вызывает изотермическое расширение воздуха от объема V до объема V + AV. Поэтому р* i • V = pk(V + AV), откуда V Pk — Ph 1 ---. И И V + A V Итак, при откачке газа постоянной величиной является не раз- ность давлений до и после каждого качания (ср. с задачей 9.21), а отношение этих давлений. Другими словами, теперь значения давления образуют не арифметическую, а геометрическую прог- рессию. Это происходит потому, что количество захватываемого насосом воздуха с каждым качанием уменьшается. После X качаний
347 Молекулярная физика 9.23. Ответ: в 1,2 раза. Решение. Обозначим давление в нагретых шинах pt. Тогда соглас- _ Т но закону Шарля — = Здесь Ti и Тз — абсолютные температу- pi 11 ры, соответствующие ti и tz. Заметим, что сила давления шин на дорогу в обоих случаях одна и та же: она равна весу автомобиля (при этом надо учесть, что речь идет об избыточном давлении, т. е. о разности между давлением в шинах и атмосферным): (pi - p«)Si = = (ра - pa)St. Из двух выписанных уравнений получаем: S1 _ р2 - р. _ Р1Т2 - рЛ\ _ 1 2 Sa р, - р. Ti(pi - р„) 9.24. Ответ: о = 1,1 м/с. Решение. За время т при скорости и через любое поперечное сечение газопровода проходит объем газа V = vxS. Этот объем связан с массой газа уравнением Менделеева-Клапейрона pV=t~RT. М _ mRT . . , Следовательно, и =----= 1,1 м/с. MpxS 9.25. Ответ: см. рис. а и б. Решение. Основное затруднение в данной задаче связано с необ- ходимостью одновременно прослеживать характер изменений трех величин (р, V, Т). Советуем сначала составить краткую сводку изменений этих трех величин на каждом из этапов (значок ~ означает прямую пропорциональную зависимость). Этап 1-2: Т — растет, V — растет (~ Т), р = const. Этап 2-3: Т = const, V — убывает, р — растет (^ 1/V)- Этап 3-1: Т — убывает, V = const, р — убывает (~ Т). Рис. а Рис. б
Ответы, указания, решения 348 Характер изменения Т и V виден прямо из графика; характер же изменения р можно определить из уравнения Клапейрона (или воспользоваться тем, что каждый отрезок на графике изображает хорошо известный изопроцесс). Теперь нетрудно построить необходимые графики (см. рис. а и б). 9.26. Ответ: см. рис. а и б. Указание. См. задачу 9.25. 9.27. Ответ: см. рис. а и б. Указание. См. задачу 9.25. 9.28. Ответ: Vz > Уь Решение. Переход из состояния 1 в состояние 2 можно осу- ществить с помощью изопроцессов (заданный в условии процесс к ним не относится). Один из вариантов показан на рисунке. Мы видим, что на участке 1-3 объем газа растет (происходит изобарное нагревание), а на участке 3-2 объем не меняется. Следовательно, Vi > Vi.
349 Молекулярная физика Такой же ответ можно получить, если провести через точки 1 и 2 соответствующие им изохоры а и Ь. Остается вспомнить, что большему объему соответствует нижняя изохора. 9.29. Решение. Проведем через различные точки графика соответ- ствующие им изобары (см. рисунок). Наибольшему давлению соответствует самая нижняя изобара, поэтому максимальное дав- ление соответствует точке касания В. Аналогично минимальное давление соответствует верхней точке касания А. Таким образом, на участке графика АСВ давление газа возрастает, на участке BDA — убывает. 9.30. Ответ: наибольшая температура до- стигается в точке А (см. рисунок) Указание. Проведите в координатах p-V семейство изотерм (гипербол). Найдите среди них изотерму, касающуюся гра- фика в точке А (см. рисунок). 9.31. Ответ: от точки 2 к точке 1. Решение. При отсутствии утечки газа график (изобара) представлял бы собой отрезок прямой, проходящей через на- чало координат. Проведем изобары че- рез концы отрезка на графике (см. ри- сунок). Посмотрим, чем определяется угловой коэффициент (а, значит, и на- клон) изобары. Из уравнения Менделее- ва-Клапейрона получаем: V mR Т ~ Мр’ Таким образом, лежащая выше изо- бара соответствует большей массе газа, поэтому лежащая на ней точка графика представляет начальное состояние. На- правление процесса указано на рисунке.
Ответы, указания, решения 350 9.32. Ответ: Тг - ^Т1Тя. Решение. Воспользуемся уравнением Клапейрона, позволяющим связать между собой параметры всех четырех состояний газа* PiVi _ ргУг _ рУг _ p,Vl Tt ~ Тг ~~Тг “ Т, • Согласно графику рг = рг, pi = pi, Уг = Vi, Vi = Уз. Учитывая, что Ti = Тг, запишем систему уравнений: Р1У1 _ рзУ1 рзУз _ Р1Ул Tt ~ Тг’ То ~ Тг' Перемножая почленно эти уравнения, получаем TiTs = Тг2. Следовательно, Тг - ^TiT.i. 9.33. Ответ: а) охлаждается; б) нагревается; в) нагревается. Решение. Чтобы найти связь между V и Т, надо в заданном соотношении выразить р через V и Т, пользуясь уравнением Т Клапейрона: р - const • р. Таким образом, получаем: a) TV = const; при расширении газ охлаждается; Т б) у =. const, при расширении газ нагревается; Т в) - const, при расширении газ нагревается. Отметим, что только в случае б мы имеем дело с изопроцессом. В случаях а и в в ходе процессов изменяются все три макрос- копических параметра (р, V и Т). 9.34. Ответ: Т,„«х = (v = 1 моль). 4vjR Решение. Согласно уравнению Менделеева-Клапейрона pV = vRT (v = 1 моль). Значит, следует отыскать наибольшее значение произведения pV в ходе процесса. Уравнение, описывающее про- Р V цесс 1-2, имеет вид — + ту- = 3 (см. математическое приложение}. Ро V о Если сумма двух положительных величин постоянна, их произве- дение максимально, когда эти величины равны друг другу (см. математическое приложение). Следовательно, pV максимально р V 3 ЭроУо при — = , что соответствует температуре Т„,.х =--. ро Vo 4vjR 9.35. Ответ: 1,0 кг. Решение. Давление газа в аэростате практически не отличается от давления воздуха снаружи (тонкая оболочка не может выдержать сколько-нибудь значительного перепада давлений). При подъеме газ внутри аэростата расширяется, заполняет всю оболочку и начинает выходить через клапан (ведь оболочка практически
351 Молекулярная физика нерастяжима, а для дальнейшего уменьшения давления газ дол- жен продолжать расширяться). Уравнение Менделеева-Клапейрона позволяет найти массу во- дорода в оболочке в начале (mi) и в конце (mz) подъема: Р1аГ=^ЛТ„р2И=^ВТ2, где М — молярная масса водорода. Отсюда определяем массу потерянного водорода: MVia.pi р2\ Am = mi - т2 = - ъгI = 1>° кг- Л ’ 1 1 12' 9.36. Ответ: увеличится на 8% . Решение. Напомним молярные массы газов: Mt = 0,004 кг/моль (гелий), Мг = 0,002 кг/моль (водород), Мв = 0,029 кг/моль (воздух). Водород вдвое легче гелия. Однако подъемная сила F возрастает отнюдь не вдвое! Она определяется разностью между архимедовой силой Fa, действующей на шар со стороны воздуха, и силой тяжести mrg наполняющего шар газа: F - Fa - mrg = (тв - mr)g. Здесь тв — масса воздуха в объеме шара (т. е. «вытесненного» воздуха). Значит, отношение подъемных сил в случае заполнения шара водородом и гелием равно: F2 _ тв - гп.г Fi тв - mi Согласно уравнению Менделеева-Клапейрона PaV = ~RT - ~RT = ~RT. r Mb Mi Mz Здесь V — объем шара, рл — атмосферное давление. Мы считаем температуру Т газа в шаре равной температуре наружного возду- ха, а давление — равным атмосферному (см. решение задачи 9.35). „ Mi М2 Отсюда получаем mi = Г7~тв, т2 = ^гт.в. Мв Мв Fz Мв — Mz . Следовательно, -=- =---------- 1,08. -ci Мв - Ml Итак, подъемная сила возрастет всего на 8%. Дело в том, что она определяется разностью между плотностью газа в шаре и плотностью окружающего воздуха. Оба газа — гелий и водород — намного легче воздуха, поэтому существенного увеличения подъ- емной силы при замене гелия водородом не происходит. Водород рейчас в воздухоплавании практически не используется, его заме- няют гелием, что полностью исключает возможность пожара. :
Ответы, указания, решения 352 9.37. Ответ: опустится на 19 см. Решение. Запишем условия равновесия поршня в начальном и конечном положениях: /nig + p»S — piS, mig + mig +p«S = piS. Здесь pi и pi — начальное и конечное давление воздуха в цилиндре. Эти величины связаны между собой уравнением Кла- пейрона: Здесь Vi = Shi и Vi = S(hi + х) — соответст- /1 12 венно начальный и конечный объем воздуха в цилиндре, х — перемещение поршня (х > 0 соответствует поднятию поршня, х < О — опусканию). Отсюда mig + p,S 1 -----------------1 = -19 см. /nig + msg + p.S’ х = Ц-1 + уг ' Таким образом, поршень опустится на 19 см. 9.38. Ответ: 8,05 1025. Решение. Количество N молекул воздуха в комнате можно полу- N чить из соотношения р = nkT, где п = -у — концентрация моле- PaV кул, k — постоянная Больцмана. Отсюда N = ~г= иМ - Na = К 1 = N, - Ns = - ~) = 8,05-1025. К / 1 12 9.39. Ответ: сосуд с влажным воздухом. Решение. В равных объемах газов при одинаковых температурах и давлениях содержится одинаковое число молекул (это следует, например, из соотношения р = nkT). Значит, во влажном воздухе молекулы воды просто заменили такое же количество молекул воздуха. Однако средняя молярная масса воздуха — 0,029 кг/моль, а молярная масса воды — лишь 0,018 кг/моль. Поскольку моле- кулы воды легче, то и сосуд с влажным воздухом тоже легче. Часто неправильно отвечают, что сырой воздух тяжелее сухого: к весу сухого воздуха добавляют вес водяного пара. На самом же деле водяной пар вытесняет из данного объема часть воздуха, масса которого больше массы пара. 9.40. Ответ: р = 190 кПа. Решение. До диссоциации количество молекул азота в сосуде было Ni = ^Na, где Na — число Авогадро, а М = 0,028 кг/мрль — молярная масса молекулярного азота. Диссоциируют Na = aNi (а = — 0,3) молекул. Каждая из них диссоциирует на два атома, тем i
353 Молекулярная физика самым общее количество молекул стало JV2 = М(1 - а) + 2aNi = М(1 + а). Давление в сосуде определяется теперь из соотношения р = nkT - _ z< pnkNtT pnRT , „ ,,A5TT Отсюда p = (1 + a) = U + o)'д/у = 1 ’9 10 Па- 9.41. Ответ: масса азота в 7 раз больше. Решение. При нагревании смеси давление растет не только за счет увеличения скоростей молекул, но и за счет увеличения их концентрации. Обозначим через Пл и п.г концентрации атомов азота и водорода после полной диссоциации, а через р — давление смеси при температуре Т. Полная концентрация частиц при температуре 2Т равна /ц + щ, а при температуре Т она составляла Hi + (так как атомы водорода были объединены в молекулы Нг). ^4 (п 2\ ni + Зр = (ль + п2)Л 2Т. Отсюда пг = 2пь Масса газа в сосуде объемом V”выражается через концентрацию ,М t 'о молекул: m = nVm> - nV^~, поэтому отношение масс азота и водорода — = где Mi = 0,014 кг/моль и М> = 0,001 кг/моль m.2 ПъМ.2. — молярные массы соответственно азота и водорода в атомарном т_ mi nMi Mi „ состоянии. Итак, — = —гт - = 7. ГП2 ПгМг 2Мг 9.42. Ответ: р = ро + pgh - 0,20 МПа (здесь р — плотность воды). Решение. Согласно условию, при всплытии объем пузырька не может измениться — ведь это привело бы к изменению объема воды или общего объема бака. Температура в системе, очевидно, тоже неизменна. Но тогда и давление в пузырьке не может измениться! Оно останется равным ро. Значит, таким же станет давление у верхней крышки бака. А давление на дно бака станет р = ро + pgh = 0,20 МПа. Напомним, однако, что «несжимаемая жидкость» — это идеализация, модель реальной жидкости, а всякая модель имеет ограниченную область применимости. В данном случае, если бак достаточно велик, изменение объема воды окажется вполне сравнимым с объемом пузырька или даже большим (см. задачу 9.43). Тогда, конечно, считать воду несжимаемой нельзя. 9.43. Ответ: S < 1 см2. Решение. Сжимаемость воды приводит к расширению пузырька При всплытии. Поэтому давление в нем несколько уменьшается,
Ответы, указания, решения 354 так что увеличение давления в жидкости Др оказывается мень- шим, чем для несжимаемой жидкости: Др < pgh (см. задачу 9.42). Общее уменьшение объема воды (т. е. увеличение объема пузырь- ка) составит ДУ = рй«Др < (рЛ-S) • (pgh). Воду можно считать несжимаемой при ДУ « Уо. Для этого достаточно выполнения условия (РЛ5) • (pgh) « Уо, или S « Уо - « —— ® 1 СМ . pgfih Итак, чтобы воду в баке можно было считать несжимаемой, бак должен представлять собой очень узкую высокую трубку. 9.44. Ответ: 0,20 МПа; 0,18 МПа. Решение. Легче ответить сразу на второй вопрос. При всплытии обоих пузырьков возникает ситуация, описанная в задаче 9.42. При этом давление на дно станет р = ро + pgh = 0,20 МПа. Что же изменится, если всплывет один из двух пузырьков? Очевидно, теперь нельзя утверждать, что объемы пузырьков неиз- менны: неизменным должен оставаться только их суммарный объем. Всплывший пузырек находится под давлением р - pgh, оставшийся у дна — под давлением р; поэтому верхний пузырек должен расшириться до объема Vi = Уо ———, а нижний — Р - pgh сжаться до объема У> = У> — (здесь Уо — начальный объем каж- Р дого пузырька). Из условия У1 + У> = 2Уо получаем 2 , . ч popgh pgh+р0 ± у[р?~+(pgh? Р - p(pgh + р0) + = 0 и р = --- (знак перед корнем выбирается с помощью простого соображения: при h -> 0 должно быть р -> ро). Подстановка численных значений дает р = 0,18 МПа. Границы применимости такого решения проанализированы в решениях задач 9.42 и 9.43. 9.45. Ответ: г > 0,15 м. Указание. Условие плавания шара в воздухе состоит в том, что сила Архимеда, равная весу rn.Bg вытесненного воздуха, уравно- вешивает силу тяжести оболочки mg и наполняющего шар гелия mrg. Массы воздуха и гелия в объеме шара можно выразить через г, воспользовавшись уравнением Менделеева-Клапейрона; массу _ т S „ оболочки — из соотношения — = где S — площадь поверх- ТПй Ъо ности оболочки, a So = 1 м2.
355 Молекулярная физика Решение. На первый взгляд может показаться, что pi и рг одина- ковы, потому что части баллона сообщаются, а равенство давлений является необходимым условием теплового равновесия. Но ведь в данном случае теплового равновесия нет — наоборот, в разных частях баллона поддерживаются различные температуры! Чтобы состояние системы не изменялось, число молекул газа в каждой из частей баллона также не должно меняться. А для этого должны быть одинаковы количества Zi и Zz молекул, пролетающих через отверстие в одну и другую сторону за одно и то же время At. Величину Z можно записать в виде Z = in|px|SAt, где п — концен- трация молекул, S — площадь отверстия; ось х направлена нормально к плоскости Р П - НИЯМИ п = И Ш ~ V kT отверстия. Воспользовавшись соотноше- [ВТ р , получаем Z ~ (мы не выпи- сываем коэффициенты, одинаковые для Zi и Zz и потому не влияющие на ответ). Из условия Zi = Zz получаем = ~р==, откуда Интересно, что отношение концентраций моле- кул оказывается при этом обратным отношению давлений: П) _ Рг П л ~ Pl т. е. давление больше в той части баллона, где концентрация молекул меньше! 9.47. Ответ: т = Указание. При малых смещениях х поршня от положения равно- весия (когда х « I) уравнение та = F = -SAp (где S — площадь поршня, Др — разница давлений газа по обе стороны от поршня) приводится к виду а = -®о2х. А это — стандартное уравнение гармонических колебаний. 10.1. Ответ: фа = 29%. Решение. Согласно определению относительной влажности ф2 - р2 = — • 100%, где р2 — парциальное давление водяного пара при Р«2 температуре ta, а р„2 — давление насыщенного пара при той же температуре (см. таблицу в приложении). Водяной пар с хорошей 12*
Ответы, указания, решения 356 точностью описывается уравнением состояния идеального газа, -- 4. Р* ' ТТ поэтому при V = const получаем — = =-, т. е. рг = рг^-. Парциаль- pi 11 -<1 ное давление pi пара при температуре ti можно найти из соотно- шения ф1 = • 100%. Подставляя полученные выражения в фор- рл мулу для фг, получаем <р2 = = 29%. Задачу можно решить проще, если воспользоваться таблицей зависимости плотности р. насыщенного пара от температуры и формулой1’ ф = -2- • 100%, где р — плотность пара. Поскольку Р= плотность пара при изохорном нагревании не изменяется, ф1 р.2 откуда ф2 = - 29%. Рн2 10.2. Ответ: воздух надо увлажнить, испарив 22 кг воды. Решение. Масса водяного пара в помещении mi = piV, в заби- раемом снаружи воздухе /Пз = рзУг (здесь pi и рз — плотность УТг соответствующего водяного пара, V2 = — объем забираемого 1 1 снаружи воздуха). Таким образом, массу' пара в забираемом воз- Тг духе следует изменить на Лиг = mi - тп2 = V(pi - р2ут). Если Лиг > > 0, то воздух надо увлажнять; если Ат <0 — осушать. Плотность водяного пара легко выразить через относительную влажность и плотность р/ насыщенного пара при той же температуре: _ ф_____ р ’ рв1б0% • — Отсюда Am = |qq0/0 (Р»1ф‘ - Р»гфг у-)* Подставляя табличные зна- чения p.i и р.г, получаем Лиг = 22 кг, т. е. воздух надо увлажнить, испарив в него дополнительно 22 кг воды. 10.3. Ответ: фз = 63%. Решение. Параметры водяного пара приближенно удовлетворяют уравнению состояния идеального газа. Начальное парциальное *’ Эту формулу легко вывести из определения относительной влажности, если воспользоваться уравнением состояния идеального газа.
357 Молекулярная физика ф1 давление пара pi = где Рн — давление насыщенного пара при 30°С. Из уравнения Менделеева-Клапейрона находим началь- МрУ RT ную массу пара: mt = (здесь М — молярная масса воды, R — универсальная газовая постоянная). Предположим, что вся добавленная вода испарилась, тогда масса пара составит m-i = mi + rrtiRT niRT + m, а его давление р2 = ~ Р> + Относительная влаж- ность ф2 = 100% = ф! + 100% = 63% . рн MpHV Получилось фг < 100%; следовательно, вся внесенная вода действительно испарилась. Если бы мы получили фа > 100%, это означало бы, что вода испарилась не полностью. Правильный ответ в таком случае был бы фг = 100% (пар стал насыщенным). Эту задачу можно решить и короче, используя табличное значение плотности рн насыщенного пара при 30°С: фг = — 100% ; (р2 = 100% = фх + --- 100% = 63%. pHF pHV p„V 10.4. Ответ: тп = 0,59 г. „ т Решение. Вода занимает объем %>=-—, где о — плотность воды. Р Тогда пар занимает остальной объем: Vi = V - Vo. При 100°С давление насыщенного водяного пара рн = 101 кПа, т. е. равно нормальному атмосферному давлению. Остается воспользоваться уравнением Менделеева-Клапейрона, из которого получаем MpuV, Мрн т т" RT RTV р) °’59 Г‘ Обратите внимание на то, во сколько раз пар легче воды! 10.5. Ответ: h = 0. Решение. Напомним принцип действия поршневого насоса (простейшая его мо- дель показана на рисунке). При под- нятии поршня между ним и поверх- ностью жидкости образуется разреже- ние и вода начинает подниматься под действием атмосферного давления ра. Предельная высота h подъема воды - - - - - обычно определяется из условия р& = pgh, где р — плотность воды;
Ответы, указания, решения 358 отсюда h ® 10 м. Однако при этом предполагается, что давлением насыщенного пара под поршнем можно пренебречь. При комнат- ной температуре такое допущение справедливо: например, ’при 20°С давление насыщенного пара составляет около 2% атмосфер- ного. Однако при повышении температуры давление насыщенного водяного пара быстро возрастет и при кипении сравняется с атмосферным. Поэтому кипящую воду насос не поднимет вообще. 10.6. Решение. На первый взгляд может показаться, что каждая жидкость начнет кипеть при «своей» температуре и потому t = = 77°С. Рассмотрим, однако, механизм процесса кипения. Кипе- ние начинается, когда давление паров в пузырьках становится равным внешнему (мы считаем, что в жидкостях достаточно много центров парообразования). В пузырьках, образующихся на гра- нице двух жидкостей, находятся насыщенные пары обеих жидкос- тей; поэтому давление в этих пузырьках (равное сумме давлений насыщенных паров обеих жидкостей) больше, чем в пузырьках, которые возникают внутри каждой жидкости. Следовательно, при нагревании раньше всего сравняется с атмосферным рл давление в «пограничных» пузырьках, и кипение на границе раздела жидкостей начнется при t < 77°С. При этом нижняя жидкость, как более летучая, будет выкипать быстрее. Чтобы определить t, надо знать зависимость давления насы- щенных паров каждой из жидкостей от температуры. Тогда можно построить график зависимости от температуры давления р„ в «пограничных» пузырьках (рп = рш + рп2). Температура кипения определяется из условия р» - р„ Поскольку зависимости рш(0 и рнз(0 в задаче не приведены, определить температуру кипения t по приведенным в условии данным нельзя. 10.7. Ответ: ср = 54%. Решение. Напомним, что точка росы — это температура, при которой водяной пар становится насыщенным. Воспользуемся формулой <р = — - 100% (здесь р — плотность водяного пара, Phi содержащегося в воздухе, а рж — плотность насыщенного пара при температуре ti). При охлаждении р = const вплоть до начала конденсации. Следовательно, начальная плотность водяного пара равна рнг. Таким образом, ф = • 100% = 54%. рН1 10.8. Ответ: фг = 2,9%. Решение. Относительная влажность ф2 = 100%, где рг — парциальное давление водяного пара после нагревания и сжатия,
359 Молекулярная физика а риг — давление насыщенного водяного пара при t2 = 100°С. Поскольку tz совпадает с температурой кипения воды при нор- мальном атмосферном давлении ра, получаем рН2 = р.. Давление рг определим из уравнения Менделеева-Клапейрона: pzV = -д-КТ/ (здесь т, М — соответственно масса и молярная масса водяного пара, V — конечный объем воздуха). Аналогичное соотношение гп можно записать для начального состояния пара: pi 3V = Из двух последних уравнении получаем р2 - —• Воспользо- вавшись формулой <pi = — 100%, находим Phi ЗршТ 2ф1 Р2 = тПоо%и<Р2 = З/РшТ^гф! РнгГ 1 = 2,9%. При более сильном сжатии и более высокой температуре ti мы могли бы получить ф2 > 100%. Это означало бы, что на самом деле произошла частичная конденсация пара. Правильным ответом в этом случае было бы ф2 = 100%. 10.9. Ответ: V = 1,0 м3, т = 7,7 г. Решение. Конденсация пара начнется в момент, когда пар станет насыщенным, т. е. при puV = рТЛ> (здесь рн — плотность насыщен- . фо ного пара при температуре t, р - рн тххзт” — плотность пара до 1UU /о \ хч ту кофо , - Л о , , сжатия). Отсюда V ~ jqqo^ 1,0 м. Масса пара в воздухе изме- няется от mi = рКо до пи = Pirp Значит, масса выпавшей росы т = mi - т> 100% 10.10. Ответ: р = 0,59 кг/м3. Решение. 100°С — температура кипения воды при нормальном атмосферном давлении р» = 101 кПа. Значит, давление насыщен- ного пара при этой температуре тоже равно атмосферному. При- меняя уравнение Менделеева-Клапейрона и учитывая, что т =рК, получаем р = = 0,59 кг/м3, где М — молярная масса воды. 10.11. Ответ: р2 = 233кПа. Решение. Очевидно, давление р2 = р'г + рп, т. е. складывается из парциальных давлений сухого воздуха р'г и водяного пара рв.
Ответы, указания, решения 360 Поскольку V = const (мы пренебрегаем объемом налитой воды), Pi 'f*2 7^ 2 — = Отсюда рг = Чтобы определить рв, нужно сначала Pt J1 Jt выяснить, вся ли вода испарится. С помощью уравнения Менде- леева-Клапейрона можно найти (см., например, решение задачи 10.4), что масса насыщенного пара в сосуде при HXFC равна 5,9 г. Следовательно, часть воды останется в жидком состоянии, а пар будет насыщенным. Его давление равно рп = р„ = 101 кПа. Итак, Т2 рг - pt-ffr + р, = 233 кПа. Типичная ошибка при решении таких J 1 задач — отождествление массы образовавшегося пара с массой налитой воды. 10.12. Решение. Весь вопрос в том, повысится или понизится относительная влажность воздуха в кухне, если открыть фор- точку: ведь скорость сушки белья зависит только от относительной влажности. На первый взгляд — повысится: ведь относительная влажность на улице близка к 100%! Однако заметим, что, во-пер- вых, в кухне относительная влажность тоже достаточно велика, а во-вторых (и это главное!), температура в кухне намного выше. Поэтому давление насыщенного пара при Комнатной температуре воздуха намного выше, чем при температуре наружного воздуха. Даже ненасыщенный водяной пар в кухне имеет большее дав- ление, чем насыщенный на улице. Поэтому при открытой форточ- ке пар будет выходить из кухни и белье будет сохнуть быстрее. 10.13. Решение. Давление на дно второго сосуда, естественно, больше, чем на дно первого. Поэтому после открывания нижнего крана вода начет перетекать в первый сосуд. Это перетекание будет происходить вплоть до выравнивания уровней воды в обоих сосудах. Давление же паров в обоих сосудах будет оставаться неизменным (за счет конденсации некоторой части пара в первом сосуде и испарения воды во втором), так что оно не влияет на процесс перетекания воды (если бы в сосудах был воздух, то его сжатие в сосуде 1 и расширение в сосуде 2 создали бы перепад давлений, не позволяющий уровням воды выравняться). Что произойдет, если открыть верхний кран? Казалось бы, ничего — ведь давление насыщенного пара в обоих сосудах одинаково. Но это означает только то, что давление пара у поверхности воды в сосудах 1 и 2 одинаково. С высотой же давление пара падает. Поэтому на уровне верхнего крана давление со стороны сосуда 1 меньше. Если открыть кран, начнется медленное перетекание пара из Сосуда 2 в сосуд 1. В результате преобладающим процессом
361 Молекулярная физика в сосуде 1 окажется теперь конденсация, а в сосуде 2 — испарение. Так будет продолжаться до полного выравнивания уровней в сосудах. Естественно, при открывании верхнего крана скорость протекания процесса намного меньше: в этом случае заметное изменение уровней воды в сосудах может произойти только через несколько месяцев. 10.14. Ответ: h = 19 см. Решение. Для нагревания воды до t2 = 100°С потребуется количес- тво теплоты Qi = mc(t2 - it) = 12,6 кДж. Следовательно, оставшееся количество теплоты Q2 = Q - Qt = 12,4 кДж затрачено на па- 02 рообразование. В пар перейдет = -=- = 5,4 г воды (здесь L — удельная теплота парообразования воды при 100°С). Поскольку mi < т, пар является насыщенным и его температура не подни- мется выше 100°С. Объем пара определим из уравнения Менделе- ева-Клапейрона: V = т.ЛТ Мр = 9,3 л. Высота подъема поршня h = = = 19 см. Может возникнуть вопрос: почему не учтена работа по подъему поршня, т. е. работа расширяющегося пара А' = рЛУ7 Дело в том, что эта работа уже учтена в табличном значении L. Ведь если бы даже поршня не было, пар при расширении должен был бы преодолеть то же самое давление = 10s Па. 10.15. Ответ: уменьшится. Решение. Состав паров у соленой воды тот же, что у пресной. Давление паров также одинаково (оно совпадает с внешним). Воспользовавшись формулой р = (см. решение задачи 10.10), приходим к выводу: поскольку Т увеличивается, плотность паров уменьшится. 10.16. Ответ: ср = 72%. Решение. Влажный воздух в трубке испытывает изотермическое расширение, при этом его давление падает от р„ до рЛ - Г(в мм рт. ст.). Согласно закону Бойля-Мариотта объем воздуха увеличива- ть* т, ется в ------ раз. Во столько же раз уменьшаются давление и р. - I плотность водяного пара. Значит, <р = фо——- = 72%. Р‘ 10.17. Ответ: I > р,(^^-— - 1) = 190 мм. ' Ф° ’
Ответы, указания, решения 362 10.18. Ответ: Л = 31 см. Решение. Конденсация начнется, когда относи- _ тельная влажность достигнет 100%. Для этого объем воздуха в трубке должен уменьшиться в 100% ______ ------= 1,25 раз. значиТ) длина воздушного стол- - - “ ~ Ф . I -с_ бика в трубке будет при этом .3V = 0,8/. Давление < в трубке, выраженное в см рт. ст., р = р, + h - 0,21 ------ (см. рисунок). Тогда согласно закону Бойля-Мариотта pd = (ра + Н h - 0,21)' 0,81. Отсюда h = 0,25ра + 0,2/ = 31 см. 10.19. Ответ: рг = 153 кПа. Решение. Будем считать объем газа постоянным, т. е. не будем учитывать небольшого изменения объема воды за счет конден- сации. Давление насыщенного водяного пара при 100° С равно атмосферному. Следовательно, парциальное давление воздуха пер- воначально pi - pR = 2р„. В результате изохорного охлаждения гр парциальное давление воздуха станет 2р, Давление насыщен- 11 ного водяного пара при t-г = 10°С составляет рт = 1,23 кПа. Таким Т образом, полное давление в котле р2 = рт + 2р.~ =153 кПа. 1 ь 10.20. Ответ: р — 60 кПа. Решение. В смеси находилось Vi = = 0,25 моль водорода и v2 = = 0,25 моль кислорода. Первоначальную температуру Т в Л7 2 баллоне можно определить из уравнения Менделеева-Клапейрона: PiV Т =-------~~ = 283 К. В реакцию 2Н2 + О2 -* 2Н2О вступит весь (vi + v-i)R водород, но только половина кислорода. В результате реакции образуется Vs = 0,25 моль воды и остается vj = 0,125 моль кислорода. Масса воды m =УзМ = 4,5 г намного превышает массу насыщенного водяного пара в объеме баллона при данной темпе- ратуре (/Пн = рн!7 = 0,047 г), поэтому большая часть воды окажется в жидком состоянии, а водяной пар в баллоне будет насыщенным. Его давление при температуре Т составляет рн = 1,23 кПа. Пар- ViRT pi циальное давление кислорода рк = —у— - (объемом, который занимает вода, можно пренебречь). Итак, р = р0 + рк = 60 кПа. Интересно сравнить этот результат с полученным, в задаче 9.9.
363 Молекулярная физика 10.21. Ответ: ср = 50%. Решение. Уже при первом сжатии давление выросло менее, чем в четыре раза: пар стал насыщенным и произошла частичная его конденсация. Поэтому давление пара после первого и второго сжатия одинаково и равно рп — давлению насыщенного пара при данной температуре. Парциальное же давление воздуха менялось при изотермическом сжатии обратно пропорционально объему. Если начальное давление воздуха было рв, то после первого и второго сжатия оно стало равным соответственно 4рв и 8рв. Обоз- начим первоначальное парциальное давление пара рп. Тогда пол- ное давление влажного воздуха сначала было рп + рв, после первого сжатия стало рв + 4рв, а после второго рв + 8р„. Согласно условию Рв + 4рв = 3(р„ + Рв), Рв + 8рв = 5(рп + Рв). Исключая из этих уравнений рв, получаем рв = 2рп. Значит, начальная относительная влажность воздуха ср = — 100% = 50%. p.i 10.22. Решение. Очевидно, вода в плавающей кастрюле нагрева- ется до температуры кипения. Однако это еще не означает, что она закипит: ведь для того, чтобы жидкость, достигшая темпера- туры кипения, и в самом деле закипела, необходим дальнейший подвод теплоты к этой жидкости. Вода в большой кастрюле получает необходимое количество теплоты от нагревателя; вода же в плавающей кастрюле может получать теплоту лишь от воды в большой кастрюле. Однако при достижении температуры кипе- ния теплообмен между жидкостями прекращается, поскольку их температуры становятся одинаковыми! Поэтому вода в плавающей кастрюле кипеть не будет. 10.23. Решение. Температура кипения существенно зависит от внешнего давления. Поэтому, если поместить воду в герметичный сосуд и откачивать из него воздух, можно заставить воду закипеть. Например, чтобы вода закипела при 20°С, т. е. при комнатной температуре, надо понизить давление до 2,33 кПа (это давление насыщенного водяного пара при 20°С). Такое давление несложно получить с помощью довольно простых насосов. Если производить откачку достаточно быстро, то температура воды заметно понижа- ется. Ведь интенсивное парообразование при кипении происходит за счет уменьшения внутренней энергии жидкости. Охлаждение вполне может привести к замерзанию некоторой части воды. При этом вода, замерзая, будет кипеть! 10.24. Решение. На первый взгляд — нельзя: ведь температура плавления свинца ti = 327°С намного превосходит температуру кипения воды t2 = 100°С. Однако вода кипит при 100°С лишь при
Ответы, указания,решения 364 нормальном атмосферном давлении! Повышая внешнее давление, можно нагревать воду без кипения намного выше 100°С, а именно до так называемой критической температуры is = 374°С. Посколь- ку ii < is, свинец расплавить в воде можно! Для этого нужно только нагревать оба вещества в герметично закрытом сосуде. Однако стенки сосуда должны выдерживать очень высокое давление, создаваемое насыщенными парами воды при высокой температуре. 10.25. Решение. Замерзание происходит при 0°С только при нали- чии центров кристаллизации — например, каких-то нераство- римых частиц. В стакане воды практически наверняка найдется несколько таких центров. Этого уже достаточно, чтобы замерзла вся вода. Если же вода раздроблена на мельчайшие капли, то центры кристаллизации найдутся лишь в некоторых каплях. Эти капли и замерзнут при 0°С, а остальные (т. е. подавляющее большинство) останутся жидкими вплоть до значительно более низких температур. Заметим, однако, что во многих случаях облака (например, перистые) состоят не из капелек воды, а из кристалликов льда. 10.26. Ответ: г = 1,5 мм. Решение. Силы поверхностного натяжения тянут капиллярцую трубку вниз. Они приложены к внутренней и внешней поверхнос- тям по окружностям радиуса г, поэтому полная длина линии, вдоль которой действует сила поверхностного натяжения F, равна 4лг. Следовательно, F = 4лгст. С другой стороны, согласно условию _ mS 1 К F = mg, отсюда г =---= 1,5 мм. 4кст 10.27. Ответ: F = (си - стг)/; сила действует перпендикулярно стороне ВС (на рисунке в условии задачи — вверх). Указание. На любой прямолинейный участок рамки длиной L действуют перпендикулярно этому участку две силы поверхност- ного натяжения: сила Ft = oiL направлена наружу и сила Рг = oaL — внутрь рамки. 10.28. Ответ: около 1 л. Решение. Вода не будет выливаться из решета, если нити не смачиваются водой и силы поверхностного натяжения компен- сируют силу тяжести. Рассмотрим столб воды, в основании кото- рого лежит один квадратик, образованный нитями решета. При максимальной высоте воды h силы поверхностного натяжения на линии соприкосновения воды с каждой из нитей направлены вверх, тогда их равнодействующая равна. 4od. Она уравнбве-
365 Молекулярная физика шивает силу тяжести столба воды pd2gh. Отсюда Л = ~ = 3 см; pgd ,, 4ЛСТГ2 ' п. полная масса воды т = пгпр = & = 0,94 кг. Оказывается, решете можно перенести не так уж мало воды! Однако требуется' осторожность: нельзя нарушать пленку воды, «провисающую» внизу, и наклонять решето — ведь при этом глубина в каком-то месте увеличится и решето станет протекать. Разумеется, получен- ный результат следует рассматривать как весьма приближенный. 10.29. Ответ: F = mg + 4ст(а + Ь). Указание. Сила F должна уравновесить силу тяжести и силы поверхностного натяжения, действующие на проволоку с двух сторон. 10.30. Решение. И материя, и бумага Пронизаны капиллярами. Жир смачивает материю и бумагу. Поэтому расплавленный го- рячим утюгом жир «втягивается» в капилляры — тем сильнее, чем уже эти капилляры (ср. с задачей 10.40). Капилляры же в бумаге тоньше, чем в материи. 10.31. Ответ: снаружи. Указание. Нагревание жидкости уменьшает поверхностное натя- жение. Поэтому в капиллярах, пронизывающих кожу, жидкость , втягивается в сторону более холодной части. ' 2ст 10.32. Ответ: см. рис. а, б, в (при Л <--) и рис. г, д, е (при pgr Л > ). Решение. Обозначим через Нк и Н высоту уровня воды соответст- венно в капилляре и в широком сосуде; АН = Нк - Н. Будем считать широкую трубку настолько высокой, что вода не достигает ее края. Рассмотрим два случая. . . 2ст _ 1. Л <---. В этом случае капилляр заполнится, когда широкий pgr сосуд будет еще практически пуст. Только после этого начнет заполняться (естественно, намного медленнее) широкий сосуд. При этом форма мениска в капилляре будет меняться от вогнутой до плоской (при А Н = 0) и затем выпуклой (при АН < 0). Когда ТТ , -2ст ___ Н достигнет значения Л +-----, вода начнет выливаться из капил- Р4ГГ ляра и повышение уровни воды прекратится. Графики HK(t), H(t), A H(t) приведены на рис. а, б, в.
Ответы, указания, решения 366 2. h >--. В отличие от предыдущего случая теперь, начиная pgr с некоторого момента, уровень воды растет с одинаковой ско- * гт ростью в капилляре и в широком сосуде, так что АН -------= pgr = const Так продолжается, пока вода не заполнит капилляр полностью. Соответствующие графики приведены на рис. г, д, е. Обратим внимание, что |ДЬГ| во всех случаях не превышает-. pgr 10.33. Ответ: h = 3,0 см. Решение. Свободная поверхность жидкости граничит со стенками трубки по двум окружностям радиуса г (см. рисунок). Верхний и нижний мениски при максимально возможной высоте столба
367 Молекулярная физика воды представляют собой полусферы, поэтому силы повер- хностного натяжения Fnn направлены вертикально вверх. Они уравновешивают силу тяжести, действующую на воду: 2Гпн = 4кстг = mg. Учитывая, что т = pV = pnr^h, находим: 4ст h = — = 0,03 м. pgr Разумеется, это вдвое превосходит обычную высоту 2ст капиллярного подъема-----: ведь воду здесь «держат» два pgr мениска, а не один. 10.34. Ответ: h = 2,3 см. Решение. Свободная поверхность жид- FA кости граничит со стенками кубика по периметру квадрата со стороной а. Ку----------------- бик тянет вниз сила поверхностного натяжения Гпн = 4ста (см. рисунок). _ Условие равновесия: FA = mg + Fnn. Z Z J- Учитывая, что Fa = p,,gV„ = p,ga2h, где -Z < Z ~Z J rmg - Z ~Z pB — плотность воды, a VB — объем Z Z^Z ZeF Z Z Z погруженной части кубика (т. е. объем Z~ Z Г Z- Z~’Z- Z~ вытесненной воды), получаем: mg + 4ста п h =--------— = 0,023 м. p»g<r Итак, нижняя часть кубика погружена, на глубину 2,3 см. В «отсутствие» поверхностного натяжения (при ст = 0) эта глубина была бы на 1 мм меньше. 10.35. Ответ: натертый парафином кубик плавает на 6,0 мм выше. Указание. См. задачу 10.34. Учтите, что парафин не смачивается водой. 10.36. Ответ: г = Л/--------= 2,25 мм. V g(2p. - рв) Указание. Парафин не смачивается водой; см. также задачу 10.34. 10.37. Ответ: г « 5 мм. Решение. Для грубой оценки можно принять, что висящая на потолке капля имеет форму полусферы радиуса г. Тогда действу- ющая на нее сила поверхностного натяжения равна ст * 2лг, а сила 2 , Л /Зст . _ тяжести капли mg = p-^nrg, откуда г = Л/— = 4,7 мм. 4 V pg
Ответь}., указания, решения 368 10.38. Решение. Рассмотрим ситуацию, в которой образуется цилиндрический мениск жидкости: в жидкость на не- большом расстоянии 2R друг от друга опущены две параллельные вертикаль- ные смачиваемые пластины (см. ри- сунок). На столбик жидкости, подняв- шейся между пластинами на высоту Л, действует вверх сила поверхностного натяжения 2<з1 (Z — длина пластин), уравновешивающая силу тяжести mg — р • 2Rlh • g. Из равенства 2oZ = р • 2Rlh • g получаем h =---. Следовательно, давление в жидкости под поверхностью pRg мениска меньше атмосферного на величину Др = pgh = -=. /С Полусферический мениск возникает в круглом капилляре, опущенном в жидкость. В этом случае условие равновесия стол- бика жидкости в капилляре с внутренним радиусом R имеет вид „ >4 2ст о • 2nR mg = р • %Rhg, откуда Др = pgh = Л Если жидкость не смачивает поверхность, давление под выпук- лой Поверхностью мениска увеличивается на величину Др. Вообще Давление всегда меньше с той стороны мениска, куда обращена выпуклость (искривленная поверхность жидкости ведет "себя в этом отношении как упругая пленка). Величину Др называют лапласовским давлением. У 10.39. Ответ: к узкому колену, Др «= 510 Па. Решение. После выравнивания уровней ртути дав- ление р под ее поверхностью в обоих коленах станет одинаковым. Давление воздуха в узком колене pi = 2ст 2 ст = р---, в широком р2 = р - — (см. задачу 10.38). Г1 Га Следовательно, разность давлений в коленах трубки Др = pa - pi = _ = 510 Па. Поскольку Др > 0 (т. е. рг > pi), вакуумный насос следует присоединить к узкому колену трубки. 10.40. Ответ: в сторону сужения капилляра; в сторону расцщрения капилляра. Решение. Для случая смачивания форма капли показана на рис. а.
869 Молекулярная физика Очевидно, радиус кривизны сферического мениска 1 меньше радиуса кривизны мениска 2, т. е. и < гг. Давление в жидкости 2ст 2ст , у менисков соответственно: pi = р, - —, р2 = р. - — (см. задачу 10.38; здесь рЛ — атмосферное давление). Очевидно, pi < р2. Жидкость будет перетекать в сторону более низкого давления; значит, капля втягивается в более узкую часть капилляра. С «энергетической» точки зрения такое поведение жидкости обус- ловлено тем, что она «стремится» увеличить площадь соприкос- новения со смачиваемой поверхностью. В случае несмачивания 2 ст 2ст (см. рис. б) pi = р» + —, р2 = р» + —. Капля будет перемещаться в Г1 Гг более широкую часть капилляра, поскольку pi > рг. Заметим, что рассмотрение сил, действующих вдоль окружнос- тей с радиусами и и гг, привело бы к противоположному (не- правильному!) ответу. Дело в том, что стенки капилляра действу- ют и на боковую поверхность жидкости: «тянут» ее в случае смачивания и «отталкивают» в случае несмачивания. 10.41. Ответ; в обоих случаях г = 1,5 мм. В первом случае выпуклость мениска обращена внутрь, а во втором — наружу. Решение. В обоих случаях радиус кривизны верхнего мениска равен (верхний мениск — полусфера, касающаяся внутренних |Ы стенок капилляра). Поэтому давление внутри воды непосредствен- но под верхним мениском р* = рЛ - 4a/d. Давление внутри жидкости у нижнего мениска р„ = р, + pgh, где Л — длина столбика воды. Это давление отливается от атмосферного на величину . , 4ст _ . ар = ря- р„ = pgh - -&• Величина Др связана с г соотношением I I 2ст 2ст |Др| = —, откуда г = ;---—. При Др > 0 выпуклость нижнего
Ответы, указания, решения 370 мениска направлена вниз, при Др < О — вверх; при Др = О нижний мениск плоский. Разумеет- ся, при допустимых значениях Л, т. е. при Зст d h <---, получаем г > Таким образом, при h = pgd z = 2,0 см радиус сферического мениска г = 1,5 мм и мениск вогнут внутрь (см. рис. а); при h = = 4,0 см радиус мениска г = 1,5 мм и мениск обращен выпуклостью вниз (см. рис. б). 10.42. Ответ: Др = 490 Па. Решение. Непосредственно перед отрывом пу- зырька радиус кривизны поверхности воды у нижнего края капилляра равен г (см. рисунок). Давление в воде в этом месте равно р0 + pgh, 2ст о внутри капилляра оно равно р» + pgh + —. Зна- чит, необходимое избыточное давление состав- 2ст ляет Др = pgh + — = 490 Па. г 10.43. Решение. Вода может подняться и заполнить достаточно тонкую трубку полностью. Однако вытекать из трубки она не будет: воду удержат те самые силы поверхностного натяжения, которые втянули ее в капилляр. 10.44. Ответ: выпуклость мениска обращена вниз, его радиус 2 ст г =---= 0,74 мм. Pgh Указание. См. решение задачи 10.41. 10.45. Ответ: р = р0 + It Решение. Очевидно, давление внутри мыльной пленки (см. рисунок, на ко- тором толщина пленки намеренно пре- ч 2ст увеличена) pi = ро + давление же xt внутри пузыря р = Р1 + 2ст R-h Поско- льку толщина Л стенки мыльного пу- зыря ничтожно мала (в сотни раз мень-
371 Молекулярная физика ше толщины волоса), получаем р = р0 + Таким образом, избы- а точное давление в мыльном пузыре вдвое больше, чем внутри воздушного пузырька того же радиуса в мыльной воде — ведь в мыльном пузыре избыточное давление создается двумя поверхнос- тями. 10.46. Ответ: F « 700 Н. Решение. Почему «слипаются» стекла? У их краев образуется цилиндрический мениск, вогнутый внутрь, с радиусом кривизны d , , — (величина d определяется высотой неровностей и возможным изгибом стекол). Значит, давление внутри воды (между стеклами) 2ст меньше атмосферного на Ар = (см. задачу 10.38). Именно за РЛС 2ctS счет этой разности давлении и создается сила F - &р • S = , прижимающая стекла друг к другу. В данном случае она равна 730 Н. Такую силу необходимо приложить, чтобы «оторвать» стекла друг от друга. Заметим, что чем лучше отполированы стекла и чем они более плоские, тем труднее их разъединить (.F ~ Можно, конечно, заставить стекла скользить друг отно- сительно друга, уменьшая тем самым площадь их перекрытия S. Для этого потребуется преодолеть лишь силу трения, намного меньшую F. Однако так стекла можно поцарапать. Лучше всего погрузить стекла в воду — мениск исчезнет, и стекла разъеди- нятся без труда. 10.47. Ответ: d = 0,15 мм. Решение. Поскольку ртутные шарики расположены симметрично, пластинка .. mg давит на каждый из них с силой В ШШ77777ШЛ 2R результате каждый из шариков сплющивается в «блин» толщиной d и радиусом R (см. рисунок). Будем предполагать, что d « R. Объем «блина» nR2d должен совпадать с начальным объемом шарика: 4 itR2d = (1) О Давление внутри «блина» (избыточное над атмосферным) р =---С другой стороны, это давление обусловлено кривизной 4nRr боковой поверхности блина. Поскольку R > d, давление р будет
Ответы, указания, решения 372 таким же, как для цилиндрического мениска (/радиусом т. е. р = -j- (см. задачу 10.38). Итак, — — = —г. Решая это уравнение а 4тг2? “ совместно с уравнением (1), получаем 2лСТГо _ . _ -z—- = 0,15 мм, Smg R = = 3,0 мм. ' бтго Предположение d « R вполне оправдалось. Нижняя поверх- ность пластинки находится на высоте 0,15 мм от горизонтальной плоскости. 10.48. Решение. Непосредственное использование формулы для давления р = pgh здесь затруднено. Ведь при остывании плотность р воды возрастает, зато высота столба воды Л уменьшается. Воз- никает «конкуренция» этих двух факторов, и ответ определяется тем, какой из них окажется важнее. Проще всего ответить на этот вопрос для сосуда 1 (цилиндричес- . „ mg кого). В этом случае можно записать р = —=-, где т — масса О жидкости, S — площадь дна. Все величины в правой части этой формулы от температуры не зависят (мы пренебрегаем тепловым расширением самого сосуда по сравнению с тепловым расши- рением жидкости). Итак, в первом сосуде давление не изменится, хотя уровень воды понизится. Сравним теперь уровни остывшей воды во всех трех сосудах. Очевидно, при одинаковом уменьшении (в результате остывания) объема воды уровень ее в сосуде 2 понизится меньше, а в сосуде 3 — больше, чем в сосуде 1. Поскольку температурная зависимость плотности воды для всех сосудов одинакова, при остывании воды в сосуде 2 давление на дно возрастет, а в сосуде 3 — уменьшится. 10.49. Ответ: 34 см и 24 см. Решение. Пусть при 0°С длина железного стержня lai, медного 102. Тогда при температуре t длины стержней: h = Zoi(l + cut), I2 ~ 2ог(1 + + a2Z), где ai и аз — температурные коэффициенты линейного расширения железа и меди соответственно. Разница этих длин AZ = Zi - I2 = Z01 - Z02 + (aJoi - a2Zo2)Z. Чтобы величина A Z не зависела от температуры, выражение в скобках должно равняться нулю. Таким образом, приходим к системе уравнений: {lai — Z02 = AZ, UiZoi ~ U2Z02 = 0. Отсюда lai - М——— = 34 см, Z02 = AZ——— = 24 см. a2 - di а2 - ai
373 Молекулярная физика 10.50. Решение. Обычный металл с поликристаллической струк- турой является изотропным. Следовательно, при однородном на- гревании металла любой отрезок между двумя его точками удли- няется в одно и то же количество раз, независимо от направления этого отрезка. Другими словами, в результате нагревания любого тела все его размеры увеличиваются пропорционально, т. е. полу- чившееся тело геометрически подобно исходному. Это общее ут- верждение позволяет ответить на поставленные, вопросы. Внут- ренний диаметр кольца при нагревании увеличивается (т. е. «дырка» тоже расширяется при нагревании!). Угол <р при нагре- вании не изменяется. 10.51. Ответ: F = 65 кН. Решение. При отсутствии опор стержень удлинился бы на AZ = aZx xAZ = aZ(t2 - ti), где Z — длина стержня. Значит, сила F должна вызвать сжатие стержня на такую же величину. Согласно закону F Гука ст = Е|г|, где ст = ~ — механическое напряжение в стержне, Е О — модуль упругости (модуль Юнга) стали, е = — — относительное удлинение стержня. Отсюда находим F — Ea(t2 ~ ti )S = 65 кН. Обратите внимание на то, какие значительные силы возникают в твердых телах при деформациях, обусловленных изменением температуры. 10.52. Ответ: t2 = -210°С. Решение. При охлаждении в проволоке возникает механическое напряжение ст. Проволока лопнет, когда это напряжение до- стигнет предела прочности ст,,.,. Используя выражение ст — Ea(ti — - t2) (см. задачу 10.51), получаем t2 = ti - - -210°С. Еа 10.53. Ответ: a = Ь. \ES> Решение. Обозначим силу натяжения проволоки Т. Из условия равновесия фонаря следует, что 2Tsina = mg. С другой стороны, из закона Гука следует, что Т = ES~; здесь AZ = 2l(— 1) — 2Z 'cosa ' т_ „ . 1 - cosa mg _ удлинение проволоки. Итак, 2sma •------------= -=^. При малых a cosa можно считать sina = a, 1 - cosa = 2sin2^ = -j-. Отсюда a = А Ш Разумеется, это решение справедливо при mg < ES. img\i Les)’
Ответы, указания, решения 374 10.54. Ответ: о Решение. Пусть на бетон приходится нагрузка (усилие) F<„ а на железо Рж. Как следует из закона Гука, Ft, = EeSel^l, Fx = Ежх хЗж|е| (относительное удлинение е у бетона и железа одинаково; в данном случае оно отрицательно, потому что колонна сжата). р EffSc, Отсюда vr = г о • Учитывая также, что Fa + Fx = F, получаем Fx ЕжОж F>« _ / Е& \ _ 1 F ~ I + E„Sj ~ 3’ Итак, хотя площадь поперечного сечения железа в 20 раз меньше, чем у бетона, железо «принимает на себя» треть всей нагрузки! Это обусловлено значительно большей «жесткостью» железа по сравнению с бетоном. 10.55. Ответ: не менее, чем из 100. Решение. Пусть количество проволок равно N. Тогда каждую mg проволоку растягивает сила создавая в ней механическое mg 4mg напряжение о = Минимально возможное количество проволок Nmin соответствует случаю ст = ст,,.,, где ст,,., — предел прочности стали. Отсюда IV™ = —-— = 100 (округление до бли- ка стт жайшего целого числа следует в данном случае проводить в сторону увеличения). В реальной ситуации число проволок N N выбирается всегда большим Nmin. Отношение —— характеризует i ▼ min так называемый запас прочности. 11.1. Ответ: см. рисунок. Решение. Пока нагревается лед, t(x) линейно возрастает от ti до 0°С, затем некоторое время t = 0°С (лед тает), после чего t линейно возрастает от 0°С до tz (нагревается вода). Для построения графика сравним длительности Ti, Тг, тз каждого из этапов, воспользо- вавшись законом сохранения энергии: Pti - с.,т(0Р - ti); Рт2 - Кт; Ртз = c„m(t2 - (F). Здесь Р — мощность нагревателя; с, и с. — удельная теплоем- кость соответственно льда и воды (сл = —); л — удельная теплота плавления льда. Из записанных соотношений получаем И = _* 16, - = -^Г = 2,0.
375 Молекулярная физика График t(r) имеет вид, показанный на рисунке. Обратите вни- мание: вода нагревается медленнее льда из-за вдвое большей удельной теплоемкости. 11.2. Ответ: т2 = ---~---= 82 мин. C(t2 - ti) 11.3. Ответ: О = 0°С. Решение. Эту задачу не очень удобно решать в общем виде: ведь для составления уравнения теплового баланса необходимо заранее знать, какие процессы произойдут со льдом и с водой, т. е. каким будет конечное состояние (только вода, вода и лед или только лед). А это определяется как раз численными значениями тв, тл, tB, Гл. Предположим сначала, что весь лед растает, а вода несколько остынет. Тогда уравнение теплового баланса имеет вид QB + (?л = О, где QB < О — количество теплоты, отданное водой; (?л — количество теплоты, полученное льдом. Вода охлаждается от tB до 0, значит, Qb = 7ПвСв(0 - <в). Лед нагревается от t„ до 0°С, при 0°С плавится и далее (уже будучи водой!) нагревается от 0°С до 0. Значит, Q., = 7ПлСл(0°С - tB) + 1т3 + тлСв(0 - 0°С). Из уравнения теплового баланса получаем m„c„ta + - А.тл оюп 09 _-------------------— — 21 С. с„(/пл + т,) Однако полученное значение (0 < 0°С) противоречит сделанно- му предположению, что весь лед растает! Значит, это предполо- жение было неверно. Можно теперь предположить, что вся вода замерзнет. Но тогда (предоставляем читателю проверить это) температура 0 окажется положительной, что снова будет про- тиворечить сделанному предположению. Остается лишь один ва- риант ответа: 0 == 0°С, т. е. весь лед не растает и вся вода не
Ответы, указания, решения 376 замерзнет,— в калориметре будет смесь воды со льдом. К этому результату можно прийти гораздо быстрее, если заметить, что вода, даже остыв до 0°С (а остывать дальше, не замерзая, она не может!), отдаст количество теплоты znBcBtB. Этого количества теп- лоты хватит лишь на плавление льда массой т‘С‘*" = о, 38 кг, что Л меньше начальной массы льда тл (при этом мы даже не учли необходимости нагревания льда до 0°С). Значит, весь лед растаять не может, т. е. 0 < 0°С. Аналогично доказывается, что 0 > 0°С. Отсюда 0 = (FC. 11.4. Ответ: а) 0 =------------------ 13,5°С, V =-----= 1,6 л. Св(тп. + ТПл) рв е> е = о°с, v = + » ю,4 л, рл рв , m,c,t, + 7ПлСл«л _ . _ где Дтп ---------------= 0,13 кг. А 11.5. Ответ: 0 = 0°С; масса льда увеличится до 500 г. Решение. Проанализируем условие задачи (см. также решение задачи 11.3 ). При охлаждении всей воды до 0°С она выделит количество теплоты + znztz) = 16,8 кДж. Легко убедиться, что это меньше количества теплоты Хтпз, необходимого для плав- ления всего льда. Значит, 0 < 0°С. С другой стороны, на нагревание льда до (FC уйдет количество теплоты -Сл/тЫз = 50,4 кДж. Это меньше, чем выделилось бы теплоты при замерзании всей воды. Поэтому 0 > О^С. Итак, мы пришли к выводу, что 0 = 0°С. Для охлаждения воды и нагревания льда до этой температуры требу- ется количество теплоты Q — 50,4 лсДж - 16,8 кДж = 33,6 кДж. Оно может выделиться только за счет замерзания воды массой Дтп = — = 0,10 кг. Итак, при установлении теплового равновесия А. масса льда увеличится на Дтп =100 г и составит т = тз + Дтп = = 500 г. 11.6. Ответ: 0°С. Указание. См. задачу 11.3. 11.7. Ответ: 112 г. Решение. Необходимое для плавления льда количество теплоты Qa = тплА. выделяется при конденсации некоторой массы пара та и охлаждении получившейся воды до О^С. Следовательно, уран-
377 Молекулярная физика нение теплового баланса имеет вид тя 1 - пга L + т.п с(1я - tn) = О, где L — удельная теплота парообразования, ас — удельная _ X „ теплоемкость воды. Отсюда т„ = тл—-------------Полная масса L + c,(t„ - t4) образовавшейся в калориметре воды т = 7ПЛ + 7П„ = 771,(1 + --------1 =112 г. ' L + c,(tu - t„y 11.8. Ответ: тп = 3,5 кг. Указание. Уравнение теплового баланса имеет вид Qn + Qb "I" Qn = 0, где Qn = mA + 7плс(0 - ti); Qb = 77iBc(0 - ti); Qn = -m^L + тппс(0 - in). 11.9. Ответ: 6,6 г. Указание. Если обозначить массу содержавшейся в мокром снеге воды через тпв, то уравнение теплового баланса приобретает вид: -miceAt + (таг - /Пв)Х + m2C„(ti - At) — 0 (мы учли, что температура мокрого снега равна 0°С). 11.10. Ответ: t > 125°С. Решение. Пусть объем куба (и выплавленной во льду ямки) равен V. Разумеется, мы пренебрегаем изменением потенциальной энер- гии системы при погружении куба в лед — оно ничтожно мало по сравнению с количеством передаваемой теплоты. Масса куба равна PaV, масса расплавленного льда рлУ (здесь рл, рл — соответственно плотность алюминия и льда). Уравнение теплового баланса имеет вид pAVcA.t = ХрлУ, откуда t = = 125°С. При большей темпера- Сдрл туре верхняя грань куба окажется ниже поверхности льда. 11.11. Ответ: тл = 0,76 г. Решение. При замерзании части воды будет выделяться теплота, благодаря чему образующийся лед и оставшаяся вода будут нагре- ваться до 0°С. Проследить этот процесс во всех деталях очень сложно, но в этом и нет необходимости: нас ведь интересует только конечный результат, а его можно узнать, воспользовавшись зако- ном сохранения энергии в форме уравнения теплового баланса. Представим себе воображаемый процесс, при котором сначала вся вода нагревается до СРС, «занимая» при этом некоторое количество теп- лоты, а потом часть воды замерзает, отдавая в точности то же самое количество теплоты. Для такого процесса уравнение теплового балан- са имеет вид: тсв((УС - t) = Хтпл, откуда т, = = 0.76 г.
Ответы, указания, решения 378 11.12. Ответ: 12%. Решение. Механизм явления описан в решении задачи 11.11. Уравнение теплового баланса имеет вид: mJ- = (т - тп)Х, где т — начальная масса воды, та — масса испарившейся воды. Отсюда — = —-— = 0,12. Итак, испарится 12% воды. т L + Л. 11.13. Ответ: v = 840 м/с. Решение. Будем считать, что вся начальная кинетическая энергия <1 7Пу2 / - х льдинок 2 9 (тп — масса каждой льдинки) переходит в их внутреннюю энергию. Льдинки нагреваются от ti до tz = 0°С и плавятся. Закон сохранения энергии дает mv2 2—5— = 2т(с& - ti) + А.), откуда v = ^2(сл(£2 - ti) + А.) - 840 м/с. 11.14. Ответ: 36%. „ „ linVo2 Решение. Внутренняя энергия пули увелиичится на Q = тн—- ' л mv2\ п а а ---=-), где г) = 0,6. Эта энергия идет на нагревание всей пули до А ’ температуры плавления свинца tz и плавление некоторой ее части массой т*: Q = mc(tz - ti) + игА. Приравнивая оба выражения для Q, получаем ~ = ~^(у02 -и1) - = 0,36. ЛК А 11.15. Ответ: и = 2,7 м/с. Решение. Будем считать, что вся выделяемая установкой за про- межуток времени т энергия идет только на нагрев протекающей воды. Тогда Nt = тс At, где т — масса протекающей за время т тг nd2 воды, с — удельная теплоемкость воды. Поскольку m = р~^~ • 4N получаем: v =-------, = 2,7 м/с. npcAtd2 11.16. Ответ: h » 70 км. Решение. Согласно условию, на нагрев воды массой m расходуется энергия, равная ^mgh, Поэтому yrmgh = mc(tz - ti), где tz = 100°С. Л л _ , 2c(tz-ti) __ Отсюда h =----------~ 70 км. . Подученный результат показывает, сколь велика энергия, вы- деляемая и поглощаемая в тепловых процессах.
379 Молекулярная физика 11.17. Ответ: внутренняя энергия воздуха в комнате не изменяется. Решение. Напрашивается «очевидный» ответ: внутренняя энер- гия U газа изменяется прямо пропорционально его абсолютной температуре. Однако будем внимательны: это утверждение спра- ведливо только тогда, когда масса т газа неизменна! А можно ли в данном случае считать т = const? Какие параметры воздуха внутри комнаты остаются при нагревании неизменными? Оче- видно, V = const; кроме того, давление в комнате (равное атмос- ферному) тоже не должно измениться. А это возможно только вследствие утечки части воздуха из комнаты при нагревании. Если представить себе герметично закрытую «комнату», то даже при относительно небольшом повышении температуры (ДТ » 10 К) ДТ „ давление в ней повысится на Др - ~ 3 кПа, а такого перепада давлений не выдержат оконные стекла. Итак, р = const, а т уменьшается. Воспользуемся теперь тем, что внутренняя энергия идеального газа пропорциональна11 величине j^RT. Согласно уравнению Мен- т делеева-Клапейрона pV = j^RT, поэтому U ~ pV. Отсюда следует, что при нагревании U = const. Внутренняя энергия воздуха в комнате вообще не растет: всю поступающую от источника тепла энергию уносит уходящий наружу воздух! Наружный атмосфер- ный воздух становится чуть-чуть теплее — благодаря этому зимой температура воздуха в большом городе на градус-два выше, чем за городом (это особенно заметно при температуре, близкой к 0°С: в городе оттепель, а за городом лежит снег). 11.18. Ответ: большую работу газ совершит при изобарном расши- рении. Решение. Работа газа при небольшом изменении объема ЛА' = рЛУ. По- скольку при изотермическом расши- рении давление падает, работа в этом случае совершается меньшая. Особен- но нагляден этот вывод при рассмот- рении графиков обоих процессов в координатах р, V (см. рисунок). Пло- щадь фигуры, отмеченной горизон- тальной штриховкой, численно равна работе при изобарном расширении; ’Численный множитель для воздуха, преимущественно двухатомного газа, можно считать равным 6/2. Однако в данном случае знание величины этого множителя не требуется. , , ,
Ответы, указания, решения 380 площадь фигуры, отмеченной вертикальной штриховкой, соответ- ствует работе при изотермическом расширении. 11.19. Ответ: газ получал тепло на этапах 1-2, 2-3; отдавал на этапах 3-4, 4-1. Получено большее количество теплоты, чем отдано. Решение. Для любого этапа процесса можно записать уравнение первого закона термодинамики: Q = &U+A'. (!) Здесь Q — полученное газом количество теплоты (Q < О озна- чает, что газ отдает теЬяр холодильнику); Д£7 — изменение внутренней энергии газа; А' — совершенная газом работа (А' > О при расширении газа, А' < О при его сжатии). Внутренняя энергия идеального газа зависит только от его температуры, поэтому AU > О при нагревании и AU < О при охлаждении газа. Из уравнения состояния газа следует, что газ нагревается в процессах 1-2 и 2-3, а охлаждается в процессах 3-4 и 4-1. Таким образом, на различных этапах процесса имеем: 1-2: ДТ > О, \U > 0; V = const, А' = 0; 2-3: ДТ > 0, &.U > 0; ДУ > 0, А' > 0; 3-4: ДТ < 0, MJ < 0; V = const, А' 0; 4-1: ДТ < 0, Д[/ < 0; ДУ < 0, А' < 0. Из соотношения (1) получаем: Qi-г > 0, Qt-з > 0, Qa-i < 0, Qt 1 < 0. Хотелось бы предостеречь читателя от чрезмерно упрощенных рассуждений: «когда газ нагревается, он получает теплоту, когда охлаждается — отдает». Для рассматриваемых здесь изобарных и изохорных процессов это действительно так, но, вообще говоря, бывает и иначе (см., например, задачу 11.25). Для ответа на последний вопрос воспользуемся первым законом термодинамики в применении ко всему циклическому процессу. При этом Д[7 = = 0 (газ возвращается в исходное состояние) и Q = А'. Газ совершает работу только на этапах 2-3 и 4-1, причем (А')г-з = p^Va - У2) и A\-i = -р1(У3 - Vt). Значит, А' = А'2з + A'4-i = (р2 - рО(Уз - У2) > 0 (по- лезно иметь в виду, что А' численно равна площади внутри графика циклического процесса в координатах р,У). Итак, Q = =А' > 0. Таким образом, газ получает большее количество теплоты, чём отдает, — ведь иначе он не смог бы совершать работу. 11.20. Ответ: см. рисунок; воздух получает тепло на этапах 1-2-3-4; отдает тепло на этапе 4-1. Указание. Gm. решение задачи 11.19. ,
381 Молекулярная физика 11.21. Ответ: большее количество теп- лоты газ получит во втором случае. Решение. Очевидно, в первом случае \ газ испытывает изохорное нагрева- t, V ние, во втором же случае нагревание происходит изобарно. В обоих случа- „ 1 4 ях изменение температуры ДТ одно и то же. Поскольку внутренняя энер- гия идеального газа зависит только V от его температуры, изменение внут- к заЭаче 1120 ренней энергии ДС7 в обоих случаях одинаково. При изохорном процессе газ, естественно, не совершает работы; при изобарном — газ расширяется, совершая положительную работу А'. Согласно первому закону термодинамики в первом случае Qi = ДС7, а во втором случае Qz = AU + А'. Итак, Qz > Qi, т. е. изобарное нагревание требует сообщения газу большего количества теплоты. Это неудивительно: такое нагревание сопровождается расширением газа и совершением работы. 11.22. Решение. Из определения молярной теплоемкости С следует: уДТ’ где v — количество вещества. При изохорном нагревании газ не совершает работы, поэтому согласно первому закону термоди- намики Qv = ДС7. Следовательно, Cv = При изобарном нагре- vAT вании QP — AU + А', причем работа газа А' = pAV. Как следует из уравнения Менделеева-Клапейрона, рДV - vRAT, поэтому QP = ДС7 + AU + vRAT и С„ =----+ R. Отсюда и следует соотношение СР - Cv = R. vAT 11.23. Ответ: например, кислород. Решение. Воспользуемся соотношением для молярных теплоем- костей газа Ср - Cv = R (см. задачу 11.22). Из заданных в условии ( Qv величин определим удельные теплоемкости газа 1сР = —-------; Qv \ м Cv = —— I и воспользуемся связью между молярной и удельной теплоемкостями С = с№, где М — молярная масса газа. Из
Ответы, указания, решения 382 RmAT приведенных соотношении следует: М =------— = 0,032 кг/моль. Qi> - Qv Как раз такую молярную массу имеет кислород. з 11.24. Ответ: Cv = ~^R = 12,5 Дж/(моль>К); лл к С„ = -zR = 21 Дж/(моль • К); у = 1,67. £л Решение. При постоянном объеме Cv - (см. решение задачи 11.22) . Для одноатомного идеального газа, как известно, U — 3 3 = -^vRT, поэтому Cv = ~R -- 12,5 Дж/(моль-К). Отсюда СР = С + £л £ 5 5 + R = ~^R = 21 Дж/(моль*К) иу = т= 1,67. Разность между СР и £4 О Cv обусловлена работой, совершаемой газом при изобарном нагре- вании. Жидкость при изобарном нагревании тоже расширяется, но очень слабо, поэтому СР - С <с Cv; этой разностью можно пренеб- С, речь и считать 77' = 1. В справочных таблицах для удельной Cv теплоемкости газа обычно оговаривается, для какого процесса указана теплоемкость газа (например, при постоянном давлении). Для жидкостей и твердых тел такие оговорки не делаются. 11.25 . Решение. На этот вопрос часто дают утвердительный ответ, ссылаясь на известную формулу Q = стЛТ, где с — удельная теплоемкость вещества. Разумеется, эта формула применима и к газам, но для газов (см. задачи 11.21, 11.22) величина с может быть совершенно различной при разных процессах. Может быть, существует процесс, для которого с < 0? Запишем первый закон термодинамики в виде AU = Q - А’ и учтем, что для любого иде- ального газа внутренняя энергия U прямо пропорциональна абсо- лютной температуре Т. Следовательно, вопрос можно поставить так: обязательно ли AU > 0, если Q > 0? Теперь ясно, что все зависит от величины А' — работы, совершенной газом. Из первого закона термодинамики следует, что если А' достаточно велика (газ быстро расширяется), ответ па поставленный вопрос может быть и отрицательным. Подтвердим сказанное примерами: 1. При изотермическом расширении Q > Q, А! > 0, А Т = 0 (т. к. Т == const). Следовательно, в этом процессе с -> <ю! 2. При адиабатном расширении или сжатии Q = 0, А Т * 0, т. е. с = 0.
383 у Молекулярная физика 3. Если открыть вентиль баллона, содержащего газ при очень высоком давлении, газ начнет быстро расширяться. Температура его при этом может настолько понизиться, что на баллоне поя- вится иней (даже летом). Значит, в ходе такого процесса газ получает тепло от окружающей среды, но охлаждается (т. е. Q > О, А Т < 0 ). Следовательно, в этом случае с < О. 11.26 . Ответ: в процессе 3-2-4-3. Решение. В этой задаче можно обойтись без расчетов, сделав ответ геометрически очевидным. Работа газа А' имеет простой гео- метрический смысл: поскольку при малом изменении объема &V работа АА' = pAV, где р — давление газа, то А' при расширении от Vi др V2 численно равна площади криволинейной трапеции под графиком p(V) (см. рис. а). При циклическом процессе возрастание объема сменяется его убыванием, при сжатии работа А' < О. На рис. б, например, вертикальной штриховкой отмечена площадь, равная работе A'i при расширении газа; горизонтальной штрихов- кой — площадь, равная |А'2|, где А'г <0 — работа газа при сжатии. Тогда полная работа газа за цикл А' = А/ + Аг - Ai'-|A2'| представ- ляет собой просто площадь внутри графика циклического процес- са в координатах р, V; причем А' > 0, если цикл «проходится» по часовой стрелке, и А' < 0, если против часовой стрелки. Чтобы воспользоваться этим результатом, следует построить графики заданных процессов в координатах р, V (см. рис. в). Поскольку гипербола (участок графика 2-3) делит прямоугольник на две неравные части, получаем А!змз > A'imi. 11.27 . Ответ: А' = уВ(^Тз - где v = 1 моль. Решение. Температура в состояниях 2 и 4 (лежащих на одной изотерме) Тг - Т* = 'iTiTa (см. задачу 9.32). Искомая работа А' равна площади прямоугольника (см. рисунок к условию): А' = (ра ~ pi)(Va — V)) = раУз — piVs — paVi + piVi.
Ответы, указания, решения 384 Учитывая, что рг = рз, Vs = У», pi = Vi = V2, получим: А' = psV3 - рМ - P2V2 + piVi. Для любого из состояний газа рУ = vRT., где v = 1 моль; i = 1, 2, 3, 4. Поэтому А' = vR(Ts -Т<-Тг + Ti) = vR(Tt + Ti - 2^7\гГ) = - <Т\)2. 11.28. Ответ: в случае б конечная температура и работа при сжатии больше. Решение. Очевидно, в случае а сжатие происходит медленнее. Как может ско- рость сжатия повлиять на конечную тем- пературу? При сжатии газ получает энергию за счет работы А, совершенной внешними силами. Если сжатие доста- точно быстрое, то теплообмен «не поспе- вает» за этим процессом, вследствие чего процесс можно считать адиабатным (Q = = 0). Поэтому газ нагревается (ДС7 =А> 0). При медленном сжатии теплообмен обеспечивает равенство температур газа и окружаю- щей среды (процесс изотермический). Поскольку при быстром сжатии газ нагревается, его давление растет быстрее, чем при медленном сжатии. Другими словами, в координатах р, V график адиабаты имеет большую крутизну, чем график изотермы. Поэто- му работа при адиабатном сжатии от Vi до V2 (см. рисунок) больше, чем при изотермическом. Разность работ численно равна за- штрихованной на графике площади. 11 .29. Ответ: Q = 7,9 кДж. Решение. Можно просто воспользоваться формулами Q = cvmAT и m = vM, где сР — удельная теплоемкость кислорода при постоян- ном давлении, М — молярная масса кислорода. При изобарном процессе объем прямо пропорционален абсолютной температуре. Поэтому Т2 = 2Ti и ДГ = Ti = 273 К. Таким образом, Q = vc^MTi = = 7,9 кДж. Читатели, знакомые с общими выражениями для теплоемкости идеального газа, могут найти более короткое реше- ние, не требующее даже табличных сведений: Q = vCv\T = vCvTi = 7 7 = -^vRTi - 7,9 кДж. Здесь С» = -^R — молярная теплоемкость двух- атомного идеального газа при постоянном давлении. 11.30. Ответ: Q = 126 Дж. Решение. Очевидно, при закрепленном поршне газ не совершает работы и потому Qi — AU. В случае, когда поршень не закреплен,
385 Молекулярная физика газ расширяется изобарно. При этом изменение его внутренней энергии такое же: ведь внутренняя энергия идеального газа зави- сит только от его температуры. Работа газа при расширении А' = p&V, так что Q = ДС7 + А’ = Qi + рАУ. Учтем также, что mg ДУ ДТ . , Л р - —=г + р. и -ту- = -=- (т. к. при изобарном процессе объем прямо О V о 1 о пропорционален абсолютной температуре газа). Тогда Q = Qi + ,И^ = 1г63ж. U1 о 11.31. Ответ: 1,25/х>Уо; при охлаждении газ получает количество теплоты, равное О,25роУо. Решение. Согласно результату реше- ния задачи 9.34 газ сначала нагрева- _ 2р»Уо ется от температуры =-------до тем- 2р0 vR g пературы Г» = gTi (точка 3 на рисуй- те 3TZ 3 . Ро ке с координатами V3 - -^Vo ирз = TjP0), а затем охлаждается от Тз до Тг = Тъ На обоих этапах (1-3 и 3-2) газ, рас- q ширяясь, совершает работу. Работу на каждом этапе нетрудно определить как площадь соответству- ющей трапеции: . , 2ро+1,5ро V-, п ___ „ А,/ = „ ----= О,875роУо; Z а Аз-з' = 1,5jVP° • = 0,625роУо. Z а Изменение внутренней энергии одноатомного газа задается 3 выражением ДЕ7 = -=vRAT, значит, А ДСЛ-з = 1уВ(Тз - Ti) = -^vRTi = fpoVo; А 1О О 3 3 ДСЛ-а = — Тз) = — ’spoVo. а О Мы воспользовались здесь уравнением Менделеева-Клапейрона 2роУо = vRTi. Остается лишь применить первый закон термо- динамики: Qi-з = ДС71-з + Аз-» = 1,25д>Уо; Q»-» = AU»-i + А»-г = 0,25роУ>. Нетрудно проверить, что Qi-a = Qw + = Ai-i'. Ведь полное
Ответы, указания, решения 386 изменение внутренней энергии в ходе процесса равно нулю (точки 1и 2 лежат на одной изотерме). Обратите внимание: газ получает теплоту при охлаждении! Ничего удивительного в этом нет: просто газ расширяется настоль- ко быстро, что подводимой к нему теплоты недостаточно для совершения работы и газу «приходится» отдавать часть своей внутренней энергии. Заметим, что теплоемкость газа в этом про- цессе отрицательна (см. задачу 11.25). 11.32. Ответ: Т = То + 3vR Решение. При сближении поршней происходит адиабатное сжатие газа. Оно, как известно, сопровождается нагреванием. Макси- мальная температура достигается при максимальном сжатии, т. е. в момент, когда поршни перестают сближаться. В этот момент их скорости и одинаковы и могут быть определены из закона сохра- нения импульса: Зти + ти — 2 ти, откуда и = 2п. При адиабатном сжатии сумма кинетической энергии поршней и внутренней энер- 3 гии газа (С7 = ^vBT) остается постоянной: m(3v)2 mi? 3 __ , zn(2v)2 3 „ _l_l + __ + _v№ = 2. + -vRT. Отсюда T = To + 3vB Ns 11.33. Ответ: m =----- 0,22 кг (здесь Т| = 0,3; q — удельная W теплота сгорания бензина). 4FI 11.34. Ответ: т =---- 42 т (здесь п = 0,25, q — удельная теплота П<7 сгорания керосина). 11.35. Решение. Тепло, отбираемое у воздуха и продуктов внутри холодильного шкафа, может быть передано только воздуху в той же комнате. Энергия же, потребляемая холодильником из сети, также превращается во внутреннюю и приводит к повышению температуры в комнате. Так что температура в комнате в конеч- ном счете повысится! Правда, при работающем холодильнике температура в разных местах комнаты будет неодинаковой. Она будет Несколько ниже у раскрытой дверцы и выше у задней стенки холодильника^ тде расположен теплообменник и происходит ос- новное выделение тепла (попробуйте наощупь трубки теплооб- менника!). L
337 Молекул^яямг физакр 11.36. Ответ: Р = 6,3 Вт. Решение. Минимальную мощность будет потреблять холодиль- ник, представляющий собой обратимую тепловую машину, кото- * рая работает по обращенному циклу. При работе по прямому циклу КПД такой машины = А = А = - г» Qi А + Qi Pt Здесь А — совершаемая работа, Qi — количество теплоты, получаемой от нагревателя (воздуха в комнате), Q2 — количество теплоты, передаваемой холодильнику. При работе по обращенно- му циклу А —Рт, a Q2 — Q (холодильник должен отдавать такое же количество теплоты, какое он получает через стенки от окружаю- -• Т2) щего воздуха). Поэтому Р =---------= 6,3 Вт. 1Т2 В действительности современные домашние холодильники пот- ребляют мощность на порядок большую. Дело в том, что цикл работы холодильного агрегата весьма далек от обратимого. 11.37. Решение. Разумеется, никакого противоречия с законом сохранения энергии здесь нет: если за один цикл работы тепловой насос потребляет из сети энергию W и передает воздуху в комнате количество теплоты Q > W (при этом коэффициент полезного Q действия Т|т„ = рр; > 1), то недостающее количество теплоты Q' = = Q - W поступает от наружного воздуха. Так может действовать тепловой двигатель, запущенный «в обратную сторону»: отби- рающий тепло у менее нагретого тела (наружного воздуха с температурой Т’) и передающий тепло более нагретому телу (воз- духу в комнате с температурой Т > Т'). Если «обратить» работу теплового насоса, работающего по обратимому циклу, и заставить его работать как обычный тепловой двигатель, то его КПД W Т-Т' , , Цтд - 7. - —=— < 1 (из обратимости следует, что за один цикл * IT совершается полезная работа W). Тогда Ц™ = — =-— > 1. Оче- Цтд Т Т' видно, чем меньше разность температур Т - Т', тем меньше энер- гии потребуется для отоцления. Так, при t = 2GPC и t' = 1(FC величина W составляет лишь 3,4% от величины Q; при t = 2(FC И t в -20°С эта доля составляет уже 14%. На самом деле, конечно, |аи один тепловой насос не работает по обратимому циклу. Тем не |йенее экономия энергии получается очень большой. Широкому
Ответы, указания, решения 388 отобранного у со- - 0°С) 4- тХ), и распространению тепловых насосов мешает пока их большая стоимость по сравнению с нагревательными приборами. 11.38. Ответ: Q = Рт + m(ct + X), где с — удельная теплоемкость воды, X — удельная теплота плавления льда. Указание. Из закона сохранения энергии следует, что количество теплоты Q равно сумме количества теплоты, держимого холодильной камеры (|QB| = mc(t энергии, полученной от сети (W = Рт). 11.39. Ответ: t = 65°С. __ — Qb Решение. Согласно определению Т| = где яр* теплоты, полученное водой; Q* — количество теплоты, выделив- шейся при сгорании метана. Для Qb получаем Л ч npd2vc(t - (о)т Q. = m,c(t - to) =--------- Qa — количество 4 где тв — масса воды, протекающей через колонку за время т = 1 ч; р — плотность воды; с — ее удельная теплоемкость. С другой стороны, Qm = qm„, где масса метана т, определяется из уравнения Менделеева-Клапейрона pVa = ~~RTa (здесь М = 0,016 кг/моль). Отсюда t = to + = 65°С. nRTopdvci 11.40. Ответ: 17%. А' Решение. По определению КПД цикла ц - где А' — совершен- ье ная газом за цикл работа; QB — количество теплоты, полученной за цикл от нагревателя. Работа А' легко определяется — она численно равна площади внутри графика циклического процесса в координатах р, V (см. решение задачи 11,26). В данном случае А' = 2роУо (это площадь прямоугольника 1-2-3-4). Для определения QB необходимо сначала выяснить, на каких этапах процесса газ получал тепло от нагревателя. Для данного цикла это этапы 1-2 и 2-3 (см. решение задачи 11.19>. Поэтому Qb = Qi-2 + Q2-3. Для нахождения этих величин необходимо опре- делить соотношение между температурами газа в состояниях 1, 2 и 3. Воспользовавшись уравнением Клапейрона = 11 = по л учим: Тз = 2Ti, Т» = 6Ть Используя результат решения задачи ! 1.24,, можно записать:
389 Молекулярная физика о Qi-2 = vCv^Tt — Ti) — -^vRTi, 5 Qz-з = vC„(T3 - T2) = -xvR 4Ti = IOvBTi. 4Ы Здесь v — количество вещества. Подставив полученные значения A', Q1-2, Qz-з в формулу для КПД, получим: т| = —. Применяя уравнение Менделеева- 11 ,ovRTt Клапейрона poV\> — vRTi, находим: т| = 4 23 « 0,17. 11.41. Ответ: 15%. , где А' — работа газа за цикл; QH — Решение. КПД цикла Т| = ТГ количество теплоты, полученное от нагревателя за цикл. Работа газа численно равна площади треугольника 1-2-3, поэтому А. 3Vo • Зро 9роРо А = 2 = 2 • Газ получает тепло только на этапе 1-2: здесь происходит расширение газа (т. е. А'1-2 > 0) и его нагревание (т. е. ДСЛа > 0), так что Q1-2 = AtZi-2 + А'1-2 > 0. На двух других этапах газ тепло отдает (см. задачу 11.19). Работа А'1-2 вычисляется как площадь соответствующей трапеции: _ ро + 4ро _ 15роУо А1-2 - 2 ЗУ» 2 • Изменение внутренней энергии газа на этапе 1-2 составит: ДС71-2 = |vB(T2 - Ti) = |(4р« • 4Ро - p»Vo) = Следовательно, Qo = Q1-2 = ЗОроУо и ц = 0,15. 11.42. Ответ: Г| = щ + Г|2 ~ тццг. Решение. КПД любого цикла можно записать в виде QlUtrp Qxo^ ~ Q ’ <cuarp Здесь Qn»n> — количество теплоты, полученное рабочим телом от нагревателя; — количество теплоты, отданное рабочим телом холодильнику. В данном случае в цикле 1-2-3-4-1 рабочее тело получает тепло в процессах 1-2 и 2-3, а отдает в процессах 3-4 и 4-1 (iQsul = Qus). Формулы для КПД циклов принимают вид . Q123 — Q13 Q13 Q133 Q123 — Q142 Л* ~ Qiis ,т1а" ^Q« - ? Л ~ QtM- .-’Ч • г.-ч
Ответы, указания, решения 390 Qis Из первых двух формул находим Q1Z.3 = ---------, Qua - (1 - T12)Q13. 1 - Т]1 Следовательно, л = 1 - (1 - v|i)(l - т|2) = r]i + гр - Т|1Т]2. 12.1. Ответ: да, зависит; для разноименных зарядов сила взаимо- действия больше. Решение. Казалось бы, согласно закону Кулона величина силы не должна зависеть от знаков зарядов. Но вспомните: в законе Кулона говорится о точечных зарядах! Выполняется ли этот закон для заряженных металлических шаров? Следует учесть, что сво- бодные заряды в проводниках подвижны, они не «привязаны» к определенному месту. Характерное распределение зарядов для разноименно заряженных шаров показано на рис. а, а для одно- Рис. а Рис. б именно заряженных шаров — на рис. б. Очевидно, что в первом случае заряды под действием взаимного притяжения будут распо- ложены ближе друг к другу, и поэтому сила их взаимодействия (притяжение) будет больше, чем во втором случае (отталкивание). Разумеется, перераспределение зарядов на шарах влияет на вели- чину силы их взаимодействия лишь тогда, когда расстояние между шарами не слишком велико по сравнению с их радиусами. Если же шары находятся на достаточно больших расстояниях, то их можно рассматривать как точечные заряды и применять закон Кулона. 12.2. Ответ: да, могут. Решение. На первый взгляд, это противоречило бы закону Кулона. Но вспомним, что незаряженное тело притягивается к заряжен- ному (например, кусочки бумаги к наэлектризованной расческе), а это тоже формально не соответствует закону Кулона. Объясня- ется такое притяжение перераспределением зарядов внутри тела (см. рис. а). Но тогда этот же эффект может привести к притя- Рис.б ч
391Л Электричество и магнетизм жению одноименно заряженных тел: «ближняя» сторона одного из тел может изменить знак заряда (см. рис. б). Такое возможно, если тела находятся достаточно близко друг к другу и заряд одного Из тел во много раз превышает заряд другого. В этом случае у тела с меньшим зарядом описанное перераспределение может привести к появлению заряда противоположного знака. 12.3. Ответ: увеличится за счет поляризации диэлектрика (стекла). 12.4. Ответ: заряды могли быть одноименными и отличаться в 3 раза либо разноименными и отличаться по модулю в 7,2 раза. Решение. Поскольку шарики одинаковы, после соприкосновения <Ji + qz Л каждый из них имеет один и тот же заряд q = —Определим согласно закону Кулона силы их взаимодействия до и после „ . ЫЫ „ , (gi + g2)2 соприкосновения соответственно: F = k—и nF = k—jp—, где г — расстояние между шариками. Отсюда (gi + g2)2 .= 4n|<yi|[Q2|. Обоз- начим — = х и разделим последнее равенство на g22 = |д2|2. По- д2 лучим: (х + I)2 = 4п|х| или х2 + 2(х - 2п|х|)+1 = 0. Предполагая, что х > 0 (т. е. |х| = х), получим xi,2 = 2п - 1 ± -V(2n - I)2 - 1, т. е. Xi = 3, х2 = Оба решения, естественно, означают одно и то же: одноименные заряды на шариках отличаются в 3 раза. Предполагая, что х < 0 (т. е. |х| = - х), придем к решениям х, = - |(11 + V112) = -7,2; х2 = —. О Xi Оба этц решения означают, что заряды qi и дг могли быть разноименными, отличающимися по модулю в 7,2 раза. 12.5. Ответ: не может. Решение. В положении устойчивого равно- весия действующая на тело сила должна равняться нулю; при небольшом же откло- нении в любом направлении от этого поло- жения возникает сила, возвращающая те- ло в положение равновесия. Если, к при- меру, точечный положительный заряд в точке А находится в положении устой- чивого равновесия, то силевые линии поля вблизи этой то’цси имеют вид, показанный на рисунке. Тогда в
Ответы, указания, решения 392 точке А должен находиться отрицательный заряд, на котором эти линии заканчивались бы, а такого заряда там нет. Поэтому устойчивое равновесие заряда в электростатическом поле невоз- можно (это утверждение называют теоремой Иршпоу). 12.6. Ответ: заряд до = -4g нужно расположить на расстоянии а от заряда д и на расстоянии 2а от заряда -4g. Решение. Определим снача- д0 д ~4д ла, где следует расположить ~ ~ ~ 2~ заряд до, чтобы он находил- г* *т* Н ся в равновесии. Очевидно, это точка, в которой равна нулю напряженность Е поля двух первых зарядов: Е = Ei + Ег = О. Такая точка может лежать только на соединяющей заряды 1 и 2 прямой, причем вне отрезка 1-2 (в точках на этом отрезке Ei и Ег направ- лены в одну сторону). Может ли искомая точка находиться правее заряда 2? Нет, поскольку тогда она будет лежать ближе к большему (по модулю) заряду, а там Ei < Ег и Е = Ei + Ег * О. Итак, искомая точка расположена левее меньшего заряда, на расстоянии х от него (см. рисунок). Из условия Е = О следует, что Ei = Ег, т. е. ft—— - ftM ОТКуДа (а + х)2 = 4х2, или а + х = 2х (мы учли, что (а + х)2 х х > О). Таким образом, х = а. Для определения величины до достаточно заметить, что заряды до и -4д расположены сим- метрично относительно заряда д. Поэтому напряженность поля зарядов 2 и 3 в точке 1 равна нулю только при до = -4д. Итак, следует разместить заряд до = — 4д на расстоянии а от заряда д. Равновесие системы зарядов будет, естественно, неустойчивым (см. задачу 12.5). 4 12.7. Ответ: нужно расположить заряд - ^д на соединяющем первые два заряда отрезке, на расстоянии а/3 от заряда д. 12.8. Ответ: Q = - Решение. Очевидно, заряды g и Q должны быть разноименными. Для равновесия достаточно, чтобы равня- лась нулю равнодействующая сил, при- ложенных к одному из зарядов g (см. рисунок): Ft + F2 + F3 = О. Проецируя это условие равновесия на вертика- льную ось и используя закон Кулона
393 Электричество и магнетизм. (Fi - Fi = Fs = k ), получим 2Л^ cos30° = k \ a2 (_a_J y a2 ,_aj2 Отсюда |Q| = т. е. Q = - ^=. — В D С 12.9. Ответ: Q = - |(1 + 2^2); равновесие неустойчиво. Указание. См. задачи 12.5, 12.8. 12.10. Решение. Направление силовых линий совпадает с направ- лением действующей на заряд силы, а, значит, с направлением ускорения заряженной частицы. Направление же скорости при криволинейном движении не всегда совпадает с направлением ускорения: например, в однородном поле внутри конденсатора электрон может двигаться по параболе, а в поле точечного заряда — по окружности. 12.11. Ответ: нельзя. Решение. Такое поле не является по- тенциальным: можно выбрать замкну- тую траекторию, при перемещении за- ряда вдоль которой поле совершает от- личную от нуля работу. Рассмотрим, например, траекторию ABCDA (см. ри- сунок). На участках ВС и DA поле не совершает работы (сила перпендикулярна перемещению). На участках АВ и CD (АВ = CD) работа поля имеет разные знаки и модули (поскольку густота линий напряженности вблизи CD больше, там выше напряжен- ность поля и больше действующая на заряд сила). Следовательно, работа при перемещении по замкнутой траектории получается отличной от нуля. В электростатическом же поле такого быть не может: это поле потенциально. 12.12. Ответ: в случаях а и б шарик будет втягиваться в область более сильного поля, в случае в останется неподвижным. Решение. Когда шарик попадает в элек- трическое поле, в нем происходит раз- деление зарядов (в диэлектрике — по- ляризация, а в проводнике — электро- статическая индукция). В результате справа на поверхности появ- ляется положительный заряд q, а слева — равный ему по модулю отрицательный заряд. Следовательно, в поле на шарик действует сила F = F+ + F- = q(E+ -I- Е-) (на рисунке в качестве примера
Ответы, указания, решения 394 показан случай а). Здесь £+ и £. — напряженности поля в области концентрации положительного и отрицательного зарядов соответ- ственно. В случае а (справа линии напряженности сгущаются) £+ > £-, поэтому сила F направлена вправо. Аналогично можно показать, что в случае б сила F направлена влево. В обоих случаях шарик втягивается в область более сильного поля (например, притягивается к создающему поле заряду). В случае в, когда поле однородно, F = О, т. е. шарик останется неподвижным. 12.13. Ответ: Е = а 4леоа Решение. Указанная точка представляет собой вершину пра- вильного тетраэдра, в вершинах основания которого находятся заряды. Высота тетраэдра является осью симметрии системы. Поэтому напряженность £ направлена вдоль высоты тетраэдра (см. рис. а). Согласно принципу суперпозиции £ = £i + Ег + £з, (1) причем £i = Ег = £з = k^. Проецируя (1) на ось у, параллельную высоте тетраэдра (и учитывая, что £Iy = Е* - £*), получаем Е = 3Ei7. На рис. б показано сечение тетраэдра вертикальной плос- костью, проходящей через высоту и одну из вершин основания. pi h Из подобия заштрихованных треугольников получаем = —; из £1 а прямоугольного треугольника АВО находим Отсюда £>, следовательно, £ =?
395 Электричество и магнетизм 12.14. Решение. Свободные элек- троны сместились в сторону, про- тивоположную направлению по- ля (см. рисунок). Теперь поверх- ностная плотность заряда макси- мальна (по модулю) в точках А и С, плавно уменьшается при уда- лении от них и обращается в нуль в точках В и Л. На достаточно больших расстояниях от шара по- ле остается однородным. Вблизи шара линии напряженности ис- кривляются. Они входят в поверхность шара (перпендикулярно ей!) на участке BCD и обрываются; на участке BAD они выходят из шара. Эквипотенциальные поверхности перпендикулярны линиям на- пряженности. Положение трех эквипотенциальных поверхностей очевидно: это две пластины конденсатора и поверхность шара. Учитывая эти обстоятельства, получаем картину, показанную на рисунке. 12.15. Ответ: напряженность поля увеличится, потенциал умень- шится. Решение. Под действием электрического поля в шаре произойдет разделение зарядов (поляризация в диэлектрическом шаре или электростатическая индукция в проводнике). На ближней к точке А стороне шара возникнет отрицательный заряд, на дальней — такой же величины положительный. Поскольку отрицательный заряд находится ближе к точке А, его влияние является опреде- ляющим. Поэтому вопрос можно сформулировать так: как изме- нятся Е и <р, если справа от точки А поместить отрицательный заряд? Этот заряд создаст в точке А поле, направленное слева направо (туда же, куда и поле заряда +q), и отрицательный потенциал (напомним, что потенциал поля заряда +q положи- телен). На основании принципа суперпозиции можно сделать вывод: напряженность поля увеличится, потенциал уменьшится. 12.16. Ответ: Aq = 1,6 • 1€Г,в Кл. Решение. Условие равновесия пылинки имеет вид mg — А, где _ _ q<Uo _ „ mgd, . Fy = qoE - Отсюда начальный заряд пылинки q0 = ТТ....... Ана- логично находим ее конечный заряд: q = — Следовательно,
Ответы, указания, решения 396 Ag = go - q = mgdAU UdQJo + AU) 1,6 1(Г1в Кл. Другими словами, при облучении пылинка потеряла один электрон. В задаче описан классический опыт Иоффе-Милликена, доказывающий дискретность электрического заряда. 12.17. Ответ: р = 1500 кг/м3. Решение. На рис. а показаны силы, действующие на один из шариков в воздухе; на рис. б — в керосине. Хотя заряды шариков могут отличаться друг от друга по величине, согласно третьему закону Ньютона на оба шарика действуют одинаковые по модулю кулоновские силы. Поэтому оба шарика расположены симмет- рично относительно вертикальной оси, проходящей через точку подвеса. Из условий равновесия следует, что кулоновская сила в воздухе Гк = а в керосине Fk' = {mg - где Fa — сила А А Архимеда. В керосине кулоновская сила при том же расстоянии между шариками в е раз меньше: Fk = — (здесь е — диэлектричес- 8 mg кая проницаемость керосина). Таким образом, mg - Fa =-. Под- е ставив в эту формулу т = pV и Fa = pngV (здесь рк — плотность керосина), получим р = рк- 1500 кг/м3. 12.18. Ответ: Ь — 6,0 см. Решение. Что произойдет с шариками после разрядки одного из них? Отталкивание сменится притяжением, шарики соприкоснут- ся. При этом заряд q одного из них поделится между шариками пополам. В результате опять возникнет отталкивание, но слабее первоначального; поэтому после затухания колебаний шарики
397 Электричество и магнетизм разойдутся на расстояние Ь < а . Чтобы найти Ь, нужно выразить его через заряды шариков. С одной стороны, сила кулоновского отталкивания (см. задачу 12.17) Fk = с другой стороны, ее 2 величина (до разрядки) Fk = k^, где расстояние а связано с длиной 01 I нити соотношением а = 2Zsin£-. Чтобы выразить теперь а через Ct q, воспользуемся условием а « I (т. е. а « 1 рад). Считая tg^ — • a ,q2 a (2klq\i q = sm-n, получаем k2^ - mg-^, откуда а = I—— b. Заменяя q на (klq2u а получаем b = Н—-1» - — = 6,0 см. \&mg' 4з 12.19. Ответ: q = 13,5 нКл. Решение. Каждый из шариков отклонен от центра треугольника на расстояние Отсюда следует (см. рис. а), что на шарик действует горизонтальная отклоняющая сила F = mgtga » (1) Рис. а а Рис. б Мы использовали тот факт, что а < I, т. е. а < 1 рад. Сила F (см. рис. б) является равнодействующей двух кулоновских сил о2 Fki и Fk», одинаковых по модулю (Fki = Fka = Л^). Из параллело- грамма сил следует F = >/ЗдХ. (2)
Ответы, указания, решения 398 Из формул (1) и (2) получаем > «*ms=1з-ъ у Щп ol 12.20. Ответ: на 0,25 нм. Решение. При сближении электронов их скорости уменьшаются в результате кулоновского отталкивания. В какой-то момент электроны останавливаются, а затем начинают все быстрее уда- ляться друг от друга. Минимальное расстояние rmin достигается именно в момент остановки электронов. К этому моменту вся их начальная кинетическая энергия переходит в потенциальную _ mvo2 е2 энергию кулоновского отталкивания, т. е. 2 —х— = к—. Отсюда £ Гmin fee2 е2 r„,,„ = = —----- = 2,5 • 10 10 м. ТПРо 4лЕо7ПРо 12.21. Ответ: электроны сблизятся на расстояние rmi„ =-—в ЛЕоТПРо результате взаимодействия они просто обменяются скоростями. Решение 1. Обозначим скорость первого электрона через pi (она увеличивается вследствие отталкивания электронов), а скорость второго — через Рг (она, наоборот, уменьшается). Сближение закончится в тот момент, когда скорости электронов сравняются, т. е. vi = Рг. Расстояние между электронами в этот момент можно //про2 mvi2 наити, пользуясь законами сохранения энергии I—т— = —z— + ' Л £ тигг .е2 \ . + —+ k—I и импульса (тио = mvi + тпрг) при условии Vi = Рг. м Л*niin* „ „ Ро 4/ге2 е2 Из этих соотношений находим Pi = р2 = -^ и rmi„ =--= =-----. 2 mvo лЕопгРо После сближения электроны опять разлетятся на большое расстояние, где потенциальной энергией их взаимодействия мож- но пренебречь. Формулы законов сохранения энергии и импульса в применении к этому случаю имеют такой же вид, как для упругого лобового удара одинаковых шаров (см. задачу 4.36). Значит, произойдет обмен скоростями: налетающий электрон ос- тановится, а покоившийся приобретет скорость ро. Решещсе 2. Проще и красивее задача решается, если перейти .в „ ТИРо Ро _ систему отсчета Центра масс, скорость которой р =------= В т + т « « екороет.мк f Тогда можно воспользоваться результатом решения задачи 12.20,
399 Электричество и магнетизм ! . Оо в2 заменив vo на л • A*min “ -Очевидно, в системе отсчета центра 2 лБо/пио масс после упругого взаимодействия (не сопровождающегося поте- рями механической энергии) электрон!»! разлетаются с такими же Vo _ по модулю скоростями ~п, т. е. обмениваются скоростями. & ’2 □. ъ™2 A*min 2лео?П1>П2(Ц1 + Уз)2' + пи) • 2 2 /1 Отсюда Г„а„ = + 14 1 2-3 12 энергию системы 4 3-4 „ /1 2лЕот17П2(1>1 + Уг) V1 12.22. Ответ: rmi„ = (- +-----*----^-1 . 'Г qiqAmi + тг) ' Решение. В момент максимального сближения заряды будут иметь одну и ту же скорость у =-----------(см. задачу 12.21). 1огда mi + тг закон сохранения энергии примет вид: rniVt2 ШгУг2 ^дгдг _ (mi + ТПг)У' 2 г mim^Vi + Уг)2 \ 1 2kgiq2(mi + пи)* 19 94 -1). 5<72 12.23. Ответ:--------——-------. 12леоО 12л£оа Решение. Работу А следует вы- числять через изменение потен- циальной энергии Wp системы: А = AWP (напомним: А — работа внешних сил, поэтому здесь нет привычного знака «минус»). Потенциальную точечных зарядов можно представить как сумму энергий взаимо- действия всех пар зарядов: W„ = V k^^-. Для четырех зарядов 2-i Гор а.Р таких пар шесть (см. рисунок). При линейном расположении заря- дов ri-z = Ггя = Гя-4 = а, Ги = гг-4 = 2а, ri t = За. Поэтому начальная 13 q2 „ энергия системы Vv,fl = —k—. При размещении зарядов в вершинах о Л квадрата четыре расстояния в «парах» равны а и два (диагонали квадрата) равны ач2. Поэтому W,, = ^~(4 + у2), а A = WP- Wl<0 = Л/2 - |) = -^-(у/2 - К о о 4л8оа о При размещении зарядов в вершинах тетраэдра все расстояния о2 между зарядами равны а, следовательно, W„ = вк—; поэтому л.г»е. щвЕ амн ' .>
Ответы, указания, решения 400 12.24. Ответ: 5,7 • КГ12 Кл; 6,7 • КГ12 Кл. Решение. Чем ограничена величина заряда Q? Двумя факторами: отталкиванием электронов от уже заряженного шарика и возмож- ностью электрического пробоя разреженного воздуха, когда на- пряженность поля на поверхности шарика достигает Ео. Учтем, что заряд Q делится между шариками поровну (диэлектрическая проницаемость разреженного воздуха практически не отличается от единицы). Электроны перестают долетать до шарика в вакууме Л mvo2 ,e|Qi| . после накопления такого заряда Qi, что п = к ^=- (здесь т — масса электрона, е — элементарный заряд). Отсюда । । mvo2R 4momvo2R |Q1' ~ ke ~ e Электрический пробой наступает при накоплении заряда Q2, если Ео = АтШ, т. е. при |Q2| = = 8леоЯ2£0. Накопленный заряд |Q| не может превышать ни |Qi|, ни JQ2I, т. е. равен меньшей из величин |Qi| и |Qa|. При заданных числовых значениях |Qi| = 5,7 • 10'12Кл, |Qa| = 6,7 10“12Кл, так что |Q| = 5,7 • НТ12 Кл. Увеличение ио приведет к тому, что |Qi| станет больше |Qa]. При этом заряд возрастет лишь до значения |Q| = 6,7 • 12.25. Ответ: Е =----—-----<р =------------г 4ле0(Я2 + Л2)^ 4ле<>(2?2 + hfy Решение. Для определения £ и ф можно вос- пользоваться принципом суперпозиции. Разо- бьем кольцо на одинаковые малые участки, каждый из которых имеет заряд Ад. Каждый из таких участков можно рассматривать как точечный заряд, дающий свой вклад Ei и <р1 в £ и <р; т. е. £ = ^Ei, ф = ]Гф<. Будем считать 1012 Кл. q > 0 (см. рисунок). Обратим внимание на два обстоятельства: первое — поле £ из-за сим- метрии системы направлено вдоль оси у, поэто- му Е = ^ГЕ*; второе — точка А удалена от всех точек кольца на одинаковое расстояние г = (Я2 + Л2)^. Поскольку Et = a cosa = = — (см. рисунок), получаем £«, ~ Ех/оза.= При суммиро-
401 Электричество и магнетизм вании по различным участкам кольца все множители, стоящие в . этом выражении при Aq, являются постоянными и могут быть вынесены за знак суммы. Поскольку ^Ag = q, получаем г* 4тсео(2?2 + Л2)! _ . Aq При суммировании величин <р. = к— находим т _ ьд_______д г 4лео(2?2 + Л2)^ № Например, для центра кольца (Л = О) получаем Е = О, т. е. поля, создаваемые противоположными участками кольца, взаимно ком- пенсируются; <р = —-—. Заметим, что формулу (1) можно также 4яЕоЛ получить из формулы (2): Е„ = - т. е. Е равняется производной АЛ выражения (2) по переменной Л (с обратным знаком). 12.26. Ответ: v = (————г электрон не обязательно '2лЕот'л ЧК2 + Л27' пролетит через центр кольца. Решение. При решении удобнее всего вос- пользоваться законом сохранения энер- гии: = IV„+Wk, где Vv,fl = - е<р0 = - -------г — потен- 4ле0(Д2 + Л2)^ циальная энергия электрона в начальной тг/ eq точке, Wv = - -----— цотенциальная энер- 4леол ти2 гия электрона в центре кольца, W\ = —5— — кинетическая энергия А электрона в центре кольца, т — масса электрона, е — элементар- ный электрический заряд (е > О). Отсюда ^2леоЛ1^Я Vi?2 + Л2" Обязательно ли электрон пролетит через центр кольца? Каза- лось бы, симметрия начальных условий исключает любые откло- нения электрона от оси кольца. Однако устойчиво ли его дви- жение по оси кольца? На рисунке представлен вид электрического |йоля кольца (кольцо показано в разрезе). Похожее поле создают
Ответы, указания, решения 402 К&& одноименных точечных заряда. Если электрон, как показано на рисунке, случайно чуть сместится в сторону of оси, то дейст- вующая на него со стороны поля сила F «потянет» его еще дальше. Значит, движение вдоль оси неустойчиво. Если величина h не слишком мала, электрон почти наверняка не пролетит через точку О. 12.27. Ответ: Т = Решение. Рассмотрим малый элемент . кольца (см. рисунок), заряд которого Ж До - q~. Поскольку заряд кольца мал т 2л 1| » по сравнению с размещенным в цент- \ ре зарядом Q, можно пренебречь ку- \ / лоновским взаимодействием различ- них частей кольца. На рисунке пока- заны действующие на элемент коль- О ца силы. Согласно закону Кулона А = Из усло- R R 2л вия равновесия следует, что А = 2Tsin^ ® аТ (мы считаем а < 1, Ti = Тг = Т). Тогда Т = у- • 2л л 8л2еоЯ2 12.28. Ответ: шарик будет совершать гармонические колебания: х = Хосоз[(“———-Hi]; шарик притянется к кольцу. МлеотпЯ3' Решение. Согласно результату решения задачи 12.25, на расстоя- нии х от центра кольца на шарик действует сила F = qE = =-----—-------з. направленная к центру кольца. Поскольку R2 + 4лео(7?2 + х2)^ + х2 ® R2 при х « R, второй закон Ньютона для шарика принимает Qq п вид та, ------Это — уравнение гармонических колебании 4л£о/? с циклической частотой со = (—^2—-U. Максимальное отклонение ЧлеошЯ1' Хо о .’ положения равновесия достигается в начальный момент, так что ' X » ХоСОВ шГ 5= ХоСОВ[( . Если убрать спицу» проявится неустойчивость такого движения по отношению к малым боковым смещениям (см. задачу 12.26), поэтому шарик притянете#1« какой-нибудь точке кольца. -J
403 Электричество и магнетизм / 0(7 12.29. Ответ: при Ро > и™» =\ -—шарик уйдет на бесконеч- V 2тсеотл НОСТЬ; ПрИ Vo < Vmin — будет совершать колебания (при ио < Umin — гармонические) и в конце концов притянется к кольцу. Указание. См. задачи 12.26 и 12.28. Решение. Под действием электрического поля заряда +q в плас- тине произойдет разделение зарядов. На ближайшей к заряду поверхности пластины образуется отрицательный поверхностный заряд -q (на этой поверхности заканчиваются все силовые линии поля, выходящие из заряда +д). Поэтому Пластина притягивает заряд. Рис. а Рис. 6 Чтобы определить силу взаимодействия между точечным заря- дом и проводником, воспользуемся тем, что электростатическое поле не проникает через проводник. Поэтому нужно учитывать только силу взаимодействия между точечным зарядом +q и повер- хностным зарядом -q, наведенным на поверхности пластины. Следовательно, пластину можно заменить «металлическим» полу- пространством (см. область 1 на рис. а). Поле в этой области равно нулю, т. е. поля точечного и поверхностного зарядов в области 1 компенсируют друг друга. Это означает, что поле поверхностного заряда в области 1 совпадает с полем заряда -q, расположенного в той же точке, что и заряд +q. Но поле поверхностного заряда симметрично относительно плоскости! Значит, в области 2 оно совпадает с полем точечного заряда -q, расположенного сим- метрично заряду +q относительно поверхности (см. рис. б). Поэтому на заряд +q действует со стороны пластины точно такая Же сила, как и со стороны воображаемого заряда -q, т. е. Л* о4 F в h-j-J—-г as i. !..О.Н зяц..!
Ответы, указания, решения 404 Описанный выше метод решения задачи называется методом изображений: воображаемый заряд -д является как бы зеркаль- ным отражением заряда +q относительно поверхности пластины. Возможен и другой подход к решению. Рассмотрим поле двух разноименных точечных зарядов +д и -q, расположенных сим- метрично относительно некоторой плоскости на расстоянии Л от нее. Заметим теперь, что эта плоскость является эквипотенциаль- ной поверхностью. Значит, если совместить с ней тонкую прово- дящую пластину, то перераспределения заряда вдоль пластины не произойдет и поэтому картина поля не изменится. Если убрать теперь любой из зарядов, то поле с противоположной стороны пластины не изменится, поскольку проводящая пластина эк- ранирует электростатическое поле. Благодаря этому мы можем убрать заряд -д: поле со стороны заряда +q при этом не изменится. В общем случае метод изображений состоит в подборе таких дополнительных зарядов в «Зазеркалье», чтобы поверхности за- данных в условии проводников совпадали с эквипотенциальными поверхностями полученного поля. 12.31. Ответ: Е = —, при г < R; Е = - при г > 7?. График 4ПЕоЛ 4ЯЕоГ Е(г) см. на рисунке. Решение. Воспользуемся аналогией между законом Кулона и законом всемирного тяготения. При сфери- чески симметричном распределении заряда поле на расстоянии г от цен- тра создается только зарядом g(r) внутри сферы радиуса г (ср. с зада- чей 3.6). Поскольку заряд распреде- лен по шару равномерно, при г < R можно записать 9(г) г* Q Я8' Тогда ад ' 4лЕа/Г При г > R поле, создаваемое заряженным шаром, такое же, как поле точечного заряда Q, расположенного в центре шара, т. е. £(г> = А- • График зависимости Е(г) приведен на рисунке „ М . ) ' . 4яа>/г'
^05 Электричество и магнетизм /"VQ7?2 г2! О 12.32. Ответ: <р = ———- при г < Я; и> = ---при г > R. График 8n&JT 4л&>г ф(г) см. на рис. б. Решение. При г > R поле заряженного шара не отличается от поля Q Q точечного заряда, поэтому ф = k— =-----. При г — R потенциал . г 4леог Q фо =-----. Чтобы определить потенциал при г < R, подсчитаем 4тсеоЯ работу поля по перемещению заряда q из интересующей нас точки к поверхности шара. При малом перемещении ДА = qEkr. Значит, полная работа (см. рис. а) составит А = qSE. Площадь под графи- c. E(f) + E(R) __ . Qq(R2-^ „ ком Se = ' • (R- r) = —~Итак, A = C 2 ЗпеоЯ3 8neaR другой стороны, A = д(ф(г) - фо). Поэтому получаем График ф(г) приведен на рис. б. Первый участок графика (при г < R) — парабола, второй (при г > R) — гипербола. Q lol Q 12.33. Ответ: Е = 0, ф =---при г < R, Е = — ф =----------при 4лёоЯ АпеоГ 4лб0г г > R . Графики Е(}~) и ф(г) приведены на рис. а и б. Указание. См. задачи 12.31, 12.32. Учтите также, что заряд расположен только на поверхности шара. 12.34. Решение. Вид силовых линий показан на рисунке. Как известно, силовые линии начинаются на положительных зарядах. Под действием поля заряженного шара происходит поляризация Жэлектрика. На поверхностях диэлектрика появляются связан- ные заряды: на внешней поверхности — положительные, а на
Ответы, указания, решения 406 Рис. а к задаче 12.33 Рис. 6 к задаче 12.33 К задаче 12.34 внутренней (у поверхности металлического шара) — отрицатель- ные. Благодаря отрицательным зарядам’напряженность поля в диэлектрике уменьшается в е раз. Наличие сферического слоя диэлектрика не влияет на электрическое поле за пределами этого слоя. На рисунке показаны только свободные заряды. 12.35. Ответ: ю = —-—. 4ЛЕоГ Решение. Электрическое поле внутри сферы отсутствует, поэтому потенциалы всех находящихся здесь точек одинаковы и равны ф. Следовательно, достаточно определить потенциал одной точки — центра сферы. Этот потенциал, как следует из принципа супер- ст позиции, складывается из потенциала ф1 - k~ поля точечного заряда и суммы потенциалов полей всех зарядов Ад, образб- вавшихся на сфере: ф = ф, + = ф1 + Однако = 0’ (сфера в целом не заряжена), поэтому ф « ф1 = k— = . Сравните г 4леоГ этот результат С полученным при решении задачи 12,38 ДЛ*| сМуЖай г < " “ • * Rww*’7*4'1 '' < > «гтнаое
407 Электричество и магнетизм 12.36. Ответ: Q ~ - q —. Решение. Потенциал заземленного шара равен нулю. При разме- щении вблизи шара положительного заряда нулевой потенциал поддерживается за счет перетекания на шар отрицательного заря- да Q. Независимо от распределения заряда Q по поверхности (см. q Q задачу 12.35) потенциал центра шара ф = k— + k-^ (разумеется, остальные точки шара имеют тот же потенциал). Приравнивая ф R нулю, получаем Q = - q —. 12.37. Ответ: A q = 19 нКл. Указание. Перемещение заряда будет продолжаться, пока потен- циалы шаров не сравняются. Поэтому можно записать: к<Ь ~ М = k<h + М. jRi Яг Отсюда А? - = 19 нКл. 7?1 + Т?2 12.38. Решение. 1) На внутренней повер- i хности сферы индуцируется отрицатель- v т j ный заряд, а на внешней — равный ему положительный. Все силовые линии, вы- ходящие из заряженного шарика, закан- чиваются на внутренней поверхности сфе- f ры (внутри проводника поля нет), зна- | \/ чит, индуцируемые заряды по модулю ъСТ'У'уС* равны Q. На внутренней поверхности сфе- ры заряд распределяется неравномерно ' 1 (см. рисунок, на котором сфера условно ’ заменена сферическим слоем). Распределение заряда на внешней Поверхности определяется лишь собственным полем этого заряда (ведь поле заряженного шарика компенсируется полем заряда -Q на поверхности полости). Значит, заряд +Q распределится по внешней поверхности сферы равномерно. 2) Вид силовых линий показан на рисунке. Поскольку Поле вне сферы совпадает с полем точечного заряда +Q, расположенного в Q центре сферы, получаем ф =-----. 4леоЯ 3) Поле вне сферы имеет такой же вид, как поле заряда +Q, размещенного в ее центре. Поэтому на точечный заряд вне сферы » Qq ' j будет действовать сила отталкивания F = Интересно, что 47ГЕоГ2
Ответы, указания, решения 408 эта сила не изменяется при перемещениях шарика внутри сферы. Фактически заряд q взаимодействует не с шариком, а с зарядами, индуцированными на внешней поверхности сферы. 4) В результате заземления потенциал сферы обратится в нуль за счет стекания на землю заряда с внешней поверхности сферы. Поле вне сферы при этом исчезнет. Распределение зарядов и поле внутри сферы не изменятся. 12.39. Ответ: сфера имеет заряд -q, равномерно распределенный по внешней поверхности. Поле внутрь сферы отсутствует; вне сферы оно совпадает с полем точечного заряда -q, размещенного в центре сферы. Указание. См. задачу 12.38. 12.40. Ответ: ф = 450 В. Решение. Если бы шар не был заземлен, то потенциалы сферы и Q шара были бы одинаковы: <pi = k-= (внутри сферы поле отсутство- /С вало бы, см. задачу 12.33). Вследствие заземления шар получит от Земли такой заряд q, что его потенциал обратится в нуль: о Q Qr k— + k-= = 0, откуда q = - -=р Тогда, согласно принципу супер- Г Ли Ли позиции ф = kQ fi-q = fe^(l - -£) = . (1 - 4) = 450 В. /С £\,г 4л8оЯ' /С/ 12.41. Ответ: Е = 0 при г < Ri; — QRi(R» — Rz) о _ , . Е = -—— ----------—- при Ri < г < Rz (поле направлено к центру); 4ле<>1?2(Я> - ЯОг2 _ QR»(Rz — Ri) _ ,, . . Е = -———--------—— при Rz < г < Rz (поле направлено от центра); 47С£о£12(*™8 ~ Ri)r Е = 0 при г > Rz. Здесь г — расстояние точки от центра сфер. Указание. См. задачу 12.40. Условия заземления первой и третьей Ci Q q> сфер имеют, соответственно, вид ~ = 0, qi + Q + q» = 0. XVI ГЬ2 XV8 Напомним, что равномерно заряженная сфера не создает поля в той области пространства, которую она окружает. Решение. Какой заряд дм останется на первом шарике после первого заземления? Этот заряд можно определить, приравняв нулю потенциал фм этого шарика: фм = + ki = 0 (см. задачу
409 Электричество и магнетизм 12.35) . Отсюда дм = ~9bi аналогично находим заряд второго шара ль после его первого заземления: дгм = • Отсюда видно, что при дальнейших заземлениях заряды шаров убывают в гео- метрической прогрессии и после N заземлений каждого шара 9>.n = -gl-W; gn.N = g(^)2N. 12.43. Ответ: ф1(1 - ЛЬ2 ' ЛЬ2' Решение. Заряд Q шара можно определить из соотношения q>i = = А™-; после соединения шар и оболочка образуют единый про- Л1 водник, все точки которого имеют одинаковый потенциал фа. Поскольку весь заряд перейдет на внешнюю поверхность этого . Q р. -Ri . проводника, q>2 = k^r. Отсюда фг = фг=- (потенциал шара умень- ЛЬ2 ЛЬ2 шится). Если оболочку заземлить (не соединяя ее с шаром), то она получит от Земли такой отрицательный заряд q, что ее потенциал Q q k^- + k~- обратится в нуль. Значит, q = -Q (при этом система в ЛЬ2 ЛЬ2 целом электрически нейтральна и не создает поля снаружи). Поле Q Q заряда q обеспечивает оболочке (и шару) потенциал А-~- = -А-=-. ЛЬ2 ЛЬ2 Согласно суперпозиции потенциал шара L Q ,Q к Ri\ фз = А=- - fr=- = ф111 - -=-1. ЛЬ1 ЛЬ 2 ' ЛЬ 2' К ответу на последний вопрос можно подойти и иначе. До Q заземления оболочка имела потенциал ф' = А-=- = фг. Заряд, при- ЛЬ2 шедший на сферическую оболочку после заземления, уменьшает ее потенциал до нуля, но не изменяет поля внутри сферы, а значит, разности потенциалов между шаром и оболочкой. Поэтому Ф1 - фг = фз - 0, откуда фз = Ф1(1 - ' ЛЬ 2' 12.44. Ответ: |g| = ——. о + а Решение. До заземления тонкие незаряженные оболочки не влия- ли на вид поля; шары имели одинаковые потенциалы. После заземления одинаковые (нулевые) потенциалы имеют уже сфери- а • ' ........‘
Ответы, указания, решения 410 ческие оболочки. Перетекание заряда по проводу прекратится, когда сравняются потенциалы шаров. После заземления первый шар имеет заряд Q + q, а второй шар заряд Q ~ q. Напряжен- Q + q ность поля у поверхности первого шара Ei = k—=2^-, у поверхности л Q - q второго Е2 = k—Б2 • /С Поле в тонком зазоре между шарами и оболочкой можно считать однородным, поэтому потенциалы ша- ров найдем из соотношений q>i - О = Eia; фг - 0 = ЕгЬ. Приравнивая b — а Ф1 и фг, получим q = ---. При Ь > а заряд перетекает к первому Ь + а шару; при Ь < а — ко второму. Q2 згтЛоЯ4’ 12.45. Ответ: f = Решение 1. Рассмотрим малый элемент повер- хности AS, на котором находится заряд Ад = Q 4л7?2 AS. Действующая на этот элемент сила AF = Ag*\Ei, где Ei — напряженность поля, которое создается всеми остальными заряда- ми, не находящимися на площадке AS. Обоз- начим через Ег напряженность поля заряда Ад непосредственно у площадки AS и рассмотрим две очень близкие точки А и В, лежащие по разные стороны от площадки AS. Определим полную напряженность поля в этих точках (см. рисунок; Q считаем, что Q > 0): Ел = Ei + Ег = k-^; Ев = Ei - Ег = 0. Отсюда /С Q Ei = Ег = т. е. очень малый элемент заряженной поверхности создает вблизи себя такое же поле, как и все остальные заряды, вместе взятые! (Этот вывод справедлив для заряженного про- Q Q водника любой формы.) Таким образом, AF -------AS • тог- 4tcjR да f = — ss ft—-— = .... . AS 8л/Г ЗглЪаЯ4 Q Решение 2. Потенциал оболочки ф = k-^, а Потенциальная энергия кулоновского взаимодействия зарядов оболочки WP (мно- , житель 1/2 введен, чтобы не учитывать дважды энергию взаимо- i действия каждой пары точечных зарядов). Представим, что под
fjjl Электричество и магнетизм действием растягивающей силы F ^f’4nR2 радиус оболочки уве- ; дичился на малую величину ЛЯ. При этом работа А =* FAR соверша- "ется за счет уменьшения потенциальной энергии: А — -AWP. Вы- числим эту величину: AV7,, = л(й^) = (ft^)'AR = ~^да* • Отсюда , Q2 Q2 f — £—2 = ____ ' 8лЯ* 32п2ЕоЯ4’ 12.46. Ответ: р = 660 Па. Решение. Заряд Q распределяется по поверхности капли. При этом (см. задачу 12.45) за счет кулоновского отталкивания различных участков поверхности создается растягивающее каплю давление Q2 pi = -—-—С другой стороны, благодаря поверхностному натя- 32я ъол жению создается сжимающее каплю давление р2 = 2&/R (так на- зываемое- лапласовское давление, см. задачу 10.38). Давление внутри капли р = рг - pi = 660 Па. Заметим, что при pi > рг сферическая капля теряет устойчивость и может существенно де- формироваться или распасться. 12.47. Ответ: нс изменится; увеличится в 1,5 раза. Решение. Возникшую после введения пластин систему можно рассматривать как батарею из трех последовательно соединенных конденсаторов, у каждого из которых емкость Ci = EoS d/3 = ЗСо, где Со — первоначальная емкость конденсатора; емкость батареи составляет С = -^Ci = Со. Соединив проводом введенные пластины, О мы «закорачиваем» обкладки одного из трех конденсаторов. Тогда 1- 3 емкость оставшихся двух С = -^Ci = ^Со. л & 12.48. Ответ: С = 250 пФ. Решение. Пластины 1-2 и 2-3 образуют два одинаковых конден- ~ EocS - . сатора емкостью Со = —каждый (здесь е — диэлектрическая проницаемость слюды). Эти конденсаторы соединены между собой параллельно (на них одинаковые напряжения, поскольку <pi = фз). , Поэтому С = 2Со = = 250 пФ. а 12-49. Ответ: gi - 30 мкКл, д» = 12 мкКл, gt = 18 мкКл. ^Решение. Найдем полную электрическую емкость С системы кои-
Ответы, указания, решения 412 денсаторов. Емкость соединенных параллельно конденсаторов Сг и Сз составляет1* Сгз = Сг + Сз. Этот участок соединен последо- _ _ Ci Саз Ci(Ca + Сз) вательно с Ci, поэтому С = —--— = —---------—. Ci + Саз Ci + Сг + Сз Полный заряд этой батареи конденсаторов q = 6’С = ЗОмкКл. Именно такой заряд находится на конденсаторе Ci и на батарее конденсаторов Саз (т. е. qt = дгз = q — 30 мкКл). Чтобы найти распределение этого заряда между Сг и Сз, проще всего определить ТТ ТТ ? ^С о т> п напряжение на них: (7г = из = =----------— = 3 В. Отсю- Сгз , Сгз С1 + Сг + Сз да дг = (УгСг = 12 мкКл; дз = U3C3 = 18 мкКл. 12.50. Ответ: Со = Ci. Решение. Поскольку верхняя и нижняя ветви цепи одинаковы, задачу расчета емкости батареи можно существенно упростить. Вследствие симметрии цепи потенциалы точек АиВ одинаковы: Фа = фв. Напряжение U на включенном между этими точками конденсаторе равно нулю (U = фд - фв); значит, конденсатор емкостью Сг не заряжен и его можно изъять из цепи2*, не изменив ее емкости. После этого расчет становится несложным: Со = Ci. 12.51. Ответ: Со = 2С. Решение. Точки А и В, D и F соединены проводниками, поэтому фд = фв, фо = фг (напомним, что это — условие отсутствия электрического тока). Соединим точки с равными потенциалами и преобразуем нашу цепь в другую, не допуская нигде разрывов и новых соединений (см. ри- сунок). Обведенная штриховой линией часть этой схемы уже рассмотрена в за- даче 12.50. Ее емкость равна С. Еще один конденсатор емкостью С подключен па- раллельно к этой части схемы. Значит, Со = С + С = 2С. ~ CU 4CU 12.52. Ответ: gi = дз = -5-; дз = д« = —=—; фо - Z О фв = — 0,3 и Решение. Емкость верхней ветви цепи составляет С,Сз С Ст + С2~ 2’ ’* Здесь и далее Сл,..л обозначает емкость участка цепи, включающего элементы с номерами i, k, ... I. , 4 Здесь и далее подразумевается, что при изъятии какого-либо элемента цепи на - его месте возникает разрыв. ' 4
413 Электричество и магнетизм. D С, Сг В + Е С3 С. „ „ си А заряды на конденсаторах верхней ветви <ji = q2 = CiJJ = -g-. Ана- _ С.С< 4С 4CU логично находим См = —----— = q» = qt = CuU = Ct + Ct о о Найдем напряжения на конденсаторах Ci и Сз: Ci = 77- = 77 Ci 4 . - v ТТ 9» 4U (вполне очевидный результат) и Ut = 7г = Сз о Учитывая, что Ut = <рп — <рд и Ut = <рв — <рл, получаем <рп - <рв = = Ui - Uz = -0,3 U. 12.53. Ответ: Со = 6С. Решение. Данная цепь не содер- жит ни последовательных, ни па- раллельных соединений. На пер- вый взгляд, в ней отсутствует и симметрия между верхней и ниж- ней ветвями цепи. Заметим, одна- ко, что емкости конденсаторов нижней ветви вдвое больше соответствующих емкостей верхней Ci Сз ветви. Докажем, что при выполнении условия 77 = тг (см. ри- С-2 С-4 сунок) потенциалы точек D и Е одинаковы, если изъять конден- сатор-«перемычку». Распределение напряжений в верхней и ниж- Ui ней ветвях цепи описывается в таком случае уравнениями — = Ct Ut Ct Q Ui Ut __ TT TT tt , rr = -=- и 77- = 77. Значит, 77- = tt-. Поскольку Ui + Ut = U3 + Ut, Ci U4 Сз C/2 U4 получаем Ui = Ut (это можно доказать и методом, описанным в решений задачи 12.52). А эти напряжения не что иное, как разности потенциалов: Ui = <рп - <рл, Сз = <рв - <рл. Таким образом, <рп = (рв и поэтому включение конденсатора-«перемычки» между точками D и Е ничего не изменит: заряжаться он не будет, а значит, его можно изъять из цепи. После этого подсчет Со уже не вызывает затруднений: С12 = qC IS = 2С’ См = fS = 4С’ Со = С12 + См = 6С- ЗС + 6С 6С + 12С 12.54. Ответ: Со = 1,2С. Решение. Схема обладает симметрией относительно пространст- венной диагонали куба 1-7. При повороте куба относительно этой оси на 120° схема в целом не меняется, но точки 2, 4, 5 меняются
Ответы, указания, решения 414 между собой местами (так же, как й точки 3, 6, 8). Значит, при включении в цепь <ра = <р< = <ps и <рз — <рв = <рв. Каждую группу из трех точек можно объединить в одну точку. Тогда эквивалентная 1 схема приобретет вид, показанный на рисунке. Емкости последо- вательных участков этой схемы равны ЗС, 6С, ЗС. Полную емкость 1111 Г 1 ог найдем из соотношения „ - откуда Со = 1,2С. Со <5С ОС оС 12.55. Ответ: U = 120 В. Решение. Если первоначально емкость каждого конденсатора бы- ла равна С, то емкость батареи конденсаторов была 2С, а полный заряд батареи q = 2CUo. После сближения пластин одного из конденсаторов его емкость составит ЗС, а всей батареи 4С. Значит, заряд батареи конденсаторов q = 4CU. Однако заряд батареи конденсаторов, отключенной от источника, не мог измениться (вследствие закона сохранения электрического заряда). Значит, 2CUo = 4CU, откуда U = = 120 В. £ С + 1 12.56. Ответ: в первом уменьшится в —д— раз, во втором увели- 4W раз. 2е чится в----- Е + 1 Решение. Обозначим первоначальную емкость каждого конденса- тора через Ci. Начальное напряжение на каждом конденсаторе было равно U/2. Заполнение диэлектриком увеличило емкость первого конденсатора в е раз. Емкость же батареи конденсаторов _ еС1 С1 £ _ „ Л . стала С = —-----= —:—Ci. Заряд батареи (и каждого из конден- eCi + Ci Е + 1 Сатаров) q - UC = —UCi. Напряжение на конденсаторе, запол- ненном диэлектриком, составляет Ui = -7- = —, т. е. уменЬ- eCi е + 1
415 Электричество и магнетизм Е + 1 _ шилось в —=— раз. Во столько же раз уменьшилась и напряжен- лг ность электрического поля в этом конденсаторе. Напряжение на конденсаторе, не заполненном диэлектриком, составляет Uz = q eU 2е = ^- =----. Оно увеличилось в----раз по сравнению с первона- е + 1 Е + 1 чальным. Во столько же раз увеличилась и напряженность элект- рического поля в этом конденсаторе. 12.57. Ответ: a) Umax — 144 В, б) Umax — 36 в. Решение. Напрашивающийся простой ответ Uma = Ui + Uz спра- ведлив лишь при условии CiUi = CzU., т. е. когда конденсаторы способны накапливать одинаковые максимальные заряды (ведь при последовательном соединении одинаковы именно заряды на конденсаторах). Когда к батарее приложено напряжение U, заряд батареи (а значит, и каждого из конденсаторов) составляет CiCzU Ci + Сг q = Напряжения на конденсаторах составят соответствен- но Ui' = ^- = ———U hU2' = %- = ———U. Напряжения Ui иЩ Ci Ci + Сг Сг Ci + Сг не должны превышать предельно допустимых значений для каж- дого из конденсаторов, т. е. UV < Ut и Uz. Следовательно, приложенное к батарее напряжение U должно удовлетворять системе неравенств: СгС Ci + Сг < Ui, CdJ Ci + Сг < Uz. тт "* СгТТ Из первого неравенства получаем U < ——Ui, из второго тг . Ci + Сгтг __ - U < —-р,—Uг. Поскольку оба эти неравенства должны выполнять- С1 ся одновременно, 17 не должно превышать меньшего из двух Ci + Сг выражений: *—~—Ui и V2 Ci + Сг С1 Uz (нетрудно проверить, что при CiUi — CJUz оба эти выражения совпадают с суммой Ui + Uz; такой «очевидный» ответ уже обсуждался выше). Подставляя числовые данные условия, получаем: а) С7п»ж = С2С/1 - 144 В; б) = ^~^Uz ~ 36 В. V2 С1
Ответы, указания, решения 416 12.68. Ответ: Со = 2,5С. Решение. Попробуем упростить схему до того, как станем ее изображать. Учтем, что те три точки, к которым не подклю- чена внешняя цепь, «взаимозаменяемы»; следовательно, их потенциалы одинако- вы. Поэтому включенные между этими точками конденсаторы не заряжены и могут быть изъяты из цепи: на емкость системы они не влияют. Заряжены лишь семь конденсаторов (см. рисунок). Ем- кость каждой из трех одинаковых ветвей цепи равна С/2, а полная емкость сис- г темы Со = + С = 2,5С. Этот результат можно обобщить: при произвольном количестве точек п получаем Со = ^С. 4W C1C4 — СгСа 12.59. Ответ: фА - фв = ё---—-------; после включения резис- (Ci + С2)(Сз + С<) _ _ (Ci + Сз)(Сг + С,) тор.фл-<р.-0,С.= С1 + С1 + С1 + с<. Решение. Батарея из конденсаторов Ci и Сг, имеющая емкость _ С1С2 е _ С12 = — заряжена до напряжения, равного 6. Значит, ее заряд qi2 = §С12. Такой же заряд и у конденсатора Ci; значит, напряжение на нем = —-----6. Аналогично находим U» =--------©. Cl С1 + С2 с., + с< Учтем теперь, что Ui = фи - фл, С7з = фп - фв. Вычитая друг из друга эти соотношения, получаем т р С1С< - С£з фд — фв С/з — Ui ~ с>-----------• (С, + С2)(Сз + с,) Ci Сэ Заметим, что фл - фв = 0, если тг = тг (СР- с задачей 12.52). С2 Сч Если соединить резистором точки А и В, то цепь все равно останется разомкнутой и поэтому постоянный ток по ней течь не будет. По резистору пройдет лишь кратковременный всплеск тока, в результате чего потенциалы фл и фв уравняются (независимо от величины Я). Эквивалентная схема Получившейся системы кон- денсаторов показана на рисунке. Полная емкость системы со- !
,417 Электричество и магнетизм „ (С, + Сз)(Сг + Сз) . ставит Со = --——---------' (до введения в цепь резистора она рав- Ci + Сг + Сз + Сз CiCz СзСз нялась —•--—- + —----— ). Ci + Сз Сз + Сз С161 — С262 12.60. Ответ: фА - фв -- —-—-— VI + V2 под напряжением <§1 + 62; значит, ее заряд q „ С1С2 Решение. Батарея конденсаторов емкостью С - —----— находится Ci + Сг С1Сг(ё1 + ёг) „ =----—:-----Таким Ci + Сг поэтому напряжение же будет заряд каждого из конденсаторов, rr q Сз на конденсаторе Ci составит Ui = U Ci + Сг ток в цепи отсутствует, напряжение на источнике тока равно его ЭДС. Итак, (6’1 + 6’2). Поскольку фи — <рв = 6’1 фо — фл = Ui = —---+ 62). VI + V2 Вычитая второе равенство из первого, получаем С1ё1 — Сгёг = с, + с. • 12.61. Ответ: а) S; б) С; в) ЗС; г) д) е) С. о О <5 Указание. На рис. б, в, д, е приведены некоторые эквивалентные схемы систем конденсаторов, заданных в условиях. 12.62. Ответ: Ut = 100 В, Ui = 150 В, Из = 300 В. Решение. Из закона сохранения заряда следует, что заряды на соединенных обкладках соседних конденсаторов одинаковы по модулю и противоположны по знаку1* (см. рисунок). Наличие между конденсаторами источников тока в этом смысле ничего не К " Эго справедливо, если только какая-то обкладка не была заряжена до включения I в цепь (см., например, задачу 12.63).
Рис. е К задаче 12.61 меняет: источник тока разделяет заряды, но не создает их. Напряжения на конденсаторах: Ui - Uz = U3 77-. Вводя Ci Сг Сз потенциалы <р точек, обозначенных на рисунке, можем записать: фг - Ф> = Q Сг ; фз - фг = - &; фз - фз = JL. Сг’ фз — ф« — — 6з; фв — фз — тт*; ф1 — фв — - <Ь1. Сз Складывая почленно все шесть записанных выше равенств, полу- К задаче 12.62
419 Электричество и магнетизм q q q с- г л п С1СгС»(<51 + Аг + <эз)_ g + S + а ’ 61 ’ Отсвда ’ = с,с. + сл + с.сГ’ следовательно, напряжения на конденсаторах СгСз(ё>1 + + (эз) С1С2 4- СгСз + СяС1 С1Сз(6>1 + + 6>з) С1С2 + СгСз + СзС! = 100 В, U2 = = 150 В, LZ. = ия = = 300 в. 2±= 2 CiC2 + с2с.з + ас. Если с самого начала заметить, что результат не зависит от порядка следования элементов цепи, то задачу можно решить гораздо проще: достаточно рассмотреть последовательное соеди- нение трех конденсаторов, подключенных к источнику напря- жения А1 + #2 + Аз. 12.63. Ответ: Г. = U2 = С/, = U< = U. = U6 = 0. о о 4 о Решение. Определим сначала емкость батареи, состоящей из конденсаторов 2-6. Эквивалентная схема этой батареи приведена на рисунке. Конденсатор 6 включен между точками с равными по- тенциалами (закорочен), поэтому U& = 0, и на схеме этот конденсатор не показан. „ с „ „ с’ зс Очевидно, С.5 = Смъ = С + = -у-; С • 15С ЗС '—- = -=~. После замыкания ключа имевшийся на пер- С + 1,5С 5 вом конденсаторе заряд CUo распределяется между этим конден- сатором и батареей Сгш (напряжение th на них одинаково). Согласно закону сохранения заряда CU0 = (С + откуда Ui = О ЗСС7о Тогда заряд батареи Сгз« составит Qans = q2- CzmsUi = —5—. Следо- О ТТ q2 ЗС7о тг тг т Uo т т Uo вательно, U2 = -^ = -5—; U2 = Ui - U2 - -т~; Ut = Us = v о 4 о Qd 12.64. Ответ: U = —на внешних поверхностях пластин одина- г Е°а -Новые заряды +2Q, на внутренних — заряды +Q и -Q. |гешение. Удобно начать как раз с выяснения характера распре- деления зарядов на пластинах (см. рисунок, на котором пластины
Ответы, указания, решения 420 схематически утолщены). Напряженность поля каждой из четырех заряженных плос- ы костей задается формулой Ei = ——- (i = 1, 2 Боа 2, 3, 4). С учетом знака заряда проекция напряженности на ось х составит справа от соответствующей плоскости Ей = -~—z, 2еоН слева — Ей =-------. Как известно, на- 2Еоа2 пряженность поля внутри каждой из металлических пластин (суперпозиция всех четырех полей) равна нулю. Поэтому (с учетом направления полей) 1 z ч л -—fa - qz - q3 - g<) = О, ~—^qi + qz + q3- q,) = 0. 2EoO Кроме того, из условия задачи получаем еще два уравнения для величин дг. Г gi + дг = 3Q, I дз + qt = Q. Решая эту систему четырех уравнений с четырьмя неизвест- ными, получаем: gi = g< = 2Q; дг = -дз = 9- Таким образом, заряды на наружных поверхностях пластин ность поля между пластинами Е = —-(д 2&>а2 одинаковы, а заряды на внутренних поверхностях противополож- ны (типичная для конденсатора ситуация). Определив напряжен- 1 + qz — q3 — g«) — , по- ЕоО2 Qd лучим U - Ed = — гм\ 12.65. Ответ: U = 10 В. Решение. Прежде всего представим схему так, чтобы были видны параллельные и последовательные соединения (см. рисунок). Надо найти напряжение между точками Е и F, если известно, что напряжение между точками А и F равно Uo. Благодаря условию Сг = 2С1 находим, что емкость цепи между точками Е и F, D и F, В и F одна и та же: она равна Сг. Следовательно, напряжение между точками В ulF равно половине напряжения между А и F, то есть С7» = -Х-. Аналогично Uor = -5- = ~z~, и, наконец, требуемое „ и„ и. в задаче напряжение CZ® = -%- = -g- = 10 В. - ; Л
421 Электричество и магнетизм К задаче 12.65 К задаче 12.66 12.66. Ответ: а) С = EoS<L + ^Е 1»; б) С = ; Ld cd - (е - 1)й . _ EoS(L - I) EoSlE в) С =---" +------------------• Ld L(Ed — (e — 1)й) Решение. В случае в конденсатор можно представить как батарею из трех различных конденсаторов — двух воздушник и одного полностью заполненного диэлектриком (см. рисунок). Очевидно, EoSl ~ EoESl EoS(L — Г) - Ci = — -----, С2 = -.т-, Сз =-Vs—тогда емкость батареи со- L{d — й) Ln, Ld — _ С1С2 EoS(Z< — I) EoSZe ставляет С = Сз +-----= —~ ;---------• Отсюда при Ci + Сз Ld L(Ed - (е - 1)й) й = d получаем случай а, а при I — L — случай б. 12.67. Ответ: б) С = в) С = d-h L\d-h d ! Указание. Внутри проводника электрическое поле отсутствует, поэтому объем конденсатора просто уменьшается на величину объема проводника, заряд переходит с обкладки на внутреннюю поверхность проводника. Приведенные ответы можно формально получить предельным переходом е -» <ю в ответах задачи 12.66, потому что с точки зрения электростатики проводник можно рассматривать как диэлектрик с бесконечно большой диэлектрической проницаемостью (поле внутри проводника исчезает, т. е. уменьшается в бесконечно много Раз).
Ответы, указания, решения 422 12.68. Ответ: 1) фз = фз = О О и 2) ф1 = 0, ф2 = фз = , ф4 = U; напряженность поля в промежут- ках 1-2 и 3-4 увеличится в 1,5 раза, в промежутке 2-3 поле исчезнет. 3) Емкость конденсатора и заряды на обкладках возрастут в 1,5 раза. Решение. 1) Внесение тонких незаряженных пластин не меняет вида электрического поля внутри конденсатора. Оно остается однородным, напряженность поля Ео во всех трех промежутках одинакова. Значит, потенциал изменяется с расстоянием линейно, U 2U , и поэтому фз = -Х-, фз - (разумеется, фх = О и ф4 = U). О О 2) В результате замыкания пластин 2 и 3 между ними проис- ходит такое перераспределение заряда, что их потенциалы урав- ниваются, а поле в промежутке 2-3 исчезает. При U = const последующее размыкание пластин 2 и 3 уже ничего не меняет. Очевидно, при этом фз = фз = (пластины 2 и 3 расположены симметрично относительно средней плоскости конденсатора). Зна- чит, разности потенциалов между пластинами 1-2 и 3-4 возросли в полтора раза по сравнению с первоначальными. Во столько же раз выросла, конечно, и напряженность поля в этих промежутках. Итак, Ei-z — Ез-4 — 1,5 Ео; Ез-з = 0. 3) В результате замыкания пластин 2-3 поле исчезает в одном из трех промежутков, что равносильно уменьшению расстояния между обкладками в полтора раза. Значит, емкость возрастает тоже в полтора раза: С = 1,5Со. Во столько же раз (поскольку U = = const) возрастают и заряды на обкладках. 12.69. Ответ: энергия конденсатора уменьшается; увеличивается энергия, запасенная в аккумуляторе; энергия отключенного от источника конденсатора при раздвигании пластин растет. CU2 Решение. Энергия заряженного конденсатора W„ = —, где С = вл EoS = -j-. Пока конденсатор соединен с источником постоянной ЭДС, U = const. Емкость С при увеличении расстояния d между плас- тинами уменьшается. Значит, уменьшается и WP. Куда же «исче- зает* энергия конденсатора и вдобавок энергия, потраченная на' раздвигание пластин (ведь нужно было преодолеть их взаимное
423 Электричество и магнетизм притяжение)? Противоречие с законом сохранения энергии, разу- меется, кажущееся: ведь конденсатор не является замкнутой системой, он подключен к аккумулятору. При уменьшении емкос- ти конденсатора уменьшается и его заряд. Уходящий с конденса- тора заряд проходит через аккумулятор, совершая работу против сторонних сил. Вследствие этого увеличивается запасенная в аккумуляторе энергия, т. е. аккумулятор подзаряжается1’. Если же цепь разомкнуть, то напряжение на конденсаторе уже не будет постоянным. Теперь постоянным будет заряд q конденсатора. Поэтому формулу для энергии заряженного конденсатора удобно о2 записать в виде WP = При уменьшении С (раздвигании плас- тин) энергия конденсатора увеличивается: внешние силы совер- шают положительную работу, а энергии «уйти» некуда! 12.70. Ответ: a) q = const; U, Е и W уменьшатся в £ раз; б) U = const; Е = const; q и W увеличатся в £ раз. Указание. См. решение задачи 12.69. 12.71. Решение. Если бы конденсатор не был заряжен, то при вытекании из него жидкий диэлектрик приобрел бы кинетичес- кую энергию Wk. При заряженном конденсаторе вытекание замед- ляется (его тормозит сила, «втягивающая» диэлектрик в поле конденсатора). Значит, кинетическая энергия Wk’, приобретенная диэлектриком в этом случае, меньше Wk. Разность Wk - Wk’ как раз и представляет собой добавочную энергию заряженного кон- денсатора. 12.72. Ответ: в случае б, когда конденсатор отключен от источ- ника. Решение. Вначале в обоих случаях необходимо преодолевать оди- наковую силу F притяжения пластин. В случае б заряд q = const, поэтому напряженность’ поля Е = const и F = const. В случае а постоянным остается напряжение на конденсаторе: U = const. Поскольку емкость конденсатора при раздвигании пластин умень- шается, уменьшается и его заряд, а значит, и электрическое поле. Следовательно, в случае а потребуется совершить меньшую работу (при одинаковом перемещении пластин). Ср. эту задачу с задачей 12.69 ** ** Цели аккумулятор заменить источником, который нельзя подзарядить, то «лиш- ил*» энергия, конечно, Перейдетво внутреннюю. '
Ответы, указания, решения 424 CR2 12.73. Ответ: Q = О Решение. Полная емкость батареи конденсаторов после переклю- чения не изменилась (и до, и после переключения два одинаковых конденсатора соединены параллельно, а третий такой же подклю- 2 чен к ним последовательно): Со = 75С. Значит, энергия электричес- С°<52 кого поля в батарее также не изменилась: она равна - g—. Означает ли это, что выделившееся количество теплоты Q = 0? Нет, потому что по цепи некоторое время шел ток (произошло перераспреде- ление заряда между конденсаторами), а это значит, что должно было выделиться некоторое количество теплоты. Необходимая для этого энергия могла поступать только от источника ЭДС, т. е. Q — — Act = &q8 (здесь Аст — работа сторонних сил в источнике ЭДС; Aq — заряд, прошедший через источник от его отрицательного полюса к положительному). На рисунках показано распределение -----1,С£----- __-С8/3 J_-C8/3 J_+2C8/3 “ ~+Сё/3 ~[~+Cg/3 ~-2С8/3 Рис. а 1L ’ 1 2 _ _~2С8/3^ _+С8/3 _+С8/3 +2С8/з" -С8/3 ~-С8/3 Рис. б зарядов на обкладках конденсаторов до переключения (а) и после CS (б) . Мы видим, что заряд Ag = -5- «ушел» с верхней обкладки О левого конденсатора и прошел через источник. Отсюда получаем Сё2 Q = —. Интересно, что при обратном переключении снова выде- о ляется такое же количество теплоты, и необходимая для этого энергия опять поступает из источника. Переключая ключ туда и обратно, мы каждый раз «берем» из источника одну и ту же «порцию» энергии. 12.74. Ответ: <р = N* Решение. Потенциал заряженной капли (проводящего шара) фо Q Q у = k~, заряд каждой, из маленьких капель q = Радиус г ма>
425 Электричество и магнетизм леньких капель найдем из условия неизменности объема ртути: 4лВ* .. 4ЛГ8 — = лг-з“- R откуда г = —1. Тогда потенциал образовавшихся капель составит Ns . 9 . Q Фо „ Ф = п— = п—«— - 2, т. е. уменьшается. Отметим, что этот ответ r N»R Ns справедлив только в том случае, когда образовавшиеся капли разлетятся на большое расстояние (чтобы можно было пренебречь энергией их взаимодействия). 12.75. Ответ: уменьшается в Ns раза. Решение. Первоначальная энергия электрического поля Wo = - 4фо<? (см. задачу 12.74 и решение 2 задачи 12.45). Энергия поля 44 после распада капли тю- »г 1 хт 1 фо Q W = N • ^Ф? = N • g • • -77 = —г. При распаде капли, однако, возрастает поверхностная энергия жидкости, поэтому самопроизвольный распад не всегда возможен. 12.76. Решение. Шарики отталкиваются друг от друга, и поэтому все четыре нити натянуты. Следовательно, при равновесии шари- ки располагаются в вершинах ромба. Если угол между стороной ромба и одной из диагоналей равен а, то длины диагоналей ромба 2Zsina и 2Zcoaa (здесь I — длина нити). В положении устойчивого равновесия потенциальная энергия WP кулоновского взаимодейст- вия шариков должна быть минимальной. Поскольку расстояния между соседними шариками фиксированы, достаточно рассмот- реть энергию взаимодействия между шариками, лежащими на одной диагонали: а2 WP = k- — + k 2Zsina Производная WP по углу а равна g2 2Zcosa Ag2(sin3a - cos3a) _ ——-------------. Отсюда 2Zsm acos a следует, что величина WP убывает при изменении угла а от 0 до 45° и возрастает при изменении а от 45 до 90°. Минимальное значение Wp достигается при a = 45°, когда ромб превращается в квадрат. „ 32лЕо7П^Е2 12.77. Ответ: Q >---------—. i q | Решение. Разумеется, кулоновская сила должна, по крайней кере,
Ответы, указания, решения 426 уравновешивать силу тяжести: , Qq ^>те. (1) Однако этого мало. В верхней точке шарик должен иметь положение устойчивого равновесия, т. е. потенциальная энергия WP при смещении шарика из верхнего положения должна увели- чиваться. Эта энергия складывается из энергии mgh притяжения к Земле (высоту h можно, например, отсчитывать от уровня центра Qcr сферы) и энергии кулоновского отталкивания А— (см. рисунок): Wp = mgh + Q Рассмотрим изменение Wp при смещении шарика из верхнего положения (шарик не отрывается от поверхности сферы!). Вели- чину смещения будем характеризовать углом а. Тогда (см. ри- а сунок) Л = 2?cosa; г = 2/?cos^. Потенциальную энергию Wp удобно , a записать как функцию величины х = cos-^; тогда получим и Wv = mgR&x* - 1) + Нас интересует поведение этой функции вблизи a = О, т. е. вблизи х = 1. Для того, чтобы равновесие шарика в верхней точке было устойчивым, необходимо, чтобы WP увеличивалась при уве- личении а, т. е. при уменьшении х. Это значит, что производная ТРр'.при х = 1 должна быть отрицательной: W'p'k.j = [ixmgR - < 0.
427 Электричество и магнетизм Отсюда получаем 8mgR2 _ 32n&>mgR2 > kq ~ q ’ Мы видим, что условие устойчивости является более сильным, чем просто условие равновесия, т. е. неравенство (1). Тот же результат можно получить иначе. Равновесие шарика будет ус- тойчивым, если при малых углах а модуль проекции кулоновской силы Fk на касательную к окружности (см. рисунок) превышает модуль проекции силы тяжести mg на то же направление: „ . а FkSin— > mg’s та. и тт .al. Поскольку при малых а можно считать sinr - типа, получаем и и Fk 2mg, откуда Q > 32nsomgRi q 12.78. Ответ: парабола; Л = 2,7 мм. Решение. Электрическое поле внутри конденсатора однородно, его и напряженность Е - Оно придает электрону постоянное уско- еЕ eU рение а - — = направленное под прямым углом к пластинам (см. рисунок). Движение электрона подобно движению горизон- тально брошенного тела под действием силы тяжести. Координаты at2 „ электрона изменяются со временем по закону х = vt, у = -у. Ис- ключая из этих соотношений время, получаем а 2 eU г v 2v 2mv2d Это уравнение параболы. Подставляя в него х = /, находим Л = еШ2 = —тз - 2,7 мм. 2mv d
Ответы, указания, решения 428 12.79. Ответ: Н = 2,47 см. Решение. После вылета из конденсатора электрон движется пря- молинейно по инерции. Направление движения определяется скоростью электрона в момент вылета из конденсатора. Из подо- бия треугольников (см. рисунок) следует: Н - h _ щ L ~ и/ „ I eUl , К моменту вылета из конденсатора и» = и, vy = а— = mv(^ (см. задачу 12.78). Тогда „ , Т eUl eUl /I .л „ _ Н = Л + L---гу = —+ L) = 2,47 см. mv2d mv2dX2 > Разумеется, основной вклад в Н дает движение электрона вне конденсатора. Значит, смещение точки на экране обусловлено не столько смещением электрона при движении внутри конденсато- ра, сколько изменением направления его движения. 12.80. Ответ: траектория электрона — парабола; ,i„ mi?osin2a\ _r znvo2sin2a _ _r _ znuo2sin2a S = 41 - 2(7е ~) ПРИ U > ~ 2Г ’ 8 = ° ПрИ U < ~2Г~- Указание. См. задачу 12.78. 12.81. Ответ: Ui = 150 В; при Z = 10 см задача не имеет решения. Решение. Энергия электрона соответствует разгону его напря- жением Uo = 1500 В, т. е. mva2 2 = eUo, (1) где но — начальная скорость электрона в конденсаторе. Проекция начальной скорости но на плоскость пластин равна vocosa, на нормаль к ним uosina. Ускорение электрона в конденсаторе Ue ~ md а (см, задачу 12.78), движение происходит по параболе (см.
429 Электричество и магнетизм рисунок). Время пролета электрона через конденсатор t = —-—. Носова Вертикальная проекция скорости электрона на вылете из конден- сатора должна равняться нулю: uosina - at = 0, откуда dsinacosa I ‘ ти2 е dsin2a Г~и°- Uel л гг Uosina-----------= О и U = zndvocosa Учитывая уравнение (1), находим rr rrd- 2sinacosa U - U о j - При I = Zi = 5 см получаем C7i — 150 В, а при I = 1г = 10 см U2 =75 В. Однако не будем торопиться: решение задачи еще не закончено. Определим вертикальное смещение Л электрона за время пролета в конденсаторе. Среднее значение вертикальной Vosina + 0 , Z. проекции скорости иср =-----5----, значит, Л = глР • t - 7?tga. 44 4 Если Z = Zi = 5 см, то h = hi = 0,67 см; если же I = h = 10 см, то h = /12 = 1,34 см. Как видим, Л2 > d! Значит, при I2 = 10 см задача не имеет решения — электрон не сможет пролететь через весь конденсатор, так как непременно попадет на положительную пластину. При I = h = 5 см электрон должен входить в конденсатор достаточно далеко от отрицательной пластины. 12.82. Ответ: и = = 2,7 • 107 м/с; скорость нйправле- на под углом a = arctg^j^ = 41° к плоскости пластин. Указание. См. задачи 12.78, 12.79. 2d2 12.83. Ответ: U2 = =^-Ui = 100 В. I2 Указание. См. задачи 12.78 и 12.81. Электроны не будут вылетать at2 d из конденсатора, если ~s~ > 12.84. Ответ: U = d\ 2pgh V бо(б - 1) Решение. Как известно (см., например, задачу 12.12), диэлектрик втягивается в область более сильного электрического поля, а у края конденсатора возникает как раз резко неоднородное поле. Однако непосредственное вычисление силы, действующей на жид- кость со стороны неоднородного поля, — непростая задача. Поэ- тому применим ♦энергетический подход»: воспользуемся тем, что потенциальная энергия WP системы в положении устойчивого
Ответы, указания, решения 430 равновесия минимальна. Эта энергия складывается из потен- mgh , циальнои энергии —столбика втянутой в конденсатор жид- л ч кости (высота ее центра тяжести ) и энергии поля заряженного и qЕ 2 * * * * „ конденсатора Заряд q конденсатора остается при подъеме жидкости неизменным, а емкость конденсатора возрастает от Со = ~~ до С = ^-(1 + (е - 1) ~) (см- задачу 12.&6). Учитывая, что ф т = pdah, записываем: w = pgdah2 + q2d Р 2 2еоа(а + (е - 1)Л)’ Приравнивая нулю производную WP по Л, получаем , й g2rf(e-l) _ pgdah ------------------ = 0. 2гм(а + (е - 1)й) / 2pgh Поскольку q = UC, находим U = d"\--------. V £о(£ - 1) 13.1. Ответ: I = 340 м; S — 0,10 мм2. Решение. Выразим массу и сопротивление проволоки через I и S: т = dlS, R = р-~, О где d — плотность меди, р — ее удельное сопротивление. Из этих = 340 м, S = л/^ = 1,0 • 107 м2. у dR уравнений находим I = Е 13.2. Ответ: j = где р — удельное сопротивление проводника. Р Решение. Из определения плотности тока, закона Ома для участка I цепи и формулы для сопротивления R = ру следует: О . и 1 ~ рГ „ г. и . Е п Поскольку Е = -г, получаем j = —. Это соотношение часто на- 1 Р зывают дифференциальной формой закона Ома. 13.3. Ответ: схема на рис. а дает меньшую погрешность для больших сопротивлений Rx', схема на рис. б — для малых сопротивлений Rx. тт „ U „ тт „ RvU Для схемы a: R, = -у - Ла. Для схемы б: R, =- IRv — U
431 Электричество и магнетизм Решение. Рассмотрим сначала схему а, в которой амперметр включен последовательно с резистором. Амперметр при этом дает правильные показания: он измеряет ток 1а =,1х, Текущий через резистор. Однако вольтметр не включен параллельно резистору и поэтому «лжет»: он измеряет не напряжение на резисторе, а сумму напряжений на резисторе (JZX) и амперметре (CZA). Следовательно, U UA + U, UA U, о о = ---г-- - ~Г~ + = Да + Rx, 1 1 1А ix и то есть отношение — равно сумме сопротивлении резистора и амперметра. Таким образом, измерение по этой схеме даст малую погрешность, если Ra < Rx (сопротивление вольтметра при этом несущественно). Рассматривая аналогично схему б, приходим к результату: tz _ Ду I Rx + Rv' В данном случае погрешность мала при Rv > Rz (сопротивление амперметра несущественно). Таким образом, для больших сопротивлений Rx предпочтитель- нее схема а, для малых — схема б. 13.4. Ответ: R* - —= 61 Ом; AR, = Rx - у » 1 Ом. 1 ~ R~v 13.5. Решение. Можно, например, начать с измерения сопротив- ления вольтметра Rv (рис. а): Rv = ^. Тогда для нахождения Rx можно воспользоваться схемой на рис. б: Рис. а Рис. б
Ответы, указания, решения 432 Такой метод можно использовать только в том случае, когда чувствительность микроамперметра достаточна для измерения малых токов h и /а. 13.6. Ответ: i = 0,80 мкА. Решение. Пусть пластина вдвинута в конденсатор на расстояние х. Тогда емкость конденсатора „ Еов2/, , *х\ с=vi1+<' - (см. задачу 12.66), где е — диэлектрическая проницаемость стек- ла, а = 10 см. Заряд конденсатора q = CU. Сила тока в цепи i= м = а(я-1>аГЛ . to = ^-l)gt7v = икД At d At d 13.7. Ответ: точки подключения должны делить кольцо в отно- шении 1 : 3. Решение. Два участка кольца представляют собой параллельно соединенные проводники. Пусть длина кольца I, а длины участков кольца (между точками подключения) х и Z - х. Сопротивления RoX Ro(l - х) ' ветвей цепи равны —и----------. Из соотношения RoX Ro(l - х) _ I I = J>x(l Rax R^l- х) I + I х I R получаем -7 = 0,5 ± Л/0,25 -Заметим, что решения Xi и ха 4 V /Со удовлетворяют условию Xi + хг = I, т. е. фактически мы получили не два разных решения, а одно. Подставляя численные значения R и Ro, получаем у- = 0,25, = 0,75. Значит, точки подключения £ t должны разделить кольцо в отношении 1 : 3. Л> 13.8. Ответ: -г = 8 Ом. 4 Указание. Воспользуйтесь тем, что выражение х(1 - х) максималь- I , . но при х = -= (см. математическое приложение). Л л 13.9. Решение. Всего можно получить 15 значений Я: -j- = 1,5 Ом, Яо л Г\ ^Яо л Л п Л „ n &Ro nd П 2Яо . _ -х- = 2 Ом, = 2,4 Ом, -«- = 3 Ом, = 3,6 Ом, -х- = 4 Ом, о о _ л о о
433 Электричество и магнетизм = 4,5 Ом, Ro = 6 Ом, ~ = 8 Ом, ™ = 9 Ом, ~ = 10 Ом, 4 о А О * Ro Ro 2Ro 3Ro Сопротивления Ro, 2Ro, -z-, 3Ro, -5-, —5-, —5- можно получить. £г О О & используя один, два или три резистора (что, разумеется, допуска-, ется). Этих схем мы не приводим. Остальные приводятся на рисунке. 13.10. Ответ: четыре резистора (см. рисунок).
Ответы, .указания, решения 434 13.11. Ответ: R = 18 Ом; Л = h = 2 A, h = 1,2 A, I2 = It = П = 0,8 А. Решение. Прежде всего необходимо определить типы соединений резисторов в цепи. Резисторы R2, R<, Ri соединены последователь- но; l?24s = R2 + Rt + Ri = 30 Ом. Резистор Яз подключен параллельно _ _ RiRui , Л _ к K24s; поэтому лгз45 = —-— = 12 Ом; резисторы Ri н R3 подклю- R» + Л245 чены к Ягз45 последовательно. Поэтому полное сопротивление цепи R = Ri + Re + Rwa = 18 Ом. Полная сила тока в цепи согласно закону Ома I = — = 2 А. R Поскольку резисторы Ri и R« стоят в неразветвленной части цепи, через них идет весь ток: h = It = 2 А. На участке же R3 - Rzh ток разветвляется (ток в каждой из параллельных ветвей обратно пропорционален сопротивлению этой ветви). Для нахождения токов в различных ветвях можно составить соответствующую пропорцию. Можно поступить и иначе: найти напряжение на параллельном участке цепи U3 = I Rzui - 24 В. Тогда, согласно U3 U3 закону Ома 13 = = 1,2 A; I2 = It = Д = — = 0,8 А. Как и следо- -TUI -TV245 вало ожидать, Д + Д = Д 13.12. Решение. Возможных схем с двумя резисторами не так уж много, и простым перебором можно проверить, что ни одна из них не.удовлетворяет условию. Схем с тремя резисторами тоже немно- го; условию задачи могут удовлетворять только две схемы, пока- занные на рис. а и б. Покажем, что эти схемы действительно удовлетворяют условию и найдем соответствующие сопротивле- ния резисторов. Схема на рис. а при подключении к любой паре клемм пред- Рис. а Рис. 6
435 Электричество и магнетизм ставляет собой последовательное соединение двух резисторов. Поэтому Нлв = И + Г2, Яве = Г2 + Га, Яле = П + Ха. ОтСЮДа Ялв + Яас — Яве, _ л -------2------= * ®м» Ялв + Яве — Яас — Га =-----„------= 2 Ом; Яве + Яас — Яав „ _ Га = -----2------ = " Ом. Схема на рис. б при подключении к любой паре клемм предс- тавляет собой параллельное соединение одного из резисторов с двумя другими, соединенными между собой последовательно. Поэтому Я1(Яз + Яз) Яав = —---—---—, Я1 + Яз + Яз D Яз(Я1 + Яз) Я1 + Яз + Яз Яз(Я1 + Яз) Яас = —---—---— • Я1 + Яз + Яз Умножая каждое из этих уравнений на Я1 + Яг + Яз и складывая все три уравнения, получаем Я1Яз + Я1Яз + ЯзЯз = + Д--+ Д~(Я1 + Яз + Яз). Тогда Я 1Яз = + Д~~ Дас(Я, + Яз + Яз); £ ЯзЯз - Д-+ Да-~АВ(Я, + Яз + Яз); А _ _ Яав + Яас — Явс,_ _ _ . Я1Яз = ------„-----(Я1 + Яз + Яз). £ Разделив почленно эти уравнения друг на друга, найдем отно- шения неизвестных сопротивлений: Я. Яав + Яас - Яве Я1 Яав + Яве — Яас Подставляя Яг = 2 Я1, Яз = ЗЯ1 в любое из уравнений системы, находим: Я1 = 6 Ом; Яг = 12 Ом; Яз = 18 Ом. 13.13. Решение. Для схемы на рис. а, очевидно, Я = г. В схеме на рис. б части реостата с сопротивлениями г и Яо - г соединены параллельно: 1 д _ гСДо ~ г) _ ~ Г) -г+(Яо-г)- До
Ответы, указания, решения 436 В схеме на рис. в соединены параллельно проводники с сопро- тивлениями г и Во: Графики этих зависимостей приведены на рисунке (график а — прямая, б — парабола, в — гипербола). В начале координат все три графика касаются друг друга. 13.14. Ответ: Ri = 20 Ом; Rz = 60 Ом; Вз = 40 Ом. Решение. Резисторы Rz и Rs при подаче напряжения на вход цепи С7> соединены между собой последовательно. Поэтому Iz = —------—, Ня 4- Ня и, = IzRz. Отсюда R» = = 40 Ом; Rz = - R» = U' ~ = 60 Ом. 12 12 12 При подаче напряжения на выход соединены между собой после- довательно резисторы Ri и Rz. Поэтому сила тока через них I'i = = ——-— и US = I'zRi = U'z—^—. Отсюда Bi = Bz * = 20 Ом. Ri + Rz R, + Rz U'» - £7'i 13.15. Ответ: a) R = 6) R = в) R = Ro. О О Решение, а) Поскольку точки А и С, В и D закорочены, их потен- циалы одинаковы: (рл = (рс, (рв = <ри. Объединив эти точки, получим эквивалентную схему (см. рис. а). „ Ro Значит, R = Л Рис. а
, 437 Электричество и магнетизм б) Эквивалентная схема имеет вид, показанный на рис. б. _ Rt> ЗЯо _ ЗЯо Очевидно, ЧТО Лгз = 5Г, Л234 = —, R = —=—. Z Z О в) Наиболее удобная эквивалентная схема получится, если «вытянуть» в одну линию резисторы 1 и 5, 3 и 4 (см. рис. в). Симметрия схемы делает очевидным равенство фв = фс. Следова- тельно, ток через Ri не идет и «перемычку» можно изъять из цепи. Поэтому Hu = Ria = 2J?o, R = Ro. Советуем сравнить это решение с решением задачи 12.61. 13.16. Ответ: R = Л Указание. На рисунке приведена схема цепи. См. также задачу 13.15 (случай в). ; Указание. Ср. с задачей 12.58. |13.18. Ответ: R = О,8/?о. |Решение. Воспользуемся тем, что цепь обладает симметрией, и
Ответы, указания, решения 438 «разрубим* узел в центре, как показано на рисунке. Покажем, что сопротивление этой цепи такое же, как и исходной. Дей- ствительно, точки С, D, Е (лежащие на вертикальной оси сим- метрии) имеют равные потенциалы. Поэтому их можно объе- динить, и мы получим исходную цепь. Расчет «разрубленной» цепи не представляет сложности, т. к. здесь мы имеем дело с комбинацией последовательных и параллельных соединений. Пол- 1 3 ное сопротивление цепи определяется из соотношения -= = + /С о/Со К задаче 13.21
439 Электричество и магнетизм 13.19. Ответ: R = о Указание. Используйте эквипотенциальность точек A, Bi, Bi и В, D, Ci (ср. с задачей 12.54). 13.20. Ответ: R = 4 Указание. Из симметрии цепи следует, что потенциалы точек А, В, Ci, Bi равны между собой. Объединив все эти точки, получим эквивалентную схему (см. рисунок). Все резисторы на схеме имеют сопротивление Ro. 7 13.21. Ответ: R = ууйо. X Л Указание. Система симметрична относительно плоскости AAiCCi. Значит, потенциалы точек В и D, Bi и Di попарно совпадают. Объединяя эти точки, получаем эквивалентную схему, показан- ную на рисунке. 13.22. Ответ: R = Указание. Эквивалентная схема показана на рисунке. 13.23. Ответ: R = Указание. Для получения эквивалентной схемы можно или «ра- зорвать* схему по вертикальной оси симметрии (см. рис. а) или объединить точки с равными потенциалами (на рис. б они обозна- чены одинаковыми буквами). Соответствующая эквивалентная схема приведена на рис. в, сопротивления всех изображенных на Ней резисторов равны Ro. ЧО /А— D -R1 + Я.З/1 . Г.4Rz j 13.24. Ответ: R - ——(1 + \ 1 +-------- . 2 ' V R, + R3 > Решение. Поскольку рассматриваемая в задаче цепь бесконечна, удаление одной «ячейки» не влияет на ее сопротивление. Значит, вся цепь, находящаяся правее звена AiBi, тоже имеет сопро- тивление R. Это позволяет использовать эквивалентную схему цепи, показанную на рисунке, и записать уравнение R = Я1+ + R3 + . Отсюда R2 - (Ri + Rs)R - (Ri + Яз)Яг = 0 и R — R2 + R + Rl(Ri + R») • Если Rs Ri + Ro, получаем R = Ri + Rs (резистор с сопротивлением Rs практически «зако- ротит» всю последующую бесконечную цепь). .Ri + Яз Л l(R, + R3y 2 + У 4
Ответы, указания, решения 440 Рис. а к задаче 13.23 Рис. 6 к задаче 13.23 Рис. в к задаче 13.23 К задаче 13.24 К задаче 13.25 13.25. Ответ: Rx = (43 - 1)7? = 0,737?. Решение. Предположим, к цепи добавили одну ячейку (можно считать, что это последняя ячейка перед точками С и D). При этом сопротивление цепи не изменится в том случае, если сопротив-
441 Электричество и магнетизм ление последней ячейки вместе с Rx (см. рисунок) равно Rx, т. е. Я(2Д + R.) 3R + Rx " ~ Отсюда Rx = (<3 - 1)Д = 0,73Д. Очевидно, при таком Rx добав- ление любого числа ячеек не изменит полного сопротивления цепи (также равного Rx). 13.26. Ответ: в (2 + V3)N раз. Решение. Найдем сначала напряжение Ui на выходе первой ячей- ки. Эквивалентная схема цепи показана на рисунке. Согласно этой 17, Rx <3-1 _ _ схеме =----------= т=----= 2 - \3. Однако напряжение С/, явг Уав я, + 2Я V3 + 1 ляется входным для второй ячейки цепи, значит, 7^ = 2 - <3 U1 (эквивалентная схема цепи без первой ячейки соответствует тому же рисунку). Значит, по мере изменения номера k ячейки напря- жение на ее выходе изменяется в геометрической прогрессии: 17k - 17лв(2 - <3)к. Напряжение между точками С и Л Ucd = 17n = = 17лв(2 - <3)N - ~ х,. Рассмотренный в этой задаче принцип /9 1 МЧ F построения делителя напряжения применяется в так называемых аттенюаторах, осуществляющих ступенчатое уменьшение напря- жения в геометрической прогрессии. Знаменатель прогрессии можно задавать, подбирая резисторы в цепи. 13.27. Ответ: R = 297 Ом. Решение. Сопротивление шунта Rm (см. рисунок) должно быть таким, чтобы через гальванометр шел ток Ir = — (I — полный ток Л в цепи). Тогда Im = I - 1Г = -—-I = (п - 1)7.. Поскольку = 4s, П Л» - 1т
Ответы, указания, решения -442 D Я, „ находим л,и =----—. Полное сопротивление показанного на рисун- п - 1 RrRm __ —. - ке участка цепи--------+ R. Общее сопротивление цепи не меня- Rr + л,0 ется, если + R = R„. Отсюда R = ) = 297 Ом. Rr + Л™ ' п' Этот результат можно получить и не вычисляя Rm‘. если полный г г I ток в цепи I, то L = и напряжение на гальванометре с шунтом IR, Ur Rr Ur =--. Значит, общее сопротивление этого участка Rnn = ~ = —. п 1 п Сопротивление всей цепи остается неизменным, если R -= /Ц1 - у). 13.28. Ответ: Rn = 2,2 кОм. Решение. При последовательном соединении миллиамперметра и добавочного сопротивления сила тока и напряжение будут связа- ны соотношением U = I(Ra + Яд), где Ra — сопротивление миллиамперметра, которое, в свою очередь, определяется из соот- ,Да ношения R,,, =---(см. задачу 13.27). Из этих двух соотношений п - 1 получаем Яд = у - Л„,(п - 1) « у = 2,2 кОм. 13.29. Ответ: увеличить на (п - 1)(т?д + ) ' Rr + Rin Решение. Пусть стрелка гальванометра отклоняется на одно деле- ние при силе тока Z. При этом ток через шунт составляет 7Ш = Rr i Rr \ = а ток через 7?д равен Л1 + —I. К цепи приложено напря- жение IR, + Z-Rj/l + После замены 7?д на R'a это напряжение ' xvnr * должно увеличиться в п раз: IRr + ZR„'(1 + 4^) = + 7Яд(1 + — 1 ^RrRni Отсюда Ra = пЛд +-----------. Rr + Rm 13.30. Ответ: q = - .—------,, 4 Я1 + Я2 Решение. До замыкания ключа сумма зарядов на правой обкладке ’’ конденсатора Ci п левой обкладке конденсатора Сг равнялась нулю ’ Г . ‘'1
443 > Электричество и магнетизм (они были соединены последовательно). После замыкания ключи каждый из конденсаторов подключен параллельно к соответству- ющему резистору, поэтому напряжения на них Ui = U——— и Ri + Ri Ri Uг = U-------Сумма зарядов на обкладках конденсаторов, соеди- Ri + R2 ненных с точкой А, составит теперь q = C2U2 - СЫ = Ri + Ri Именно этот заряд и прошел через ключ (при q > О ток через ключ шел от В к А, при q < О — наоборот). 13.31. Ответ: Фа - фв = '/11 4- Г12 Vi + Сг' , 13.32. Ответ: Rx = Л2 Решение. Ток не идет через гальвано- метр только тогда, когда прибор вклю- чен между точками с равными потен- циалами. Изымая из цепи гальвано- метр, получаем (см. рисунок) <рд = фв. Отсюда Ux = Ui, Us = U2. Поскольку Ux Rx Л Rx Rx = получаем = — и L/ 3 2V3 U 2 Jb2 /ьз /Ь2 _ RiRs a = 7rT- Советуем сравнить эту задачу с задачей 12.53. 13.33. Указание. Воспользуемся результатом, полученным при решении задачи 13.32. Рассмотренная в этой задаче схема фак- тически представляет собой мостик Уитстона, если под Ri и R2 понимать сопротивления отрезков провода длиной h и h. Для измерения Rx подбирают такое положение движка D, при котором ток через гальванометр равен нулю (в этом случае мостик назы- вают сбалансированным). Н3.34. Решение. При сбалансированном мосте ток-между точками "С и D не идет независимо от того, замкнут ли ключ. В противном [случае замыкание или размыкание ключа изменяет распреде- ление токов в цепи. Значит, нужно подобрать такое положение Цвижка В, при котором показания гальванометра не зависят от В°го, замкнут или разомкнут ключ. Сопротивление R* вычисляют to формуле, приведенной в задаче 13.33. Заметим, однако, что в
Ответы, указания, решения 444 реальной цепи такая перестановка приборов создает опасность выхода из строя гальванометра (не рассчитанного, как правило, на большую силу тока). 13.36. Ответ: Л = А = /, = 4 А; Д = I, = I» = 2 А; I» = П = О. Решение. Простейшая эквивалентная схема получается,если «вы- тянуть» в одну линию резисторы 1, 2, 3 и 4, 8, 7 (см. рисунок). Поскольку Я1 : Я2: Яз = Я< : Я8 : Ят, нетрудно показать (ср. с зада- чей 13.32), что потенциалы точек А и С, В и D совпадают: (рд = (рс, (рв (pD. Поэтому ток через резисторы Я5 и Яз не течет: Л = I» = О. После изъятия из цепи этих резисторов расчет токов не вызывает затруднений: U = 4 А, Я1 + Rz + Яз U = 2 А. Rt + Rj + Яз 13.36. Решение. Очевидно, до соединения точек Аз и Вз было (pAi = (рлг, <рвз = (рвг. Однако потенциалы проводников А1А2 и В1В2 отличаются друг от друга (иначе не шел бы ток, например, через
445 Электричество и магнетизм участок А1В1); (pAi > <pm. Значит, флз > фв>, поэтому при соединении точек Аз и Вз по проводнику АзВз пойдет ток (см. рисунок). Следовательно, должен появиться и ток в проводниках AiAz и В1В2. Ток течет в сторону убывания потенциала, поэтому после соединения точек Аз и Вз соотношения между потенциалами таковы: (рлз > фаз; <рлз < <рль флз < флз; фаз > срвь фаз > фвг. Разности по- тенциалов между точками Ai и Bi, Аг и Вг уменьшаются из-за перераспределения токов в цепи. Если сопротивления провод- ников А1А2, В1В2, АзВз пренебрежимо малы, то потенциалы всех шести отмеченных на рисунке точек сравняются. При этом исчез- нут токи через резисторы на участках А1В1, АгВг. 13.37. Ответ: ток /а = течет от точки А к точке В. □л Решение. Непосредственное вычисление тока через амперметр (применение закона Ома) невозможно, т. к. сопротивление ампер- метра Ва считается пренебрежимо малым (Вл О). Однако именно поэтому можно объединить точки А и В в одну (фл = фв) и найти силу тока через каждый из резисторов. Последовательно соеди- ненные участки 1-3 и 2-4 имеют одинаковые сопротивления, U m поэтому напряжение на каждом из них равно Токи через „ г и и Т и и резисторы Bi и Вг соответственно h = и 12 = = g^. Зная токи h и I2, легко определить и ток /а, текущий через амперметр: вследствие закона сохранения электрического заряда I = Ii- I2, откуда 1а = (ток течет от А к В). 13.38. Ответ: 1 = 4 А. Решение. Эквивалентная схема цепи представлена на рисунке.
Ответы, указания, решения 446 Вычислим сопротивление цепи: Лю — = 4 Ом; Л12з = Л13 + Лг =12 Ом; Л1 + Лз полное сопротивление внешней цепи Л = = 8 Ом. Л123 + Ri Ома для замкнутой цепи сила тока через Согласно закону амперметр I - — 13.39. Ответ: I = 0,1 A; U = 14,6 В. Решение. Сопротивления участков цепи: 7? 3 2Л 9R тэ “Г* тэ *5 п п /?12 — ~2^9 -**123 = ТуГЦ = 2R + ~R и 6„ „ 13„ — ^Л; Е1-в — Л19 7Л-ЗЛ = ^л+./34 73 полное сопротивление внешней цепи Ло = Ли> + Л = ^4Л =146 Ом. 6’ Найдем силу тока через источник: I =---= 0,1 А и напряжение на источнике: U = IRo = 14,6 В (или U = £' - Ir = 14,6 В ). 13.40. Ответ: Zi = 2 А. Решение. При произвольных значениях сопротивлений резисто- ров такая схема требует достаточно сложного расчета: ведь в ней нет ни параллельных, ни последовательных соединений. В данном случае, однако, сопротивления в верхней и нижней ветвях цепи пропорциональны друг другу, т. е. мы имеем дело со сбалан- сированным мостиком (см. задачу 13.33). Значит, при р;.< чете цепи резистор R можно просто изъять (ток по нему не идет) Тогда полное сопротивление внешней цепи составит 2 Ло = "х(Л1 + Ла) = 8 Ом. О ё Следовательно, полная сила тока в цепи Z =------= 3 А, на- Ло + г пряжение во внешней цепи U = IRo = 24 В, сила тока через первый и резистор Zi = —---- = 2 А. R1 + /?2
447 Электричество и магнетизм 13.41. Отрет: /а = 6 А. Указание. Если «выбросить» из схемы амперметр, эквивалентная схема примет показанный на рисунке вид. Используя затем закон сохранения заряда (см. задачу 13.37), получаем Ia = I - It или 1а = 1г + Ii. 13.42. Ответ: R = 2 Ом. gR Решение. В первом случае согласно закону Ома 17г = 1Л = —-. R + г Во втором случае полное сопротивление внешней цепи равно R/2, так что c-R U = 2 = 6R Л R . r R + 2г 2 + г Л Ui R+2r „ 2U2-Ui Отсюда -77- =----и R = г • —--= 2 Ом. 02 R + Г U1 - U2 13.43. Ответ: ё = 20,4 В. Решение. Заметим, что Ui + U2 > U'i, следовательно, сопротив- ления вольтметров Ri и 7?г сравнимы с внутренним сопротив- лением г батареи. Тогда из соотношения------= следует, что Ri + г Ri Л I х* ё = U'i—. Значит, для решения задачи достаточно установить Л1 соотношение между Ri и г. При последовательном соединении L СА и* t вольтметров через них текут одинаковые токи; значит, -=- = -g- и ' Л1 /и
Ответы, указания, решения 448 Rz Uz „ SRt 8R1 = 77-. Поскольку Ut = —-----, a U1 = —---, можно записать: Rt Ut Rt + Rz + r Rt + r U't _ Bi + Д2 + r Ut Bi + r Отсюда r = B.(-- U* — 1). Подставляя это выражение в формулу для 8, получаем 8 = & = 20,4 В. Uri — C/i 13.44. Ответ: в обоих случаях амперметр показывает 0,67 А. Я2Я3 Решение. Полное сопротивление цепи R = Я> +----—- = 9 Ом. СИ- Яг + ЛЗ r 6 5 . „ ла тока в цепи I = А, напряжение на Rz и Rz составляет ft о U23 = 8 - IRt = 8 В; амперметр показывает силу тока через резис- . _ Т т Uzs 2 А _ тор Rz: 1з = 1л = А. Если поменять местами амперметр и /Сз о источник ЭДС, то R' = Rz + = 14,75 Ом, = 1 А, 17'12 = Ri + Rz Гъ U'iz 2 = 2,8 В, Га - Г, - А. Является ли случайным совпадение lit О этих двух результатов? Если не торопиться с подстановкой в формулы численных значений, то получим выражение 8Rz RiRz + R1R3 + R2R3 в выражении для Гл величины Ri и Rz меняются местами; при такой замене значение выражения не изменяется, поэтому Гл = 1л. 13.45. Ответ: Rv = 10 кОм. Решение. При последовательном подключении вольтметра напря- жение на нем Ut=- 6Rv , г + Я + Яу гт бЯЯу С/г - “тг-z---“• RRv + гЯ + гЯу Я2 и Uz9 получаем Яу = — = 10 кОм. Следует, при параллельном Приравнивая Ui однако, отметить, что при любых значениях Rv и R, удовлетворя- ющих неравенству г < R < Rv (а это условие, как правило, выпол- няется), разница между Ui и Uz может оказаться неуловимо малой: Ut » Ut» 8.
449 Электричество и магнетизм 13.46. Ответ: U = U =^±^£1. Fi + Гг Fi + Гг Решение. Ток в цепи идет против часовой стрелки. Сила тока _ 81 + 8г _ I -------. Для определения разности потенциалов <рл - <рв (т. е. Г1 + Гг напряжения U на источниках) можно воспользоваться, например, следствием из закона Ома для замкнутой цепи: U = 61-IR^6ir-~-ri. Г1 + Гг Обратим внимание, что U = О при — = —, т. е. когда у обоих Г1 Гг источников одинаковы токи короткого замыкания. Естественно, U = О и для одинаковых источников. Если изменить полярность включения второго источника, в формуле для U знак перед 8г также изменится: _ ёлГг + 8гГ1. Г1 + Гг Для источников с одинаковыми ЭДС получим U = 8t — 8г. Отметим, что разность потенциалов точек А и В можно рас- сматривать как ЭДС батареи из двух источников (ведь ЭДС рав- няется напряжению на источнике при разомкнутой внешней цепи). Внутреннее сопротивление этой батареи составит, конечно, R =-------Разумеется, параллельное включение источников с О + Гг разными ЭДС приведет к разрядке источника с большей ЭДС. 13.47. Ответ: <pAi - (рлг = О, <pAi - <рлк = О. Решение 1. Рассматриваемая цепь обладает симметрией по отно- шению к поворотам: если мы повернем ее на одно звено по часовой стрелке, то цепь в целом не изменится, однако точка Ai займет место точки Аг. Однако потенциал любой точки цепи не мог измениться из-за поворота всей цепи, поэтому <pAi = (рлг. Отсюда сразу же следует, что <рдк = <рл1 при всех k. Решение 2. Согласно закону Ома для замкнутой цепи сила тока Т N8 8 1 = т. е. каждый элемент работает в режиме короткого замыкания. Значит, напряжение на каждом из элементов Ui — = 8 - Ir = О. Соответственно обращается в нуль и сумма любого числа этих напряжений: <рл - q>Ai = (k - l)C7i = О. Отметим, что такой же результат получится и для элементов различных. Но обладающих одинаковым током короткого замыкания.
Ответы, указания, решения 450 13.48. Ответ: £7 = 0, если между точками Ai и Аь имеется четное количество элементов; U = ± 8, если между этими точками — нечетное количество элементов. [NR [Nr~ 13.49. Ответ: п = т = \ У г у R Решение. Полная ЭДС батареи ng (здесь g — ЭДС каждого из „ „ „ „ пг _ элементов). Внутреннее сопротивление всей батареи —. Поэтому пё __ W —. Учитывая, что т = —, полу- fl П сила тока через резистор I = пг т чаем _ Nn&________NS ~ n2r+NR~ nrN ' nr -I-----------fl n Следует подобрать такое n, чтобы сумма nr + —fl была мини- мальна. Согласно известному неравенству (см. математическое приложение) получаем nr + —Я > 2Л nr^-R - 2^1 NrR = const. Ра- п у п N„ , венство достигается только при nr = —Я (при этом внутреннее сопротивление батареи совпадает с сопротивлением fl внешней / я I 7 цепи). Отсюда п = \N—, т = \ N^. Подставляя числовые зна- У г ул чения N, г и Я, получаем по = 14,1, то = 7,07. Ближайшие к по значения п, соответствующие целым zn: ni = 10 (тогда nit = 10) и пг = 20 (тогда тз = 5). Сравниваем соответствующие значения силы тока: Т 10g т 10g т Итак, задача имеет два решения: п: = 10, mi = 10 и пз = 20, тз = 5. Можно предложить и лучшее решение этой задачи: если вообще не использовать два из ста элементов, то при п = 14и/п = 7 '14g о _g получаем --------— = 3,5—, что.больше полученных выше зна- 2г + R г чений силы тока. 13.60. Ответ: N = 8. Решение. Будем рассматривать батареи типа описанной в задаче 13.49 (очевидно, последовательное соединение групп параллельно
451 Электричество и магнетизм соединенных элементов дает тот же ре- зультат). Для обеспечения максималь- ной силы тока при заданном должно . lKrR . f. выполняться условие п - (обоз- начения те же, что и в задаче 13.49). приведенной в решении задачи 13.49, Тогда из формулы (1), 8^N следует, что I = '"~т_ , откуда N = —= 8; п = 4, т = 2 (см. рисунок). © 13.51. Ответ: 8 = 8,7 В, г = 0,05 Ом. Решение. В конце зарядки и начале разрядки 8 и г аккумулятора можно считать одинаковыми. Когда ток в цепи равен нулю, напряжение на аккумуляторе совпадает с его ЭДС. При разрядке (см. рис. а) напряжение меньше ЭДС: Uг = 8 - hr. (1) При зарядке (см. рис. б) заряды движутся против сторонних сил в аккумуляторе. Это возможно лишь при напряжении, пре- вышающем 8'. Ui = 8 + hr. (2) Решая систему уравнений (1) и (2), получаем г = ——— = h + 1г = 0,05 Ом; 8 = --1——= 8,7 В. Л + Л 13.52. Ответ: h = 21 A; h = 6,3 А; 1з - 14,6 А. Решение 1. Согласно результатам решения задачи 13.51 напря- жение между точками А и В (см. рисунок) составляет U = 8t - hr\ = = 82 + /2Г2 = I3R. Из закона сохранения электрического заряда следует: h = I2 + 1з. Решая систему этих уравнений, получаем: 8i(R + Гг) - 8гЯ -11 _ —----—— _ Ji А Г1Гг + Rr, + Rn : - J > ‘ 15*
Ответы, указания, решения 452 612? - <5г(П + 7?) о д. 1г =---------—-----—— = 0,3 А; Г1Г2 + /?л + Rr2 ё1Гг + ёгГ1 Г1Г2 + Rri + Rr2 14,6 А. Решение 2. Воспользуемся «методом суперпозиции», пригодным для любых систем, где связь между токами и ЭДС линейна, т. е. все элементы цепи подчиняются закону Ома. Предположим сна- чала, что батарея «потеряла» свою ЭДС, сохранив внутреннее сопротивление. Тогда токи, обусловленные ЭДС 61, определяются соотношениями: . 61(7? + Гг) . SiR . ё1Гг 11 =-------------; i2 =------------; 13 = ---------—. ига + Rri + Rr2 nr2 + Rri + Rr2 Г1Г2 + Rri + Rr2 Если же «утратил» ЭДС генератор, то распределение токов имеет вид (положительными считаются направления токов, пока- занные на рисунке): . , 6г7? . , 6г(Л + Гх) . , 62Г1 й = -----------i2 ----------—------—; 1з =-----------—. Г1Г2 + Rri + Rr2 Г1Г2 + Rri + Rr2 пгг + Rri + Rr2 При «включении» обеих ЭДС получаем It + i't, I2 = 12 + 1'2, 1з = 1з+1з', что совпадает с результатом рещения 1. <эх + ё2 13.53. Решение. Сила тока в цепи I =----------. Напряжение на ri + Г2 + R зажимах, например, первого элемента составляет Ui = Si — Iri = Si(r2 + R) — S2ri „ = --------------. Приравнивая это выражение нулю, находим Л + г2 + R R = - гг. Полученное значение R положительно при ~ Величина Sjr дли источника ЭДС представляет собой ток корот-
453 Электричество и магнетизм кого замыкания. Значит, можно сделать равным нулю напря- жение на том из элементов, у которого ток короткого замыкания меньше. Если величина токов короткого замыкания одинакова (— - —то при R 0 напряжения на обоих элементах обраща- ются в нуль одновременно. 13.54. Ответ: ё = 0,60 В. Решение. Описанная в задаче ситуация возникает при измерении ЭДС компенсационным методом. Напряжение на резисторе с- сопротивлением Ri равно ЭДС ё аккумулятора. Поскольку ток через аккумулятор все равно не идет, эту ветвь цепи можно убрать, не изменив напряжения на резисторе. Сила тока в цепи 1 =---&------1 Го Ч" Я1 + поэтому ё = IRi = ёо--—-----= 0,60 В. Го + Rl + В2 13.55. Ответ: ё = 36 В; г = 0,50 Ом. „ Л - R1+ Г 1г Указание. Из закона Ома для замкнутой цепи следует:----= у. Rz + г -О . 13.56. Ответ: I = -------------- (г + Bi)(B + Rz) + rRi Решение. Сопротивление внешней цепи, подключенной к источ- Ri(Rz + R) _ _ нику тока, Rcd =---------напряжение между точками Сих) Ri + Rz ч* R составляет Ucd = . Следовательно, ; Г + Rcd r = UcD ==<SB1 Rz + R r(Ri + Rz + R) + Ri(Rz + В) (r + Bx)(B + Rz) + rBi Интересно, что этот результат можно записать в виде ,,, SRi . _ rBi „ ,,, где ё =-----—, г = Rz + -----. Величина ё совпадает с напря- r + Ri г + Bi жением между разомкнутыми клеммами А и В; величина г* совпадает с сопротивлением двухполюсника, показанного на ри- сунке к условию задачи, между точками А и В. Таким Ьбразом, рассматриваемая цепь по отношению ко всякой нагрузке, под- ключаемой между клеммами А и В, эквивалентна источнику тока с ЭДС, равной ё4, и внутренним сопротивлением f. Можно данйЬ
Ответы, указания, решения 454 зать, что этот вывод справедлив для любого двухполюсника, состоящего из линейных (т. е. подчиняющихся закону Ома) эле- ментов. С^С 13.57. Ответ: I = 10— Uo + IoR Указание. См. задачу 13.56. Рассматриваемый двухполюсник можно заменить эквивалентным источником тока, у которого ЭДС равна Uо, а сила тока при коротком замыкании 1о. 13.58. Ответ: Ui = <5 Ui = 8-~^—-. Ci + Сг + 2С Ci + Сг + 2С Решение. Цепь разомкнута, тока в ней нет. Поэтому потенциалы обоих концов резистора одинако- вы; эти точки можно совместить, убрав резистор. На рисунке при- ведена эквивалентная схема. Полная емкость батареи из четырех кон- „ (Ci + СХСг + С) денеаторов С. = Ci + ft+2c • Напряжения на конденсаторах: ту — .8Со — £ Сг + С 1 " С, + С “ ёС1 + Сг+ 2С’ ' 13.59. Ответ: Ui = 0, С7г = Сз = 8. Указание. См. решение задачи 13.58. 13.60. Ответ: 8 = 222 В. Решение. Определим сопротивление R внешней цепи: _ R2R3 _ _ Н1Д234 __ _ /?гз, ~ ~~ + Лз = 60 Ом; R = „ „— - 36 Ом. Ri + Ra Ri + RiM Находим напряжение на конденсаторе (а, следовательно, и на источнике): Ci + Сг + 2С U = £ = 216 В. Сг Тогда сила тока в цепи I = = 6 А, и ЭДС источника состав- ляет 8 = I(R + г) = 222 В. _ 48CR(r + R) 13.61. Ответ: о =------1-----—. 4 (2R +ТХ2Я + Зг) Указание. Эквивалентные схемы цепи при замкнутом и разомк- нутом ключе приведены соответственно на рис. а и б. Обратите
4Б5 Электричество и магнетизм внимание на то, что знаки зарядов на обкладках конденсатора изменяются. 13.62. Ответ: q = О Решение. Резисторы соединены последовательно, поэтому напря- 3(3 жения на них относятся как 3:1. Значит, фв - срв = -j-; фв - <ре = ё = -j. Найдем теперь <рд — потенциал точки А. Заряды соединенных с точкой А обкладок конденсаторов равны 2С(<рл - фв), ЗС(фд - ч>е), С(фл - фв). Поскольку эти обкладки образуют изолированную систему, сумма их зарядов равна нулю. Записывая это условие и выражая фп и фк через фв, получаем: фд - фв = Заряд на конден- _ ч Сё саторе С составляет q = С(<?л - фв) = О Предостерегаем читателя от попытки рассматривать соеди- нение конденсаторов 2С и ЗС как последовательное! Это было бы верно только в случае очень малой емкости конденсатора, вклю- ченного между точками А и В. 13.63. Ответ: Напряжение понизится на 8 В. Решение. По номинальной мощности приборов (т. е. соответству- ющей нормальному напряжению сети) определим их электричес- U2 Ц2 Кие сопротивления: Л = = 480 Ом, R2 = -=- = 97 Ом. До вклю- * 1 л 2 URi чения электрокамина напряжение на лампе было l/i « —------- = ,.. ;i. ,t . ri.„ Bo.+ Bj * 218 В. После включения электрокамина сопротивление по-
Ответы, указания, решения 456 ’ « - ' » RtRz D. „ требителеи электроэнергии в комнате составит ли = —-— = 81 Ом. Ri + Rz URn Теперь напряжение на лампе и электрокамине будет С/г = —-— = Ro + Лхг = 210 В. Таким образом, напряжение на лампе понизилось на Д U = Ui - Uz = 8 В. В действительности, однако, при таком изменении напряжения следовало бы учитывать изменение сопротивления нити накала из-за понижения ее температуры. 13.64. Ответ: R = г. Указание. В обоих случаях через нагреватель течет ток одинако- <5 NS вой силы. Значит,-------=-----—. Г . „ Nr + R N + R 13.65. Ответ: 8 лампочек. U2 Решение 1. Сопротивление каждой из лампочек Ro = -5-. Обо- Jro значим количество их jV. Тогда полное сопротивление всех парал- Во лельно включенных лампочек равно R = сила тока в цепи ё Ro I =---и напряжение на лампочках U = IR = ё—-------Отсюда г + R Nr + Ro Ц(ё~и) о гРо 8‘ Ра Решение 2. Сила тока через каждую из лампочек 1о = полная Т -тт NPo тт С т £' ХРоГ сила тока в цепи I = NIo = ~ц~- Тогда U = ё - 1г = ё-отсю- р р г г 13.66. Ответ: U = ё-----------— = 30 В; Л = = 3,75 А; Я1Й2 + r(Ri + R2) R> П = = 1,25 А; Р = и(1г + Ь) = 150 Вт. Кг 13.67. Ответ: Rx = 23 Ом. Решение. Сопротивление участка АВ при разомкнутом ключе 2 1?лв1 = 2Во, при замкнутом ключе ВАвг = -xRo. Сила тока в цепи О 7 = ,мощность на участке АВ составит Л + Лдв ' . ,.v, . .. . - . . - !' ? . Л <- - 1 : -V ' I : IaU •
457 Электричество и магнетизм Р -г» р" " " - да + W’ Из условия постоянства мощности получаем 2Ro_________3До (R. + 2R,)2 = 2До)2- % о г- 2 2Ro Отсюда Rx + 2-Ro = v3(Rx + ^Ro), значит, R, - -т=- = 23 Ом. 3 УЗ 13.68. Ответ: г = ^RiRi. Решение. Обозначим ЭДС источника через 8. Резистор с сопротив- 82R лением R потребляет мощность Р = I2R =--------Согласно усло- (Я + г)2 Ri R2 вию Pi = Рг, откуда--------- =--------. Из этого уравнения полу- (Ri + г)2 (Ri + г)2 , чаем Г = >/Я1Й2. 13.69. Ответ; г - 'iRiRi = 3,0 Ом. Указание. См. задачу 13.68. 13.70. Ответ: 1Ю = 25 А. ё Решение. Ток короткого замыкания = —. Внутреннее сопротив- ление источника мы можем определить из условия равенства мощностей (см. задачу 13.68): г = ^RiR2. Величину 8 можно теперь определить из выражения для мощности при подключении любого из нагревателей, например, Р = 82Ri (Ri + г)2’ Отсюда получаем (с уче- том г = <RiR2): 8 = Vp(a®T + Vr^) и = 25 А. 13.71. Ответ: первая лампочка потребляет мощность 9,6 Вт, вторая ; 14,4 Вт. Полная мощность 24 Вт. и2 Решение. Мощность определяют по формуле Р = —, где R — К сопротивление участка цепи, U — напряжение на этом участке- Значит, сопротивление второй лампочки больше: Rz> Ri. Очевид- > но, при последовательном включении обе лампочки горят вепря-* ным накалом, т. к. напряжение на каждой из них не достигает номинального (220 В). Более того, даже полная потребляемая, мощность Р меньше номинальной мощности Рг = 40 Вт более слабой лампы (ведь при последовательном соединении сопротив-
Ответы, указания, решения 458 ление цепи R = Ri + R2 превышает сопротивление каждой из лампочек). Кроме того, при последовательном соединении ярче будет гореть лампочка с большим сопротивлением, т. е. Рг > Р/, ведь сила тока I является одинаковой для обеих лампочек, а Р = = 72Л. Произведем расчет, считая сопротивление лампочек неизмен- U2 U2 ним, не зависящим от накала1’: Ri = Рг = При последова- Г1 Г 2 r и Р1Рг тт „ тельном включении / = —----- - ------—. Найдем мощность каж- R1 + ЩРх + Рг) дой из лампочек при таком включении: Р1Рг2 Р12Рг PS = PRi = = 9,6 Вт; Рг = - = 14,4 Вт. (Р1+Р2)2 (Рх + Рз)2 Р1Р2 Полная потребляемая мощность Р = Рх' + Рг = ~--—- = 24 Вт. Рх + Рг Ярче горит лампочка, на которой написано «40 Вт». 13.72. Ответ: см. рисунок. Указание. Достаточно обратить внимание, что при параллельном включении трех лампочек по 4 Вт или двух по 6 Вт мы получаем «потребителей» с одинаковой расчетной мощностью — 12 Вт. Значит, эти участки цепи имеют одинаковое сопротивления и при последовательном включении на них будут одинаковые напря- жения. 13.73. Ответ: = 20 А. Решение. Из уравнения Р = I2R определяем сопротивление на- грузки в обоих случаях: R Си — Кг — ” На самом деле при увеличении напряжения на лампочке от 0 до 220 В ее сопротивление может возрастать примерно в десять раз.
459 Электричество и магнетизм Из закона Ома для замкнутой цепи следует: & = Ii(r + Bi) = I-Ajr + Ba). _ IiBi — I2R2 c, IiIARi — B2) _ Отсюда г =---------—, 6 ---Тогда I2 ~ 11 22 — 11 _ Ii/JRi - В2) _ P1I2* - P2Z12 _ r I1R1 — I2R2 P1/2 — P2/1 13.74. Ответ: через 8 минут; через 36 минут. Решение. Обозначим сопротивления обмоток Bi и Вг, напряжение в сети U. Количество теплоты, необходимое для доведения воды до U2 U2 U2 (U2 U\ _ Bi кипения, Q = -5-ti = -g-t2 = —-~tmJ, = I-5- + -б~П1ч>. Отсюда — = ill Л2 + /?2 'Л1 £12' £12 tl = —; время закипания воды при последовательном включении 12 tnoM = ti + <2 = 36 мин, при параллельном включении обмоток tn»P = Й<2 о =-----= 8 мин. tl + t2 При решении подобных задач обычно предполагается (и даже не всегда оговаривается), что необходимое количество теплоты во всех случаях одинаково. На самом деле теплопередача от чайника к окружающему воздуху достаточно велика. Потери теплоты тем больше, чем медленнее происходит нагревание. Поэтому они мак- симальны при последовательном включении обмоток и минималь- ны — при параллельном. Значит, t^ > 36 мин и #Мр < 8 мин. 13.75. Ответ: d = 2,7 мм. Решение. Падение напряжения на проводах равно kU (k = 0,01); значит, напряжение на выходе линии электропередачи равно (1 - k)U. Из соотношения Р = (1 - k)UI получаем силу тока в Р Р kU2 линии: I = ———- « уу. Значит, сопротивление линии В = — (1 - K)U и Р Считая линию двухпроводной, записываем: м где р — удельное сопротивление меди. Отсюда d - •== 2,7 мм. 13.76. Ответ: увеличить в 20 раз. Указание. См. решение задачи 13.75, где приведено соотношение между A, U и Р.
Ответы, указания, решения 460 13.77. Ответ: — = 1,6. Рг Решение. При равномерном движении по горизонтальному пути сила тяги Ft = jimg (т — масса электропоезда); при равномерном движении на наклонном участке Fz = mg[ji + k) (см. задачу 4.16; мы учли, что k « 1). Соответственно развиваемая механическая мощность на горизонтальном участке Ni = FiVi = ]ungvi, на подъеме Nz = FzVz = mgvz(\i + k); потребляемая электрическая мощность Pi = Uh, Pz = UIz, где U — напряжение в сети. Считая КПД двигателя неизменным, получаем: М _ М Pi “ Рг' _ V1 /1/, k\ - Отсюда — = — 1 + — = 1,6. Vz 12' р/ 13.78. Ответ: I = 47 А. Решение. Выделившееся тепло равно I2Rx, следовательно, I2Rx = mc(t„„ - to), где R — сопротивление проволоки, т — ее масса, с — удельная теплоемкость железа, t™ — температура плавления железа. Учи- „ I 41 кГР тывая, что R = р-;; = р—; и т = dl—r-, получим о л.О 4 = 47 А. Pt Здесь р и d — соответственно удельное сопротивление и плот- ность железа, I и S — длина и площадь поперечного сечения проволоки. 13.79. Ответ: алюминиевая проволока. Решение. Фактически вопрос сводится к тому, какая из трех проволок раньше нагреется до «своей» температуры плавления. Указание на высокое напряжение означает, что процесс нагре- вания происходит быстро, так что теплообменом и потерями тепла можно пренебречь. Определим время, в течение которого прово- лока нагревается до температуры плавления: lTtD2\2 dc(ta„ - to) Т 4/ ) ’ р (см. задачу 13.78). Поскольку I и D для всех трех проволок _ dc(tпл ~ to) одинаковы, перегорит та, у которой величина —1------ наимень- шая. Подставив табличные данные, увидим, что эта величина
461 Электричество и магнетизм минимальна у алюминия. Распространенная ошибка при решении этой задачи — подсчет количества теплоты, необходимого для того, чтобы расплавить целиком одну из проволок. В действи- тельности же при перегорании проволоки плавится очень ко- роткий ее отрезок. 13.80. Ответ: первой перегорит алюминиевая проволока, затем медная и, наконец, железная. Указание. См. задачи 13.78 и 13.79. В данном случае одинаковым для всех трех проволок является напряжение. Первой перегорит pdc(tM - to)Z2 та проволока, для которой время т =-------минимально. 13.81. Ответ: т = 0,21 с; температура проводов повысится всего на = 0,1°С. Указание. См. задачу 13.78. Рис. в Рис. г
Ответы, указания, решения 462 График этой зависимости (гипербола) приведен на рис. а. Напряжение U = IR = ё - Я + г График (также гипербола) — см. на рис. б. Мощность во внешней цепи полная мощность Ро = Г{П + г) = . R + г Графики зависимостей Р(Я) и Po(R) приведены на рис. в. КПД Р R . . „ , цепи и = — =-----(см. рис. г). Исследуем на максимум функцию Ро R + г Р(Я), вычислив ее производную: р'да = (лЬ(<Л + г),-л'2(Л + г)* = 1^- Значит, P(R) возрастает при R < г и убывает при R > г, максимум достигается при R = г (такую нагрузку называют согласованной). При этом КПД цепи составляет всего 50%! 13.83. Ответ: т ~ RC; см. рисунок. Решение. По мере уменьшения на- пряжения на конденсаторе уменьша- ется и сила тока i в цепи; следователь- но, скорость разрядки конденсатора также уменьшается. Вид графика U(t) показан на рисунке. Под временем разрядки понимают время т, за кото- рое напряжение заметно изменится, например, вдвое. Оценить т можно различными способами. Простейший из них — метод размерностей. Из физических соображений ясно, что т может зависеть лишь от В, С и, возможно, от начального напряжения Uo на конденсаторе. Единицы измерения этих величин соответствен- но: 1 Ом, 1 Ф = 1 Кл/В = 1 с/Ом и 1 В. Из величин R, С и Uo можно построить величину с размерностью времени единствен- ным способом: умножить R на С. Следовательно, т ~ RC (точное выражение для т может содержать еще численный коэффициент порядка единицы). Как видим, величина т не зависит оТ Uo. Можно получить тот же результат и иначе: напряжение заметно
463 Электричество и магнетизм уменьшается, когда выделившееся в цепи количество теплоты становится сравнимым с начальной энергией заряженного конден- сатора: г2Дт ~ С17о2 2 • „ • и-° Считая, что i ~ получаем /С т ~ RC. Наметим и точное решение. За малое время At напряжение на конденсаторе Ag IM UM U уменьшается на величину -77 = -77 = -=77, откуда следует и' = - -57;. Решение этого С С гСС лС дифференциального уравнения (с учетом условия U = Uq при t — 0) имеет вид i 1 U = Uge ?, где т = За время т напряжение убывает в е » 2,7 раз, за время 5т — гСС в 150 раз! 13.84. Ответ: I =------------, U = г + Дх(1 - х) 6’2Дх(1 - х) (г + Дх(1 - х))2 ; см. рис. а, б, в. Р = Указание. Обе части реостата сое- динены параллельно, поэтому со- противление внешней цепи равно Дх(1 - х); следовательно, все гра- фики симметричны относительно прямой х = 1/2. Характерный «дву- горбый» вид графика Р(х) можно было предсказать заранее. Дело в том, что максимум Р(х) достига- ется для согласованной нагрузки 6'Дх(1 - х) г + Дх(1 - х)’
Ответы, указания, решения 464 (см. задачу 13.82), т. е. при сопротивлении внешней цепи 2 Ом. При перемещении же движка реостата от любого края к середине сопротивление внешней цепи изменяется от нуля до Л/4 = 4 Ом, поэтому существуют два симметричных положения движка, соот- ветствующих сопротивлению внешней цепи 2 Ом. 14.1. Ответ: v — 0,45 мм/с. Решение. Согласно условию задачи концентрация п свободных электронов совпадает с концентрацией атомов меди. За время х электроны перемещаются на расстояние их; значит, через сечение проводника площадью S пройдет nSvx электронов, которые пере- несут заряд Ад = enSvt, где е — элементарный заряд. Поскольку г Ад „ сила тока 1 = — = enSv, плотность тока связана со средней ско- т ростью v соотношением j = = епи. Концентрацию п атомов меди ъ можно определить из табличных данных следующим образом. В объеме V сосредоточена масса pV (здесь р — плотность меди); масса Af одного атома равна где М — молярная масса меди, Na — In а постоянная Авогадро. Определим число N находящихся в объеме V атомов: М ’ их концентрация N pWA П V М • Отсюда получаем и = = 0,45 мм/с. Заметим, что скорость упорядоченного движения свободных электронов (она называется дрейфовой скоростью) ничтожно мала по сравнению со скоростью беспорядочного движения электронов и атомов. 14.2. Ответ: t = 22ОО°С. Решение. Обозначим сопротивление лампы в рабочем режиме R, а в «исследовательском» режиме Ri. Из данных, приведенных в условии, получаем: R = ~ = 28,8 Ом; Л, = = 2,5 Ом. Г XI Большее сопротивление в рабочем режиме обусловлено более
465 Электричество и магнетизм высокой температурой нити накала. Из формулы зависимости сопротивления проводника от температуры R = Я1(1 + a(t - ti)j , где a — температурный коэффициент сопротивления, получаем , , R - Ri t — ti ч----. a • R\ Подставляя в эту формулу значения R, Ri и табличное значение а, получаем t = 22ОО°С. 14.3. Решение. Часть спирали, на которую попадет вода, заметно охладится за счёт испарения воды. Сопротивление этой части существенно уменьшится (у металлов сопротивление с температу- рой растет) и сила тока в спирали возрастет. Вследствие этого накал той части спирали, куда не попала вода, увеличится (из-за этого спираль может перегореть!). 14.4. Ответ: железный стержень должен быть длиннее угольного в 62 раза. Решение. Сопротивление каждого из стержней при температуре t составит Я = ро-~(1 + at), О где ро — удельное сопротивление при 0°С, a — температурный коэффициент сопротивления, I — длина стержня. Мы пренебрега- ем изменением размеров стержней, т. к. коэффициент линейного расширения твердых тел на один-два порядка меньше температур- ного коэффициента сопротивления. Полное сопротивление соеди- ненных стержней: Я = Р01“оО + ait) + ро2^(1 + azt) — рогтз + po27j + -^(poiZiai + рог/гаг), О о о о о где индекс 1 соответствует угольному стержню, а индекс 2 — железному. Эта величина не зависит от температуры, если коэф- фициент при t в последней формуле обращается в нуль, т. е. при у = - P°‘ai. Значения рм и рог выразим через величины удельных рогаг сопротивлений pi и рг при ti = 20°С (см. приложение, таблица 10): pi,г ро1,ог — —----. 1 + ai,2ti Следовательно, h piOh(l + a»ti) __ у* ---------------= 045. « Ргаг(1 + aiti) Итак, железный стержень должен быть длиннее угольного в 62
Ответы, указания, решения 466 раза. Разумеется, решение существует лишь благодаря тому, что аз/ai < О (т. е. у одного из стержней сопротивление при нагре- вании растет, а у другого — уменьшается). 14.5. Решение. При включении нагревателя ток в проводах, подво- дящих электроэнергию к квартире, резко возрастает. Соответст- венно возрастает потеря напряжения на этих проводах и из-за этого уменьшается напряжение на лампочках. По мере нагре- вания сопротивление спирали увеличивается в несколько раз, поэтому ток в цепи уменьшается. В результате, уменьшается и потеря напряжения на проводах, напряжение на лампочках воз- растает, хотя и не достигает первоначальной величины. 14.6. Ответ: I = 0,34 м. Решение. Сопротивление нити накала в рабочем состояний и* R - С другой стороны, оно равно „ I 4Z B-ps = ₽rf- где р — удельное сопротивление накаленной нити. Учитывая, что = =~, получим I =-----= 0,34 м. Здесь р0 — удельное сопро- ро * ° 4Рр0Т тивление вольфрама при температуре То = 293 К (см. табл. 10 в приложении). 14.7. Ответ: q = pj2. Указание. За время At во всем объеме проводника выделяется количество теплоты Q = I2R&t. Воспользуйтесь соотношениями: q = V = SI; I = jS; R = р L 4 VM 1 S 14.8. Ответ: 171 = 525 В, Uz = 131 В. Решение. Нить перегорит, когда нагреется до температуры плав- ления вольфрама Т = 3650 К. Удельное сопротивление вольфрама Т при этой температуре р = ро^г (см._ решение задачи 14.6), сопро- „ I 4TpoZ тивление нити R = ртг = —выделяющаяся мощность >Si Toiuh2 р _ 41 _ I^Todi2 ~ R~ 4TpoZ ’ Практически вся выделяющаяся мощность расходуется на теп- ловое излучение. Значит, (1) Р =t0T*ndil, (2)
467 Электричество и магнетизм где luiil — площадь поверхности нити. Приравнивая выражения (1) и (2) для мощности, получаем 171 = = 525 В. X 1 0U1 Более толстая нить перегорит при более низком напряжении: 17г = С71Л/^ = 131 В. Это объясняется тем, что при увеличении диаметра нити выде- ляющаяся в ней мощность согласно формуле (1) возрастает про- порционально d2 (ср. с задачей 14.7; заметьте, что объем нити пропорционален d2), а площадь поверхности, с которой проис- ходит теплоотдача, растет медленнее (пропорционально d). 14.9. Ответ: Л = —МИ— =15 мкм (здесь М — молярная масса neNApS никеля, р — его плотность). 14.10. Ответ: W = 130 МДж = 37 кВт-ч. Решение. Согласно закону электролиза масса полученного алюми- М ния т = kl&t, где k = — электрохимический эквивалент алюминия, М — его молярная масса, ап — валентность. КПД установки т] = где А = UIAt — работа электрического тока. т Подставляя в формулу для КПД Ibt = -г, получаем R Ат] mUeNtji Мг\ = 1,3 108 Дж = 37 кВт-ч. 14.11. Ответ: Т = - 940 К (здесь В — универсальная газовая постоянная, п = 2 — валентность кислорода). Указание. Воспользуйтесь законом электролиза и уравнением Мен- делеева-Клапейрона; не забудьте, что выделившиеся в результате электролиза атомы кислорода образуют двухатомные молекулы. 14.12. Решение, а) При последовательном соединении сила тока через обе ванны одинакова (см. рис. а). Поэтому согласно закону 1 2 Puc.t
Ответы, указания, решения 468 электролиза массы выделившейся на катодах меди также одина- ковы: mi - m2. б) Поскольку сопротивление первой ванны меньше (в ней более высокая концентрация свободных носителей заряда), при парал- лельном соединении через эту ванну идет более сильный ток: h > > I2 (см. рис. б). Следовательно, в этой ванне меди выделится больше: mi > m2. 14.13. Решение. В соединении CuCl валентность меди ni = 1, а в соединении CuCh валентность меди пг = 2. Значит, в ванне с C11CI2 ионы меди имеют вдвое больший заряд. Однако за время электролиза через обе последовательно соединенные ванны про- ходит одинаковый заряд. Следовательно, через раствор CuCh проходит вдвое меньшее количество ионов меди, чем через раствор CuCl, и поэтому на катоде ванны с CuCh выделится вдвое меньше меди. То же самое следует и непосредственно из формулы m = (закон Фарадея). 14.14. Решение. На поверхности металла всегда есть неровности. У выступов поверхностная плотность заряда, а следовательно, и напряженность электрического поля больше, чем у гладких участ- ков или впадин. После изменения направления тока образо- вавшийся слой металла становится анодом и начинает растворять- ся. Этот процесс быстрее всего происходит у выступов (плотность тока прямо пропорциональна напряженности электрического по- ля, см. задачу 13.2), в результате выступы растворяются быстрее других участков, и поверхность металла выравнивается, стано- вится более гладкой.
469 Электричество и магнетизм 14.15. Ответ: I = 8,1 • 1(Г14 A; U = 8,1 1(ГП В. Решение. Читатели, знакомые с вольт-амперной характеристикой газового разряда, уже поняли: речь идет об определении так называемого тока насыщения. Высокое напряжение вызывает быстрое упорядоченное движение ионов между обкладками кон- денсатора. Будем считать, что все рождающиеся ионы достигают обкладок, не успев рекомбинировать. Сила тока при этом макси- мальна. Она численно равна общему заряду ионов одного знака, рождающихся ежесекундно: I — enSd. Здесь п = 5 • 104 см’3с-1; емкость конденсатора С = откуда / = —= 8,1 • 10 14 А. Тог- d Ео да U = IR = 8,1 10 11 В. Полученные значения I и U не зависят от величины приложен- ного к цепи высокого напряжения (пока напряжение это недоста- точно велико для превращения разряда в самостоятельный). 14.16. Решение. Горение дуги поддерживается за счет сильной термоэлектронной эмиссии с раскаленного катода. Поэтому охлаж- дение катода прекратит дуговой разряд; охлаждение же анода на характер разряда существенно не повлияет. 14.17. Ответ: и = 2700 км/с. Указание. Кинетической энергией, переданной электроном атому, можно Пренебречь вследствие большого различия их масс. Поэто- му можно считать, что энергия Wк электрона равна энергии mv2 ионизации: Wk = = е<р. & 14.18. Ответ: Wk = 43 эВ = 6,9 КГ18 Дж; v = 20 км/с. Решение. Поскольку рассматривается столкновение частиц с прак- М ,,, тически равными массами т» = (Az — молярная масса неона), далёко не вся энергия Wk может быть затрачена непосредственно на ионизацию: обе частицы после столкновения будут иметь скорости, сравнимые с начальной скоростью vo иона, а следова- тельно, надо учесть их кинетическую энергию после столкновения (ср. с задачей 14.17). Наибольшая часть энергии будет затра- чена на ионизацию при неупругом ударе (см. задачу 4.48), когда после удара обе частицы имеют скорости -jj-. При этом их суммар- т°\2/ Wk ная кинетическая энергия 2—и из закона сохранения
Ответы, указания, решения 470 энергии следует: Жк = е<р + Отсюда Wk = 2еср = 43 эВ — 6,9 • 10 18 Дж и v = = = 2,0« 104 м/с. Как видим, ион должен иметь вдвое большую кинетическую энергию, чем электрон (см. задачу 14.17). 14.19. Ответ: Т = 1,7 105 К. 14.20. Решение. Дело не только в том, что для ионизации молекул газа ионы должны иметь кинетическую энергию большую, чем электроны (см. задачи 14.17 и 14.18). Главная причина неэф- фективности ударной ионизации с помощью ионов в другом: ионы из-за больших размеров сталкиваются с другими частицами го- раздо чаще, чем электроны, т. е. имеют намного меньшую длину свободного пробега. В результате при столкновениях ионы теряют кинетическую энергию и не могут приобрести в электрическом поле такую же большую кинетическую энергию, как электроны. 14.21. Ответ: уменьшается. Решение. Количество образующихся ежесекундно в единице объема германия свободных электронов при легировании практически не меняется (оно определяется температурой полупроводника). А вот вероятность «гибели» свободного электрона в результате встречи с дыркой и рекомбинации резко возрастает — ведь дырок стало намного больше. Следовательно, динамическое равновесие между процессами рождения и гибели свободных электронов достигается при значительно меньшей концентрации свободных электронов, если в полупроводник внесена акцепторная примесь. 14.22. Решение. При нагревании увеличивается скорость беспоря- дочного движения свободных носителей заряда в полупроводнике. В результате происходит диффузия свободных носителей заряда от нагретого конца к холодному, ведущая к разделению зарядов. У холодного конца полупроводника возникает повышенная кон- центрация свободных носителей заряда. Если основными носи- телями заряда являются дырки (т. е. полупроводник принадлежит К p-типу), то потенциал выше у холодного конца. Если же основ- ные носители заряда — электроны (полупроводник n-типа), то более высоким оказывается потенциал нагретого конца, 16.1. Решение. Согласно принципу суперпозйции В = Bi + Ва; направление векторов Bi и Вг определяется правилом буравчика
471 Электричество и магнетизм (см. рисунок). Поскольку Bi = Вг, вектор В образует с плоскостями обоих обручей углы по 45°. К задаче 15.1 15.2. Ответ: В = О; нулевой вектор не имеет направления. Решение. Радиальные провода не создают магнитного поля в центре кольца. Поле в центре кольца создается токами, текущими от А к В по малой и большой дугам кольца (см. рисунок). Поскольку обе дуги соединены параллельно, токи в них обратно пропорциональны сопротивлениям дуг, а, значит, и их длинам: h /г Iz h' Т‘ е‘ Illi = I2I2. (1) Согласно правилу буравчика ток h создает в центре кольца магнитное поле, направленное вниз; ток I2 — поле, направленное вверх. Чтобы сравнить эти поля по величине, разобьем каждую из дуг АВ на одинаковые малые участки. Величина магнитной индукции поля, создаваемого в центре кольца каждым участком, пропорциональна току, протекающему по этому участку; количес- тво же участков в дуге пропорционально ее длине. Значит, маг- нитная индукция поля каждого из токов h и 12 прямо про- порциональна произведению II. Согласно уравнению (1) величины индукции полей обоих токов одинаковы. Поэтому в центре кольца поля полностью компенсируют друг друга, т. е. В = О. 15.3. Решение. На рис. а показано распределение токов в ребрах куба. Все 12 проводников можно разбить на пары, состоящие из параллельных проводников одинаковой длины (1-3, 2-4, 5-11, 6-12, 7-9, 8-10). По обоим проводам в паре текут одинаковые токи; центр куба (точка О) расположен посередине между проводами пары (см. рис. б). Очевидно, созданные этими токами поля в точке О компенсируют друг друга. Поэтому магнитное поле в точке О отсутствует.
Ответы, указания, решения 472 Рис. а, б к задаче 15.3 15.4. Решение. Предположим, рам- ка расположена в однородном маг- нитном поле, как показано на рис. а. Тогда на противоположные сторо- ны рамки действуют согласно пра- вилу левой руки равные по модулю и противоположно направленные силы Ампера (на рис. б показан вид сверху). Возникает пара сил, разворачивающая рамку. Враща- ющий момент этой пары сил обра- щается в нуль лишь тогда, когда плоскость рамки образует прямой Рис. а угол с направлением вектора магнитной индукции В. Равновесие является устойчивым, если направление тока в рамке связано с направлением вектора В Рис. в Рис. в
473 Электричество и магнетизм правилом буравчика (при этом силы Fi и Рг являются для рамки растягивающими). Итак, однородное магнитное поле (например, поле Земли) оказывает на рамку с током только ориентирующее действие. Поле магнита, в отличие от поля Земли, нельзя считать однородным. У магнита оно заметно усиливается по мере при- ближения к полюсам. В этом поле (см. рис. в) силы F> и Fz имеют отличную от нуля равно действу ющую. В результате рамка не только повернется, но и втянется в область более сильного поля, т. е. притянется к полюсу магнита. Совершенно аналогично ведет себя и магнитная стрелка. 15.5. Решение. После замыкания ключа по цепи пойдет ток. Соседние витки пружины, по которым ток протекает в одну сторону, притягиваются друг к другу. В результате пружина сжимается; если ток короткого замыкания источника достаточно велик, нижний конец пружины выходит из ртути и цепь разры- вается. Когда ток исчезает, пружина опять расправляется и замы- кает цепь. Весь процесс повторяется. Таким образом, система совершает колебания, период которых зависит от характеристик пружины, силы тока и глубины погружения в ртуть нижнего конца пружины.. Если ток короткого замыкания источника недо- статочно велик, чтобы «выдернуть» нижний конец пружины из ртути, то пружина просто сожмется. 15.6. Ответ: I = 2,0 А. Ток должен идти слева направо. Решение. Натяжение гибких проводов исчезает, если действую- щая на горизонтальный проводник сила Ампера Fa компенсирует силу тяжести mg. Из правила левой руки следует, что ток по проводнику должен идти слева направо (тогда сила Ампера направ- лена вверх). Поскольку Fa = IBl = mg, получаем I = -==- = 2,0 А. 15.7. Ответ: В = = 98 мТл. Указание. Стержень сдвигается с места, когда сила Ампера Fa = = IBl превышает максимальную силу трения покоя. 15.8. Решение. Пусть магнитная индукция поля равна В, сила тока в проводнике I, концы проволоки расположены в точках Ai и Аг. Выберем систему координат, как показано на рисунке, и рассмотрим силу AF, действующую на малый элемент проволоки Длиной AZ, а также ее проекции на оси координат: AF = IB&h AFx = AFsina; AFW = AFcosa.
Ответы, указания, решения 474 Тогда F = F, = £aFx = /B^AZsina; F, = £aF„ - /B^AZcosa. Учтем, что AZsina = (AZ)W; ^(AZ)„ = z/A2 - j/ai = 0; аналогично AZcosa = (AZ)X; ^(AZ)X - xA2 - xAi = |AiA2|. Итак, Fx = 0; Fv = /В|А1Аг|. Такая же сила действовала бы на прямолинейный провод AtAz. Из полученного результата следует, в частности, что в однородном магнитном поле для замкнутого проводника с током сила F равна нулю. 15.9. Ответ: ст = 42 кПа. Решение. Рассмотрим малый элемент кольца (см. рисунок). На него действует сила Ампера Fa — IBM = IBRa и две равных по модулю силы упругостц. Из условия равновесия следует: _ Fa IBRa Fym, =----=-------. 2sin<y 2sin^ При малых a можно считать, что sin|^ = Следовательно, V Т D D yup IBR AQ ТТ Fyap = IBR и су = -х- = - 42 кПа. о о 15.10. Ответ: a = arctg^^ = 45°. mg Указание. См. рисунок (ток по проводнику идет от нас). 15.11. Решение. Проводники с током в целом электрически нейт- ральны, и взаимодействие между ними — только магнитное. Согласно правилу буравчика магнитное поле первого проводника в области, где находится второй проводник, направлено от нас (см. рисунок). Применяя правило левой руки, получаем: сила Гн (действующая на второй проводник со стороны первого) направ-
475 Электричество и магнетизм К задаче 15.10 К задаче 15.11 лена влево, т. е. магнитное взаимодействие приводит к притя- женйю проводников (сила Fi? рассматривается аналогично). Магнитное взаимодействие электронных пучков тоже приводит к их взаимному притяжению. Однако между электронами есть еще и кулоновское взаимодействие! Оно, разумеется, приводит к отталкиванию. Известно, что для магнитных и кулоновских сил Fm v2 t выполняется соотношение jr ~ ~г (и — скорость электронов в пучке). Понятно, что Fk > Fm; отталкивание преобладает. Под действием кулоновских сил пучки не только отталкиваются друг от друга, но и расширяются, «расплываются». „ „ „ „ Л „ mv 15.12. Ответ: по окружности радиуса R = в плоскости, пер- СИ пендикулярной В. Решение. На электрон действует сила Лоренца Fn = eBv, пер- пендикулярная вектору В. Поскольку начальная скорость элект- рона также перпендикулярна В, траектория электрона лежит в перпендикулярной вектору В плоскости. Работа силы Лоренца равна нулю, поэтому v = const. Значит, электрон движется по плоской траектории с постоянной по модулю скоростью v и w Fn eBv , Постоянным по модулю ускорением а = — = (ускорение обра-
Ответы, указания, решения 476 зует прямой угол с вектором скорости). Названным условиям удовлетворяет только одна траектория — окружность. Из соотно- си2 _ mv шения а =- определяем радиус окружности R = — Л 15.13. Ответ: не зависит. Решение. Электрон движется с постоянной скоростью по окруж- ности радиуса R = (см. задачу 15.12). Период его обращения „ 2nR 2ят равен Т - - = и не зависит от v (при большей скорости электрон опишет за то же время окружность большего радиуса). Независимость периода от скорости позволила создать первый циклический ускоритель заряженных частиц — циклотрон. При больших (релятивистских) скоростях появляется зависимость Т от v (механика Ньютона применима лишь при малых скоростях v « с). 15.14. Ответ: электрон будет двигаться по винтовой линии радиуса „ m usina , Ягстписова R =---s— с шагом п = ———------. еВ еВ Решение. Поскольку действующая на электрон сила Лоренца Гл перпендикулярна его скорости, эта сила не совершает работы, и кинетическая энергия электрона (а значит, и модуль его скорости) остается неизменной. Кроме того Гл -L В, поэтому проекция ско- рости электрона на направление поля также не изменяется. Удоб- но представить скорость электрона в виде векторной суммы (см. рис. a): v = vi + Vj. Поскольку v = const и щ = const, получаем a = = const. Составляющая скорости vi = osina, оставаясь неизменной Рис. а Рис. 6
477 Электричествоимагнетизм по модулю, может лишь поворачиваться в плоскости, перпендику- лярной В. Таким образом, электрон одновременно участвует в двух движениях: равномерном перемещении вдоль В с постоянной скоростью и - ocosa и движении по окружности в плоскости, перпендикулярной В, со скоростью l>i = usina. Траектория такого движения представляет собой винтовую линию (см. рис. б). Най- дем ее радиус В и шаг h (расстояние между соседними витками). Ускорение электрона соответствует движению по окружности V12 радиусом R со скоростью о±: а - -=. Значит, Л „ О • ти^2 Fa = euBsina = та = —=—, л _ znosina откуда R =------—, где е — модуль элементарного заряда, т — еВ масса электрона. Один оборот по винтовой линии электрон делает 2пВ 2пт за время Т = —— = Шаг винтовой линии представляет собой перемещение электрона вдоль направления В за время одного , , 2 тг mucosa оборота: й = ---. 15.15. Решение. Как и при решении задачи 15.14, разложим скорость электрона на две составляющие: вдоль полей (i>i) и перпендикулярно им (i>±). В результате действия электрического поля изменяется только ц. Сила Лоренца зависит только от Vi = usina, и поэтому «поперечное» движение электрона будет снова происходить по окружности, радиус которой можно найти так же, как и в задаче 15.14. «Продольное» же движение элект- рона в данном случае будет равноускоренным с ускорением еЕ ' а = —, поскольку в этом направлении на электрон действует постоянная кулоновская сила Fk = еЕ. Это означает, что расстоя- ния между соседними витками «винтовой линии» не будут одина- ковыми: они составят арифметическую прогрессию (как пути, проходимые при равноускоренном движении за последовательные равные промежутки времени). Разность этой прогрессии d = аТ\ _ 2лт , еЕ /2itm\, 4it2mE гДе Т = (см. задачу 15.13), то есть d = — • = gg2 • Прогрессия может быть как возрастающей, так и убывающей (в зависимости от направления Е); в последнем случае, если протя-
Ответы, указания, решения 478 женность I области, занятой полями, достаточно велика (I > )> то иц может обратиться в нуль и затем изменить знак: электрон как бы отразится от некоторой плоскости. 15.16. Решение. Какие силы действуют на частицу, влетевшую в прибор? Для определенности будем считать заряд q частицы положительным. Тогда сила Лоренца Fn = qBv направлена вверх, а кулоновская сила Л = qE — вниз. Если эти силы не уравно- вешивают друг друга, частица отклонится вверх или вниз и не попадет в выходное отверстие. Пройдут через прибор только те частицы, для которых сила Лоренца практически уравновешивает Е кулоновскую. Это возможно при v = —. Для отрицательно заря- Е женных частиц получим такой же результат (в этом случае обе силы поменяют направления на противоположные). Заметим, что для прохождения сквозь фильтр никакие характеристики час- тицы роли не играют — лишь бы она была заряженной и двигалась Е со скоростью v = перпендикулярно обоим полям. 15.17. Решение. На рис. а показано направление вектора маг- нитной индукции В поля магнита вблизи разных участков прово- да. Согласно правилу левой руки сила Ампера на участке 1 направлена от нас (перпендикулярно плоскости рисунка); на учас- тке 2 — к нам; на участке 3 сила Ампера отсутствует. В результате S Рис. а Рис. б Рис. в
479 Электричество и магнетизм провод обвивается вокруг магнита (см. рис. б). Обвившийся вокруг магнита провод притянется к магниту (см. рис. в). 15.18. Решение. Предположим для простоты, что второй проводник за- креплен, а первый свободен. На рисун- ке показаны линии магнитной индук- ции поля второго проводника (ток I2 направлен от нас). Из правила левой руки следует: на верхнюю часть перво- го проводника сила Ампера действует в направлении от нас, а на нижнюю — к нам. Значит, ток h будет ориен- тироваться в ту же сторону, что и ток I2. Следовательно, про- водники станут притягиваться (см. задачу 15.11). 15.19. Ответ: жидкость начнет вращаться по часовой стрелке. Указание. Поскольку раствор медного купороса — проводник, в нем потекут токи от центрального электрода к медному кольцу. На положительные и отрицательные ионы, движущиеся навстре- чу ДРУГ ДРУГУ> со стороны магнитного поля будут действовать силы, направленные в одну сторону (по часовой стрелке). 15.20. Ответ: т = 2TtRBIsina g = 3,2 г. Указание. См. задачу 15.4. 15.21. Ответ: UAC = —. enb Решение. При упорядоченном движении электронов (в направ- лении, противоположном направлению тока) на них действует сила Лоренца. В результате они отклоняются в сторону поверх- ности СС1Е1Е, на этой поверхности происходит накопление отри- цательного заряда, а на противоположной — положительного. Процесс разделения зарядов продолжается до тех пор, пока воз- никающее электрическое поле Е не скомпенсирует действие на электрон силы Лоренца: еЕ = evB, или Е = vB (здесь v — скорость упорядоченного движения электронов). Учитывая, что фл - фс = Еа и I = envS = envab, получим ГТ 1В UaC = фл - фс = Рассмотренный эффект дает возможность достаточно точно измерять концентрацию свободных заряженных частиц в про- воднике или индукцию магнитного поля.
Ответы, указания, решения 480 15.22. Ответ: Е = U = 0,11 мкВ, В = 5,7 • 1(Г’ Тл. е Решение. При вращении цилиндра свободные электроны за счет центробежного эффекта отбрасываются к поверхности цилиндра, образуя вблизи нее избыточный отрицательный заряд. Это разде- ление зарядов прекращается, когда возникшее электрическое поле способно сообщать свободным электронам центростремитель- 2 „ „ „ тагг ное ускорение а = аг, т. е. когда еЕ = та. Отсюда Е = —-—. Напряженность электрического поля с ростом г линейно возрас- тл ЕП1НХ ma2R _ тает, поэтому ее среднее значение Еср = —х— = —х—. Значит, и — Л = В, Л = = 1,1 • кг в. Если магнитное поле направлено вдоль оси цилиндра, сила Лоренца, направленная по радиусу, может сама сообщить элект- ронам необходимое центростремительное ускорение: Вл = та. В таком случае электрическое поле не возникает и разделения зарядов не происходит. Учитывая, что Вл = evB, v = саг и а = со2г, получаем еаВг = та2г, значит, В = -у = 5,7 • 10"9 Тл. Разумеется, направление В должно быть согласовано с направ- лением вращения (чтобы сила Лоренца была направлена к оси вращения). 15.23. Ответ: х = 3,5 мм. Решение. Все ионы попадают в Е поле Bi со скоростью v - -= (см. задачу 15.16). Радиус R описыва- емой окружности определяется _ mov таЕ из соотношения R - —=- = . eBi eBBi В. „ М Здесь та - -гг- — масса иона, М Na — молярная масса соответствую- щего изотопа, Na — постоянная Авогадро. Очевидно (см. рисунок), что х — 2(Я2 - Я.) = - Мх) =. 3,5 • 10-2 м. Измерив радиус описываемой ионами окружности, можно с высокой точностью определить массу иона. Прибор, в котором используется этот принцип, называется масс-спектрометром.
481 Электричество и магнетизм 15.24. Ответ: а = arctg------. 2(Ло + Л) Решение. Воспользуемся тем, что магнитное поле является вихре- вым, т. е. все линии магнитной индукции замкнуты. Следователь- но, магнитный поток Ф через любую замкнутую поверхность равен нулю (количество входящих в поверхность и выходящих из нее наружу линий магнитной индукции одинаково). Рассмотрим про- ходящую через точку А замкнутую поверхность, имеющую форму цилиндра высотой Az (см. рисунок). Представим вектор магнитной индукции в точках на боковой поверхности цилиндра в виде суммы вертикальной и горизонтальной составляющих: В = В„,„ + Втр. В данном случае В«,„ = В,. Тогда Ф = В;(г + Аг)лВ2 - ВДг)лД2 - 2лЯДгВго„. Поскольку Ф = 0, получаем В^, = ~Г~(Вг(2 + Az) - B,(z)). 2Az' ' Учи- тывая, что величина B,(z + Az) - B,(z) Az представляет собой при ма- RBo лых Az производную Вг по z, находим Вгор =-хт-. Следовательно. Ana tga = Вп>|> _ R В. ~ 2(Ло + Л)’ 16.1. Решение. Изменение магнитного поля всегда вызывает появ- ление вихревого электрического поля. В результате в диэлект- рическом кольце возникает поляризация диэлектрика, а в прово- i дящем — индукционный ток. Вихревое электрическое поле исче-
Ответы, указания, решения 482 зает, когда магнит останавливается. При этом поляризация ди- электрика1’ также исчезает, а ток в проводнике затухает из-за сопротивления. Энергия вихревого поля переходит во внутреннюю энергию кольца (оно нагревается), а небольшую ее часть уносит электромагнитное излучение. Согласно правилу Ленца направление индукционного тока в проводящем кольце таково, что магнит и кольцо во время вдви- гания магнита отталкиваются. 16.2. Ответ: но часовой стрелке, если смотреть со стороны магнита. Решение. Направление магнит- ного поля магнита показано на рисунке сплошными линиями (магнитное поле «входит» в юж- ный полюс). При приближении магнита к кольцу магнитпыц поток через кольцо увеличива- ется. Согласно правилу Ленца магнитное поле индукционного тока li в кольце (его линии пока- заны на рисунке пунктирными стрелками) должно быть направ- лено противоположно увеличивающемуся внешнему магнитному полю, т. е. оно «стремится» воспрепятствовать увеличению маг- нитного потока через кольцо. Зная направление магнитного поля индукционного тока, можно по правилу буравчика определить направление индукционного тока: он идет по часовой стрелке, если смотреть на кольцо со стороны магнита. 16.3. Ответ: когда рамка находится в плоскости а; когда плоскость рамки перпендикулярна плоскости а. Указание. Согласно закону электромагнитной индукции величина ЭДС индукции определяется скоростью изменения магнитного потока через рамку. Эта скорость (и ЭДС) равны нулю, когда рамка и провод лежат в одной плоскости (при этом боковые стороны рамки не пересекают линий магнитной индукции, а движутся вдоль них). 16.4. Ответ: не возникает. Указание. Линии магнитной индукции поля, создаваемого током, текущим в одном из обручей, не пересекают второго обруча. 16.5. Решение. При движении магнита возникает вихревое элек- трическое ноле, которое вызывает вихревые электрические токи в трубе. Взаимодействие этих токов с падающим магнитом при- *’ Мы не рассматриваем случай, когда кольцо сделано из сегнетоэлектрика.
483 Электричество и магнетизм водит согласно правилу Ленца к воз- никновению силы, тормозящей маг- нит; причем эта сила возрастает с увеличением скорости падения. В са- мом начале "падения эта сила мала, и магнит движется с ускорением сво- бодного падения. Затем ускорение плавно уменьшается, и через достаточно длительный промежуток времени сила взаимодействия магнита с вихревыми токами прак- тически уравновесит силу тяжести магнита. Дальнейшее падение происходит почти равномерно. Характер изменения скорости со временем показан на графике (см. рисунок). Движение магнита в трубе напоминает падение тела в жидкости или газе, где сопро- тивление движению также возрастает с увеличением V. 16.6. Ответ: колебания быстро затухнут. Указание. После замыкания ключа в цепи возникнет индук- ционный ток и на нить маятника начнет действовать сила Ампера. Из правила Ленца следует, что эта сила будет тормозить движение маятника. Кроме того, в соответствии с законом электромаг- нитной индукции, сила прямо пропорциональна скорости шари- ка. Значит, сила Ампера аналогична силе вязкого трения (см. задачу 16.5) и приводит к более быстрому затуханию колебаний. 16.7. Ответ: в первом случае плоскость рамки перпендикулярна вектору индукции магнитного поля; во втором случае — парал- лельна. Решение. Необходимым условием равновесия является обращение в нуль момента сил, действующих со стороны магнитного поля на рамку, в которой возникает индукционный ток. Момент сил обращается в нуль в двух случаях; 1) индукционный ток вообше отсутствует (это возможно, когда плоскость рамки параллельна магнитному полю); 2) плоскость рамки перпендикулярна маг- нитному полю. Во втором случае равновесие является устой- чивым, если индукция Во магнитного поля индукционного тока совпадает по направлению с индукцией В внешнего магнитного поля. Согласно правилу Ленца это происходит, когда внешнее магнитное поле убывает. При возрастании В такое положение рамки соответствует неустойчивбму равновесию, а устойчивое равновесие достигается в случае 1. В этом можно убедиться, рассмотрев произвольное промежуточное положение рамки: воз- никающий момент сил поворачивает рамку к положению, при котором ее плоскость будет параллельна магнитному полю. Рас-
Ответы, указания, решения 484 смотрение промежуточного положения поможет проанализиро- вать также и случай убывания внешнего магнитного поля. 16.8. Ответ: Ф = лгЛВ. Решение. До выключения магнитного поля ток в кольце отсутст- вовал; плоскость кольца пронизывал магнитный поток Фь = пг*В. Во время выключения поля согласно закону электромагнитной индукции в кольце возникает ЭДС индукции <5, = - Но в At соответствии с законом Ома & = IR, и при R -» 0 получаем & -* 0. Это возможно только при ДФ = 0, т. е. когда Ф = const. Значит, магнитный поток через сверхпроводящий контур вообще не может измениться: он остается равным Фо и после выключения внешнего магнитного поля. Теперь его создает индуцированный в кольце незатухающий ток. 16.9. Указание. См. задачу 16.1. В сверхпроводящем кольце, в отличие от проводящего, индукционный ток будет незатухающим. Магнитный поток, создаваемый этим током (см. задачу 16.8), в точности компенсирует магнитный поток, создаваемый магнитом. Отталкивание между магнитом и кольцом будет наблюдаться и после прекращения вдвигания магнита. Сверхпроводник всегда выталкивается из области более сильного поля (от полюса маг- нита), т. е. ведет себя как идеальный диамагнетик. Он может даже ♦зависнуть» в воздухе над полюсным наконечником магнита. TtNBr2 16.10. Ответ: I = - —. jL Указание. Воспользуйтесь условием Ф = const (см. задачу 16.8) и соотношением Ф = LI для магнитного потока, создаваемого током в катушке. 16.11. Ответ: В = 4 Во. Указание. Магнитный поток через сверхпроводящий контур не может измениться (см. задачу 16.8). Площадь контура умень- шилась в 4 раза; следовательно, индукция магнитного поля долж- на во столько же раз увеличиться. 16.12. Ответ: увеличится вдвое. Указание. Радиус меньшего кольца составляет половину радиуса исходного кольца; площадь каждого из меньших колец стала в 4 раза меньше первоначальной, а их общая площадь — в 2 раза меньше первоначальной. Заметим, что после складывания «вось- мерки» токи по меньшим кольцам текут в одну сторону. Значит,
485..Электричество и магнетизм магнитные потоки, пронизывающие оба кольца, складываются, причем полный магнитный поток остается неизменным (см. зада* чу 16.8). 16.13. Решение. Заметим, что в результате такого складывания ток должен был бы идти по двум сложенным кольцам в противо- положные стороны! Поэтому суммарное магнитное поле контура, казалось бы, должно обратиться в нуль. Но это несовместимо с условием Ф = const (см. задачу 16.8). В чем же дело? Рассмотрим последний этап складывания «восьмерки». На этом этапе маг- нитные поля, создаваемые каждым из двух колец, направлены почти противоположно друг другу. Их сближение привело бы к уменьшению суммарного магнитного потока через контур; но согласно правилу Ленца уменьшение потока приводит к уве- личению силы тока в контуре! Поэтому соотношение Ф = const выполняется, но сила тока при складывании «восьмерки» неог- раниченно растет. При этом неограниченно возрастает и сила отталкивания, которую нужно преодолевать для складывания «восьмерки». Реально это означает, что на некотором этапе про- цесса сверхпроводимость будет утрачена (для каждого сверхпро- водника существует предельный ток, называемый критическим). После этого ток в контуре затухнет и «восьмерку» нетрудно будет сложить. Магнитное поле контура обратится в нуль. 16.14. Ответ: q = 0,57 мкКл. Решение. В каждом из витков катушки при изменении магнит- ного поля индуцируется постоянная ЭДС Si. Ее модуль |Si| = = Суммарная ЭДС в катушке S = n |Si|. Через конденсатор At постоянный ток не может идти, поэтому сила тока в цепи равна пулю. Значит, равна нулю и полная напряженность электричес- кого поля в проводах. Она представляет собой сумму напряжен- ности Ei вихревого электрического поля и напряженности £* электростатического (кулоновского ) поля; таким образом, £, + £* = 0, т. е. Ek = -Ei. Следовательно, при прохождении заряда через катушку работа кулоновского и вихревого полей отличается только знаком: <pi - фг = -S. Заряд конденсатора q = С|ф1 - фг| = = ппСг*^- = 0,57 мкКл. At 16.15. Ответ: Р = )2 = 5,0 • 10-5 Вт. 16.16. Ответ: заряд одинаковый; количество теплоты больше при быстром движении магнита. Решение. Пусть сопротивление катушки R. Если за небольшой
Ответы, указания, решения 486 промежуток времени At магнитный поток через контур изменяет- ся на АФ, то при этом в катушке возникает ЭДС индукции ,, ЛФ „ „ Т & 1 ЛФ о <?i = - —Индукционный ток / - к = _ S • —• 3й время At по At tt tt At цепи проходит заряд Ag = /At = ЛФ R ' Полный проходящий ЦО цепи заряд q - ^Лд = - ^^ЛФ = ~ Здесь Ф — конечное зна- чение магнитного потока (начальное равно нулю). Значит, g не зависит от скорости процесса. Выделившееся в цепи количество теплоты Q равно работе сторонних сил: Q = qt\. Поскольку заряд g в обоих случаях одинаков, а больше при быстром движении магнита, количество теплоты в первом случае больше. К такому выводу можно прийти и другим путем: Q = А = Fs, где А — совершаемая при введении магпита механическая работа. Пере- мещение магнита s одинаково в обоих случаях, a F больше в первом случае (F — сила отталкивания магнита от катушки вследствие появления индукционных токов). 16.17. Ответ: g = —J~(l - cosa). Указание, g - - ™ (см. задачу 16.16). 16.18. Ответ: g„,iut = 32 мкКл- |ЛФ| Решение. Из соотношения g = - -г- (см. задачу 16.16) следует, что /С для прохождения максимального заряда провод должен охватить площадку наибольшей площади S. Как известно, из всех фигур с заданным периметром наибольшую площадь имеет круг. Из формул Z2 JB S I = 2лг, S ~ пт* получаем S = —; следовательно, g„,«x = ~±~ = 4тс tt =----= 32 мкКл. 4nR 16.19. Ответ: I = 4R Решение. Фактически ЭДС наводится только во вращающемся стержне. Ее можно найти, применяя закон электромагнитной индукции к любому из двух секторов, на которые «стрелки» делят круг: ЛФ _ -AS „Аг) ве Ла ВшР 6, _ _ - в— « - В— - - -2- • — = - -2-.
487 Электричество и магнетизм Здесь S — площадь сектора, а — угол между стрелками (® = ). At _ ' Т |6',| ыВ12 Согласно закону Ома I = rii-. 16.20. Решение. Характер движения определяется равнодейству- ющей сил, действуюхцих на стержень. Она равна векторной сумме приложенной силы F и силы Ампера Fa со стороны магнитного поля (при движении стержня в нем индуцируется ток силой 1). Чтобы найти Fa при движении стержня со скоростью v, исполь- Г- Г к’*! |<Ч АФ dAS а , зуем соотношения Fa = В/l, I = ,| ---= В — = Bvl. К At At Из них следует: Fa R V‘ Согласно правилу Ленца сила Ампера направлена противопо- ложно движению стержня. И по направлению, и по характеру зависимости от скорости эта сила напомипает силу сопротивления при движении в жидкости или газе. Отсюда ясен характер движе- ния (ср. с задачей 16.5): ускорение стержня максимально в начальный момент, когда Fa — 0, а затем постепенно уменьшается до нуля. Стержень в конечном счете приобретает постоянную скорость, определяемую из условия Fa = F: FR v ~ В~Г Заметим, что при установившейся скорости движения стержня выполняется соотношение Fi> At = PR At: работа внешней силы равна количеству теплоты, выделившейся в резисторе. При уве- личении R скорость v растет из-за уменьшения силы индук- ционного тока, а, следовательно, и силы Ампера, направленной противоположно движению. 16.21. Ответ: Стержень будет двигаться равноускоренно под дей- ствием силы F, пока не достигнет скорости Vi = Затем уско- рение начнет уменыпаться и станет равным нулю при скорости FR „ vz = i>i + Далее стержень движется равномерно. Указание. При v < vi индукционный ток в цепи отсутствует (диод включен «против» источника ЭДС). См. также задачу 16.20. «л лл TtPdBa , 16.22. Ответ; i - — = const; разность потенциалов между лю- •Л быми точками кольца равна нулю.
Ответы, указания, решения 488 Решение. Поскольку — = аВа = const, в кольце индуцируется At постоянная ЭДС: |<5| = = itr^aBo. At Значит, и сила тока в кольце постоянна: . _ иг^аВо l~ R~ R ' На любом участке АВ кольца индуцируется ЭДС 6Ав, пропор- циональная длине участка (а, значит, и его сопротивлению Дав): Г, Gab = е „ . К Из закона Ома для участка цепи с ЭДС следует: фА - фв = = (Sab - i/?ab = ё—^ - ^Яав = 0. Можно пояснить этот результат и л л иначе: плотность заряда всюду в кольце равна нулю (это следует из соображений симметрии). Значит, кулоновское электрическое поле в кольце отсутствует, а разность потенциалов может быть обусловлена только кулоновским полем (вихревое электрическое поле, поддерживающее ток в кольце, непотенциально). 16.23. Ответ: U > 380 В. Решение. Обозначим сопротивления верхнего и нижнего колец «восьмерки» соответственно Ri и Яг, а индуцируемые в них ЭДС Si и ёг. Эти ЭДС действуют в цепи навстречу друг другу. Пробой возможен между точками А и В на перемычке «восьмерки». На рис. а представлена эквивалентная схема цепи. Согласно закону Ома
489 Электричество и магнетизм В соответствии с законом электромагнитной индукции 6'1 = 2В „ 2В Ri Гг = ял -—, е>2 = лг2—. Учитывая, что = —, получаем At At Ry П и„.,в гг+nr, = «в™. = щ 1 * - м Гг Возникает вопрос: в чем же отличие этого изогнутого «восьмер- кой» провода от кольца, рассмотренного в задаче 16.22? Почему тогда не возникало разности потенциалов между любыми двумя точками кольца, а теперь эта разность потенциалов есть? Для ответа на этот вопрос рассмотрим, как действует вихревое элек- трическое поле на свободные заряды в проводе. На рис. б показано, в какую сторону это поле «гонит» положительные заряды в верхней и нижней частях «восьмерки» (мы предположили, что возрастающее внешнее поле направлено к нам). В результате действия вихревого поля произойдет разделение зарядов: на ближ- ней к нам части перемычки возникнет положительный заряд, а на дальней — отрицательный. Вследствие разделения зарядов возникнет кулоновское (электростатическое) поле — оно-то и соз- даст разность потенциалов. 16.24. Решение. ЭДС самоиндукции = - L—, поэтому, согласно At закону Ома для всей цепи IR = ё' + St = <5 - L—. Перепишем это At соотношение в виде L— = ё - IR. (1) At ' ' Сразу же после замыкания ключа 1 = 0, поэтому можно считать AZ ё „ <S — и —, т. е. ток возрастает с постоянной скоростью (I = —t; см. At ь L рисунок). При увеличении тока правая часть в уравнении (1) уменьшается, следовательно, уменьшается и скорость возрастания тока. Через достаточно длительное время (о том, каким оно должно быть, — чуть ниже) правая часть уравнения (1) становится пренебрежимо малой. Это означает, что ток стремится к постоянной ве- ё личине I = -=: (см. рисуцрк). «Харак- Л терное» время т для рассматривав- мой системы можно определить, оце-
Ответы, указания, решения 490 нив, когда слагаемое IR станет сравнимо по величине с <S. Восполь- зовавшись для оценки зависимостью I = ft, получаем утЛ L Li откуда т — L/R. Для тех, кто уже изучил дифференциальное и интшральвое исчисление в объеме школьного курса: уравнение (1) является простейшим дифференциальным урав- i Н ? R нением Г - - -fl. Покажем, как оно решается. Введем переменную у = у - у/. L L L х? д f, Тогда паше уравнение примет вид: у' = - - р. Его решение у - Се l , где С — произвольная постоянная. Значение этой постоянной определяется из условия 7=0 ,< j1; в при t = 0 (т. е. у = при t = 0). Таким образом, для I получаем: I = ^(1 - е д'). Из этой формулы легко увидеть, что при t » , когда второе слагаемое в скобках мало, I стремится к постоянному значению Чтобы «увидеть» линейный рост I R при малых t, надо воспользоваться формулой е R (см. математическое приложение). При < 1 , R г к 1 - jt, откуда I х j-t. ' к 1 + х, применимой при |х| < 1 L *, (т. е. при t « —) получаем е l а 16.25. Ответ: см. рисунок; т ~ л Решение. Зависимость силы тока от времени показана на рисунке (ср. с задачей 16.24). Оценить время т мож- но различными способами. Простей- ший из них — метод размерностей. Физически очевидно, что т может за- висеть лишь от L, от R и, возможно, от силы тока То в момент to. Единицы измерения этих величин: 1 Гп = 1 Ом • с, 1 Ом и 1 А. Нетрудно убедиться, что величину с размерностью времени можно получить только одним способом: разделив L па R, Значит, т ~ L/R (точное выражение для т может содержать еще численный коэффициент порядка единицы). Как видим, т не зависит от /о. Тот же результат можно получить и иначе: ток затухает, когда начальная энергия магнитного нота тока W* - а перейдет во внутреннюю энергию Л проводов, нагреваемых током: Жм = Q ~ I02Rt. Отсюда также L , 1 v получаем т ~ (множитель — при оценке можно опустить). л 2
491 Электричество и магнетизм Наметим и точное решение (ср. с задачей 13.83). Закон Ома rAZ r, Rr при t > t\> приводит к соотношению L— -- -1R, или г = - -y-Z. At Ь Решение этого дифференциального уравнения, удовлетворяющее условию I = 1о при t = to, имеет вид I - 1ое где т = 16.26. Ответ: L пропорциональна №. Решение 1. Индуктивность катушки определяется из соотношения 6' = At где А' —- ЭДС самоиндукции. Эта ЙДС является суммой ЭДС всех витков. В замкнутом сердечнике рассеиванием магнитного потока можно пренебречь, поэтому каждый из витков пронизывается одинаковым магнитным потоком Ф — BS. Значит, в каждом витке р |АФ| ~|АВ| г, наводится одна и та же ЭДС Ai =--—. Согласно принципу 1 At' At1 суперпозиции магнитная индукция В поля в сердечнике прямо пропорциональна N, т. е. В N. Значит, и А ~ N. Полная же ЭДС А = Поэтому А' ~ N'2, откуда L ~ №. Решение 2. Индуктивность — это коэффициент пропорциональ- ности между магнитным потоком, пронизывающим контур, и силой тока: Ф = LI (магнитное поле создается током, текущим в контуре). Таким образом, наша задача сводится к вопросу: как зависит величина магицтнохю потока, пронизывающего катушку, от числа ее витков (при неизменной силе тока)? Заметим, что Ф = BS, где В — индукция ноля, a S — суммарная площадь всех витков. Оба множителя (В U.S) прямо пропорциональны числу витков N (величина В потому, что поле в катушке является суперпозицией полеТй всех витков, а величина S — потому, что все витки пронизываются одним и тем же магнитным полем). Следо- вательно, Ф ~ № и L ~ №. 16.27. Ответ: I = уменьшится вдвое. А Решение. Магнитный поток Ф через сверхпроводящее кольцо не может измениться (см. задачу 16.8). В начальном положении этот поток составляет Ф = L/o, где L — индуктивность кольца. После сближения поле соседнего кольца дает в магнитный поток такой же вклад, как и собственное магнитное поле кольца; значит, Ф = 2LZ. Отсюда I = Л
Ответы, указания, решения 492 В начальный момент, когда магнитные поля колец практически не перекрывались, энергия магнитного поля была Wo = • 2. А Сблизившиеся вплотную кольца, по каждому из которых течет ток 2", создают такое же магнитное поле, как одно кольцо с током г W ТЛ 1о; энергия этого поля W = = —-Жуда девалась половина м £ начальной энергии? В результате взаимного притяжения кольца при сближении увеличивали свою скорость; значит, «исчезнув- шая» энергия поля перешла в кинетическую энергию колец (некоторую часть энергии уносит электромагнитное излучение, возникающее при ускоренном движении колец). 16.28. Решение. Будем для определенности считать, что начальная сила тока в первом кольце больше: lot > Кг. Кольца притягиваются и сближаются. В соответствии с условием Ф = const (см. задачу 16.8) токи в них уменьшаются. В некоторый момент ti сила тока во втором кольце обращается в нуль. Однако еще до этого (в некоторый момент to) магнитный поток через первое кольцо, создаваемый током 1г, начнет уменьшаться, несмотря на сближе- ние колец. Поэтому, начиная с момента to, ток h начнет возрас- тать. К моменту ti ток h достигает начальной величины 1м (иначе не выполнялось бы условие Ф = const для первого кольца). И хотя в момент ti сила магнитного взаимодействия обращается в нуль, кольца продолжают сближаться по инерции; ток I2 меняет направ- ление на противоположное, в результате чего кольца теперь отталкиваются и их сближение замедляется. Отталкивание станет настолько велико, что не позволит кольцам сблизиться вплотную: ведь тогда магнитные потоки через них сравнялись, бы, что противоречило бы условию Ф = const (напомним, что начальные значения магнитных потоков различны). После сближения на .минимальное расстояние кольца начинают удаляться, затем опять сближаются. Возникнут затухающие колебания (затухание обус- ловлено потерями энергии на трение и электромагнитное излу- чение). В положении равновесия ток во втором кольце отсутствует, но пронизывающий кольцо магнитный поток остался тем же: этот поток обусловлен теперь полем тока, текущего в приблизившемся первом кольце. ’ Ьг ЬР* 16.29. Ответ: Е = -5- при г < R; Е = -5— при г > R. А 1г Решение. Силовые линии вихревого электрического поля пред-
499 Электричество и магнетизм ставляют собой окружности с центрами на оси цилиндрического сердечника. Вследствие осевой симметрии величина напряжен- ности поля вдоль каждой из силовых линий постоянна. Применим закон электромагнитной индукции к контуру радиуса г, совпада- ющему с одной из силовых линий: S, = 1^1. 1 At1 Величина магнитного потока Ф равна В • пг2 при г < R и В • kR2 при г > R. Величина ЭДС индукции St представляет собой работу вихре- вого электрического поля при перемещении по контуру единич- ного положительного заряда: & = 2лгЕ. Итак, 2пгЕ = дг2* при г < R, 2nrE = nR2k при г > R. Отсюда _ kr _ _ kR2 _ Е = при г < R; Е = -5— при г > R. 16.30. Решение. Разность потенциалов между любыми двумя точками кольца равна нулю (см. задачу 16.22). Ведь в кольце создается только вихревое элек- трическое поле, которое и поддержи- вает индукционный ток, а разность по- тенциалов может создаваться только ку- лоновским полем. Итак, разность по- тенциалов между точками А и В равна нулю: фл - фв = 0. Однако можно ли утверждать, что вольтметр, подключен- ный к этим точкам, покажет нуль? Нельзя: ведь по кольцу идет ток. Часть тока, хотя и ничтожная, может ответвиться и идти через вольтметр (см. рис. а), вызывая отклонение стрелки от нулевого деления. Вспомним, что показание вольтметра опреде- ляется силой проходящего по нему тока; а ток ведь может воз- никать как за счет кулоновского поля, так и за счет вихревого электрического поля (при наличии ЭДС ток может идти и в отсутствие разности потенциалов). Из-за большого сопротивления вольтметра его подключение практически не приводит к изме- нению силы тока в кольце; значит, и возникающим кулоновским полем, можно пренебречь (в отличие от ситуации, описанной в задаче 16.31). ЭДС индукции S равномерно распределена вдоль кольца. Зна-
Ответы, указания, решения 494 чит, на участке АВ, составляющем четверть кольца, 6'Ав = -j# яг2 АВ nfer2 = —— = —j—. Рассмотрим теперь заштрихованный на рис. а контур. Магнитный поток через него равен нулю0; следовательно, полная ЭДС в этом контуре также равна нулю. Это означает, что <Savb + <5вс* = 0, т. е. |6'avb| = |<SbcaI = |6ав|. Значит, сила тока через вольт- <5ав тт Т „ р nkr2 „ метр/v = -д-, и вольтметр показывает Uv = IvRv = Оав = Однако полученный ответ нельзя считать исчерпывающим: показание вольт- метра зависит не только от того, к каким точкам кольца он под- ключен, но и от расположения самого прибора и подключенных к нему проводов! Предлагаем читателю убедиться, что в ситуации, „ гг 3,, ЗяЛг2 . соответствующей рис. о, вольтметр показывает Uv = = —j—. А ведь возможен еще вариант, когда провод от вольтметра образует виток (или несколько витков) вокруг сердечника (см. рис. в)! Поэтому окончательный ответ следует записать в виде Uv = ~-(1 + 2N), где может быть равно 0, 1, 2... 16.31. Ответ: в меди 2,9 В/м, в латуни 12 В/м. Решение. В кольце действует постоянная ЭДС & = яг3 • —. Зна- At чит, в нем протекает постоянней ток. Разумеется, сила тока одинакова во всех сечениях кольца, поэтому одинакова всюду и плотность тока ]. Однако поскольку удельные сопротивления меди 4 Имеется «иду внешнее магнитное поле. Магнитное поле индукционного тока в данном случае является постоянным, так что явления самоиндукции не наблю- дается.
495 Электричество и магнетизм И латуни различны, напряженность электрического поля разная в меди и латуни (Е„ и Ея). Для определения Е„ и Е„ воспользуемся Е Е соотношениями j = —- = — (см. задачу 13.2) И 6'i = кг\Еч + Е„) (см. г рм р.1 задачу 16.29). Из этих соотношений находим Е„ = —= 2,9 В/м, Е, = —£1—= 12 в/м. р„ + рл Af р„ + р„ At Заметим, что это поле является суперпозицией вихревого и кулоновского полей (ведь вихревое поле одинаково во всех точках кольца). Как же возникает кулоновское поле? Дело в том, что возникшее вихревое поле вызывает разные токи в меди и латуни (см. рис. а): из-за лучшей проводимости меди сила тока в медной части кольца в первые моменты больше. Это сразу лее приведет к разделению зарядов и, следовательно, к возникновению кулонов- ского поля (см. рис. б). Напряженность Ek этого ноля одинакова по модулю в меди и в латуни; но в латуни Ek совпадает по направлению с напряженностью Е» вихревого поля, а в меди эти поля направлены противоположно. Процесс разделения зарядов заканчивается, когда силы тока в меди и латуни становятся одинаковыми. 16.32. Решение. Разумеется, ток в проводнике может возникнуть под действием вихревого электрического поля и в отсутствие разности потенциалов (возникает же он в самом кольце!). Рассмотрим два примера. 1. Проводники между А я В направлены, как показано на рисунке. Поскольку они перпендикулярны линиям вихревого
Ответы, указания, решения 496 электрического поля, ток в них не возникает. Ток в кольце от вклю- чения таких проводников не изме- няется, и по-прежнему фл = фв. 2. Дополнительный проводник между точками А и В практически совпадает с частью кольца АВ и изготовлен из той же проволоки, что и само кольцо. В этом случае через него пойдет половина силы тока I, текущей через кольцо. Пусть сопротивление кольца R, тогда из закона электромагнитной индукции и закона Ома для замкнутой цепи следует „ ,ЛВ 3 R <5i = пг— = -zRI + -г At 4 4 £ 2’ Отсюда / = 86, 7R Применим к дополнительному проводнику закон Ома для участка цепи с ЭДС: I R 6, _ . _ _ £дв + фв _ фА = — + фв _ фА> Л 6, 6, 36, Злг2 АВ „ Отсюда фА - фв = -z-=- = -55- = -55-. Итак, включение до- 4 ( йо 40 полнительного проводника может нарушить равенство потенция- лов фл и фв. Причиной возникновения кулоновского поля является возникновение зарядов противоположных знаков в точках А и В ветвления цепи (ср. с задачей 16.31). 16.33. Ответ: U = Р-~ Р- — = 0,43 В. рл + р» 2 At Указание. См. задачи 16.31 и 16.32. . г 4яг* АВ , л яг2 16.34. Ответ: h - h = • —• Ь = О, U = -г- ол At 4 АВ At ‘ Решение. ЭДС индукции на участках кольца составляют „ Злг2 АВ „ пг1 АВ 4 At 4 At (см. задачу 16.30). Поскольку магнитный поток через контур, заштрихованный на рис. а, равен нулю, ЭДС на участках 2 и 3 одинаковы (62 •= 6з). Эквивалентная схема цепи показана на рис. б. дд д Здесь Ri = -j-, Яз = -j. Из закона сохранения заряда следует соот-
497 Электричество и магнетизм Рис. а ношение Ii = I2 + 1з. Применяя закон Ома к контуру, образован- ному ветвями 2 и 3, получаем /2 = 0 (т. е. четверть кольца полностью шунтируется сверхпроводником). Тогда . Т т 6’1 4- ёз АПГ2 АВ 11" /з ” т?1 зя ’ &' Разность потенциалов между точками А та В совпадает с ЭДС , лг2 АВ на сверхпроводящем участке цепи: и = &з = 16.35. Решение. Пока поезд продолжает по инерции двигаться, в обмотках электродвигателей индуцируется ток. Другими словами, вращающиеся двигатели превращаются в генераторы. Замкнув их на реостаты, мы осуществляем следующую цепочку превращений энергии: кинетическая энергия поезда превращается в электро- энергию, а затем переходит во внутреннюю энергию нагреваемых индукционным током реостатов. Разумеется, при этих превра- щениях скорость поезда быстро уменьшается (согласно правилу Ленца при работе генератора возникает вращающий момент, тормозящий его вращение; в данном случае это приводит к тор- можению поезда). Существует и другой, более экономный режим торможения, при котором вырабатываемая энергия не расходует- ся в реостатах, а возвращается в сеть (при этом также использу- ется обратимость электрических машин постоянного тока). 16.36. Ответ: Р - 0,16 Вт. Решение. При вращении диска в нем происходит разделение зарядов за счет перераспределения свободных электронов. Таким образом, диск можно рассматривать как источник тока. Сопро- тивление диска пренебрежимо мало. Определим ЭДС такого источ- ника, т. е. напряжение U на нем при разомкнутой цепи (при подключении резистора это напряжение практически не изме- нится, поскольку сопротивление диска намного меньше R даже
Ответы, указания, решения 498 при малой толщине диска). При разомкнутой цепи процесс разде- ления зарядов заканчивается, когда напряженность электричес- кого поля Е в диске достигает такой величины, что действующие на свободный электрон со стороны электрического и магнитного полей силы Fk и Fn сообщают ему центростремительное ускорение а. В любой точке диска должно выполняться соотношение та = Fn + Ft, (1) где т — масса электрона, а — центростремительное ускорение данной точки диска. Если обозначить расстояние от данной точки до оси вращения через х, то а = ю2х, Fn = eBv = eBox, Fk = еЕ. Заметим, что та то 9 отношение = —=- -6 10 одинаково для всех точек диска и гл СИ очень мало. Это означает, что центробежные эффекты в данном случае несущественны. Пренебрегая величиной та в соотношении (1), получаем Fn = Fk, откуда Е = Box. Поле линейно нарастает от гг г. Вог2 _ оси диска к его окружности, значит, и = Вер г = —-—. Выделяе- 2 U2 В2©2^ мая на резисторе мощность Р = =0,16 Вт. „__ .. Bol2 mgR . 16.37. Ответ: U = +..- - sincot. 2 &L Решение. Будем отсчитывать время t от момента прохождения стержнем нижнего положения. Тогда угол а отклонения стержня от вертикали удовлетворяет уравнению а = ю t. Угловая скорость вращения будет сохраняться, если моменты приложенных к стер- жню сил (силы тяжести mg и силы Ампера Fa = ВИ) уравно- Zsina ПТ, I „ . вешивают друг друга, т. е. mg —-— = Bit у;. Сила Ампера и сила 2 2 тяжести равномерно распределены по длине стержня, поэтому точкой их приложения можно считать середину стержня. Итак, „ r mgavaot индукционный ток I должен изменяться по закону I = ——• Согласно закону Ома для замкнутой цепи IR = U - &, где <Si = (ср. с задачей 16.19) — ЭДС индукции, которая согласно правилу Ленца стремится затормозить вращение. Отсюда Bol2 mgR . . U - -J- + -gj- sinraZ, т. е. приложенное напряжение должно иметь постоянную и пере- менную составляющие.
499' Электричество и магнетизм 16.38. Ответ: мощность двигателя должна уменьшиться. Решение. Частота вырабатываемой) переменного напряжения за- висит от частоты вращения ротора генератора. Значит, частота Вращения должна остаться неизменной. Не Изменится поэтому и ЭДС генератора. Однако сила тока в цепи нагрузки при уве- личении сопротивления этой цепи уменьшится в соответствии с законом Ома. Значит, уменьшится и вращающий момент, кото- рый возникает в генераторе за счет взаимодействия магнитного поля и индукционного тока и который стремится затормозить вращение ротора. Следовательно, мощность двигателя должна быть уменьшена. Конечно, этот же вывод следует непосредственно из закона сохранения энергии: ведь при увеличении сопротив- ления нагрузки потребляемая ею мощность уменьшается. 16.39. Ответ: когда совершает работу. Решение. Нагревание обмотки двигателя зависит только от силы протекающего по нему тока. При вращепии двигателя в его обмотке наводится ЭДС индукции, вызывающая согласно правилу Ленца уменьшение суды тока. Поскольку двигатель под нагруз- кой вращается медленнее, ЭДС индукции в пем меньше, и поэтому сила тока больше. Значит, в этом случае двигатель нагревается сильнее. Обмотка двигателя может даже перегореть, если ротору включенного двигателя не давать вращаться. 16.40. Ответ: = 300 Вт, 10 - 5 А. Решение. Потребляемая от сети постоянного тока мощность всегда равна UI, а выделяющаяся в обмотке тепловая мощность всегда равна I2R. Для резистора эти величины равны, поскольку U = IR. Однако в данном случае U ФIR-. при работе электродвигателя в его обмотке возбуждается ЭДС индукции б, так что IR = U - в < U. Из закона сохранения энергии следует, что развиваемая двигате- лем механическая мощность N = UI - I2R. Эта величина зависит не только от напряжения в сети и сопротивления обмотки, но и от скорости вращения двигателя: при увеличении нагрузки вра- щение двигателя замедляется, ЭДС индукции уменьшается, и поэтому сила тока растет. Она монотонно возрастает до величины г V Л™. = s, которая достигается при неподвижном роторе двигателя it (т.е. при заклинивании). Как следует из формулы N = UI - I2R, U2 максимальную мощность = 300 Вт двигатель развивает при Л = ух = б А (см. Математическое приложение).
Ответы, указания, решения 500 16.41. Ответ: v = • rn.gR. 'itdxBo' Решение. Скорость кольца перестает увеличиваться, когда сила Ампера, действующая на индукционный ток в кольце, уравно- вешивает силу тяжести: I • nd • Bmp = mg. Здесь Вгор — проекция вектора В на горизонтальную плоскость. Величина этой проекции: _ d ЛВг _ d 4 ‘ д2 “ 4йоЯо 4 h»m g (см. задачу 15.24). Отсюда находим I = —-—; ЭДС индукции в ndBa кольце равна & = IR. Согласно закону электромагнитной ин- „ |ДФ| nd2iAB,i nd2iAB, Azi ndxBoV _ дукции = — = —j—------- = -~т~----• — = —tv—. Следовательно, * 'At1 4 I At I 4 1 д2 At 4Ло v = —~~~ = ( )2 mgR Заметим, что при установившейся ско- ndBo \nd2Ba> рости падения I = const, поэтому ЭДС самоиндукции равна нулю. 17.1. Ответ: а) Т = 1,26 мкс, v = 800 кГц; б) Т = 2,5 мкс, v = 400 кГц. Указание. Согласно формуле Томсона Т = '2n^LCo, где Со — емкость С батареи конденсаторов, равная в случае а и 2С — в случае б. 17.2. Ответ: а) 5,0 А; б) 10 А. Решение. Максимальная сила тока 7м достигается в тот момент, когда конденсатор полностью разряжен. Если пренебречь поте- рями энергии на нагревание контура и излучение электромагнит- ных волн, то энергия заряженной батареи конденсаторов Wo — СМ = —5— полностью переходит к этому моменту в энергию Wm = А __ = магнитного поля в катушке: Wo = Wm. Отсюда 7м В случае а получаем 7м = 17м в случае б находим 7м = 17м 17.3. Ответ: а) 7,0 А; б) 8,7 А. Решение. В начальный момент напряжение на конденсаторе мак- симально, а сила тока i равна нулю. Следовательно, эти величины
501 Электричество и магнетизм изменяются согласно формулам и - С7мСОЗ(0< = L/mCOS-^-, (1) г г . 2тг£ I = /MSincaf = /Msin-^- (2) 2л (о = — циклическая частота колебаний). Величины 1м, Т определены при решении задач 17.1 и 17.2. . . т . 2itti a) ti = 7Msin-y~ = 7,0 А. б) Чтобы выразить силу тока i через напряжение и, следует исключить время t из соотношений (1) и (2). Из тождества sin2<ot + + cos2cd£ = 1 следует: т. е. i = ЛчаА - X -и*)= 8,7 А. V С/м V Lt Последний результат можно получить и непосредственно из CoUh См2 Li2 закона сохранения энергии: —х— = ы ы 17.4. Ответ: раздвигать пластины, уменьшать площадь перекры- тия пластин. Решение. Из формулы Томсона и соотношения X = сТ (X — длина электромагнитной волны, с — ее скорость) следует, что X = 2ncVZc. Следовательно, емкость С конденсатора нужно уменьшать. Соглас- „ E»eS но формуле С = Для уменьшения емкости плоского конденса- тора следует либо увеличить расстояние d между пластинами, либо уменьшить площадь S их перекрытия. 17.6. Ответ: £* = где k = 1, 2, .... N. X™» = Х,ш(^)г. 'Си/ 'Си/ Решение. Определим сначала, при какой индуктивности Li ка- тушки нижняя граница диапазона длин волн радиоприемника совпадает с Xmta. Поскольку Xmi„ = 2nc'lLtCi, находим Li = Xniin 4л4с2С1" Индуктивности катушек L* (k = 1, 2, .... N) обеспечивают макси- мальную ширину диапазона и полное перекрытие его частей при условии 2xcVl»C2 = 2nc'lLbiCi. Следовательно, Lm = (т. е. по- ч } VI
Ответы, указания, решения 502 следовательность L* составляет геометрическую прогрессию: L* = "'(J ). Наибольшая индуктивность Ln , так что = 2лСУ1ЬпСг - 2лс 17.6. Ответ: 4 катушки. (Сг\2 —I , можно подо- Ci' брать катушки так, чтобы соответствующие им участки диапазона перекрывались. 17.7. Ответ: 2000 колебаний. Решение. За один период Т.„ = — звуковых колебаний произойдет v Т„ А N = электромагнитных колебаний с периодом Тм = —. Таким 1 ЭЛ С образом, N = — = 2 • 10я. vA 17.8. Ответ: увеличить в 9 раз.. 17.9. Ответ: частота уменьшится; увеличится. Решение. Изменение собственной частоты обусловлено измене- нием индуктивности катушки. Ферритовый сердечник, намного усиливая магнитное поле, увеличивает индуктивность катушки. Согласно формуле Томсона период свободных колебаний при этом растет; следовательно, частота уменьшается. Если внести в катуш- ку медный сердечник, то в нем при периодических изменениях магнитного поля катушки возникнут вихревые индукционные токи (токи Фуко). Согласно правилу Ленца магнитное поле этих токов уменьшает магнитное Поле катушки, а значит, и ее индук- тивность. Частота свободных колебаний возрастет. Правда, воз- растет и затухание колебаний за счет нагревания сердечника индукционными токами. Заметим, что на величину постоянного магнитного поля медный сердечник практически не повлиял бы (магнитная проницаемость ц меди оЧень близка к единице). А вот у железа ц > 1. Поэтому при введении железного сердечника возникли бы конкурирующие факторы: усиление магнитного поля за счет ферро- магнетизма и его ослабление за счет индукционных токов. Однозначно ответить иа поставленный в задаче вопрос в этом случае нельзя. 17.10. Ответ: Д<р = рад. Решение. Прохождение переменного тока по линии электропере-
503 Электричество и магнетизм дачи представляет собой волновой процесс. Волна проходит линию I за время t = -; за это время фаза колебаний напряжения в начале линии изменяется на Дф = cot = 2nvl п —- = -рад. 17.11. Ответ: Smflx — 18 км, п < 8,3* 103 с *. Решение. Радиолокаторы работают в У КВ-диапазоне, поэтому их излучение распространяется практически прямо- линейно. Радиолокатор действует в пре- делах прямой видимости. На рисунке А — радиолокатор, АВ 8шах ---- мак- симально возможная дальность обнару- жения плота, OB — R — радиус Земли. Учитывая, что ОА = R + й, запишем теорему Пифагора для прямоугольного треугольника OABt (R + й)2 = R2 + sLx. Отсюда Snuuc - >lh(2R + й) ~ ^2Rh = 18 км (2R + й « 2R, посколь- ку й « R). Отраженный радиоимпульс должен вернуться до того, как начнется излучение следующего импульса, т. е. 2sтих с £ п Отсюда п < — = 8,3 • 10’ с 1. max 17.12. Ответ: 9000; А„,„ = ., 225 м. Указание. Радиолокатор способен принимать отраженные сиг- налы лишь после окончания излучения очередного имнульса. 17.13. Решение. Радиосвязь и радиолокация возможны только в том случае, когда плотность нотока I энергии, поступающей к приемному устройству, превышает некоторую величину 1о, харак- теризующую чувствительность приемника. Поскольку I обратно пропорциональна квадрату расстояния R от передатчика, выпол- Р няется соотношение Io = k-jg (здесь Р — излучаемая мощность, k — безразмерный коэффициент; для передатчика, равномерно из- 1 1<Л2 лучающего по всем направлениям, k = — ). Отсюда Р - —г—, т. 4п « е. Р ~ Я2. При радиолокации цель отражает пришедшим к ней радио- сигнал, т. е. сама становится вторичным источником радйоволЙ,
Ответы, указания, решения 504 Р „ мощность которого Р пропорциональна Интенсивность вернув- л шегося к радиолокатору отраженного сигнала пропорциональна Р' Р т. е. -=i. Отсюда Р ~ R4. Таким образом, для увеличения «К /с дальности радиолокации в три раза потребуется увеличить мощ- ность передатчика в З4 = 81 раз. 17.14. Ответ: на 7,8 см. Решение. Сигнал, отразившийся от крыши, проходит на пути к приемной антенне «лишних» 3 км; поэтому он запаздывает по сравнению с сигналом, распространявшимся прямолинейно, на At = 10'5 с. В результате второй контур изображения смещен относительно первого на AZ = oAt, где и — скорость перемещения электронного «луча» по строке. Эту скорость можно найти из условия, что за одну n-ую долю секунды луч пробегает N строк длины I, откуда v = nNl. Таким образом, AZ = nNl&t = 7,8 см. 17.15. Ответ: ZM = (7 А/ --С-—. VL(Ci + С2) Решение. После замыкания ключа в цепи возникают свободные электромагнитные колебания. При этих колебаниях изменяются заряды qi и дг на конденсаторах и сила тока Z через катушку. Воспользуемся законом сохранения электрического заряда и за- коном сохранения энергии (ведь потери энергии отсутствуют, колебания незатухающие): qi + дг = CiUi, д,2 д22 Li2 C,Ui2 2С, + 2Сг+ 2 2 ' „ Т-2 ^гг 2 9>г (С.СЛ-дО2 „ Отсюда Lr = CiUi--^-------—-—. Это выражение макси- Ci с2 мально, когда производная от правой части по gi равна нулю: 2д>м 2(jCiC7> - д>м) п С>гСЛ „ —-----------------= О, т. е. при gw =-—. Величину q,n мож- О2 Ci + С2 но найти и без дифференцирования. Достаточно заметить, что i = /м при прохождении системой «положения равновесия» (при механических колебаниях скорость также максимальна при про- хождении положения равновесия), т. е. дш — «равновесный» заряд первого конденсатора при замкнутом ключе. Он определя- ется из условия равенства напряжений на обоих конденсаторах и закона сохранения заряда:
505 Электричество и магнетизм фм _ фи Ci Сг ’ фм + дгм — CiCJi. Подставляя полученное значение фм в формулу для i, находим Заметим, что при Сг -* со получается /м = U• Это соответ- ствует результату для обычного колебательного контура (см. за- дачу 17.2), как и следовало ожидать. 17.16. Решение. Величину U нетрудно найти из закона сохранения Uo электрического заряда: CUo = 2CU, откуда U = -х-. Следовательно, £ энергия электрического поля в конденсаторах после замыкания с(—V |AUz1 CUo2 Ч 2 f CUo2 _ ключа уменьшается на величину |AW| = —~---2—s— = ——. Та- ^4 ким образом, ровно половина начальной энергии «теряется». Может ли «потерянная» энергия перейти во внутреннюю энергию проводов? Проверим это. Увеличение внутренней энергии прово- дов Q ~ Io2Rx, где 1о = — начальная сила тока в цепи, R — сопротивление проводов, т ~ RC — время перераспределения зарядов между обкладками конденсатора (см. задачу 13.83). Под- ставляя значения 1о и т, получаем1’ Q — CUo2, т. е. Q ~ |ДЖ|. Обратим внимание на то, что полученный результат не зависит рт R, т. е. формально он применим и при сколь угодно малом сопротивлении R. Но ведь сверхпроводящие провода вообще не нагреваются при протекании тока! В этом случае на первый план выступает другой фактор. Всякая замкнутая цепь обладает индук- тивностью, поэтому рассматриваемая цепь представляет собой колебательный контур. После замыкания ключа в цепи возникнут электромагнитные колебания, затухающие вследствие излучения электромагнитных волн. Эти-то волны и унесут «потерянную» энергию. 1) [ 9 2Г Болеелочный подсчет выглядит так: Q = I iRdt, где I = /ое~ яс (формула для 'о силы тока i выводится аналогично задаче 13.81). Подставляя i в выражение для CU % Q, находим Q = = |AW|.
Ответы, указания, решения 506 17.17. Ответ: 7м = ил]~у. Решение. Сразу после замыкания ключа, до начала электро- магнитных колебаний, происходит очень быстрое перераспреде- ление заряда между конденсаторами (ведь замкнутая цепь из конденсаторов практически не обладает индуктивностью). В ре- зультате перераспределения заряда получается батарея конденса- торов общей емкостью 2С, заряженная до напряжения уу. В этом Л процессе «теряется» половина начальной энергии первого конден- сатора (см. задачу 17.16). Именно учет этой «потери» энергии и представляет основную трудность в задаче. В дальнейшем энергия из цепи не уходит, так что 2С Lilt I 2 / 2 2 Отсюда 7м = US\r~y. 17.18. Ответ: п — 8,0 с*1. Решение. Будем отсчитывать время t от момента, когда нормаль к плоскости рамки совпадает по направлению с вектором В. Тогда в произвольный момент времени угол между нормалью и В сос- тавляет а = <£>t — 2nnt, где © — угловая скорость вращения. Магнитный поток через рамку Ф = BSNcos2nnt. Согласно закону электромагнитной индукции ЭДС в рамке с?Ф , е = —тг = - Ф' = 2nBSWnsin2;cn£ = 6Msin2flnf. at Следовательно, £м = 2nBSNn и п - = 8,0 с 1 == 480 об/мин. 2itBSN 17.19. Ответ: С7мВ = 7мй, иж = ~,иж = 1мыЬ; ©С 1 I-------------- WL ~ ©С Un = 7м*уН2 + (©L--—)2, и = Uncos(at + <р), где ф = arctg1--- Решение. Амплитуда напряжения на резисторе Ur = InR; ампли- туда напряжения на конденсаторе Unc = InXc = амплитуда ФО напряжения на катушке Uml = 7mYl = 7M©L. Здесь Хе = — ем- ©С костное сопротивление, Хь = ©£ — индуктивное сопротивление.
507 Электричество и магнетизм Казалось бы, при последовательном соединении 17м = Umr + С7мс + + Uml. Но это не так, Потому что в цепи переменного тока мгно- венные значения напряжения на отдельных элементах — это функции времени, а не постоянные величины! По существу речь идет о сложении гармонических колебаний. При этом очень важно, что фазы трех складываемых гармонических колебаний различны: ин(<) совпадает по фазе с силой тока, Uc(t) отстает от тока на Ui.(t) опережает ток на Запишем закон изменения каждого из напряжений: Ur = IvRcosat, Uc - IxXeCOsitot - Ui = 7mXi.COs(©^ + Мгновенное значение приложенного к цепи напряжения и = ur + u< + Ul = EAutCoscot + (СТмс - СТмОзтсЦ = - ^Umr + (CZml - E7Me)2cos(ot + cp), , E7ml — 17mc . . где (p - arctg———— (см. математическое приложение). U MR Итак, при сложении мгновенных значений периодически изме- няющихся величин (в данном случае — напряжений) их амп- литуды не всегда складываются. Выражение (1) можно записать в виде U - Е7мсоэ(®4 + <р), где амплитуда напряжения во всей цепи С7м = /м^Т?2 + (Xi. - Хс)2 = ImZ. Выведенное здесь соотношение обыч- (1) но записывают в виде 7м = Uh/Z и называют законом Ома для цепи переменного тока, а величину Z — полным сопротивлением цени переменного тока. Величина ср характеризует сдвиг фаз между колебаниями силы тока и напряжения в цепи. Ее можно записать в -Xl ~ Хс виде ср = arctg--—. Полезно также иметь в виду, что costp = R/Z. К 17.20. Ответ: см. рисунок. Решение. Как показано в задаче 17.19, 7м = —, где I ....... 2U Поскольку второе слагаемое под корнем в выражении для Z неотрицательно, Z > R (т. е. 7м < U^/R ). Величина Z принимает минимальное значение Zmin = 7? при &L —— = 0, что соответствует ©С © = ~г=^ = о» (здесь ©о — циклическая частота свободных коле-
Ответы, указания, решения 508 баний в рассматриваемом контуре). При О < со < шо функция И(со) моно- тонно убывает (7м возрастает), а при со > соо функция И(со) возрастает (7 м убывает). График зависимости /м(со) для двух различных значений R (Я1 < Яг) приведен на рисунке (поскольку (7Мн = 7мЯ, зависимость С7мн(со) имеет такой же вид). Высота максимума при со = соо и фиксированном 17м зависит от величины Я. Резкое увеличение 7м при со = соо называют резонан- сом в цепи переменного тока. Очевидно, при резонансе XL = Хг = = л/—, ср = 0 (отсутствует сдвиг фаз между током и напряжением), С7мн = С7м (напряжение на резисторе совпадает с приложенным ко всей цепи напряжением). Однако это не значит, что отсутствуют напряжения на конденсаторе и катушке: ТТ . II <l/ctt иMl. - тт Л 'Е/С - е. При больших значениях величины —~, называемой доброт- я ностью контура, напряжения на конденсаторе и катушке при резонансе многократно превосходят (7м (что может привести, например, к пробою конденсатора). Но поскольку напряжения ис и ul изменяются в противофазе, в любой момент ис + ul = 0. Таким образом, напряжение на участке цепи переменного тока может во много раз превосходить напряжение на всей цепи! 17.21. Ответ: Хи = 3,2 кОм, Хи = 157 Ом, Zi = 3,2 кОм. Хсг = 16 Ом, Xi.2 = 31,4 кОм, Zi = 31,4 кОм. v0 = 1 2лл/£С = 225 Гц. Указание. См. задачи 17.19 и 17.20. 17.22. Ответ: Рв = 4гЯ = 72Я, где I = -fe Pc = Pl = 0. 2 У2 Решение. Мгновенная (т. е. средняя за очень малый промежуток времени) мощность на любом участке цепи р = ut, где и, I — мгновенные значения напряжения и силы тока. Пели i = lucosat, то напряжение на резисторе изменяется по закону uR = IsJlcosojt, на конденсаторе ис = 7mXcCos (cot - xl = 7мХс8пко£, а на катушке Ul = /nXicos/cot + тп = - PsiXtsincot. При нахождении средних зна-
509 Электричество и магнетизм чений произведений ui воспользуемся тем, что —-а—- 1 + cos2cot 1 ---------——- 1-. д созоП =------5-----= coscotsincot = -~sm2cof = О £ лл (черта сверху означает здесь усреднение за время, равное периоду Zm2 Zm колебаний). Тогда Рк = рй - ~^~R - FR> где I - — действующее значение силы тока; Рс = рс = 0 и Pt = pi. - 0. Таким образом, конденсатор и катушка в среднем не потребляют энергии (на- помним, что речь идет об идеализированных элементах цепи, не обладающих активным сопротивлением). Конденсатор четверть периода заряжается, запасая энергию электрического поля WP = сиг =..9 , но следующую четверть периода он разряжается, пол- нвстью возвращая энергию в цепь. При возрастании силы тока в катушке, т. е. также в течение четверти периода, она запасает Li2 энергию магнитного поля однако за следующую четверть & периода эта энергия также полностью возвращается в цепь. Толь- ко в резисторе (элементе цепи, обладающем активным сопро- тивлением) происходит необратимое превращение электрической энергии во внутреннюю. 17.23. Ответ: Р = U/созф. Решение. Величина Р совпадает со средней мощностью, потребля- емой резистором: Р = Pr = I2R (конденсатор и катушка, как пока- зано в задаче 17.22, вообще не потребляют энергии). Воспользо- Т и R- вавшись соотношениями/ = 77 и coscp = (Z — полное электричес- Zi Zi кое сопротивление цепи), получаем. Р = ly^R = = U/созф. Be- Z Z личину совф называют коэффициентом мощности. 17.24. Ответ: Из = 15 В. Решение. Разумеется, из-за сдвига фаз между напряжениями на различных участках цепи Us * Ui + Us. Вольтметры переменного тока показывают действующие значения соответствующих напряжений. Значит, амплитуда напряжения на конденсаторе Umc = V2Ui, а ам- плитуда напряжения на резисторе Umr = V2 Us. Если сила тока в цепи изменяется по закону i = /мсозю/, то Uc = V2 UiCOs(c>t - Ur = V2 UzCOSCOt. Следовательно, полное напряжение в цепи равно (см. матема-
Ответы, указания, решения 510 тическое приложение) и = ur + Uc = V2 U&osat + V2 LAsin©t = ^2(СЛ1 2 + Uz) cos(©t + <p). Итак, Um = t/2(CA2 + Uz*). Третий вольтметр показывает дейст- вующее значение полного напряжения ЕЛ = + Uz = 15 В. 17-25. Ответ: Zt = 50 Ом, Zz = 321 Ом. срх = 53° = 0,92 рад, фг = = 85° = 1,5 рад. Указание. См. задачу 17.19. 17.26. Ответ: L = 0,13 Гн, Pi = 30 Вт, Рг = 11 Вт. 17.27. Ответ: сначала возрастает, затем убывает. Указание. Максимально возможная сила тока будет при резонан- се, когда © = 2?tv = т. е. L - -——— = 0,51 Гн. Накал лампы 4xVC возрастает при приближении к резонансу и убывает при удалении от него. 17.28. Ответ: после подключения второго конденсатора накал лампы возрастет; после подключения третьего — уменьшится, но останется больше первоначального. Указание. См. задачу 17.27. При неизменной индуктивности резо- нанс в цепи наблюдается при емкости Со = £— = 20 мкФ = 2С. 4 7t v L После подключения третьего конденсатора общее реактивное со- противление цепи (т. е. сопротивление катушки и конденсатора) равно 2ttvL -------= 51 Ом, что примерно втрое меньше первона- чального реактивного сопротивления цепи |2tivL--I = 160 Ом. 1 2л vC 1 2 17.29. Ответ: п = 100 с ; лампа горит периода. О Решение. Пусть напряжение в сети изменяется по закону и = [/маш©# (амплитуда напряжения 17м = =^2 • 220 В = 311 В). Неоновая лам- па горит, когда выполняется усло- вие |u| > U*. На рисунке штрихов- кой отмечены промежутки времени, в течение которых длятся вспышки лампы. За период происходят две вщщшхки, поэтому п = 100 с"‘. Мо-
511 Электричество и магнетизм менты времени ti к tt определяются из условия U. = C7Main<Bt: , 1 . U, Т . U, . Т . ti = —arcsin^T- = —arcainyy-, ti = - ti. со vm 2л мм 1 Продолжительность каждой вспышки « Л , , Т Т . U.. Д? = #2 - = тг---arcsin-—; а л «и значит, лампа горит в течение доли периода 2Д£ 2 . U, = 1 - — arcsinr— Т п Un = 0,67. 17.30. Ответ: а) I = 10, б) I = м в) 7 = Решение. «Ступенчатые» сигналы такого типа широко применя- ются в современной электронике. Для них, разумеется, в общем — * и случае I -т=-. Величина I представляет собой значение силы постоянного тока, при прохождении которого в проводнике выде- ляется такое же количество теплоты, как и при прохождении переменного тока за это же время. Это означает, что I2 = I2 (усред- нение производится за период). а) Квадрат силы тока в любой момент равен /о2. Поэтому I = 1о. б) I^T = I& + (-7о)25, откуда I = у. о о Z в) I*T = Ia2x, т. е. 1~~ 1о 17.31. Ответ: C7,ux = 51 В; не менее 102 В. Решение. Рассмотрим, сначала работу выпрямителя без нагрувки. Если пренебречь обратным током полупроводникового диода (а этот ток обычно ничтожно мал), то положительные заряды могут поступать на верхнюю обкладку конденсатора (при усло- вии фд > фв), но не могут с нее стекать. Значит, после включения выпрямителя в сеть конденсатор быстро зарядится до напряжения Um = V2 • 36 В = 51 В, равного амплитуде напряжения в цепи. Что изменится после подключения к выпрямителю нагрузки? Прак- тически ничего: постоянная времени т — RC = 0,5 с (см. задачу 13.83) цепи разряда конденсатора намного превышает период Т = 0,02 с переменного напряжения, в сети. Значит, за время между двумя «всплесками» тока, идущего через диод и подзаря- жающего конденсатор, напряжение на конденсаторе снизится очень незначительно. Таким образом, ЯС-цепочка хорошо сгла- живает напряжение на выходе. Резистор выполняет и другую
Ответы, указания, решения 512 функцию — защиты от перегрузки при возможном коротком замыкании выхода выпрямителя. Максимально возможное напря- жение на выходе (при разомкнутой цепи нагрузки) совпадает с напряжением на конденсаторе, т. е. равно 51 В. Чтобы определить, в каких пределах изменяется за период напряжение на диоде, запишем его в виде фп - Фа = (фп - фв) + + (<рв - фа). Остается заметить, что фп - фв (напряжение на конден- саторе) постоянно и равно С/м, а разность фв - фл (мгновенное напряжение в сети) удовлетворяет неравенству -Um < фв - Фа < Um. Значит, фп - фл < 2Um (равенство достигается в моменты, когда потенциал точки А становится минимальным). Следовательно, диод должен быть рассчитан на обратное напряжение, не меньшее, чем 217м = 102 В. 17.32. Ответ: в первичной обмотке сила тока увеличится, во вторичной — уменьшится. Решение. При удалении части сердечника магнитное поле, прони- зывающее обе катушки, уменьшится. По этой причине умень- шится и ЭДС индукции во вторичной обмотке, а следовательно, и сила тока в этой обмотке. В первичной же обмотке из-за ослаб- ления магнитного поля уменьшатся ЭДС самоиндукции и индук- ции, приводящие к уменьшению тока. В результате сила тока в первичной обмотке увеличится. 17.33. Решение. При замыкании витка обмотки реостата ток просто не пойдет по нему. В результате сопротивление реостата чуть уменьшится (число витков обычно достаточно велико) и сила тока немного увеличится, что не опасно. У трансформатора же в каждом (в том числе и в замкнутом) витке создается одна и та же ЭДС индукции. Поскольку сопротивление одного витка ничтожно мало, в нем индуцируется очень большой ток. Чрезмерный нагрев может расплавить провода или разрушить изоляцию и вызвать замыкание соседних витков. 17.34. Ответ: первичная обмотка имеет = Ui/U = 440 витков; вторичная — Na = Uz/U = 84 витка. Указание. Провод с вольтметром можно рассматривать как до- полнительную обмотку трансформатора из одного витка (этот контур пронизывается таким же магнитным потоком, как и любой из витков). 17.35. Ответ: а) во вторичной обмотке; б, в) нельзя. Решение, а) Напряжения и токи в первичной и вторичной обмот- h U, тт ках связаны соотношением v ® -=z-. Поэтому трансформатор, по- 11 Ui
513 Электричество и магнетизм нижающий напряжение, во столько же раз повышает силу тока. Необходима^ толщина провода определяется именно силой тока, поэтому во вторичной обмотке провод должен быть толще. б) При подключении к источнику постоянного напряжения в U первичной катушке идет ток силой I = —. При переменном токе полное сопротивление катушки Z = ^R2 + XI, причем индуктивное сопротивление Хъ намного превышает активное сопротивление R. Значит, при подключении к источнику постоянного напряжения сила тока резко возрастет. Катушка может выйти из строя. в) В отсутствие сердечника резко уменьшаете^ индуктивность катушки, а, значит, и ее индуктивное сопротивление. Сила тока опасно возрастает. Поэтому включать в сеть снятую с сердечника катушку нельзя. 17.36. Ответ: 17АС = 120 В. Решение. Напряжение «7Лс равно сумме напряжений на выходе каждого из трансформаторов: Uac = Uab + Ubc (поскольку Uab и Ubc совпадают по фазе). Эти напряжения можно выразить через напряжения Ui и U2 на входе трансформаторов: ТТ ТТ Na Т. тт Ni Uab — Ubc — Uzz-=-. N2 M Итак, задача свелась к определению Ui и U2. Пренебрегая активным сопротивлением обмоток трансформаторов, можно за- т * т U писать силу тока I в первичных обмотках в виде I =---------- coLi + coLz (Li,i — индуктивность катушки с числом витков TVi.a). Тогда IZi = 7<bLi = U———, Ui = U — * Li 4* L2 L\ + L2 _ Lz NI Для катушек, отличающихся только числом витков, — = (см. задачу 16.26). Поэтому Uab = Ubc = U-^-, Uac = 2U~^~ = 120 В. Ni + NI NI + NI Интересно, что при любых значениях Ni и Nz получаем Uac < U, причем равенство достигается лишь при Ni = Nz. Это следует из неравенства 2NiNz < Ni + NI. 18.1. Ответ: Н = 4,0 М. Решение. Если обозначить через а первоначальное расстояние от фонарного столба до шеста, то из подобия треугольников получаем
Н а + h , . Н а + s + 1г . „ __ ~г = —;— (см. рис. а) и -г- =--------z—— (см. рис. о). Из этих двух fl 11 fl Iz „ Tr S + lz — h _ уравнении получаем: H = h---------------= 4,0 м. lz — li 18.2. Ответ: около 7000 км. Решение 1. Луна отбрасывает расширяющийся конус полутени (см. рис. а). Поскольку вершина этого конуса (точка А) лежит намного ближе к Луне и Земле, чем к Солнцу, можно считать, что угол а п 2гс практически совпадает с малым углом р - под которым виден хСс солнечный диск с Земли. На рис. а поверхность Земли рассматрива- ется как «плоскость», нормальная к оси конуса полутени (т. е. считается, что Солнце в зените и область полутени захватывает небольшую часть поверхности Земли). Тогда диаметр области полу- тени BE = CD + 2ВС = 2гл + ссйл = 2(гл + » 7000 км (это еще ’ 2Сс' л Рис. и ' Рис. б
515 Оптика не настолько много, чтобы при оценочном расчете учитывать кривизну Земли). Если, как обычно, Солнце в момент затмения не в зените, то «плоскость» поверхности Земли образует с осью конуса угол, отличный от прямого. Из-за этого пятно полутени удлинится, но его максимальная ширина и в этом случае около 7000 км. Решение 2. Воспользуемся тем известным фактом, что видимые с Земли размеры Солнца и Луны практически совпадают, поэтому вершина В конуса полной тени от Луны (заштрихованного на рис. б) находится вблизи поверхности Земли. На рис. б изображен слу- чай, когда Солнце стоит в зените. Считая лучи 1 и 2, 3 и 4 практически параллельными из-за большой удаленности Солнца, получаем: АВ = ВС = 2т. Тогда ширина АС полутени примерно равна удвоенному диаметру Луны, т. е. АС я 7000 км. 18.3. Решение. Разобьем щель на малые участки, каждый из которых можно рассматривать как точечное отверстие. Солнеч- ный свет, проходящий через каждое из таких отверстий, будет давать на стене круглое светлое пятно — изображение Солнца (как DI в камере-обскуре). Диаметр этого пятна d - -f- ® 3 см (см. рис, а), где D — диаметр Солнца, L — расстояние от Земли до Солнца. Рио. 6
Ответы, указания, решения 516 Пятно от всей щели является результатом наложения светлых кругов от каждой из точек, поэтому оно имеет форму полоски с закругленными краями (см. рис. б). Ширина полоски d » 3 см, она не зависит от ширины щели, пока ширина щели намного DI меньше -у-; длина полоски а + d « 9 см. Уменьшение ширины щели практически не влияет на размеры светового пятна, а ведет лишь к уменьшению его освещенности. Уменьшение же длины щели до величины, заметно меньшей d, приближает форму свето- вого пятна к круговой. Однако если во время опыта случайно начнется солнечное затмение, то форма пятна станет серповидной, потому что это пятно — изображение Солнца. Тот же эффект можно изучать, наблюдая летом за формой светлых пятен на земле под кронами деревьев. Эти пятна имеют форму эллипсов, хотя они образованы солнечными лучами, прошедшими через промежутки неправильной формы в листве (см. задачу 18.4). 18.4. Ответ: Н = 9,7 м, а = 49°. Решение. Солнечные лучи, прой- дя сквозь маленькое отверстие (см. рисунок), образуют в пространст- ве круговой конус; поэтому самые крупные пятна созданы светом, прошедшим через отверстия вбли- зи вершины дерева. Угол при вер- шине конуса равен 0. Диаметр кругового сечения этого конуса на уровне земли равен Ь; из-за наклонного падения световых лучей пятно на земле имеет форму эллипса с большой осью а = ——. Учитывая, что b = L0 и Н = sina - Lsina получаем Н = — = 9,7 м и sina = — = 0,75, откуда a = 49°. a0 а 18.5. Ответ: L « 900 м. Решение. В момент наибольшего сближения боцман видит борт теп- лохода А1А2 под прямым углом (на рисунке изображен вид сверху). Из подобия треугольников ОА1А2 и ОВ1В2, где О — большой палец боцмана, Bi и Вг — его глаза, сле- L а 1а дует, что — = -г. Отсюда L = -г- » 900 I о о м.
517 Оптика Боцман в данном случае использует параллакс — изменение направления на какое-нибудь тело вследствие перемены точки наблюдения. Именно благодаря параллаксу расстояния до пред- метов гораздо точнее оцениваются с помощью двух глаз, чем одним глазом. Параллакс используют при определении рассто- яний до небесных тел, в оптических дальномерах и для создания стереоэффектов. 18.6. Ответ: 0 = 69°. Решение. Из закона отражения света сле- дует, что падающий и отраженный лучи образуют одинаковые углы с плоскостью зеркала (на рисунке эти углы обозначе- ны у). Из рисунка видно, что у = р - а = = 90° - р, откуда р = 45° + £ = 69°. Л 18.7. Решение. Рассмотрим траекторию АЕВ (см. рис. а), где Е — произвольная точка пластины CD. Длина этой траектории совпа- дает с длиной ломаной AiEB (Ai — точка, симметричная точке А относительно прямой, на которой лежит отрезок CD). Минимально возможная длина ломаной AiEB достигается, когда точка Е лежит на прямой AiB. При этом, как видно из рис. б, угол падения луча (Z AEF) равен углу его отражения (Z BEF).. Таким образом, длина траектории будет минимальной именно при зеркальном отра- жении. Это свойство отражения открыл Герои Александрийский (о дальнейшем развитии этой идеи см. задачу 18.19). 18.8. Ответ: изменится на противоположное. Указание, а + 0 = (см. рисунок), следовательно, 2а + 20 = л. А D 18.9. Ответ: на угол 2<р; этот угол не зависит от угла а; Указание. При первом отражении (см. рисунок) луч отклоняется
Ответы, указания, решения 518 К задаче 18.8 на угол 81 = it - 2а; угол падения луча на второе зеркало равен ср - а (при <р > а); при втором отражении луч отклоняется на угол 8г = тс - 2(ф - а). Угол полного отклонения 8 = 81 + 82 = 2тс - 2ф, что равносильно отклонению на угол 2ф. При ф < а луч испытает более двух отражений. Независимость угла отклонения луча от угла его падения используется, например, в секстанте — приборе для определения угловой высоты светил над горизонтом. 18.10. Решение. Воспользуемся механической аналогией — тем, что при упругом ударе шарика о неподвижную гладкую стенку также Происходит «зеркальное» отражение. Значит, вместо свето- вого луча можно рассматривать траекторию движения упругого шарика. Направим оси координат х, у, г вдоль ребер двугранных углов. Пусть проекции начальной скорости v шарика на эти оси равны vx, vy, Vx. Тогда после отскока от зеркала, лежащего в плоскости yz, скорость шарика имеет проекции -vx, Vy, v2; два последующих отскока приводят к изменению знаков vu, иг. Следо- вательно, после трех «ударов» о зеркала вектор скорости шарика имеет проекции на оси координат -vx, -vv, -Vx, т. е. этот вектор равен —и. Таким образом, направление движения, шарика из- менилось на противоположное. То же самое произойдет и с на- правлением распространения света. Уголковые отражатели (катафоты), в которых используется этот принцип, устанавливаются сзади на велосипедах и авто- мобилях, а также на дорожных знаках. Отражая обратно пада- ющий на них свет фар, катафоты хорошо видны водителю даже ночью на неосвещенной дороге. Расстояние от Земли до Луны наиболее точно было измерено с помощью отражения лазерного луча от доставленного на Луну уголкового отражателя.
519 Оптика 18.11. Ответ: см. рисунок. Искомая область отмечена двойной штриховкой. Решение. Можно воспользовать- ся тем, что зеркало играет как бы роль «окна», через которое на- блюдатель смотрит на изображе- ние экрана. На рисунке AiBi — изображение в зеркале экрана те- левизора; вертикальной штри- ховкой отмечена область, откуда можно видеть в зеркале отражение точки А; горизонтальной штриховкой — соответствующая область для точки В. Из области, отмеченной двойной штриховкой, можно видеть отражения точек А и В, а, значит, и всего экрана телевизора. Однако читать рекламу даже из этой области будет довольно неудобно! 18.12. Ответ: Н/Ч', при этом высота нижнего края зеркала должна быть вдвое меньше расстояния от глаз до пола. Решение. На рисунке отрезок АВ — «человек»; AiBi — его изображение в зеркале, висящем на стене DF-, точка С расположена на уровне глаз челове- ка. Отрезок DE представляет собой среднюю линйю треугольника AiBiC; следовательно, DE = Н/2. Аналогич- но ЕЕ = ВС/2. Этот результат, каза- лось бы, не зависит от расстояния между человеком и зеркалом. Однако когда это расстояние ста- новится сравнимым с толщиной человека, следует учитывать, что тело человека является объемным, а не плоским. По этой причине может потребоваться зеркало большей высоты, чем половина роста человека. 18.13. Решение. Падающие на поверхность воды солнечные лучи частично отражаются, а частично преломляются и уходят в воду. Коэффициент отражения света от границы раздела любых Проз- рачных сред убывает с уменьшением угла падения. Рано утром для наклонных солнечных лучей этот коэффициент близок к единице, а к полудню сильно уменьшается (особенно летом, когда Солнце в полдень стоит высоко). По этой же причине рано утром, хотя Солнце уже взошло, в воде еще темно. 18.14. Решение. Когда луч света проходит сквозь толченое стекло, ему приходится пересекать мнойсество границ раздела между
Ответы, указания, решения 520 стеклом и воздухом. На каждой из этих границ происходит отражение и преломление света. Когда же толченое стекло нахо- дится в воде, отражение света на границах вода-стекло резко уменьшается, потому что показатель преломления воды мало отличается от показателя преломления стекла. По той же причине отклонение лучей при преломлении тоже значительно уменьша- ется. Вот почему толченое стекло, находящееся в воде, почти прозрачно. 18.15. Решение. И лужу, и дорогу освещают ночью только фары автомобиля. Однако от этих двух поверхностей свет отражается по-разному: от гладкой поверхности воды — зеркально (т. е. вперед), а от шероховатой дороги — диффузно, т. е. отраженный свет частично попадает в глаза водителя. При появлении встреч- ного автомобиля ситуация может измениться: отражая свет «чу- жих» фар, лужа воспринимается как яркое пятно на темном фоне. Разумеется, в сильный дождь или ветер поверхность лужи пере- стает быть зеркально гладкой и описанные эффекты ослабляются. 18.16. Ответ: 3 изображения; см. рисунок. Решение. Два изображения Ai и Аг (см. рисунок) расположены симметрично точке А относительно зеркал 1 и 2. Эти мнимые изображения образованы лучами, отразив- шимися от одного из зеркал. Но часть лу- чей, отразившихся сначала от зеркала 1, отражается затем и от зеркала 2. После первого отражения пучок этих лучей как бы «исходит» из тючки Ai (в этой точке пересе- каются их продолжения). Значит, после второго отражения появится еще мнимое изображение Аз точки Ai в зеркале 2. Изображение точки Аг в зеркале 1 тоже попадает в точку Аз . Более двух отражений не испытывает ни один луч; следовательно, других изображений нет. Это видно и из того, что точка Аз уже не может отразиться от какого-либо зеркала: для обоих зеркал она находится в «Зазеркалье». 18.17. Ответ: 5 изображений; см. рисунок. Указание. См. задачу 18.16. 18.18. Ответ: а = 40°. „ „ sina п. _ Решение. Из закона преломления-----= — и равенства a + В = 90° sinp Пт (см. рисунок) следует: tga = —; отсюда a = arctg— = 40°. z Лет Лет
521 Оптика К задаче 18.17 18.19. Решение. Пусть vi и V2 — ско- рости света соответственно в первой и второй средах; а и Р — углы падения и преломления луча. Минимальное время достигается при прохождении света по некоторой ломаной: при vi * vz «выгодно» уменьшить длину тра- ектории в оптически более плотной среде за счет удлинения той части траектории, которая проходит в оп- К задаче 18.18 тически менее плотной среде. Выразим время t прохождения света из точки А в точку В через длину х отрезка AiC (см. рисунок): АС ВС _ j + (<* ~ xf Vi Vz Vi Vz' _ „ , x d-x Производная этой функции t' = —- *----, . ViNfti2 + x2 Vzyhz2 + (d - x)2 Воспользовавшись тем, что x . d - x . „ ПГТ----г = SUla’ li. г ....~ Sln₽’ УЙ12 + х2 Nhz + (d - х)2 , sina sinP _ получим t ---------------. Эта производная обращается в нуль Vi Vz sina Ui когда-----= —, т. е. как раз при выполнении закона преломления, sinp Уг Для точек, лежащих левее точки преломления, f < О; для точек, лежащих правее, f > О. Значит, функция t(x) принимает на ис- тинной траектории минимальное значение. Таким образом, свет «выбирает» траекторию, прохождение которой требует наимень- шего времени (принцип Ферма). Этот принцип распространяется
Ответы, указания, решения 522 и на отражение света (ср. с задачей 18.7). Он является общим принципом геометрической оптики1’. 18.20. Ответ: 1,7 м; 3,4 м. Решение. Обозначим длину тени на поверхности воды через h (см. ри- сунок). Тогда Zi = ftictga = 1,7 м; длина тени на дне 1г = h + ftztgp. Здесь р — угол преломления солнеч- ных лучей в воде. Согласно закону sin(90° - a) преломления-------------= п, где п sinp — показатель преломления воды. Следовательно, sinp_______cosa & Vl - sin2p "Jn2 - COS4J ’ откуда 1г = ftictga + -г- ' = = 3,4 м. уп - cos a 18.21. Ответ: h = 28 мм. Решение. Смещение луча а = ABsin(a - Р), где Р — угол преломления луча в стекле (см. рисунок). Толщина пластинки . „ „ acosP h = A BbosP = —----------—. smacosp - cosasinp Пользуясь законом преломления света, . „ sina получаем sinp = ——; тогда cosp = = 28 , aVn2 - sin2a откуда h = - - . ==------- sina(Nn - sin a - cosa) 18.22. Ответ: не сможет. Решение. На первый взгляд может показаться, что слой подсолнечного масла (оптически более плотного, чем вода) делает, возможным выход луча в воздух: ведь после прелом- ления на границе вода-масло луч па- MM. «I Воздух Масло l^/pi Вода ”В более строгой формулировке принципа Ферма говорится не о «минимальности* траектории, а о ее «экстремальности».
523 Оптика дает на границу масло-воздух под углом р, меньшим а (см. рисунок). Однако следует учесть, что предельный угол полного отражения у масла меньше, чем у воды! Согласно закону прелом- sina п, . „ n,sina „ . 1 ления ——- = —, откуда sinp =------. По условию sina > —, еле- sinp п» довательно, sinp > —, т. е. угол р превышает предельный угол полного отражения для масла, и, значит, выйти из масла в воздух луч по-прежнему не сможет. 18.23. Ответ: р = arcsin^^—^j. Решение. В оптически неоднородной среде, где показатель пре- ломления меняется с г точки к точке, траектория светового луча искривляется. Найти ее довольно трудно, но в данном случае в этом и нет необходимости. Разобьем объем жидкости на большое число N тонких слоев, каждый из которых можно считать опти- чески однородным (см. рис. а). Обозначим показатели прелом- ления слоев ni =п„, пг, пз, ..., Пх = пь. При преломлении на границе z sina*-i nk , , _ двух слоев (см. рис. б) —;—- = —-, т. е. zu-isma^i = n*sina*. Отсюда sina* п* следует важный вывод: при переходе от слоя к слою произведение n<sina( остается неизменным! Поэтому nisinai = nssinas, а посколь- ку ai s а и aN s р, получаем n,sina = ribSinp. Описанное в задаче явление происходит не только в неоднород- ной жидкости, но и в газе — например, в земной атмосфере, плотность которой убывает с высотой. Искривление световых лучей в оптически неоднородной среде называют рефракцией. 18.24. Решение. Воздух в пустыне нагревается в основном не прямыми солнечными лучами, а от горячего песка; поэтому нижние слои воздуха оказываются днем самыми горячими. По
Ответы, указания, решения 524 этой причине плотность (а, следова- тельно, и оптическая плотность) нижних слоев воздуха меньше, чем верхних. Свет, отраженный от какого-либо предмета, может испытать в такой атмосфере на- столько большую рефракцию (см. задачу 18.23), что это приведет к полному отражению (см. рису- нок). В результате возникнет иллюзия, будто свет отражается от зеркальной поверхности — эту поверхность и принимают за поверхность водоема. Аналогичный эффект возникает на разогре- той солнцем дороге: водитель видит впереди на дороге «лужи» воды, в которых отражается голубое небо. 18.25. Ответ: Н - 5,2 м. Решение. Луч, изображенный на рисун- ..J?-,... — ке, падает на поверхность воды под уг- '1 >“о|ао\ лом Оо, равным предельному углу пол- / к Щ 1 СА 1 X ного отражения: sinao = —. Тогда из ра- П| л IE \А ’' П > II ) >‘>11 > > > Mj) ) > JTT. венства AD = АЕ + DE следует s sinao 2Н - h s = Htgao + (Н - ft)tgao = (2Н - ft) 2 = г- У1 - Sin2Oo УП2 - 1 Отсюда Н = - 1 - 5,2 м. 18.26. Ответ: 5 м х 2,5 м; 2,7 м х 0,2 м. Указание. В ясный день плот освещен практически параллельным пучком солнечных лучей; в пасмурный же день рассеянный свет падает со всех сторон, так что в воде он распространяется по всем направлениям, для которых р < р0 = arcsin—. 18.27. Ответ: не могут. Решение. Пусть угол падения луча све- та на поверхность капли в точке А равен а (см. рисунок). Тогда угол прелом- ления Р определяется соотношением . а sina 1 гт sinp = —- < —. Поскольку треугольник АВО равнобедренный, угол падения луча на поверхность капли в
525 Оптика точке В также равен р. Так как sinp < —, полного отражения в точке В не произойдет: луч выйдет в воздух, отразившись в точке В лишь частично. 18.28. Решение. Заметим, что от влаги темнеет не только асфальт: вспомним, как темнеет, например, намокшая ткань. А вот поли- рованные поверхности от влаги не темнеют. Отсюда мы можем сделать вывод, что существенную роль играют неровности поверх- ности, и переформулировать вопрос задачи так; почему увлаж- нение шероховатой поверхности делает ее темнее? Дело в том, что неровности поверхности покрываются тонким слоем воды (см. рис. а). В результате луч, однократно отраженный от поверх- ности, не обязательно выйдет в воздух: он может испытать полное Рис. б отражение на границе вода-воздух и повторно отразиться от поверхности (на рис. а пунктиром показан ход луча при отра- жении от сухой поверхности, а сплошной линией — от мокрой). Однако при каждом отражении от поверхности часть света погло- щается, поэтому мокрая шероховатая поверхность кажется более темной. На гладкой же поверхности слой воды плоский, и полное отражение невозможно (см. рис. б). Аналогично объясняется и увеличение насыщенности цвета окрашенной шероховатой поверх- ности при ее увлажнении. Тот же эффект наблюдается, когда поверхность покрывают прозрачным лаком или пленкой — поэто- му художники лакируют свои картины, а яркие книги оклеива- ются тонкой пленкой. 18.29. Ответ: п = 1,5. Решение. Согласно условию при каждом из двух преломлений луч отклоняется от первоначального направления на угол 8/2, т. е. а - р = 8/2(см. рисунок). Из рисунка видно, что Р = <р/2. Следова- тельно, . <р + 8 sin-’-s— sina 2 , „ n = —— =-----------= 1,5. sinp . ф sin2
Ответы, указания, решения 526 18.30. Ответ: 8 = 47°. Решение. При первом преломлении луча угол преломления „ . /sina\ о Р1 = агсэпЦ——) = 19,5°. Значит, при первом преломлении луч от- клонился от первоначального направления на угол 8i = a - Pi = 10,5° (см. рисунок). Из треугольника АВС получаем <р + (90° - РО + + (90° - аг) = 180°, откуда угол падения луча на вторую боковую грань аг = <р - Pi = 40,5°. Тогда угол преломления на этой грани Рг = arcsin(nsinaz) = 77° и угол отклонения луча при втором преломлении 8г = Рг - аг = 36,5°. Таким образом, суммарный угол отклонения луча составляет 8 = 81 + 8г = 47°. 18.31. Ответ: а) 8 ?= 23°; б) 6 = 60°. Решение. Угол падения луча на вторую грань призмы равен преломляющему углу призмы: а =ф (см. рис. а). Угол преломления в случае а составляет р = arcsin(nsina), а угол отклонения 8 = = Р - a = 23°. В случае б sina > поэтому произойдет полное отражение от второй грани призмы (см. рис. б). Отраженный луч падает на Puc. a Рис. б
527 Оптика третью грань призмы перпендикулярно к поверхности, так что преломления на этой грани не происходит. Как видно из рис. б, угол отклонения 8 = 60°. 18.32. Решение. Это возможно только в том случае, если луч испытает, помимо двух преломлений на боковых гранях призмы, полное отражение от основания призмы (см. рис. а). Тогда из закона отражения можно вывести, что Pi = аз. Отсюда следует, что ai = Рз, т. е. вышедший из призмы луч также параллелен основанию. Если точка А лежит посередине основания призмы, то луч не только сохраняет направ- ление, но вышедший луч лежит на продолжении падающего. Выясним те- перь, при каком соотношении прелом- ляющего угла ср призмы и ее показателя преломления п от- носительно окружающей среды возможен такой ход луча. Полное отражение происходит при az > arcsin^. Из соотношений 90° + Pi — Ф Ф о sinai „ф • 1 пг»о = ± + а2, at = и sinPi =------ получаем Pi > ^ + arcsin--9(У, ЛЛ ЛЛ П ы U откуда sin^ > - ncos(^ + arcsin—). Заметим, что для справедли- Z п> вости этого неравенства достаточно выполнения условия + + агсзпД < 90°, т. е. < соз^. Чаще всего такие призмы исполь- зуют при ф — 90°, тогда условие п > соДу = выполняется практически для всех сортов стекла. На рис. б показан результат
Ответы, указания, решения 528 прохождения параллельного пучка лучей через рассмотренную нами призму. Ее называют оборотной, потому что такая призма позволяет переворачивать изображение. 18.33. Ответ: 6 = ф(п - 1), этот угол не зависит от а. Решение. Углы падения и преломления луча при входе в призму и при выходе из нее малы (см. рисунок), поэтому Pi = ^, р = ал. Поскольку ai = ср — Pi, получаем ai = ф - —, р - (рп - а. При первом преломлении луч отклоняется на угол 5i = a - Pi, при втором — на угол 5г = Р - ai. Следовательно, 5 = 81 + 62 = ф(п -1). К задаче 18.33 К задаче 18.34 ючл и а^п2 ~ sin2“ 01 18.34. Ответ: h ------------= 3,1 см. sin2a Решение. Падающий на пластинку луч частично отражается от ее верхней поверхности, а частично — от нижней (см. рисунок). Из рисунка видно: а = ABcosa = 2ftcosatgp. Согласно закону прелом- . _ sina ления света sinp = ——, тогда , „ sina , a>/n2 - sin2a a^n2 - sin2a tgP = « h = ——---------=-----—-------= 3,1 cm. Nn - sin a 2smacosa sin2a 18.35. Ответ: в 1,33 раза. Решение. Кажущееся уменьшение глубины объясняется прелом- лением света на поверхности воды. Пусть Н — действительная глубина водоема, а h — его кажущаяся глубина (см. рисунок). Точка А на дне водоема рассеивает попадающие на нее солнечные лучи; узкий вертикальный пучок лучей, преломившись на повер- хности воды, попадает в глаза наблюдателя (на рисунке пучок
529 Оптика сильно расширен). Определим глубину точки Ai пересечения вертикального луча с продолжением преломленного луча CD. Согласно закону преломления sina 1 ----= —; из прямоугольных треуголь- sinP п ВС ников АВС и AiBC находим tga = и tgp ВС ВАС Синус и тангенс малого угла д можно считать равными друг другу, поэтому BAi = —. ‘Эта вели- чина, как видим, не зависит от а: она одинакова для всех лучей, близких к вертикальному. Значит, именно в точке Ai находится Д' мнимое изображение точки А дна водоема. Итак, h = BAi = Глубина водоема кажется уменьшенной в и == 1,33 раза. 18.36. Ответ: I = hi +nhz = 3,0 м. Указание. См. задачу 18.35. 18.37. Ответ: h = 3,2 м. Решение. Отраженные от рыбы солнечные лучи испытывают: а) преломление при выходе в воздух, б) отражение от зеркала, в)‘преломление при обратном переходе из воздуха в воду. На каждом из этих этапов формируется новое мнимое изображение рыбы (обозначенной на рисунках- точкой А). а) Изображение Ai находится на глубине йг/n (см. задачу 18.35), т. е. на расстоянии йз = hi + йз/n от зеркала. б) Изображение Аг (см. рис. а) находится на высоте й» = йз + hi = = 2й1 + йг/n над водой. в) Изображение Аз (которое и будет наблюдать рыба) находится
Ответы, указания, решения 530 на высоте h = nht = 2hin + hz = 3,2 м над водой (см. задачу 18.36 и рис. б). _ ~ ~ о л . /sina\ 18.38. Ответ: 8 = л + 2a - 4arcsm|—j. Указание. См. рисунок: при каждом из преломлений луч отклоняется на угол 81 = a - р, а при отражении — на угол <• zsinai _ 82 = л - 2р, где Р = arcsm|——j. Таким образом, 8 = 281 + 82 = л + 2a - 4р = _ . . /sinai = л + 2a - 4arcsin|——J. Заметим, что именно благодаря преломлению и отражению солнечных лучей в каплях дождя или тумана возникает радуга. 19.1. Ответ: см. рис. а, б, в; а) линза собирающая, изображение действительное; б) линза собирающая, изображение мнимое; в) линза рассеивающая, изображение мнимое. Указание. Все лучи, вышедшие из точки А и прошедшие через линзу, попадают в точку Ai. Во всех трех случаях построение удобно начи- нать с нахождения точки О (она находится на пересечении прямых MN и AAi, так как луч, проходя- щий через оптический центр тон- кой линзы, не преломляется). За- тем можно построить луч АВ, па- раллельный MN. Этот луч (или его продолжение) пересекается с MN в точке F.
531 ' Оптика 19.2. Ответ: см. рйс. а, б. Указание. На рис. а, б использована при построении побочная оптическая ось линзы, параллельная падающему на линзу лучу. Она пересекается с прошедшим через линзу лучом в фокальной плоскости. Второй фокус симметричен первому относительно оп- тического центра линзы. На рис. в пока- зан другой метод решения для случая собирающей линзы: на падающем луче выбирается произвольная точка А и с помощью луча АО строится ее изобра- жение Ai. После этого легко найти ход луча АВ, параллельного MN, и фокус F линзы. 19.3. Ответ: см. рис. а, б. Решение. Сложность этой задачи в том, что все три основных луча, с помощью которых обычно строится изображенйе, в данном случае совпадают друг с другом и с главной оптической осью. Однако для построения можно использовать, например, выхо- дящий из точки А «наклонный» луч и параллельную ему побоч- ную оптическую ось (см. рис. а): они пересекаются в фокальной плоскости. Другой метод построения показан на рис. б. 19.4. Решение. Учтем, что изображение Ai точки А создано всеми г лучами, вышедшими из точки А и прошедшими через линзу (см';
Ответы, указания, решения 532 рисунок). Закрыв верхнюю часть линзы, мы перекрываем часть лу- чей, но остальные по-прежнему пересекаются в точке At. Поэто- му положение и размер изобра- жения не изменятся, однако оно станет менее ярким. 19.5. Решение. Закроем непро- зрачными полосками все слои из стекла одного сорта. Оставшиеся слои «работают* (см. задачу 19.4) как целая линза из однородного К задаче 19.4 стекла. Открыв теперь эти слон и закрыв остальные, получим линзу из другого сорта стекла, а, значит, с другим фокусным расстоянием. Если открыты все слои линзы, она дает два изобра- жения одного источника света. На экране, перпендикулярном главной оптической оси линзы, лишь одно из изображений может быть четким. Лучи, преломленные слоями из стекла другого сорта, образуют вокруг этого изображения светлый ореол. 19.6. Ответ: не всегда. Решение. Направление отклонения луча при преломлении зависит от соотношения показателей преломления граничащих сред. Дво- яковыпуклая линза является собирающей, если показатель пре- ломления Пл материала линзы больше показателя преломления пс окружающей среды (стеклянная линза в воздухе или воде). Если же Пл < пс (воздушная линза в воде или стекле), то двояковыпук- лая динза является рассеивающей. 19.7. Ответ: F = 12 см. Решение. На экране может быть получено только действительное изображение. Значит, линза собирающая, а предмет и экран расположены дальше от линзы, чем ее фокусы. Следовательно, d = = F + at, f = F + аг, и согласно формуле тонкой линзы 1 11 F + at F + аг F’ Отсюда F = ylaiaz = 12 см. 19.8. Ответ: Zmin = 4F, Г = 1. Решение. Согласно формуле тонкой линзы , Fd 1 1 1 л ^ + у = р. Отсюда , и для расстояния между предметом и изображением d — F
533 Оптика получаем t=d+/= d-F' Это выражение достигает минимума при d = 2F (когда произ- водная I по d обращается в нуль); при этом Zmin = 4F. Тот же результат можно получить и иначе, воспользовавшись if + d\2 l2 z неравенством fd < ) = -j (см. математическое приложение). Тогда > p или I > 4F. Равенство достигается при d = f = = 2F. Итак, предмет и его изображение должны быть расположены f симметрично относительно линзы. При этом Г = -7 = 1. 19.9. Ответ: d = 0,37 мм. Решение. Яркое (и горячее) све- товое пятно от зажигательного стекла представляет собой изоб- ражение Солнца. Солнце нахо- дится настолько далеко, что его изображение расположено в фо- кальной плоскости. Из оптического центра О линзы (см. рисунок) это изображение видно под тем же углом 0, что и солнечный диск (ведь солнечные лучи, проходящие через точку О, не меняют своего направления). Поскольку угол 0 мал, d = 0F = 0,37 мм. 19.10. Решение. На рисунке АВ — предмет, AiBi — его мнимое изоб- ражение; ОВ = d, OBt = |/|. Линей- т- AiBi ное увеличение Г = . Из подо- ** бия треугольников ОАВ и QAiBi I «I следует: Г = -?. Из подобия треу- С* гольников A1B1F2 и COFi следует: „ AiBi _ AiBi F + |/| АВ “ ~СО ~ ~F~‘ Приравнивая правые части обеих формул для Г, получаем:
Ответы, указания, решения 534 Отсюда А - та = 4;; если считать f < 0, то И = -f, и формула , - г 111 принимает обычный вид формулы тонкой линзы -j + -7 = a f г 19.11. Решение. На рисунке АВ — предмет, А1В1 — его мнимое изображение; OB = d, OBt = И, OFt = OF 2 = |F| (точки Fi и F2 называют мнимыми фокусами, поскольку в них собираются не сами пучки лучей, пада- ющих на линзу параллельно ее главной оп- тической оси, а продолжения этих лучей). Из подобия треугольников ОАВ и ОА1В1 следует, что линейное увеличение Г - АЛ । «I равно Из подобия треугольников F1A1B1 и FtCO следует: Г_^1В, _ 1*1-И АВ СО |F| ’ Отсюда и Если считать f < О и F < О, то |/| = -f и |F| = -F. Полученное уравнение тогда приобретает вид 1 1_1 d + / ” F' 19.12. Ответ: / = 12 см; изображение дей- ствительное (см. рисунок). Решение. Воспользуемся обратимостью хода световых лучей. Тогда вопрос при- нимает такую форму: на каком расстоянии / от линзы следует разместить точку At, чтобы ее мнимое изображение оказалось на расстоянии а от линзы? Уравнение тон- кой линзы принимает вид / а ~ F (1) (см. задачу 19.10), откуда / — 12 см. Ход лучей показан на рисунке. Изображение действительное, поскольку в точке Ai пересекаются сами лучи, а Не их продолжения. Заметим, что формула (1) представляет собой формулу тонкой линзы, в которой d - -а.
535 Оптика Точку А можно рассматривать как «мнимый* источник света или «мнимый» предмет — в этой точке пересекаются не сами лучи, а их продолжения1’. 19.13. Ответ: в 60 см от линзы; изображение мнимое (см. рисунок). Указание. См. задачу 19.12. Считая d = -а, получаем f = -60 см. 19.14. Ответ: d = 10 см; см. рисунок. Решение. Воспользуемся формулой тонкой линзы Д + 4 = Д и а Т г соотношением Г = |^|, где Г линейное увеличение (в данном случае равное Д ). Подставляя в первое уравнение f = -rd, полу- и , Г(Г-1) чаем d =----=10 см. Г 19.15. Ответ: D = -8,3 дптр. Решение. Мнимое изображение и предмет находятся по одну сторону от линзы, так что l = d-\f\ = d + f. Увеличение lh , 1Н „ -—r7, f = - -—Согласно форму- ft - Н h- Н (h-Hf ------= — о,о ДПТр. Г = ^ = 4 = - 4. Отсюда d = nd, fl п 1 1 ле тонкой линзы D = + -г =-- d f Hnl 11 Суммируя результаты задач 19.10 19.12, можяо утверждать, что формула тонкой линзы в виде J + у = р применима ко всем случаям получения изобра- жений с помощью тонкой линзы, если величины, соответствующие мнимым точкам, считать отрицательными: в случае «мнимого» предмета (т. е. когда на линзу падает сходящийся пучок лучей) d < О, в случае мнимого изображения f < 0, в случае мнимого фокуса (т. е. для рассеивающей линзы) F < 0. Увеличение линзы во всех случаях Г •= |Ч. Ji'
Ответы, указания, решения 536 19;16. Ответ: Н = 60 мм. Указание. Каждая половина линзы действует как целая линза (см. задачу 19.4), ось которой находится на расстоянии з/2 от источ- ника света А. Изображение источника находится на расстоянии , s f sF m h = g • —— от этой оси (см. рисунок). Тогда расстояние между изображениями Ai и Аа равно Н = 2Л + з - sd d-F 60 мм. Тот же результат можно получить из подобия треугольников АА1А2 и АО1О2. 19.17. Ответ: 15 см или 10 см. Решение. Существуют две возможности получить на расстоянии а от линзы пучок света радиуса г (см. рис. а, б). В первом случае действительное изображение источника света получается на рас- Рие. б , aR . стоянии f =----= 60 см от линзы, а во втором — на расстоянии R - г aR f =----= 20 см. Величину F определяем из формулы тонкой R + г линзы: в первом случае F = 15 см, во втором F = 10 см. 19.18. Ответ: г = 40 мм. dF Указанве. Расстояние от линзы до экрана f - ——— = 60 см. Если d-F _ г d + f убрать линзу, то — = у-
537 Оптика 19.19. Ответ: a) F = 9,1 см; б) F = 20 см. Решение, а) Поскольку Г = 0,1, расстояние от оптического центра объектива до пленки f -Vd. Согласно формуле тонкой линзы „ fd rd F = —-— =-------= 9,1 см. f +d Г+ 1 rd б) Разумеется, и в этом случае F = ——- (в данном случае Г = ^qqq, d = h). Поскольку Г < 1, можно считать F = Vd = 0,20 м. Обычно при фотографировании условие Г « 1 выполняется, поэтому можнЬ считать F = f = Vd. 19.20. Ответ: At < 0,83 мс. f F Решение. Увеличение при фотографировании равно Г = (см. задачу 19.19). Разумеется, Г < 1, т. е. изображение получается уменьшенным. За время экспозиции автомобиль перемещается на расстояние vAt, так что размытость контуров изображения, обус- ловленная движением автомобиля, равна TuAt. Из условия ГиА4 < а получаем At < = 8,3 • 10-4 с. 19.21. Ответ: L > = 15 м; т < 2 мс. F Указание. См. задачу 19.20. Фотографируемая поверхность дви- жется относительно объектива со скоростью, равной первой кос- ,— F мической скорости Ui « ygR. Увеличение фотоаппарата Г = -ту. /j 19.22. Ответ: F = Н1<^ ^г- = 5,6 см. Hl — 112 Н f F Указание. Из формулы тонкой линзы следует: Г = -г- = -у =-, Л a d — F где h — высота фотографируемого предмета. 19.23. Ответ: 1,8 м < d < 2,3 м. Решение. Если d * do, изображение точки получается за пленкой или перед ней. На пленке при этом возникает не точечное изобра- жение, а размытое круглое пятно. Согласно формуле линзы плен- „ ___ , Fdo , Ка расположена на расстоянии fo = —---- от объектива. Если do - F расстояние от объектива до изображения равно f, диаметр пятна
Ответы, указания, решения 538 на пленке b = D (см. рис. а, 5). Из условия Ь < а получаем fi < f < /2, где fi = taD— и f2= Значит, dz < d < di, где D + a D - a Ffi Ffz di = ---- w. dz- --Подставив в соотношения для di и dz fi-F fz-F выражения для ft, fz и /о, получим окончательно 1 a(do - F) . _ ado 1 FD~ FD и dz =-----7---—- » —^° , - 1,8 м (мы учли, что F < do). a(do-F) , ado 1 + ~ fd ’ FD 19.24. Ответ: F < 0,67 cm. Указание. См. задачу 19.23. В данном случае di -* 00 при F — = = 0,67 см (при этом d2 = тг = 1 м, т. е. изображение получа- ется достаточно резким при фотографировании с любого рассто- яния, большего 1 м). 19.25. Ответ: d ₽ 2,5 м; L = 2,0 мм. Указание. Преломившиеся при выходе из воды в воздух световые лучи дают мнимое изображение краба в натуральную величину на глубине Н = — (см. задачу 18.35), т. е. на расстоянии d = Н + Нг~ Не — 2,5 м от объектива. 19.26. Ответ: ft = 30 см, fz =“ 60 см. Решение. Запишем систему двух уравнений Id + f “ I, d* f г
539 Оптика Здесь d — расстояние между лампой и линзой. Эта система приводит к квадратному уравнению - If + IF — О, из которого находим Л,2 = ± Заметим, что di = fa и da = fi. Итак, во втором решении расстояния от линзы до стены и до лампы просто меняются местами по сравнению с первым. Наличие двух таких решений следует из обратимости хода световых лучей. h2 19.27. Ответ: Л2 = V- Л1 Решение. Во втором случае (см. задачу 19.26) расстояния до предмета и изображения меняются местами: /а - di, dz = fi. Согласно , hi fi ht h di h , формуле увеличения линзы = ~ = ~Г ~ Второе изоб- ражение уменьшено во столько же раз, во сколько первое уве- личено. Его высота Ла = -г*. Л1 ' /Vn 19.28. Ответ: F = —т=—— = 10 см. (\п + I)2 Указание. См. задачи 19.26 и 19.27. Г2 = di — I + 1’ I'fn 'fn + 1' х г. Vn; следовательно, Л - 19.29. Ответ: D = 5/3 дптр. Указание. См. задачу 19.26. 19.30. Ответ: на расстоянии 8 см от одного и 24 см от другого источника. Решение. Очевидно, одно из изображений будет мнимым, т. е. . х ГТ 1 1 1 1 _ fi = -fa. Поэтому можно записать соотношения -г - у = di /2 г Cfa 11 + у = -=, где di,a — расстояния от линзы до источников света. /2 г Ffa Ffa Отсюда di = ----, da - —-. Из условия di + da ~ I находим fa + F fa ~ F I i /I расстояние между линзой и изображениями: l/J = F\lЗна- ___________________________________ - 2F F^Tt ) = 8 см 24 см. чит, di = И da
Ответы, указания, решения 540 19.31. Ответ: см. рисунок. Решение. Изображение в данном случае будет иметь необычный вид — оно будет разорванным: дейст- вительное изображение отрезка AF находится с противоположной сто- роны линзы, а мнимое изображе- ние отрезка BF — по ту же сторону линзы. Обе части изображения уходят на бесконечность (расстояние от линзы до изображения точки неограниченно возрастает по мере приближения точки к фокусу). Но какую форму имеет изображение стрелки АВ? За- метим, что если из любой точки стрелки выходит луч в направ- лении от А к В (см. рисунок), то после преломления он идет по прямой, проходящей через точку С параллельно главной оптичес- кой оси линзы. Значит, изображения всех точек стрелки лежат на этой прямой! Дальнейшие построения уже несложны. 19.32. Ответ: a) L = 20 мм; б) L = 40 мм. « X L f F г lF Решение, а) -г = j - ~,—откуда L ——— = 20 мм. ‘ d d-F d-F б) Концы стержня находятся на оси линзы на расстояниях di = d- ^Hd2 = d + 4 от линзы. Их изображения находятся на расстояниях Л = 38,2 см и /г = 34,2 см от линзы, поэтому L = = Л - fz = 40 мм. Итак, «продольное» увеличение Г1 линзы не совпадает с «попе- речным» увеличением Г2. Можно доказать, что для небольших отрезков вблизи главной оптической оси Г1 = Г1. 19.33. Ответ: см. рис. а, б, в. Рис. а Рис. б
541 Оптика Указание. В условии не указано; о каких линзах идет речь. На рис. а, бу в приведены все возможные варианты (из двух рассеивающих линз построить телескопическую систему нельзя). Разумеется, па- дающие под углом к главной оп- тической оси параллельные пуч- ки также сохраняют свою парал- лельность. 19.34. Решение. Пусть точка А размещена на некотором рассто- янии d сут линз. Тогда лучи после прохождения первой линзы формируют изображение At на расстоянии ft от линз, причем 4 + у- - Di. Заметим, что если Л > 0 (изображение действитель- U /1 ное), то на вторую линзу падает сходящийся пучок лучей, т. е. для второй линзы точкаА1 является «мнимым» источником. Если же fi < 0 (изображение At мнимое), то для второй линзы точка At является действительным источником, из которого исходит рас- ходящийся пучок лучей. Таким образом, во всех случаях dz = —fi. Тогда расстояние f до изображения Аг, создаваемого системой линз, удовлетворяет соотношению - у + -т = Dz. Складывая урав- /1 / нения, записанные для обеих линз, получаем -т + -т = Di + Dz, т. е. U / уравнение для линзы с оптической силой Di + Dz. Нетрудно обобщить этот результат на случай большего количе- ства линз: D = ^D; однако при этом необходимо, чтобы толщина всей оптической системы была намного меньше фокусных рассто- яний каждой из отдельных линз и фокусного расстояния всей системы в целом. 19.35. Ответ: по ту же сторону от линзы, что предмет, на рассто- янии 60 см от линзы; изображение действительное. Решение. Лучи от предмета АВ проходят через линзу, отражаются от зеркала и снова проходят через линзу. Проследим, какая последовательность изображений при этом возникает. Линза дает мнимое изображение А1В1 на расстоянии |/| = dF F-d = 30 см (см. рис. а); это изображение находится на расстоянии |/| + а = 45 см от зеркала, поэтому зеркало дает изображение АгВг на таком же
расстоянии от своей плоскости (т. е. на расстоянии dz = |/| + 2а = = 60 см от линзы). Итак, AzBz находится на двойном фокусном расстоянии от линзы и является для линзы действительным источником (т. е. на линзу справа падает расходящийся пучок лучей). Следовательно, изображение АзВз является действитель- ным и расположено на расстоянии 2F == 60 см левее линзы. Такая цепочка вычислений и построений, когда каждое очередное изоб- ражение является «предметом» для следующего элемента оптичес- кой системы, позволяет рассматривать довольно сложные опти- ческие системы. Другой подход состоит в том, чтобы выбрать два луча и проследить за их прохождением через систему (см, рис. б). 19.36. Решение. Лучи, исходящие из точки А, после преломления на сферической поверхности образуют внутри линзы параллель- ный пучок. Этот пучок после отражения от плоского зеркала (слоя серебра), образует такой же угол с главной оптической осью, как и до отражения. Значит, изображение At расположено симмет- рично точке А относительно фокуса. Можно прийти к этому результату и иначе: благодаря зеркалу луч проходит линзу дваж- ды, поэтому оптическая сила системы удваивается (см. задачу 19.34). Фокусное расстояние системы равно F/2, т. е. точка А расположена от системы на двойном фокусном расстоянии. Поэ- тому изображение At расположено на таком же расстоянии F от линзы. 19.37. Ответ: см. рисунок; изобра- жение действительное, расположено между линзой и ее фокусом. Указание. См. задачу 19.35. 19-38. Ответ: на расстоянии 2F. Указание. Линза дает первое изо- бражение AiBi на расстояний 3F от линзы, а второе изображение AzBz зеркало должно дать в фокусе лин- зы. Следовательно, зеркало должно К ялОлчв 10.37
543 Оптика находиться посередине между этими изображениями. При этом на зеркало падает сходящийся пучок лучей; в результате зерка- ло дает действительное изображение АгВг, находящееся перед зеркалом. 19.39. Ответ: со стороны собирающей линзы на расстоянии 90 см от нее либо в плоскости рассеивающей линзы. Решение. Проще всего восполь- зоваться обратимостью хода све- товых лучей. Направив со сто- роны рассеивающей линзы пу- чок параллельных лучей (см. рисунок), получим точечное изображение А на расстоянии 3 3Fi = 90 см от собирающей линзы (а + |F2| = ^Fi). Направив параллельный пучок лучей с противоположной сто- роны, получим точку А как раз в центре рассеивающей линзы. Естественно, источник можно смещать в плоскости, перпендику- лярной оси системы. При этом меняется направление выходящего из системы пучка лучей, но он остается параллельным. 19.40. Ответ: f = 60 см. Решение. Первая линза дает действительное изображение на рас- dFi стоянии Л = -у = 16 см от своей плоскости, т. е. на расстоянии dzFz dz = а - fi - 20 см от второй линзы. Следовательно, f = --— = dz - Fz = 60 см. Отметим, что для подобных задач нецелесообразно искать решение в общем виде, потому что формулы получаются чрезмер- но громоздкими. 19.41. Ответ: в 30 см за второй линзой; Н - 25 мм. Указание. Первая линза дает уменьшенное в 4 раза действитель- ное изображение на расстоянии 6 см от второй линзы. Это изоб- ражение является «предметом» для второй линзы (см. задачу 19.40). 19.42. Ответ: нужна собирающая линза с фокусным расстоянием F = 40 см. Решение. Первая линза дает изображение Солнца диаметром PFi (см. задачу 19.9; Р — угловой размер Солнца) в своей фокальной плоскости, т. е. за второй линзой на расстоянии Fi - I от нее. Это изображение является «мнимым» предметом для второй линзы
Ответы, указания, решения 544 (см. задачу 19.12). Вторая линза дает действительное изображение . 1 11 на расстоянии /, определяемом из уравнения----------+ -т = -=-. _ , ад - о. Отсюда/ = Г 1 + Г 2 “ 4 диаметр изображения равен рг f 'f.-i ---FlFl--- + Л - I Приравнивая эту величину РГ, где F — фокусное расстояние интересующей нас одной линзы, получаем F = F\Fi Ft + Fi — I = 40 см. 19.43. Ответ: F = 15 см. Решение. Если солнечные лучи (которые можно считать парал- лельными) сначала падают на собирающую линзу, а изображение Солнца получается на расстоянии Л от второй линзы (см. рис. а), 111/> , т, то------+ ^ = - — (Z — расстояние между линзами; I < F, по- F - I Л скольку рассеивающая линза может дать действительное изобра- жение только при падении на нее сходящегося пучка лучей). После перестановки линз изображение Солнца получается на расстоянии fi от второй линзы (см. рис. б), где —-— + I = F + I /2 г Из записанных соотношений получаем . F(F-Q . F(F + I) fi i 'I2 i • Во втором случае экран должен быть размещен дальше (по- скольку fi > fi); согласно условию fi - fi = а, откуда F = 4 = 15 см.
545 Оптика 19.44. Ответ: перед рассеивающей линзой на расстоянии 3,5 см от нее или за рассеивающей линзой на расстоянии 5 см или 35 см. Указание. См. задачи 19.40 и 19.43. 19.45. Решение. Глаз как оптическая система, преломляя лучи несколько раз, дает четкое изображение предметов на сетчатке, когда мы наблюдаем эти предметы в воздухе. При этом основное преломление происходит на внешней поверхности роговицы. По- казатели преломления воды и роговицы довольно близки, поэтому преломление на внешней поверхности роговицы в воде резко ослабляется. Оптическая сила глаза сильно падает, и глаз ста- новится настолько дальнозорким, что аккомодация с помощью деформации хрусталика уже не позволяет сфокусировать изобра- жение на сетчатке. При нырянии в маске роговица граничит не с водой, а с воздухом в маске. Переход же света из воды в воздух, находящийся под маской, происходит на плоском переднем стекле маски. При этом меняется лишь кажущееся расстояние до пред- мета (см. задачу 18.35), но четкость изображения сохраняется. 19.46. Решение. Предположим, человек смог стать невидимым. Это означает, что показатель преломления всех тканей его тела стал очень близким к показателю преломления воздуха (т. е. к единице). Но в результате на поверхности роговицы и хрусталика свет перестанет преломляться, а, значит, на сетчатке не будет формироваться изображение! Поэтому человек-невидимка должен стать слепым, и это, к счастью, помешает ему стать неуловимым преступником. Кроме того, сетчатка обязательно должна погло- щать хотя бы часть падающего на нее света — ведь именно за счет поглощенной световой энергии в сетчатке происходят фотохи- мические реакции. Поэтому сетчатка не может быть совершенно невидимой. Таким образом, законы физики надежно предохраня- ют нас от козней человека-невидимки. 19.47. Ответ: D = -6,0 дптр. Решение. Человек в правильно подобранных очках читает книгу, держа ее на расстоянии наилучшего зрения do = 25 см от глаз. Для глаза «предметом» служит уже не страница книги, а создан- ное линзой очков изображение этой страницы. Значит, очки создают мнимое изображение страницы на расстоянии d = 10 см от глаз. Согласно формуле ^тонкой линзы D = - Д = —6,0 дптр. (Zo cZ 19.48. Ответ: уменьшится на 4 дптр. 1 1 Решению. Оптическая сила глаза в первом случае Di = + у (мы
Ответы, указания, решения 546 считаем, что учебник находится на расстоянии наилучшего зрения do = 25 см от глаза; f — расстояние от оптического центра глаза до сетчатки). Во втором случае, при рассматривании далеких облаков, D2 = у. Изменение оптической силы глаза (совпадающее с изменением оптической силы хрусталика) АВ = Вг-В1 = -4' = U0 = -4 дптр. Перевод взгляда за окно приятен не только потому, что мы отрываем глаза от учебника, но еще и потому, что при рас- сматривании далеких предметов не надо напрягать мышцу хрус- талика. Вот почему врачи рекомендуют во избежание переутом- ления глаз время от времени смотреть на отдаленные предметы. 19.49. Ответ: 14 см1 < d < 33 см. Решение. Нормальный глаз позволяет без напряжения разгляды- вать предметы на расстояниях от do (расстояние наилучшего зрения) до бесконечности (дальнего предела аккомодации нор- мального глаза). Значит, именно в таких пределах может изме- няться расстояние И от глаза (линзы очков) до мнимого изобра- жения предмета, которое дает линза: |/| > do. Используя соотно- ^11 Л шение В = -г + -т, получаем при f < О неравенство U / D < 4 < D + 4-, d do т. е. ——— < d < 4 (14 см < d < 33 см). 1 + Ddo 19.50. Ответ: 25 см < d < 33 см. Указание. См. задачи 19.47, 19.49. 19.51. Ответ: а = 15 см. Решение. Рассеивающие линзы очков должны иметь такую опти- ческую силу D, чтобы «собирать» на расстоянии dz от глаз продол- жения параллельных лучей от удаленных предметов, поэтому D = - 4-. При чтении (на ближнем пределе аккомодации глаза) dz _ 1 1 „ 111 did2 ,Е D -----у. Отсюда — = -т---гиа =--------= 15 см. a di a dt d2 d2 - di 19.52. Ответ: di =* 12 см, d2 = 53 см. Указание. См. задачу 19.51. 19.53. Решение. Как следует из условия задачи, в данном случае наибольшее изменение оптической силы глаза при аккомодации АВ = = 4 дптр. У системы «глаз + очки» величина АВ такая
Оптика 547 JL же. Однако для того, чтобы хорошо видеть все предметы на расстояниях от а до «бесконечности», необходимо изменение оптической силы AD™,, = = 6,7 дптр. Поэтому одной пары очков будет недостаточно: потребуются различные очки для «ближнего» и «дальнего» зрения — вот почему у пожилых людей часто две пары разных очков. Очки для «ближнего» зрения должны давать на расстоянии di от глаза мнимые изображения точек, располо- женных на расстоянии а; поэтому оптическая сила очКов для «ближнего» зрения Z>i = — - -у-»-4 дптр. Такие очки позволяют a di хорошо видеть предметы на расстояниях от а — 15 см до Ъ — d =--------= 32 см. Чтобы хорошо видеть удаленные предметы, 1 + Didz потребуются очки с оптической силой Dz = - л ~ -7 дптр. За- U2 метим, что минимальное расстояние, на котором можно хорошо dl видеть предметы с помощью таких очков, с = --——- = 24 см. 1 + Dzdi Итак, потребуются очки «для чтения» с оптической силой -4 дптр и очки для «дальнего» зрения е оптической силой -7 дптр. Когда расстояние d до предмета удовлетворяет соот- ношению с < d < b, можно пользоваться любыми из этих очков. 19.54. Решение. Лупа представ- ляет собой короткофокусную со- бирающую линзу: ее фокусное расстояние F меньше расстоя- ния наилучшего зрения do. Лу- па должна располагаться у гла- за, а предмет — вблизи ее фо- кальной плоскости. Будем счи- тать, что точка А предмета расположена точно в фокальной плоскости линзы. Тогда вышедшие из нее световые лучи после прохождения через лупу образуют параллельный пучок (см. рис. а); в результате нормальный глаз формирует на сетчатке изобра- жение точки А без деформации хрусталика (так же, как при
Ответы, указания, решения 548 рассматривании очень удаленных предметов). Поэтому часовщики и ювелиры, правильно используя лупу, подолгу работают без переутомления глаз. Следовательно, говорить о «приближении» предмета к глазу неверно: наоборот, лупа может дать изображение даже бесконечно далеко от глаза (при этом линейное увеличение лупы тоже будет стремиться к бесконечности)! Однако при исполь- зовании лупы нас интересует не эта величина: способность раз- личать мелкие детали предмета определяется не размером изобра- жения, даваемого лупой, а размером изображения предмета на сетчатке глаза. А этот размер определяется углом <р, под которым виден предмет. Невооруженный глаз без значительного напря- жения видит предмет под углом фо = Д (см. рис. б), а при исполь- do зовании лупы (см. рис. а) тот же предмет виден под большим углом h . , „ Ф = -= (мы считаем углы фо и ф малыми). При малых углах зрения Г „ Ф do „ угловое увеличение лупы Г = -^ = Именно эту величину ука- фо с зывают на оправе лупы. Например, надпись «х4» соответствует _ do „ F = -7- а 6 СМ. 4 19.55. Решение. Приближая предмет к глазу, мы увеличиваем угол, под которым виден этот предмет. Поэтому увеличивается и размер изображения на сетчатке. Однако для того, чтобы четко различать детали предмета, требуется большая деформация хрус- талика; если предмет находится слишком близко, изображение на сетчатке начинает расплываться. Маленькое отверстие в картоне играет ту же роль, что и диафрагма в фотоаппарате: оно огра- ничивает ширину светового пучка, попадающего в глаз от каждой точки предмета. Благодаря этому размер светового пятна на сетчатке, соответствующего «изображению» точки предмета, уменьшается, и изображение становится более четким. Заметим, однако, что для эффективности такого простого оптического при- бора (действующего как камера-обскура) необходимо хорошее освещение предмета. 19.56. Решение. Угловые размеры звезд столь малы, что даже в самые сильные телескопы звезды видны как точечные источники света. Поэтому говорить об «увеличенных изображениях* звезд неверно. Однако телескоп, увеличивая угловые расстояния между звездами, позволяет наблюдать раздельно звезды, которые для невооруженного глаза Сливайтся. Например, при наблюдении
549 Оптика даже в школьный телескоп видно, что Млечный Путь — это гигантское скопление звезд (наша Галактика). Кроме того, плот- ность потока световой энергии, попадающей в глаз, при исполь- зовании телескопа становится намного больше: ведь объектив телескопа намного больше зрачка. Поэтому в телескоп видны слабые звезды, невидимые невооруженным глазом. 19.57. Ответ: на 0,93 см дальше от объектива. Решение. Объектив дает изображение Луны в своей фокальной dF плоскости, а изображение предмета — на расстоянии f = -——, т. е. дальше изображения Луны на Af = f - F =-— = 0,93 см. d-F Окуляр же всегда располагается так, чтобы созданное объективом изображение находилось в его фокальной плоскости. Значит, во втором случае он должен быть дальше от объектива на А/ — 0,93 см. 19.58. Ответ: Г = 150. Решение. В задаче идет речь об угловом увеличении микроскопа (см. задачу 19.54). Оно определяется как Г = Г1 • Г2, где Г> — линейное увеличение объектива, Г2 — угловое увеличение окуля- ра. Чтобы не приходилось напрягать мышцу хрусталика, из окуляра в глаз должны идти параллельные пучки лучей, т. е. объектив должен давать изображение в фокальной плоскости оку- ляра. При этом Г2 = Ф (см. задачу 19.54). Увеличение объектива Г2 r Fi , 111 Г1 = —---(это следует из формулы тонкой линзы -у + и d-Fi d Т ri f Fidn соотношения Г> = -у ). Отсюда Г =--—— • 150. d F£d - FO 19.59. Ответ: Fz = 5,0 см. Решение. Пусть предмет расположен на расстоянии d от объектива. dFi Тогда его изображение расположено на расстоянии f = —-----от d - Fi f Fi объектива и увеличено в Г1 = -у =--------раз. Это изображение » d - Fi находится в фокальной плоскости окуляра (окуляр работает как лупа), так что f + Fz = I; окуляр увеличивает изображение в Г2 = раз (do = 25 см). Увеличение микроскопа Г2 т-, г, т-ч F . } ... Т 3= Г1Га=?'/- у.'л.• - ; ; rr • п-.. • t :rti. ,
Ответы, указания, решения_________________________________&50 тт fZ-Fl П 1 Последнее уравнение вместе с уравнением------+ Fz = I обра- d - Fi „ ddl -Fi) r _ зует систему, из которой находим Fz = —i----- = 5,0 см. do+rFi Можно предложить и приближенное решение задачи, гораздо более простое и в данном случае достаточно точное. Запишем Г гг увеличение объектива как Г1 = — = —j— и воспользуемся тем, что Гг Но ГГ1Г- для микроскопа Гг » 1. Следовательно, d « Ft и f ~ I\Fi = ———z; do rF iFz „ . _ dal ~ поэтому —v— + Fz « l и Fz «-------= 5 cm. d« do + ГГ; 20.1. Ответ: v = 12 c-1. Решение. Отраженный свет не виден наблюдателю, если за время 2Z t = — прохождения света до зеркала и обратно колесо провернется так, что на пути отраженного светового пучка окажется не про- резь, а зубец. Минимальный угол поворота колеса за время t, при котором это возможно, равен — рад. Полный оборот колесо сделает 1 с за время 2Nt. Следовательно, v = ^777 = = 12 с-1. Z/V t 42VI 20.2. Решение. Природа света и радиоволн одинакова — это электромагнитные волны. Но у света длина волны меньше 1 мкм, а у радиоволн она в десятки и сотни тысяч раз больше. Отражение от поверхности будет зеркальным, если размеры неровностей меньше длины волны, а высота неровностей на поверхности листо- вого железа больше 1 мкм, но меньше 1 мм. 20.3. Ответ: надпись станет зеленой, фон — черным. Решение. Белая поверхность хорошо отражает лучи всех цветов видимого спектра, поэтому белая надпись отразит зеленый свет и будет казаться зеленой. Красная же поверхность хорошо отражает красный свет, а зеленый свет поглощает. Значит, зеленый свет почти не отражается от красной поверхности, и фон будет казаться темным, почти черным. 20.4. Ответ: X' = 547 нм; красный; красный. Решение. Волны, в том числе и световые, при переходе из среды в среду частоты не меняют (ср. с задачей 7.27), однако длина волны изменяется пропорционально скорости света в, данцрйсре-
551 Оптика де. Поскольку скорость света в воде уменьшается в п раз по сравнению с воздухом, получаем X' = — = 547 нм. Действие света на молекулы светочувствительных веществ (на сетчатке глаза и на фотопленке) определяется частотой колебаний электромагнитного поля световой волны, а не длиной волны. Поэтому и глаз, и фотопленка воспримут свет в воде как красный. 20.5. Решение. С поверхности Земли мы видим небо голубым из-за рассеяния солнечного света в атмосфере1). Это происходит потому, что коротковолновая часть видимого солнечного света (фиолетовый, синий, голубой участки спектра) рассеивается сильнее всего. Благодаря исследованию Рзлея выяс- нилось, что рассеяние происходит на неоднородностях воздушной среды, малых даже по сравнению с длиной волны света (это быстро возникающие и исчезающие в результате хаотического движения молекул микроскопические сгущения и разрежения). Вследствие такого рассеяния голубая часть спектра света, идущего к нам прямо от Солнца, ослаблена, поэтому Солнце кажется нам не белым, а желтоватым. Возникает вопрос: чем же тогда объясня- ется белый или серый цвет облаков, в которых также происходит рассеяние солнечного света. Дело в том, что рассеяние света в облаках происходит на капельках воды или кристалликах льда, размеры которых велики по сравнению с длиной волны света. При этом световые волны различных частот рассеиваются примерно одинаково. 20.6. Решение. В этом случае свет от Солнца или Луны проходит значительно большее расстояние в атмосфере, чем в другое время суток. На более длинном пути заметно рассеиваются не только синие и голубые составляющие солнечного света, но и желтые и зеленые. Остаются лишь световые волны, которые рассеиваются слабее других. В результате Солнце или Луна приобретают крас- новатый оттенок. 20.7. Решение. На темном фоне мы наблюдаем дым в рассеянном свете, а на фоне светлого неба — в проходящем свете; поэтому речь идет о том же явлении, которое является причиной голубого цвета неба и желтого цвета Солнца при наблюдении с поверхности Земли. Достаточно мелкие частицы дыма (см. задачу 20.5) рас- сеивают коротковолновую часть солнечного излучения гораздо сильнее, чем длинноволновую. Поэтому рассеянный дымом белый 11 Все побывавшие в космосе (или по крайней мере за пределами плотных слоев атмосферы) описывают черное небо И ослепительно белое солнце.
Ответы, указания, решения 552 свет приобретает синеватый оттенок, а свет, прошедший через дым, приобретает желтоватый оттенок. 20.8. Указание. Снег в тени освещен светом, рассеянным атмосфе- рой. См. задачу 20.5. 20.9. Решение. Наше восприятие цвета обусловлено физиологией зрения: в природе же есть лишь световые волны различных частот. Монохроматическая волна не может быть получена смешиванием волн с другими значениями частот, поэтому, строго говоря, приве- денное утверждение неверно. Однако наш глаз не является таким совершенным анализатором света, как, например, спектроскоп. На сетчатке размещены «цветочувствительпые» приемники всего трех типов, максимумы чувствительности которых приходятся соответственно на красную, синюю и зеленую области спектра. Наш мозг формирует восприятие цвета, сравнивая интенсивность нервных импульсов, приходящих от «красных», «синих» и «зе- леных» светочувствительных элементов. Смешивая красный, си- ний и зеленый свет, всегда можно получить такое же цветоощу- щение, какое дает любой световой сигнал (это используется, например, в цветном телевидении). Итак, если ограничиться рассмотрением физиологии зрения, приведенное утверждение мож- но считать справедливым. Физически же оно неверно: достаточно пропустить «телевизионный» желтый свет через призму или диф- ракционную решетку спектроскопа, чтобы убедиться, что на са- мом деле это смесь красного и зеленого цветов. 20.10. Ответ: L < 1,8 м. Решение. Углы преломления лучей при входе в призму определя- . „ sina ются из закона преломления sinpi = ~‘> угол падения на вторую грань призмы (см. задачу 18.30) a.> = (р - pi; угол преломления при выходе из призмы определяется соотношением sinpz = nsina2 = nsin(<p - pi) = sin2a sina . >—5--?-ч— - ncoscp—— - sincpVn - sm'a - sinacosq). Подставляя в это выражение вместо п величины пк и п*, находим углы преломления крайних лучей спектра (красного и фиолетового): рг» = 47,4°, Рг* = 50,6°. Итак, вышедший из призмы пучок разноцветных Лучей расходится под небольшим углом 0 = Ра* - Ра< = 3,2° = 0,056 рад. Ширина пучка достигает величины s на расстоянии = — = 1,8 м (мы пренебрегаем шириной пучка = nsincp
553 Оптика при выходе из призмы, считая размеры призмы малыми). Экран должен быть перпендикулярным световому пучку, вышедшему из призмы. 20.11. Решение. Разумеется, исчезнуть энергия не может. Не происходит и перехода энергии в другие формы (например, во внутреннюю): ведь интерференция происходит и в вакууме. Поэ- тому для ответа на вопрос нужно выяснить, что происходит в соседней области, где волны усиливают друг друга. Если ампли- туда каждой из волн равна А, то в области интерференционного максимума амплитуда результирующих колебаний электромаг- нитного поля равна 2А. Поскольку плотность энергии волны пропорциональна квадрату ее амплитуды, при удвоении ампли- туды плотность энергии возрастет в четыре раза. Итак, в резуль- тате наложения волн энергия в некоторой области не удваивается (как это, казалось бы, должно быть), а учетверяется! «Лишняя» энергия как раз и есть та энергия, которая исчезла из области взаимного погашения волн. Таким образом, при интерференции энергия воля перераспределяется в пространстве. 20.12. Ответ: смещается к красному концу спектра; смещается к фиолетовому концу спектра. Решение. Окраска пластинки обуслов- лена интерференцией световых волн, отраженных от передней и задней по- верхностей (см. рисунок). Если плас- тинка кажется зеленой, это значит, что Ad пучков 1 и 2 составляет целое число k длин волн зеленого света: Ad = ЙХ (условие максимума интерференции). Оптическую разность хода лучей мож- но записать в виде Ad = | + п(АС+ ВС)-АН X Слагаемое учитывает, что при отражении пучка 1 от опти- £ чески более плотной среды фаза колебаний электромагнитного поля меняется на противоположную (А<р = л), т. е. возникает такое X z же изменение фазы, как при прохождении пути 5 (как говорят, £ «теряется полволны»). Множитель п учитывает уменьшение ско- рости света в среде: А<р = ©At = о— = —п, т. е. на пути s в среде
Ответы, указания, решения 554 возникает такое же изменение фазы А<р, как на пути зп в вакууме. h Учитывая, что АС = ВС =------, AD = A&ina = 2Asinatg[3, и ис- cosp sina пользуя закон преломления, света--— - п, получаем sinp М 4 + 2W" - singsinW = А * - зшЪ. cosp Таким образом, условие максимума интерференции принимает вид. (й - 4)А = 2h'ln2 - sin2a. Отсюда видно, что при уменьшении А угла а величина А. увеличивается (цвет пластинки смещается к красному концу спектра), а при увеличении угла а величина А. уменьшается (цвет смещается к фиолетовому концу спектра). 20.13. Решение. Цвета радуги представляют собой чистые спект- .ральные цвета (такой же спектр получается и с помощью призмы). А вот цвета тонких пленок образуются за счет полного или частичного «вычитания» из белого света некоторых его цветных составляющих. Это «вычитание» происходит за счет гашения некоторых световых волн при интерференции; поэтому свет, отра- женный от тонкой пленки, не является спектрально чистым (монохроматическим). 20.14. Ответ: d = 590 нм. Решение. При нормальном падении света (а = 0) интерферен- ционный максимум наблюдается в отраженном свете при (k - i)Ai = 2dn (см. задачу 20.12). Условие интерференционного А минимума имеет вид йХг = 2dn. Из этих уравнений находим k =-----------= 3, d =------------ 2(А. - Аа) 4п(Х! - М 590 нм. 20.15. Решение. Наличие интерференционной окраски только у достаточно тонких пленок обусловлено тремя причинами. Во-первых, если толщина пленки заметно превышает т. н. длину гармонического цуга падающего света (для солнечного света это величина порядка 1 мкм), отраженные от двух различных поверхностей пленки световые волны не являются когерентными (хотя обе они — «продолжения» одной и той же падающей волны) и интерференция этих волн невозможна. Во-вторых, при большой толщине пленки «густота» интерфе- ренционных максимумов и минимумов так возрастает, что мы Перестаем их различать. \
555 Оптика В-третьих, в обычных условиях (когда падающий свет не является монохроматическим) происходит наложение интерфе- ренционных максимумов световых волн различной длины. 20.16. Ответ: х =1,8 мм; не обязательно. Решение. Начнем со второго вопроса. Хотя расстояния от точки О до ис- .точников At и Аз одинаковы, это еще не значит, что обе световые волны приходят в точку О в одинаковых фазах, т. е. что будет наблюдаться максимум освещенности. Ведь коге- рентность источников не исключает, что они испускают волны с различ- ными фазами (когерентность означа- ет лишь то, что разность фаз Л<р оста- ется постоянной). Поэтому в задаче не хватает данных, чтобы ответить на этот вопрос. А вот найти величину х по приведенным в условии данным можно. Воспользу- емся тем, что изменение фазы колебаний одного из источников приводит к одинаковому смещению темных и светлых полос, не изменяя их ширины. Поэтому для вычисления х можно считать Лф = 0. Тогда в точке О наблюдается максимум освещенности, так как OAi — ОАз = 0; расстояние х берется до точки М следующего максимума (см. рисунок): MAi — МАз = X. Обозначив через з половину отрезка ААг, записываем MAi - 'IL2 + (х + з)2, МАз =ylL2 + (х - s)2. Решение уравнения 'II2 + (х + s)2 - 'IL2 + (х - s)2 = X довольно громоздко, однако его можно упростить, используя малость х и s по сравнению с L. Можно воспользоваться формулой из мате- матического приложения или домножить и разделить левую часть последнего уравнения на сопряженное выражение 'IL2 + (х + з)2 + + 'lb2 + (х - з)2, приблизительно равное 2L. Получаем (х + з)2 - (х - з)2 2хз -------------- = X, или = X. Л XL XL Отсюда х - = ~т~г- = 1,8 • 10 м. 2s А1А3 20.17. Ответ: х = = 1,2 мм. 2s Указание. В задаче описан известный интерференционный опыт Ллойда. Интерференция происходит между световыми волнами,
Ответы, указания, решения 556 непосредственно попадающими на экран, и волнами, отражен- ными от зеркала. Можно считать поэтому, что имеются два источника когерентных волн: источник А и его изображение в зеркале At. Далее см. задачу 20.16. Заметим, что из-за потери полуволны при отражении света от зеркала (см. задачу 20.12) источники А и Ai — противофазные. 20.18. Ответ: х = 1,1 мм. Указание. Интерференционную картину соз- дают световые волны от двух мнимых изобра- жений Ai и Аг источника А (см. рисунок). Точки Ai и Аг находятся на расстоянии AiAz = 2d(n - а) друг от друга (здесь к - а = 7t 360 рад) и на расстоянии L + d от экрана (мы учитываем малость угла тг - а). Таким образом, для расстояния между соседними интерференционными полосами получаем (см. задачу 20.16) х = + = 1,1 мм. 2</(л - а) 20.19. Ответ: концентрические темные и светлые кольца; темная полоса. Решение. Интерференционную картину на экране можно рас- сматривать как результат сложения световых волн от источника А и противофазного ему источника Ai (см. рисунок), расположен- ного на расстоянии 21 от экрана (см. задачу 20.17). Точки экрана, равноудаленные от точки О, равноудалены и от любой другой точки прямой AAi. Поэтому результат интерференции двух свето- вых волн в этих точках одинаков; интерференционная картина имеет вид концентрических светлых и темных колец с центром в точке О. .Для точки М экрана на расстоянии г от точки О
557 Оптика оптическая разность хода двух световых волн Ad = | + AtM - AM = 4 + J4lz + г2 - >/F+7. лл ы Считая г « I, получим “4+(я+3-('+й=‘4-5 Ad (см. математическое приложение). Поскольку — представляет X собой «полуцелое» число, световые волны приходят в точку М в противофазе; следовательно, через точку М проходит темная интерференционная полоса. 20.20. Ответ: h - йо(2й + 1), где йо = —= 0,10 мкм; k = 0, 1, 2, ... 4\п Решение. Ослабление отраженного света достигается за счет взаимного ослабления при интерференции двух световых волн (см. рисунок): отра- женной от внешней границы пленки и от границы пленка-стекло. По- скольку 1 < Пп < п, обе волны при отражении «теряют полволны» (см. задачу 20.12). Поэтому оптическая разность хода обеих волн равна 2йпп; она должна составлять нечет- ное число полуволн, т. е. 2hna = 7т(2й + 1), где k = 0, 1, 2, ... £ Итак, h = ~4=(2A + 1) = йо(2й + 1). 4уп Минимально возможная толщина пленки йо = 0,10 мкм (чет- вертьволновая пленка). На практике из-за сложности нанесения очень тонких пленок используют пленки с толщиной, превышаю- щей йо в нечетное число раз. 20.21. Ответ: зеленый; сиреневый. Решение. С минимальным отражением через объектив проходят световые волны с длиной волны, удовлетворяющей условию 2й^п = +. 1) (см. задачу 20.20). При k = 0 получаем X о = 1520 нм, 2
Ответы, указания, решения 558 что соответствует инфракрасному излучению далеко за пределами видимой части спектра; при k = 1 получаем Xi = 510 нм (зеленый цвет); Хг = 300 нм соответствует уже ультрафиолетовому излу- чению (на самом деле это излучение поглощается стеклом). Итак, прошедший через объектив свет приобретает зеленоватый отте- нок. Толщину пленки обычно подбирают именно для пропускания зеленого света, к которому наиболее чувствителен глаз: ведь зеленый цвет соответствует середине спектра видимого света. Наиболее сильное отражение испытывают те световые волны, для которых отраженные пучки 1 и 2 (см. рисунок к решению задачи 20.20) вследствие интерференции усиливают друг друга: 2/iVn = ЙХ'*. При k - 1 получаем X/ = 760 нм; при k = 2 получаем Хг’ = 380 нм; для остальных k будет X’* < 250 нм. Длины волн X/ и Хг' очень близки соответственно к красной и фиолетовой границам видимой части спектра. Значит, отраженный объективом свет имеет крас- но-фиолетовый (т. е. сиреневый) оттенок. Это особенно заметно в случае сложных объективов, содержащих несколько линз. 20.22. Ответ: N а 25. Решение. Билинза дает два изображения At и Аг источника А (см. , ТТ hd т т dF рисунок) на расстоянии Н =-----друг от друга и Li = L------ d-F d-F от экрана (см. задачу 19.16). Расстояние между соседними свет- XLi лыми интерференционными полосами на экране х - (см. зада- ri чу 20.16). Ширина полосы перекрытия на экране двух световых пучков (они отмечены на рисунке различной штриховкой) состав- ляет а = h~±—. Следовательно, N » — =-й (L + d)-_ d ’ х ^Ld-LF-dF) I . • 20.23. . Ответ: г» = 1,2 мм. Рещвние. Описанный эффект обусловлен наличием'тонкой воз-
559 Оптика душной прослойки между линзой и плас- тиной. Интерференция происходит между световым пучком 1, отраженным от выпук- лой поверхности линзы (отклонением этого пучка от вертикального направления мож- но пренебречь), и световым пучком 2, отра- женным от плоской поверхности пластины (см. рисунок). Оптическая разность хода X этих двух пучков Ad = 2h + где h — тол- £ щина воздушной прослойки (слагаемое Х/2 обусловлено «потерей полуволны» при отра- жении от поверхности стекла, см. задачу 20.12). Воздушная прослойка аналогична тонкой пленке переменной толщины. Зависимость h от расстояния г до центра колец можно получить, записав для прямоугольного треугольника АВО теорему Пифагора: R2 — г2 + (R — h}2, откуда h = R - VB2 - г2. Учитывая, что R > г, получим (см. математичес- г2 кое приложение) h = Величина г, соответствующая середине темного кольца с номером k, определяется из условия взаимного X ослабления двух волн при интерференции: Ad = (2k + 1)^, т. е. & 2h = йХ. Отсюда г* = "VfeXB; при k = 3 темное кольцо имеет радиус гз = V3XR = 1,2 КУ3 м. Темное пятно в центре колец соответствует случаю k = 0. В этом случае разность хода Ad = Х/2 обусловлена только «потерей полуволны», т. е. изменением фазы световой волны в пучке 2 на противоположную при отражении от опти- чески более плотной среды. 20.24. Ответ: X — 0,61 мкм. Решение. Формула дифракционной решетки dsincp = йХ позволяет опре- делить угол <р* отклонения спектра й-го порядка от первоначального на- правления (см. рисунок). Расстоя- ние на экране между спектрами й-го и нулевого порядка равно Ltg9*, поэтому 8 = Ыбфз - Ltgtpa. Согласно условию s < L, так что можно счи- тать углы отклонения малыми и
Ответы, указания, решения- 560 заменить их тангенсы на синусы: Т( . . iX s - L(smcp2 - зпмрг) = sd ОтсюдаХ = -у- = 6,1 • 10~7 м. 20.25. Ответ: Ашах = 3. Решение. Согласно формуле дифракционной решетки cfeincp d К — • X X При d - мм = 2 • 10 6 м получаем k < 3,85, т. е. Атах = 3. □Uv 20.26. Решение. Угловое отклонение лучей, прошедших через призму, находящуюся в вакууме (или в воздухе), возрастает с С увеличением показателя преломления п = —. Здесь с — скорость света в вакууме, являющаяся универсальной постоянной; v — скорость света в материале призмы, зависящая от длины волны. Для «красных» волн величина v, как правило, больше, чем для волн, соответствующих другим цветам видимого спектра. Поэтому показатель преломления материала призмы и угол отклонения для красного цвета минимальны. Отклонение света дифракцион- ной решеткой зависит от длины волны согласно формуле dsincp = = АХ; следовательно, меньше всего отклоняются самые короткие световые волны (в видимой части спектра — фиолетовые). 20.27. Ответ: в фокальной плоскости линзы; s = 57 см. Решение. Из дифракционной решетки выходят несколько парал- лельных пучков лучей. Поэтому для получения четкого дифрак- ционного спектра экран должен находиться в фокальной плос- кости линзы. При этом расстояние s связано с углом отклонения спектра третьего порядка соотношением s = Ftgcps. Из уравнения , . , sintps ЗХ азшфз = ЗХ получаем tgфз = г= = ~i~y. (угол фз в дан- VI - знгфз Ver - 9Х2 ном случае не настолько мал, чтобы считать tgфз = зтфз). Таким г 3XF образом, s = ^=== = 0,57 м. 20.28. Решение. В воде длина волны, соответствующая определен- ному цвету, уменьшается в п раз (см. задачу 20.4). Согласно АХ соотношению suup - уменьшается и угол отклонения лучей, а
561 Оптика значит — и расстояние в от соответствующей полосы спектра до центрального максимума (при малых углах отклонение а умень- шается в п раз). Если для получения дифракционного спектра используется (как это обычно и бывает) собирающая линза, то в воде ее оптическая сила резко падает. Поэтому экран окажется намного ближе фокальной плоскости линзы, и четкого дифрак- ционного спектра на нем не будет. 20.29. Ответ: перекрытия не будет. Решение. Спектры порядка k и Л + 1 перекрываются, если kfa > > (Л + 1)Х1, т. е. если k > - -1 = 10. Однако максимальный' Хз — Ai порядок спектра, который вообще может быть получен с помощью d данной дифракционной решетки, ограничен условием k < — = 8. Xi Как видно, полученные неравенства несовместны; следовательно, перекрытие спектров разных порядков в данном случае невоз- можно. 20.30. Ответ: d(sin<p - sina) = kk, где k = 0, ±1, ±2, ... Решение. При наклонном падении света разность хода между волнами, идущими от краев соседних щелей, составляет Ad = AD - ВС “ ABsincp - ABsina = d(sin<p - sina) (см. рис. a). Дифракционный максимум наблюдается, когда Ad = = fel, т. e. d(sin<p - sina) = Kk. Вд/ееь k — любое целое число; k = 0 соответствует случаю <р = а, г. е. прохождению света без изменения направления распространения (при падении белого света именно в этом направлении наблюдается белый, неокрашенный центральный максимум). Значения k > 0 и k < 0 соответствуют дифракционным максимумам по разные стороны от центрального (см. рис. б). « v = 9 Рис. в Рис. б
Ответы, указания, решения 562 21.1. Ответ: v = 0,14с = 4,2 > 10’ м/с; и - 0,87с = 2,6 • 10® м/с. Решение. Релятивистское сокращение длины описывается форму- лой I ~ la'll ~ V2/с2, где /о — длина тела в системе отсчета, в которой I /7 \ I оно неподвижно. Отсюда и = с*\/1 - . При 7- = 0,99 получаем У \ /о' 1о v = 0,14с = 4,2 • 107 м/с; при у = 0,5 получаем v = 0,87с = 2,6 • 10* м/с. 1о 21.2. Ответ: I = 2,0 км. Решение. Разумеется, в справочнике приводится среднее время жизни неподвижных частиц (или, что то же самое, среднее время жизни частиц в связанной с ними системе отсчета). В системе отсчета, относительно которой частицы движутся со скоростью о, То у , , среднее время жизни t = -;=====• («эффект замедления време- У1 - и2/с2 ни»). Поэтому I = пт = -т===^=== = 3,0сто = 2,0 км. N1 - V2/C2 21.3. Ответ: а) V = 0,98с. б) и — 0,98с. Решение, а) Согласно релятивистской формуле сложения скоростей V = = у—---7 = 0,98с. 1 + vu/<r 1 + v~/c2 V - и б) Из соотношения -V = ------— при V = и получаем “ = = °’98с- 1 + v /с Этот результат можно получить и иначе — рассматривая ракету как «неподвижную» систему отсчета, а наблюдателя — как «дви- жущуюся назад» со скоростью и. Поскольку снаряд движется относительно наблюдателя также со скоростью п, его скорость относительно «неподвижной» системы отсчета, т. е. ракеты, 2и и =----= 0,98с. 1 + V-/C2 21.4. Ответ: v = и, с2 + и1 Решение. В системе отсчета К', в которой частица до распада неподвижна, скорости обоих осколков равны по величине и. Значит, первый осколок вылетел в системе отсчета К' назад со скоростью и. Этот осколок по условию неподвижен в лабораторной системе отсчета К. Следовательно, система отсчета К движется относительно К' назад со скоростью и. Очевидно, с такой же
563 Теория относительности и атомная физика . скоростью и система отсчета К' движется вперед относительно К, т. е. и = и. Для определения скорости второго осколка в системе отсчета К воспользуемся релятивистским правилом сложения v + и 2ис2 скоростей: Vt - ------ --- —- 1 + VU/C С- 4- U 21.5. Ответ: и = --—====. 1 + Nl - с/е Решение. Очевидно, скорость и центра масс направлена в ту же сторону, что и V. В системе отсчета, связанной с центром масс, частицы имеют равные по величине и противоположно направлен- гт и - и , „с2 , ные скорости. Поэтому--------- = и, откуда и~ - 2—и + с~ - О, так 1 - Vu/c V с2 ---------- что U = — ± СУСг/и2 - 1 . V Из естественного условия и < с следует, что физический смысл имеет только меньший корень уравнения: с2 —- v и -----сус /V - 1 =----- н 1 + V1 - н /с и „ „ При v < с получаем и = т. е. классический результат. При & v « с (ультрарелятивистский случай) получаем и ~ и. 21.6. Решение. Пусть S = 0, т. е. I = ст. Это означает, что свет, испущенный при первом событии, может быть поглощен при втором событии. Это утверждение справедливо, разумеется, р любой системе отсчета. Поскольку скорость света в вакууме одина- кова в системах отсчета К и К', получаем Г = сх', т. е. S' — 0. Если S < 0, то ст < I. Это означает, что испущенный при первом событии свет не поспевает ко второму событию. Такое же утвер- ждение справедливо и в системе отсчета К', поэтому S' < 0. Аналогично получаем S' > 0 при S > Q. * 21.7. Решение. Заметим, что в данном случае речь идет не о произвольных событиях, а о двух причинно связанных событиях. Поскольку скорость передачи взаимодействия не превышает с, для таких двух событий выполняется условие I < ст, или S > 0 (см. задачу 21.6). В системе отсчета К' при т' = 0 величина S' равнялась бы - Г2, т. е. была бы отрицательной (ведь V не может обратиться в нуль вместе ст' — тогда оба события просто совпали бы). Согласно полученному в задаче 21.6 результату неравенства S > 0 и S' < 0 не могут выполняться одновременно. Следовательно,
Ответы, указания, решения 564 искомой системы К' не существует. Тем более не существует такой системы отсчета, в которой рассматриваемые события происходят в обратном порядке во времени. В такой системе отсчета нару- шился бы принцип причинности: следствие не может опережать по времени причину (а что, если после попадания мяча в окно игрок передумает наносить удар?). Как видим, выполнение прин- ципа причинности в релятивистской теории связано с отсутствием сигналов, распространяющихся со сверхсветовой скоростью. 21.8. Ответ: X = 26,25 см. Решённе. Поскольку движение излучающих атомов перпендику- лярно направлению на Землю, расстояние между источником и приемником волн не меняется. В этом случае частота и длина волны с точки зрения классической физики вообще не должны изменяться (поперечный эффект Доплера отсутствует). Так и происходит, например, для звуковых волн. А вот для электро- магнитных волн, излучаемых достаточно быстрым источником, поперечный эффект Доплера наблюдается. Он связан с различием в промежутках времени, измеряемых в «неподвижной» и «движу- щейся» системах отсчета. По этой причине период Т излучаемых радиоволн с точки зрения земного наблюдателя больше: Т = -~То— -, - v /с2 где То = —. Следовательно, X = сТ - -==== = 1,25Хо = 26,25 см. с VI - р2/с2 Заметим, что при v « с это соотношение принимает вид х = Ц1 + Х-Хо и2 , т. е. —-— = (см. математическое приложение). Отсюда можно Хо видеть, что даже для v = 30 км/с (скорость орбитального движения Земли) относительное увеличение длины волны составляет всего 5 10"’. Таким образом, поперечный эффект Доплера является чисто релятивистским эффектом. 21.9. Ответ: и = 4,0 см/сут. Решение. За время Ato в системе отсчета «Корабль» стебель удлиняется на At = uoAto. В системе отсчета «Земля» удлинение стебля такое же (поперечные размеры одинаковы в обеих системах отсчета), но прошедшее время At больше: At = л ^°, =. Поэтому VI - v/c? и = — = uo^l - и2/с2 = 4,0 см/сут. Фактически мы получили фор- Ai
565 Теория относительности и атомная физика мулу преобразования «поперечной» составляющей скорости любо- го движения при переходе в другую систему отсчета. 21.10. Решение. Описанная в задаче ситуация действительно воз- можна. Однако противоречия с постулатами теории относитель- ности при этом не возникает. Согласно этим постулатам превысить скорость света в вакууме не может ни один материальный объект (частица, волна); рассмотренное же в данной задаче светлое пятно, образованное точками пересечения световых лучей с плоскостью экрана, материальным объектом не является. В самом деле: разве при движении светлого пятна происходит перенос энергии или информации от одной точки экрана к другой? Если между двумя точками А и В экрана поставить непрозрачное препятствие, это не помешает светлому пятну пробежать от А к В — ведь на самом деле свет идет не из А в В, он приходит в точку В совсем с другой стороны — от лазера. Так что полученная сверхсветовая скорость не должна нас смущать. Советуем читателю самостоятельно убе- диться, что светлое пятно от расположенного на Земле лазера перемещается по поверхности Луны со сверхсветовой скоростью даже при довольно медленном повороте лазера. Другая ситуация, в которой геометрическая точка может двигаться быстрее света, рассмотрена в задаче 1.21. 21.11. Ответ: F =----—-----. (1 - и2/с2)1 Решение. Второй закон Ньютона в динамике специальной теории относительности записывается в виде „ dp F = где импульс тела mv В скалярном виде получаем (ср. с задачей 21.14) VI - tr/c2 dv р _ d / v \dt та п___ eEt тс 21.12. Ответ: v = с * = ~ёг~ > см. рисунок. VmV + eT't2 ЧЗеЕ р 3 Решение. Из уравнения движения F = при F = еЕ — const следует: р — eEt. Поскольку р = получаем VI — ir/<r - Рс - U '/тгсг + рг yfmV + e*EV *
Ответы, указания, решения 566 Проанализируем эту формулу. При малых t, когда eEt < тс, скорость растет линейно со временем: eEt v =---------------- т (нерелятивистское движение являет- ся, естественно, равноускоренным). Затем рост скорости замедляется, при eEt > тс получаем v» с (ультрарелятивистский случай). График зависимости u(t) приведен на рисунке. 21.13. Ответ: з = -~('bnic‘ + e2E2t2 - тс). Решение. Для ответа на первый вопрос проще всего воспользовать- ся законом сохранения энергии: тс2 2 . = тс2 + А, V1 - vl/c2 где А — работа электрического поля: А = Fs = eEs. Отсюда _ тс2/ 1 _ \ eE^Vl - v2/c2 Подставляя в это выражение формулу для v (см. задачу 21.12), получаем1’ a = a—^(^Jm2c2 + ё2ЕЧг - тс). В нерелятивистском случае (при eEt < тс) можно записать тс2/^ /„ /eEt\, eEt2 2т (см. математическое приложение). Это, конечно, соответствует еЕ „ равноускоренному движению с ускорением а = —. В ультраре- лятивистском случае (при eEt > тс) получаем з = ct. „ та 21.14. Ответ: F = . V1 - v2/c2 Решение. Очевидно, при v = const ускорение а представляет собой нормальное (центростремительное) ускорение. Воспользовавшись dp соотношением F = мы можем при дифференцировании вы- „ т « мести постоянный множитель -т==^=====. В результате получим: VI - V? с2 ” Эту формулу можао получить и с помощью интегрирования: • =
567 Теория относительности и атомная физика _ d / mv \_________т dv _ та dn>/l - v2/c2' Vl - и2/с2 dt >/i - и2/с2 Сравнивая эту формулу с ответом задачи 21.11, мы видим, что пг ♦релятивистская масса» т,«., = -f==^===~ не всегда является коэф- yl - v2/c2 фициентом пропорциональности между а и F: «поперечная» масса (при нормальном ускорении) и «продольная» (при тангенциаль- ном) — различны. Следовательно, а и F не всегда даже направлены одинаково! А это значит, что масса как коэффициент пропор- циональности между а и F теряет смысл в механике теории относительности. По этой и ряду других причин многие физики предпочитают не использовать термин «релятивистская масса». В современной научной литературе «масса» означает обычно массу покоя. Л т 2л т _ 21.15. Ответ: Т = См. рисунок. Решение. При движении в циклот- роне на протон действует сила Ло- ренца F — evB, где В — индукция однородного магнитного поля. Эта сила сообщает протону центростре- V2 мительное ускорение а = _,где к — а радиус окружности, по которой движется протон. Из сЬотношения та F = -========• (см. задачу 21.14) находим У1 - v2/cr R =____ eByl - и2/с2 Период движения по окружности _ 2nR____2пт_______ v рВу11^Ъ2/с2 _ 2кт , При псе период не зависит от скорости частицы: Т = —(см. еВ задачу 15.13). График зависимости T(v) приведен на рисунке. 21.16. Ответ: W = с^р2 + mV, Wk = W - me2. mc^ Решение. Полная энергия частицы W = а импульс У1 - v2/c2 mv „ Р - Из последнего соотношения находим У1 - v2/c2
Ответы, указания, решения 568 откуда W = с^р2 + т2с2. Кинетическая энергия (энергия движения) представляет собой разность между полной энергией частицы и ее энергией покоя: Wk = W - тс2. Она показывает, какую энергию нужно передать неподвижному телу для разгона до скорости V. 21.17. Ответ: скорость электрона должна быть на 44,5 м/с меньше скорости света в вакууме. ТП с2 Решение. Из соотношения -?======= = mvc2 получаем VI - у2/с2 TTV Поскольку —- = 5,45 10 4 « 1, можно записать ТМр (см. математическое приложение). Таким образом, с - v = тл—I = Z'mP/ = 44,5 м/с. На современных ускорителях электронам сообщается на порядок большая энергия, так что с - и < 50 см/с. И хотя при этом электрон движется почти со скоростью света, сам свет по отношению к такому электрону движется все равно со ско- ростью с! 21.18. Ответ: Am = 6,7 10"® г. Решение. Согласно соотношению Эйнштейна , Wk ти2 „ „ , Ат = —г = -7TV = 6,7 10 кг. с2 2с2 ’ Эту массу можно измерить, если речь идет о пылинке, однако такое изменение массы тысячетонного поезда наблюдать невоз- можно. 21.19. Ответ: на 3,7 • 10"® г. Решение. При замерзании вода отдает энергию Хт, где X — удельная теплота плавления льда. Согласно соотношению Эйн- штейна при этом масса уменьшается на Ат = = 3,7 • 10"" кг. 21.20. Ответ: v = 2,6 • 10® м/с. Решение. Кинетическая энергия Wk частицы связана с ее полной
569 Теория относительности и атомная физика энергией W = соотношением тс2 + Wk = W (здесь т — V1 - v2/c2 масса покоя частицы). При Wk = тс2 получаем >/1 — v2/c2 = & суЗ откуда v = —z— = 2,6 108 м/с. 2! 21.21. Ответ: оР - 1,2 • 107 м/с, о, = 2,7 108 м/с. Решение. Можно воспользоваться законом сохранения энергии 2 ТТ т<^ тс2 + eU = -т======, VI - v2/^ где е — элементарный заряд; отсюда _ с^еЩ2тс2 + eU) тс2 + eU В данном случае удобно измерять энергию в килоэлектронволь- тах: eU = 700 кэВ, энергия покоя протона тРс2 = 938’ 103 кэВ, энергия покоя электрона т„с2 = 511 кэВ. Подставляя в формулу для v эти значения, получаем оР = 0,039с = 1,2 • 107 м/с, v. = = 0,91с = 2,7 • 108 м/с. Поскольку для протона /ПрС2 > eU, можно заменить 2тпРс2 + eU на 2т₽с2 и тпРс2 + eU на /ПрС2, что приводит к хорошо известной формуле классической физики оР = *\/-(при У тР vP « с закон сохранения энергии может быть записан в виде 2 V + и V — и 21.22. Ответ: Vi = -----Vz = ~---------- , 1 + VU/C2 1 - VU/C 2nw Р V(1 - iZ/c^l -uVc2)' Решение. Для определения скоростей vi и ог проще всего восполь- зоваться релятивистским правилом сложения скоростей: V + и V - и 01 =-------Vz = ---------. 1 + vu/c 1 - VU/C2 Суммарный импульс разлетающихся шариков _ mvi mvz____________________________ 2mv________ Р ~ VI - v2/с2 + VI - vz/c2 ~ V(1 - v2/c*)(l ~ и2/с2)' До выстрела суммарный импульс шариков был меньше: _ 2mv >/1 - о2/с2 * Однако следует учесть, что сжатая пружина обладала потен-
Ответы, указания, решения 570 циальной энергией, которая при выстреле перешла в кинетичес- кую энергию шариков: W = 2тс4 -1). '<-п7с2 ' Это энергия пружины в той системе отсчета, в которой пружина неподвижна, т. е. добавка к ее энергии покоя. Энергии W соответ- W ствует добавка mi = кмассе покоя. При движении со скоростью v этой добавке к массе соответствует импульс ______Тиш____ 2mv /1 \ Р' " Vl~^7 “ ТГ-м5/? “ 1‘‘ После выстрела импульс pi передается шарикам. Как и следовало ожидать, ро + pi = р в полним соответствии с законом сохранения импульса. Отметим, что Масса покоя системы до распада больше суммарной массы покоя оОразовавшихся при распаде «осколков» (для ядерной физики это обычная ситуация). п__ ,, 2т 2ти 21.23. Ответ: М = pi=pi = ----—. VI - u 7с2 1 - u2/c Решение. Начнем с определения величины рг, совпадающей с импульсом второго осколка: _ mv, _ 2ти Р* VI - Oa/C2 1 - и1/С2 (см. задачу 21.4). На первый взгляд может показаться, что началь- 2ти л ныи импульс частицы pi - --=====, и поэтому pi < рг. Однако VI - и2/с следует учесть, что масса покоя М исходной частицы не равна 2т. Закон сохранения энергии в системе отсчета К’ (см. задачу 21.4) записывается в виде ,, 2т O^M = ^— Мс2 = 2тс2 V1 - ы7?’ > 2т. Начальный импульс частицы Ми 2ти Р' ~ V1 - и2/с2 ~ 1 - и2/е' Таким образом, pi = рг в полном соответствии с законом сохранения'импульса. 21.24. Ответ: и = 0,5 с; М = 2,3 т. Решение. Согласно законам сохранения энергии и импульса долж- ны ВЫПОЛНЯТЬСЯ СООТНОШеНИЯ!: - ...it Citi Ч » tc TOJ„!
571 Теория относительности и атомная физика ___тс2__ 2 _ Мс2 Vi - v2/^+ тс vi - u*/c*’ ту Ми Разделив почленно второе уравнение на первое, получим 1) и =;—= °*5 с- 1 + VI - v /с Это скорость центра масс (см. задачу 21.5). Подставляя зна- чение и в любое из уравнений системы, получим (1 - V2/с2)* Разумеется, при v < с мы получили бы М ® 2т. При реальных распадах нестабильных ядер и элементарных частиц расчеты усложняются: заметную часть энергии и импульса могут уносить кванты электромагнитного излучения и нейтрино. 21.25. Ответ: W = 1в4(Х) ГэВ. Решение. В обоих случаях Должна быть одинакова скорость и движения одного из протонов в системе отсчета, связанной с другим протоном. При энергии Wi скорость и протона относитель- но неподвижной системы отсчета определяется из соотношения m,,c2 ’ „ L (т}£г\г _ , W, = ~т=' Эта скорость v = v\ 1 - |-=j7-| . При Wi > mP<r VI - v /с У \ Wt > скорость v близка к скорости света. Согласно релятивистскому закону сложения скоростей = 2v = 2сУ1 - (m^/W,f U ~ 1 + v2/(? ~ 2 - (rn.e/Wx)2 ' Именно такую скорость должен иметь протон при бомбардировке неподвижной мишени. При этом его энергия w - ____ЗЖЧЛ _ 1М’) “ ~ ?п,Л 21 Wi f Г При Wi > тРс2 (в данном случае это условие выполняется) 2 ИЛ2 получаем W = —= 10400 ГэВ. Полученная формула объясняет тРс преимущества ускорителей на встречных пучках в области сверх- высоких энергий частиц. Она же показывает, что выигрыш в энергии при использовании встречных пучков существенно боль- ше для легких частиц (например, электронов), / ЦТ, у» 21.26, Ответ: W = = 196 МэВ. Указание. См. задачу 31.26. В данном случае 1У1 и W намного превышают энергию покоя электрона.
Ответы, указания, решения, 572 22.1. Ответ: 6,3 • Ю1’. Решение. Энергия излучения, испущенного за время т = 1 с, равна „ . he п Рх, а энергия каждого кванта равна hv = —. Значит, количество А РтХ испущенных фотонов N = 6,3 10”. he 22.2. Ответ: X = = 3,4 • 10* м. nWo 22.3. Ответ: 17 = 41 кВ. Решение. При разгоне в рентгеновской трубке электроны приобре- тают кинетическую энергию Wk = eU; эта энергия при ударе электрона об анод переходит в энергию излучаемых фотонов и во внутреннюю энергию анода. Очевидно, энергия излучаемого фото- на hv = ~ не может превышать Wt; поэтому = eU. Отсюда A Amin he ,, _ U = —— = 41 кВ. в Amin 22.4. Ответ: и = 580 км/с. Решение. Работа выхода А для калия равна минимальной энергии фотона, вызывающего фотоэффект: . , he А — /•’Vmin — - • Аямх Согласно уравнению Эйнштейна для фотоэффекта hv = А + —g~, л 1%he(\&ac — X)" к о <ЛВ , откуда у = Л/------------- = 5,8 • 10* м/с. ’ тХХш 22.5. Ответ: v < v0 + -ч- = 1,3 • 10м Гц. л 22.6. Ответ: Ua — 0,51, В. Решение. Запирающее напряжение U связано с максимальной кинетической энергией фотоэлектронов Wk соотношением Wk = eU. Поскольку v = приходим к системе уравнений he . „ — = A + d7i, he . — =A + eUt. het 1 11 Решая эту систему, находим Ua - Ui + —|-— —I = 0,51 В. 1 в 'Х« X»' ;,
573 Теория относительности и атомная физика I 21 22.7. Ответ: —. с с Решение. Легче всего ответить на поставленные вопросы, рас- сматривая излучение как поток фотонов с энергией Wo и импуль- Wo , сом (классическая теория электромагнитного поля дает ту ясе величину светового давления). При падении на черную поверх- Wo п ность,фотон поглощается, передавая поверхности импульс — За время Ai на поверхность площадью S падает излучение с энергией W W = ISAt, содержащее фотонов. Значит, за это время поверх- kr о ность получает импульс W Wo W ISi\t Wo с ~ с ~ с Согласно второму закону Ньютона переданный импульс равен FAt = p4S&t. Следовательно, Р- = В случае зеркальной поверх- ности падающие фотоны отражаются, изменяя свой импульс на противоположный. При этом каждый фотон передает зеркальной < 2W, „ поверхности импульс , вдвое больший, чем в случае черной поверхности. Поэтому и Давление излучения. на зеркальную 2Z поверхность вдвое больше: р, = 2рч = —. _ 2/соз2а 22.8. Ответ: р =-----. г с Решение. Давление света на поверхность обусловлено передачей импульса падающих на эту Поверхность фотонов. Пусть энергия! Wo одного фотона Wo, тогда его импульс Ро = При зеркальном Рис, а1^
Ответы, указания, решения 574 отражении от поверхности проекция импульса фотона на ось х не изменяется, а проекция на ось у изменяется на противоположную: Ру = -Pog (см. рис. а). Значит, действующая на поверхность сила направлена по нормали к поверхности, причем каждый фотон . „ „ 2 Wocosa „ передает поверхности импульс Аг = 2Ро„ =------. Согласно вто- рому закону Ньютона можно записать: 2MV<>cosa FAt =----------, с где N — количество падающих за время At фотонов. Для участка поверхности площадью S (см. рис. б) энергия падающего за время At излучения W = LSAtcosa, так что .. W IS At cosa ту —--=---------- W„ Wo _ F 2Jcos2a Отсюда находим р -----------—. о С 22.9. Ответ: зеркальной стороной к Солнцу. Решение. На первый взгляд может показаться, что необходимо сравнивать величины светового давления на черную и зеркальную половины цилиндра. Однако поворот цилиндра зависит не от силы светового давления, а от момента этой силы. Сила F, светового давления на небольшой зеркальный участок поверхности (см. рис. а) направлена нормально к поверхности, т. е. вдоль радиуса (см. задачу 22.8); поэтому момент этой силы равен нулю. Момент же силы F., светового давления на черные участки поверхности стре- мится повернуть цилиндр, как показано на рис. а. На рис. б Рис, а Рис. б-’*1'
575 Теория относительности и атомная физика показано положение устойчивого равновесия цилиндра. При очень малом трении цилиндр может совершать колебания вокруг этого положения. 22.10. Ответ: X = 2,4 • 1(У12 м, и ~ 2,1 • 108 м/с, = 1,41. Ж he Решение. Энергия кванта W, = hv = —. Приравнивая это выра- Л, жение величине тс2, где т — масса покоя электрона, получаем h А = — - 2,4 • 1012 м. Это комптоновская длина волны электрона (см. задачу 22.14). Полученная длина волны соответствует жест- кому рентгеновскому излучению. Импульс соответствующего фо- ft ,, тона р = — = тс. Скорость v электрона можно определить из соот- ношения J-r==?=~ = тс. Получаем v = -7= = 2,1 • 1(г м/с. По- VI - v2/c2 V2 тс2 W 1 скольку W = находим = 1,41. VI - и2/с2 W, Vl — о/с2 22.11. Ответ: не может. Указание. Предположите, что описанный в задаче процесс возмо- жен, перейдите в систему отсчета центра масс электрона и позит- рона и воспользуйтесь законами сохранения энергии и импульса1'. 22.12. Ответ: v > 2,47 1020 Гц. Указание. В отличие от процесса, описанного в задаче 22.11, данный процесс возможен, поскольку в этом случае импульс и энергия передаются веществу. Энергия кванта должна превышать энергию покоя пары электрон-позитрон: hv > 2m<c2. 22.13. Решение. Предположим, что этот процесс произошел и скорость электрона изменилась. Тогда в системе отсчета, в которой электрон после взаимодействия покоится, процесс выглядит так: движущийся электрон поглощает фотон и останавливается. Это очевидным образом противоречит закону сохранения энергии. Однако возможен процесс, при котором электрон, поглотив фотон, испускает другой фотой (см. задачу 22.14). 2Л * ’ €1 22.14. Ответ: ДА = —eihSv, где т — масса поКоя электрона. ” Полученный в задаче 22.11 ответ является частным случаем более общего результата (см. задачи 21.22 и 21.23): распад частицы на «осколки» возможен лишь в том случае, когда масса покоя этой частицы больше суммы масс покоя «осколков» к) ,
Ответы, указания, решения 576 (1) Решение. Применим к рассматриваемому процессу законы сохра- нения энергии и импульса: |Р> = Р + А» Л тс* + We = W + W.. h Здесь ро — начальный импульс фотона (р> = — ); р — его конечный Л импульс (р = ); А — импульс, переданный электрону; Wo = сро л + Да и W = ср — энергии фотона до и после рассеяния; W, = cVm2c2 + р* — энер- гия электрона после взаимодействия с фотоном; тс2 — энергия покоя элек- трона. Применяя теорему косинусов к треугольнику импульсов (см. рису- нок), получаем р,2 = ро2 + р* - 2рорсов®. Подставим это выражение в формулу (1):_____________ Г7~, Л* & 2Л2соа® Л Л V v (X + да.)2 х(х + дх) х х + дх Возведя обе части последнего уравнения в квадрат и упростив полученное соотношение, находим . . Д .л 2Л , .0 ДХ = —-(1 - COS®) = 91П -Х-. тс ' тс 2 Л2 = тс + — - Мы видим, что ДХ не зависит от длины волны падающего излу- чения. Поскольку величина ДХ весьма мала — она характеризу- ется комптоновской длиной волны электрона = 2,4 10~12 м), эффект Комптона наблюдается только для достаточно коротковол- новых излучений. За открытие (1922 г.) и объяснение описанного В этой задаче эффекта американский физик Артур Комптон был удостоен Нобелевской премии. 22.15. Ответ: Wk = 0,10 МэВ, v = 1,6 10* м/с, ® = 33°, а = 58°. Решение. Согласно полученному в задаче 22.14 результату X» - Xi = = - cos®). Следовательно, 0 = arccos^l - ) = 33е. Кинетическая энергия электрона равна энергии, потерянной фотоном: Wk = = 1,66 ПГН Дж - 0,10 МэВ. Ai Ла Полная энергия электрона W - тс* + Wk = 0,61 МэВ. Посколь- . • i ;; ’ • !.>Г-Г<.<ТОф T-JRMH М?'
S77 . Теория относительности и атомная физика /ПС2 «У w = получаем v = с ) » 0,55 с = 1,6 • 10® м/с. Разумеется, при такой большой ско- рости электронов применять законы клас- сической механики нельзя. Для опреде- ления угла а найдем импульс электрона vW р, = -2- и применим теорему синусов к треугольнику импульсов (см. рисунок): Р° _ Р sin® sina’ h где р = — — импульс рассеянного фотона. Отсюда Ха Ac2sin® _ ое к_0 sina = ———— = 0,85 и а - 58°. XauVF 22.16. Ответ: сила увеличится в 2,4 раза. Решение. Пусть за время At на поверхность пластины попадает . ~ , Лс h фотонов. Энергия каждого из них Wo = nv = —, импульс р» = —. X X Поскольку Wo меньше работы выхода электронов с поверхности серебра, все фотоны зеркально отражаются, передавая пластине суммарный импульс 2Npo. На пластину действует сила 2Npa 2N h At ~ At X* После поворота пластины фотоны начинают выбивать с поверх- ности лития фотоэлектроны. Из уравнения Эйнштейна для фото- эффекта находим скорость фотоэлектронов: Ft = X* v где А — работа выхода для лития. Каждый вылетающий фото- электрон передает поверхности импульс pi = nw. Следовательно, на поверхность лития при падении света действует сила Fi = Fi + —- 4) lOOAt V " X > (мы пренебрегли тем, что первое слагаемое в этой формуле должно - 1 быть уменьшено на своей величины, потому что фотон, вызвавший вылет фотоэлектрона, не отражается, а поглощается).
Ответы, указания, решения 578 Итак, he Fz _ - А, , Fi * 200ft Такое увеличение силы объясняется тем, что импульс pi фото- электрона во много раз превышает импульс ро выбившего его фотона (в данном случае — = 274). Ро Wi - W2 22.17. Ответ: о = с~-- Wi + W2 Решение. Приведенные в условии данные позволяют, применив законы сохранения, найти энергию W и импульс р частицы: W, - W2 W = Wi + W2,p = —2----- Wo (мы учли, что импульс фотона с энергией Wo равен ). Разделив друг на друга формулы для импульса релятивистской частицы , mv . тс2 (р = ТГ? ) и Для ее энергии (W = -====== ), получаем вы- VI - о /с VI - о /с2 pc2 Wi-W2 ражение для скорости частицы: о = = с——. w Wi + Wz „ sin(0i + ©а) _ 22.18. Ответ: v = с--------— при 01, 02 не равных 0 или л. sin0i + sin02 Решение. Запишем законы сохранения энергии и импульса: W = ftvi + ftv2, р = pi + Рг. Здесь W, р — энергия и импульс распав- шейся частицы, pi и рг — импульсы фотонов, vi и V2 — их частоты. Восполь- зовавшись теоремой синусов (см. рисунок), получаем sin(0i + 02) sin0i р = pi-----—----, pz = pi . 31П02 SH102 _ ftvi hvt Поскольку pi = — , р2 = можно записать: ftvisin(@i + 0з) . /< ainOii csin02 ' sin02' t on irn P^ 8in(0« + 02) Отсюда (см. задачу 22.17) P = == c--——-— •г sin0i + sin02 Заметим, что если sin®» “ sin®2 «= 0, задача не имеет решения (Й>. с задачей 22.17). !
579 Теория относительности и атомная физика и 22.19. Ответ: 0„lin = 2arccos—. с Решение. Поскольку sin©i * О и sin02 * 0, можно воспользоваться полученной в задаче 22.18 формулой sin(0i + 02) п _ с-—------— sin0i + sm©2 Записав ее в виде + 02\ СОЦ—2~~) /01-02\’ соЦ—2~) 01 - 02 , где © = 01 + 02 — угол разлета фото- 0 и I получим cos „- = -сон „ лл С нов. Из этого соотношения следует, что 0 > 2arccos—, причем Л v минимальное значение угла разлета 0mh, = 2arccos— достигается при 01 = 02, т. е. при симметричном разлете двух фотонов одина- ковой частоты. 22.20. Ответ: и = 0,27с. Указание. Мюон и нейтрино имеют равные но величине импульсы р. Энергия нейтрино равна ср, энергия мюона W2 = tr/m^c2 + р2 (см. задачу 21.16). Согласно закону сохранения энергии 7П1С2 = cVofe.2C2 + р2 + рс. тп2 - тг2 рс2 • т2 - т2 Отсюда р = с—~------, v = = ——------с = 0,27с. 2/П1 Wi /ni2 + т2 22.21. Ответ: X, - 1,7 10 14 м, Х2 = 3,6 10 13 м. Решение. Кинетическая энергия частиц после разгона одинакова: Wk = eU — 3,0 МэВ. Эта величина намного меньше энергии покоя протона, так что движение протона можно считать нерелятивист- ским. Поскольку _ m"v* _ Р"2 ТТ к — Л Л f 2 2/п„ находим Pt, = 42mi,Wk и Xi = — = 1,7 • 10-14 м. Энергия же покоя А электрона ггм* существенно меньше W, т. е. движение электрона является релятивистским. Из соотношения тл? + W> = е^р.* + т2с2 L 2nM!* . . Л .. 4 находим р. = V У1 + ~wT’’ слеД°вательио> = — = 3,6 • 10"
Ответы, указания, решения 580 Заметим, что применение для электрона формул классической механики дало бы значение Х2 = ХА/— = 7,1 • 10-” м, завьппен- X т. ное почти в 2 раза. 22.22. Ответ: v = 1300 м/с. Решение. Из условия задачи следует, что де-бройлевская длина h , „ волны электрона такая же, как длина волны света: — = X-Отсюда и = — = — = 1300 м/с. Для электронов это очень маленькая ско- тта тк рость. Обычно де-бройлевская длина волны электронов намного меньше. Так, в атоме водорода (см. задачи 22.23 и 22.24) она порядка 0,3 нм. е е2 22.23. Ответ: v = . , W = - У4пеоттаг 8л&>г Решение. Электрон вращается по круговой орбите под действием е2 силы Кулона А = -------сообщающей ему центростремительное ЧЛЕол V2 . тт ускорение а = —. Из второго закона Ньютона находим е .. тт, mv2 е2 Механическая энергия электрона W - Ич + И/, = —=-----------. * 4пеог Подставляя в это выражение значение v из формулы (1), получаем е2 W = - —----• Полная энергия получилась отрицательной, как это 8лбог и должно быть для связанной частицы: отрицательная потен- циальная энергия притяжения превышает по модулю кинетичес- кую энергию, и поэтому частица не может уйти «на бесконеч- ность», т. е. сколь угодно далеко от притягивающего центра. 22.24. Ответ: г = 5,3 • 1(ГП м, Ж = 13,6 эВ. Решение. Де-бройлевская длина волны электрона в атоме водорода X,- — - _ h+l'Litw' ~ р~ тм ~ еу т, go^2 (см. задачу 22.23). Из условия X = 2пг следует, что г = —-— = пет. •* 5,3 * НТ11 м. Для ионизации атома (т. ё. для удаления электрона на бесконечность) надо сообщить электрону такую энергию, чтобы ' _. ... .if .. . , _:
581 Теория относительности и атомная физика полная энергия электрона стала неотрицательной. Поскольку е2 . . полная энергия электрона в атоме W --------, находим Wi = |W] = 8ябог г* = = 2,2 10-" Дж =13,6 эВ. 8ябог Согласно теории Бора именно по такой орбите вращается электрон в атоме водорода, когда атом находится в основном состоянии. Некоторые из читателей, несомненно, уже знают, что у электрона в атоме нет никаких орбит: его движение вообще нельзя описывать как движение классической частицы по опре- деленной траектории. Теория Бора представляла собой промежу- точный этап между классической физикой и квантовой механи- кой, но для атома водорода уже эта теория позволила получить правильный порядок размера атома и точное значение энергии ионизации (она действительно равна 13,6 эВ), а также частоты линейчатого спектра водорода. 22.25. Ответ: 12 ч; 7,2 ч; 693 сут. Решение. Легче ответить на первый вопрос: если через сутки из каждых четырех атомов остался один, то сутки составляют ровно два периода полураспада, т. е. период полураспада равен 12 ч. В общем случае из соотношения N = No • 2~,/Т находим Т =----------, logzCVo/jV) где t = 1 сут, No — начальное число атомов, N = No - AN — число нераспавшихся атомов (здесь AN — число распавшихся атомов). При AN = 900 получаем Т = 7,2 ч; при AN = 1 получаем Т = 693 сут. В последнем случае, при AN < No, формулу для Т можно записать (см. математическое приложение) в виде у,_________t .______tNo\n2 ' -logXl - AN/No) AN 22.26. Ответ: t ° 80 сут. 22.27. Ответ: 1,8 10*°. Решение. В образце руды одновременно идут два процесса: распад урана, увеличивающий число атомов тория, и распад тория, уменьшающий число его атомов. Если эти два процесса не компенсируют друг друга, число атомов тория изменяется со временем. Каков характерный масш- таб времени такого изменения? Для ответа* на этот вопрос следует учесть, что период полураспада;урана во много разбольще, чед период полураспада тория. Поэтому масштаб времени определи-
Ответы, указания, решения 582 ется более «быстрым» процессом, т. е.. распадом тория. Следова- тельно, каким бы ни было начальное количество атомов тория, через промежуток времени, малый по сравнению с Tv, но большой по сравнению с Тп (скажем, через тысячу лет), число атомов тория перестанет зависеть от времени (и не будет зависеть от своего начального значения!). Это значит, что количество распадаю- щихся за небольшое время т будет равно количеству «рожда- ющихся» за это время атомов тория (т. е. распадающихся атомов урана). Число распадающихся за время х <Т радиоактивных атомов AN - No - No 2’,т = Nox].n2/T (см. математическое прило- жение). Приравнивая количества распадающихся за одно и то же ЛГть Nv „ время атомов урана и тория, получаем Поскольку 1 Th i U Nv HiNa М (М — молярная масса урана), находим окончательно 3? Nn = Nai-пТп MTv = 1,8 10’°. 22.28. Ответ: 39,2 МэВ и 5,6 МэВ/нуклон для ядра лития; 225 МэВ И 8,3 МэВ/нуклон для ядра алюминия. Решение. Дефект масс атомного ядра массой т, определяется из соотношения Am = Zrn.. + Nm„ - тя, где Z — атомный номер элемента, N = А - Z — число нейтронов в ядре, А — массовое число. Считая т, = т„, - Zm,, где т», — масса нейтрального атома (дефект масс при связывании электронов с ядром ничтожно мал), получаем Am = Z(m}, + т.) + Nm„ - т„ = Zma + А’т;1 - т«,. Здесь тн = т,. + т.. — масса атома водорода 1Н. Энергия связи Ес, = Ате2. Для ядра Ил получаем Ат = Зти + 4т„ - ты = 0,0421 а.е.м.; К, = 0,0421 а.е.м. • 931,5 МэВ/а.е.м. = 39,2 МэВ; ф = 5,6 МэВ/нук- А, лон. Для ядра iaAl находим Ат = 0,242 а.е.м., Ё,-., = 225 МэВ, = 8,3 МэВ/нуклон. & 22.29. Ответ: 5 а-распадов и 3 [5-распада. Решение. При Р-распаде массовое число ядра не изменяется, а при а-распаде оно уменьшается на 4. В рассматриваемом случае мас- совоечисло уменьшилось на 2Q, (Сдедо1доельно. произошло неть сьраспадов.При каждом из них атомный иомер ядоачуменьшался
583 Теория относительности и атомная физика на 2, т. е. в результате одних только а-распадов получилось бы ядро с атомным номером 90- 5*2 = 80. Следовательно, для того, чтобы образовалось ядро с атомным номером 83, должны про- изойти три Р-распада (при каждом из них атомный номер уве- личивается на 1). 22.30. Ответ: 17,6 МэВ. Решение: Запишем уравнение ядерной реакции: ?Н + ?Н > 1Не + on. Масса покоя тг образовавшихся частиц меньше массы покоя mi частиц, вступивших в реакцию1’, па Am = 1,89 • 102 а.е.м. Таким образом, при данной реакции выделяется энергия Wo = Amc2 = = 1,89 1О2 а.е.м. *931,5 МэВ/а.е.м. = 17,6 МэВ. 22.31. Ответ: ш» = 50 т. Решение. При единичном акте термоядерного синтеза выделяется энергия Wo = 17,6 МэВ и расходуется ядерное топливо массой то = 5 а.е.м. Следовательно, израсходовав топливо массой т, мы т освобождаем энергию W = Wo—. Вода при этом получает количес- тв тво теплоты Q = T]W, где т] = 0,1. Отсюда получаем Q nW» c,At c,m0At Это в 50 миллионов раз больше массы израсходованного термо- ядерного горючего! 22.32. Ответ: "В + Ше -> l<N + in; W = 0,17 МэВ. Решение. В ядерную реакцию вступают ядра с суммарным заря- дом, равным 7 элементарным зарядам, и массой около 15 а.е.м. Пользуясь законом сохранепия заряда, а также тем, что измене- ние массы пркоя при ядерных реакциях намного меньше 1 а.е.м., приходим к выводу: наряду с нейтроном образуется ядро с зарядом +7 элементарных зарядов и массой около 14 а.е.м. Это — ядро PN. Следовательно, уравнение реакции имеет вид "в + Ше -+ m + In. В этой реакции масса покоя частиц уменьшается на Ат - = 13 • 10 ‘ а.е.м. Прн этом выделяется энергия W = Ате2 = 0,17 МэВ. 22.33. Ответ: ГВ + > SL1 + Ше; W = 2,8 МэВ. • •• .. .- .'1 ” Заметим, что для вычисления Лт не обязательно из приведенных в та&ИЙЙ (<Ш* приложение 18); масс атомов вычитать массу электронов — это пряйедгг к измеяемп»т<ишэ наодну и ту лив величину (2m,) в не намелит величии** Дж.
Ответы, указания, решения 584 22.34. Ответ: W w 2,2 МэВ; v = 3,4 • 10й Гц. Решение. Дефект масс ядра дейтерия Д/и = тР + тп - т„ = тн + т„ - ть = 2,4 • 10 3 а.е.м., где т. — масса дейтрона, тн — масса атома 1Н, то — масса атома iH. Следовательно, минимальная энергия гамма-кванта W W = Ате2 = 2,2 МэВ. Частота v = ~ = 3,4 • 10я Гц. При решении задачи мы не учитывали кинетической энергии освободившихся нуклонов, поскольку она невелика (подробнее см. задачу 22.35). 22.36. Ответ: Wk = 3,3 МэВ. Решение. Может показаться, что ответ на этот вопрос (2,2 МэВ) уже получен в задаче 22.34. Однако это не так. При энергии 2,2 МэВ W импульс гамма-кванта р, = — = 1,2 • 1021 кг-м/с, а импульс про- тона р,, = >/2/nPWk = 3,4 • 1020 кг • м/с (при Wk « т.рС2 можно поль- зоваться формулами классической механики). Как видим, им- пульс протона почти в 30 раз больше. Максимальная кинетичес- кая энергия продуктов реакции между протоном и дейтроном 2 Wm.« * « 1 МэВ, что весьма близко к величине, полученной при решении задачи 22.34. Поэтому кинетическую энергию обра- зующихся в реакции частиц обязательно следует учесть (при 2 взаимодействии гамма-кванта с дейтроном Wm« * yr— ® 1(У3 МэВ, что намного меньше поглощаемой в реакции энергии; именно это и позволило не учитывать кинетической энергии продуктов реакции при решении задачи 22.34). Минимальная величина Wk~ mvvo2 2 (vo — скорость налетающего протона) соответствует случаю, когда после разрушения ядра все три нуклона движутся с одинаковой скоростью v (см. задачу 4.48). Считая тр = тп, Уо m получаем из закона сохранения импульса и = -у. Тогда О Wk = Дт • с2 + 2 Л 2 = Дт • с2 + О Отсюда Wk = ——5--= 3,3 МэВ. Как видим, протон должен иметь энергию, существенно большую, чем гамма-квант. 22.36. Ответ: Wo = 4,4 МэВ. Решение. Масса покоя продуктов ядерной реакции на Дт =
585 Теория относительности и атомная физика = 3,0 10 ’ а.е.м. превышает массу покоя вступивших в реакцию частиц. Значит, при реакции поглощается энергия Wi = 6т • сг == = 2,8 МэВ. Однако для осуществления реакции а-частица должна иметь большую кинетическую энергию: ведь часть этой энергии перейдет в кинетическую энергию W2 ядра бора и нейтрона. Согласно полученному в задаче 4.48 результату максимально возможная часть энергии а-частицы будет «использована» для ядерной реакции, если продукты реакции (ядро бора и нейтрон) будут двигаться с одинаковой скоростью v. Величина v определя- ется из закона сохранения импульса. Округляя массы частиц (до . 4vo величин, кратных а.е.м.), находим v = -jj-, где Vo — начальная скорость а-частицы. Тогда i\lma\v2 4 тмо2 4 W2 = \-l~)-2 = Т1 -2~=lj4yo; закон сохранения энергии принимает вид чг 4 Wo Wo = Wi + -j-p. Отсюда Wo = —= 4,4 МэВ 22.37. Ответ: W2 = 14,9 МэВ. Решение. Поскольку Wi намного меньше энергии покоя любой из участвующих в процессе частиц, мы можем пользоваться закона- ми классической механики. Со- гласно полученному в задаче 22.30 результату в данной реакции вы- *н деляется энергия W = 17,6 МэВ. Поэтому закон сохранения энер- гии принимает вид: Wi + W = W2 + Wa Pi (Wa — кинетическая энергия образовавшихся а-частиц). Запишем также закон сохранения импульса: pi = р2 + рз (см. рисунок). Поскольку р2 ± pt, получаем р» = 4р* + р2\ Тогда w _ ГПзУ»2 _ р»2 _ (?niUi)2 + (/П2У2)2 ’ “ 2 ~ 2т» ~ 2т» , Полагая mi = 2т2, тз = 4/пг, получаем W» = + -jW2. Под- ставляя это выражение для Wa в формулу закона сохранения энергии, находим W» - 0,4(2W + Wi) = 14,9 МэВ.
Ответы, указания, решения 586 22.38. Ответ: Wn = 0.61 МэВ. Решение. Бомбардировка литиевой мишени нейтронами вызывает ядерную реакцию JU + on -> ?Н + Ше. Масса покоя образовавшихся ядер на Ат = 5,14 • 101 а.е.м. меньше массы покоя вступавших в реакцию частиц, т. е. при реакции выделяется энергия W = Ат • с2 = 4,8 МэВ. Если не учитывать энергии, уносимой гамма-излучением, то закон сохра- нения энергии принимает вид W„ + W = Wr + w„, где ТРт — кинетическая энергия ядра трития. Поскольку вели- чины W, Wu намного меньше энергии покоя любой из рассматрива- емых частиц, движение всех частиц является нерелятивистским. Поэтому мы можем пользоваться классическими формулами для кинетической энергии и импульса частиц. Из законов сохранения энергии и импульса для лобового соударения (напомним, что а-частицы вылетают вперед) получаем систему уравнений: т„ипг m-tur „г ~2------2~ = Wa ~ W' m„v„ = m-rUr + mava. Считая mt = Зт», ma = 4/n„ и обозначив — = x, приходим к урав- нению W X1 + 4х - 14 + 6Т1Л = 0 гг а 1 Wa (мы учли, что тм = „ —~ ). Положительный корень этого Д £л & уравнения х = 0,90, т. е. <л = 0,90i>u. Следовательно, W. „ = £ . = £ = 0,61 М,В. 22.39. Ответ: W2 = 11,2 МэВ, О = 161°. Решение. В результате рассмат- риваемой ядерной реакции масса покоя частиц уменьшается на А/п = 1,86 • 1(Г2 а.е.м. Следовате- льно, при реакции освобождается энергия W - Лт-с2 = 17,4 МэВ. Согласно закону сохранения энер- гии 24Fz = Wi + W, откуда W2 *= ”* 11,2 МэВ. Импульсы а-частиц одинаковы (как и их энергии); поэтому из закона сохранения
587 Теория относительности и атомная физика импульса д. = p.i + р.2 следует, что а-частицы разлетаются сим- метрично относительно направления движения протона (см. рису*’ нок) и рР = 2pucos(здесь р„ = pai = р,2). Поскольку кинетические энергии всех частиц намного меньше их энергий покоя, можно использовать формулы классической механики: р, = от,,и,, = ^/2от,,ИЛ, р. = V2m„W2. Тогда ® - 2arccos~ Aj—- 161°. 2 V maWi 22.40. Ответ: т = 5,3 кг. Решение. Пусть за время т — 1 сут = 86400 с произошло N актов деления. При этом выделилась энергия AWo, так что полезная (электрическая) мощность Р = r^NWo/r (здесь т] ’== 0,2). Отсюда N - Рг/riWo; масса израсходованного MN МРт урана т - =------= 5,3 кг (здесь М — молярная масса ЛгА T|?/aWo урана). 22.41. Ответ: 150 000 вагонов; Лот = 4,8 г. 22.42. Ответ: на 65 К. „ „ otNa Решение. Первоначальное количество атомов полония No - где Л4 — молярная масса полония. Через время т -к Т число атомов т•1п2\ Т) (см. математическое приложение). Следовательно, число распав- шихся ядер полония N = No • 2-’/г = . хт ., .. №г!п2 AN = No - N = у—. Масса покоя ядра полония превышает суммарную массу покоя ядра свинца и а-частицы на Лт = 5,8 10 1 а.е.м., поэтому при распаде каждого ядра выделяется энергия Wo ~ Лт • с2 = 5,4 МэВ. Калориметру передается энергия П * АГ rmNА\п2 Q = WoAV = ——Wo, значит, температура калориметра поднимется на л. _ Q т/п#а!п2 Wo С~ ТМ с 65 К' Мы учли, что теплоемкость самого препарата намного меньше теплоемкости калориметра; кроме того, вылет а-частиц за преде- лы калориметра считается невозможным.
Приложения 588 МАТЕМАТИЧЕСКОЕ ПРИЛОЖЕНИЕ 1. Для любых неотрицательных чисел а пЬ выполняется неравен- ство Q * - > 'lab. Для доказательства этого неравенства достаточно привести его к виду ('la - '1b)2 > 0. Равенство достигается только при а = Ъ. Следствие А. Если а + b = s, то ab < -j, причем равенство достига- ется только при 0 = 6 = 2* Следствие Б. Если ab = р, то а + Ъ > 2^р, причем равенство дости- гается только при а = Ъ = '1р. 2. Парабола, являющаяся графиком функции у = ах2 + Ьх + с, b Ь2 „ имеет вершину с координатами х0 = - уо = с - —. При а > О величина </о представляет собой минимальное значение рассматри- ваемой функции, при а < 0 — максимальное. 3. При |х| « 1 выполняются приближенные равенства: -4- « I Т X, VT±^ « 1 ± -=== - 1 т f. 1±х 2 VI ± х 2 Эти соотношения являются частными случаями более общей формулы (1 + х)“ « 1 + ах, где а — любое действительное число. ^.2 Полезное следствие: 'la2 ± х2 ® а ± -д-. ia 4. При |х| « 1 выполняются приближенные равенства: е*х а 1 ± X, а*х « 1 ± xlna (например, 2"х » 1 - xln2), 1п(1 ± х) ® ±х, 10ga(l ± х)*±~. 5. Выражение f(a) = asina + dcosa можно представить в виде: ««)='®Г7Р(^ту81п“+ _ а Ь Сумма квадратов величин ~г^—и , ...равна единице, va2 + Ь2 va2 + Ь2
589 Приложения поэтому существует такой угол 0, что . а ь а Sinp = -7 , cos р =. , , н Va2 + Ь2 н <а2 + Ь2 (т. е. tg0 = ^). Следовательно, /(а) = ^а2 + b2sin(a + 0). Следствие А. Максимальное значение рассматриваемой функции равно 'la2 + Ь2 и достигается при а = - 0 (например, функция /(a) = sina + V3cosa приводится к виду /(а) = 2sin(a + ^); ее мак- _ л . симальное значение равно 2 и достигается при а-= g ). Следствие Б. Сумма гармонических колебаний одинаковой часто- ты представляет собой гармоническое колебание той же частоты. 6. Средние значения периодических функций за период: sintp = costp = sin2cp = соз2ф = О, --------------------------Г- 1 Sin ф = COS ф = Zi 7. Если прямая лежит в плоскости хОу и проходит через точки х и (хо, 0) и (0, по), то ее уравнение имеет вид — + — = 1. Хо I/O 8. Площадь поверхности сферы S = 4лЯ2, где R — радиус сферы. 4 Объем шара V = где R — радиус шара. О
ПРИЛОЖЕНИЯ 1. Плотность веществ Твердые тела Жидкости Газы (при нормаль- ных условиях) Вещество р, кг/м3 Вещество р, кг/м3 Вещество р, кг/м’ Алюминий 2700 Бензин 700 Гелий 0,18 Железо 7800 Вода 1000 Водород 0,09 Латунь 8500 Керосин 800 Воздух 1,29 Лед Медь Никель Свинец Сталь Стекло 900 8900 8900 11300 7800 2500 Нефть Ртуть Спирт 800 13600 800 Кислород 1,43 2. Тепловые свойства веществ Твердые тела Вещество Удельная теплоемкость, кДж/(кг • К) Температура плавления,°C Удельная теплота плав- ления, кДж/кг Алюминий 0,88 660 380 Вольфрам 0,13 3387 185 Железо 0,46 1535 270 Лед 2,1 0 330 Медь 0,38 1083 180 Свинец 0,13 327 25 Сталь 0,46 1400 82 Жидкости Вещество Удельная теплоемкость, кДж/(кг« К) Температура кипения1’, °C Удельная теплота паро- образования”, МДж/кг Вода 4,2 100 2,3 Спирт 2,4 78 0,85 11 При нормальном давлении. ” При нормальном давлении и температуре кипения.
591 Приложения Газы (при постоянном давлении) Вещество Удельная теплоемкость, кДж/(кг* К) Водород 14,3 Воздух 1,01 Кислород 0,91 3. Скорость звука при 20°С, м/с 4. Поверхностное натяжение жидкостей при 20°С, мН/м Воздух 340 Вода 1500 Вода 73 Ртуть 510 5. Зависимость давления ри и плотности рн насыщенного водяного пара от температуры t t,°C рн, кПа Рн, г/м3 Г,°C Рн, кПа рн, г/м3 0 0,61 4,8 20 2,33 17,3 2 0,71 5,6 30 4,24 30,4 4 0,81 6,4 40 7,37 51,2 6 0,93 7,3 50 12,34 82,9 8 1,06 8,3 90 70,11 423,3 10 1,23 9,4 200 1560 7870 18 2,07 15,4 300 8600 46250 6. Удельная теплота сгорания топлива, МДж/кг 7. Температурный коэффициент линейного расширения твердых тел а, 1(Г5 К~‘ Бензин 46 Дерево 10 Керосин 46 Железо 1,2 Медь 1,7 Сталь 1,2 8. Предел прочности на растяжение onq и модуль упругости Е 9. Диэлектрическая проницаемость Вещество от, МПа Е, ГПа Керосин 2,1 1 Алюминий 100 70 Парафин 2 Медь 50 120 Слюда 7 Сталь 500 200 Стекло 7
Приложения 588 10. Удельное сопротивление р при 20°С и температурный коэффициент сопротивления а проводников ' Вещество Р, Ю-’Ом-м а, К~* Вещество Р, 10-8 Ом*м а, К'1 Алюминий 2,8 0,0042 Медь 1,7 0,0043 Вольфрам 5,5 0,0048 Свинец 21 0,0037 Железо 9,8 0,006 Сталь 12 0,006 Латунь 7,1 0,001 Уголь 4000 -0,0008 11. Работа выхода 12. Показатель преломления электронов, эВ (средний для видимых лучей) Литий 2,4 Вода 1,33 Серебро 4,3 Воздух 1,00029 Цинк 4,2 Стекло 1,60 13. Относительная атомная масса некоторых изотопов1*, а.е.м. Изотоп Масса нейтрального атома Изотоп Масса нейтрального атома 1Н (водород) 1,00783 *вВ (бор) 10,01294 iH (дейтерий) 2,01410 “В (бор) 11,00931 1Н (тритий) 3,01605 *вС (углерод) 12,00000 |Не (гелий) 3,01602 (азот) 14,00307 гИе (гелий) 4,00260 *?N (азот) 15,00011 зЫ (литий) 6,01513 ’аО (кислород) 15,99491 зЫ (литий) 7,01601 ’«О (кислород) 16,99913 ®Ве (бериллий) 8,00531 вА1 (алюминий) 26,98146 «Ве (бериллий) 9,01219 14. Приставки для образования десятичных кратных и дольных единиц Кратные Дольные Приставка Обозначение Множитель Приставка Обозначение Множитель тера Т 10й пико п 10’12 гига Г 10е нано н 104* мега М 10е микро мк 10"* кило к 103 милли м 10’’ гекто г 103 санти с 10-* 11 Для нахождения массы ядра необходимо вычесть суммарную массу алектронов.
ОБОЗНАЧЕНИЯ ФИЗИЧЕСКИХ ВЕЛИЧИН, ИСПОЛЬЗУЕМЫЕ В КНИГЕ, И ЕДИНИЦЫ ИЗМЕРЕНИЯ ЭТИХ ВЕЛИЧИН В СИ Механические величины Величина Обозначение Единица Вес Р Н = КГ'М/С Время t, т с Давление Р Па = Н/м2 Импульс Р кг • м/с Коэффициент жесткости (жесткость) k Н/м Коэффициент полезного действия Т) Коэффициент трения скольжения g Масса т, М кг Механическое напряжение о Па = Н/м2 Момент силы М Н-м Мощность N, Р Вт = Дж/с Объем V м3 Перемещение S м Период (равномерного вращения, колебания) Т с Плотность Р кг/м3 Площадь S м2 Путь 1 м Работа А Дж = Н • м Сила Р, N, Т Н = кг* м/с2 Скорость и, и, V м/с Скорость угловая рад/с Ускорение а м/с2 Частота вращения п с'1 Энергия Е, W Дж = Н*м Тепловые величины и величины молекулярной физики Количество вещества V моль Количество теплоты Q Дж Концентрация частиц п м-3 Модуль упругости (модуль Юнга) Е Па = Н/м2 Молярная масса М кг/моль Относительная влажность ф Поверхностное натяжение а Н/м Температура по шкале Цельсия t, 0 °C Температура абсолютная (термодинамическая) Т К Температурный коэффициент линейного расширения а К"1 Теплоемкость С Дж/К Удельная теплоемкость с Дж/(кг • К) Удельная теплота парообразования L Дж/кг Удельная теплота плавления X Дж/кг Удельная теплота сгорания топлива q Дж/кг Энергия внутренняя и Дж
Электрические и магнитные величины Величина Индуктивность Магнитная индукция Магнитный поток Напряженность электрического поля Плотность энергии поля Плотность заряда поверхностная Плотность электрического тока Проницаемость диэлектрическая Сила тока ^Температурный коэффициент электрического сопротивления Удельное электрическое сопротивление Электрическая емкость Электрический заряд Электрический потенциал Электрическое напряжение Электрическое сопротивление Электродвижущая сила (ЭДС) Электрохимический эквивалент Обозначение Единица L Гн = В-с/А В Тл = Н/(А-м) Ф Вб = Тл-м2= В Е Н/Кл = В/м W Дж/м3 а Кл/м2 j А/м2 Е I А а К'1 Р Ом • м С Ф = Кл/В q, Q Кл = А-с Ф В = Дж/Кл и В = Дж/Кл R, Г Ом = В/А S В = Дж/Кл k кг/Кл Величины физики колебаний и воли, оптические величины Длина волны 1 м Интенсивность волны I Вт/м2 Оптическая сила D дптр = м Показатель преломления п Фаза гармонического колебания Ф рад Фокусное расстояние F м Частота колебаний V Гц = с~‘ Частота циклическая 0) рад/с ФИЗИЧЕСКИЕ ПОСТОЯННЫЕ Гравитационная постоянная G = 6,672-10'" Н-м2/кг2 Постоянная Авогадро NA= 6,022-1023 моль'1 Постоянная Больцмана k = 1,3807 • 10'23 Дж/К Универсальная газовая постоянная R — k'NA = 8,31 Дж/(моль-К) Элементарный электрический заряд е = 1,602-10'19 Кл Электрическая постоянная е0 = 8,854 • 10~12 Ф/м (1/(4лео) = 9 109 Н -м2/Кл2) Скорость света в вакууме с = 299 792 458 м/с Постоянная Планка h = 6,626 -10“34 Дж-с = 4,136-10'15 эВ-с h = Л/(2л)=1,О55-10'34 Дж-с = 6,59-10'1вэВ-с Коэффициент пропорциональности между единицами измерения массы и энергии с2 = Е/т = 8,987 • 101в Дж/кг = 931,5 МэВ/а.е.м. (1 а.е.м. = 1,66057-Ю'27 кг; 1 МэВ = 1,602 • 10'13 Дж) Масса покоя электрона гае = 9,1095 • 10~31 кг = 5,486 • 10'4 а.е.м. Энергия покоя электрона ЕОе= тес2 =0,511 МэВ Масса покоя протона тпр = 1,6726 • 10~27 кг = 1,00728 а.е.м. Энергия покоя протона ЕОр = трс2 = 938,26 МэВ Масса покоя нейтрона т„ = 1,6749-10~27 кг = 1,00866 а.е.м. Энергия покоя нейтрона Еоп= тпс2 — 939,55 МэВ
СОДЕРЖАНИЕ Механика 1. Кинематика...................................... 6 2. Законы Ньютона.................................. 17 3. Закон всемирного тяготения...................... 23 4. Импульс, работа, энергия. Законы сохранения..... 25 5. Статика......................................... 34 6. Гидростатика и аэростатика...................... 42 7. Механические колебания и волны.................. 46 8. Комбинированные задачи по механике.............. 52 Молекулярная физика 9. Молекулярно-кинетическая теория. Свойства газов .... 62 Ю. Пары, жидкости и твердые тела................... 70 11. Законы термодинамики........................... 77 Электричество и магнетизм 12. Электростатика............................... 86 13. Законы постоянного тока........................ 99 14. Электрический ток в различных средах...........116 15. Магнитное поле. Сила Лоренца................... . 119 16. Электромагнитная индукция......................124 17. Электромагнитные колебания и волны. Переменный ток. 131 Оптика 18. Законы геометрической оптики...................140 19. Оптические системы и приборы...................145 20. Волновая оптика................................154 Терия относительности и атомная физика 21. Релятивистская механика........................162 22. Кванты, атомы, ядра, частицы...................165 Тесты..............................................172 Ответы, указания, решения......................... 185 Приложения.........................................588
НЕЗАВИСИМЫЙ НАУЧНО-МЕТОДИЧЕСКИЙ ЦЕНТР «РАЗВИВАЮЩЕЕ ОБУЧЕНИЕ» И.М. ГЕЛЬФГАТ П.Э. ГЕНДЕНШТЕЙН Л.А. КИРИК КХЛ ЗАДАЧА ПО ФИЗИКЕ С РЕШЕНИЯМИ Харьков-Москва Центр «Инновации в науке, технике, образовании» 1996
Учебное издание ГЕЛЬФГАТ Илья Маркович ГЕНДЕНШТЕЙН Лев Элевич КИРИК Леонид Анатольевич 1001 задача по физике с решениями Учебное пособие Ответственный за выпуск РМДеревянченко Редактор И.Д.Клугман Технический редактор ГЛ Александрова Компьютерная верстка: И.В .Макаров, П.ВЛаливайко, М.В.Захаров Художник-оформитель ЕЛ.Таборисский Переплет художника АЛ.Удовенко Подписано к печати 13.07.95. Формат 60x90/16- Бумага газетная. Гарнитура школьная. Печать офсетная. Усл. печ. л. 37. Уч.-изд. л. 37. Тираж 30 000 экз. (2-й завод 15 001—30 000 экз.). Зак. 5-172. Независимый научно-методический центр «Развивающее обучение», Украина, 310001, Харьков, ул. Плехановская, 4-а, к 31/2 ИМП «Рубикон» Украина, 310166, Харьков, ул.Вакулина, 11 Научно-практический центр «Инновации в науке, технике, образовании» (ИНТОР) Россия, 121069, Москва, улЛоварская, 14 Отпечатано с оригинал-макета на Харьковской книжной фабрике «Глобус» Украина, 310012, Харьков, ул.Энгельса, 11.